You are on page 1of 481

CHARTERED ACCOUNTANCY PROFESSIONAL II

(CAP-II)

Compilation of Suggested Answers


Income Tax and VAT
(Dec 2003 - June 2022)

The Institute of Chartered Accountants of Nepal


CAP-II Paper 7 - Income Tax and VAT

Publisher: The Institute of Chartered Accountants of Nepal


ICAN Marg, Satdobato, Lalitpur, P.O. Box: 5289
Tel: 977-1-5530832, 5530730, Fax: 977-1-5550774
E-mail: ican@ntc.net.np, Website: www.ican.org.np

© The Institute of Chartered Accountants of Nepal

This compilation of suggested answers is prepared by the Institute of Chartered Accountants of


Nepal. Permission of the Council of the Institute is essential for reproduction of any portion of this
paper.

All rights reserved. No part of this publication may be reproduced stored in a retrieval system, or
transmitted, in any form, or by any means, electronic, mechanical, photocopying, printing,
recording or otherwise, without prior permission, in writing, from the publisher. However, students
of ICAN are allowed to print this compilation of suggested answer for their own study purpose
only.

The compilation of suggested answers is prepared by the Institute with a view to assist the students
of ICAN in their study. The suggested answers presented here are indicative and not exhaustive.

Students are expected to apply their knowledge and write the answer in the examinations taking
the suggested answers as guidance.

Due care has been taken to compile the suggested answers. In case students need any clarification,
creative feedbacks or suggestions for the further improvement on the material, or any error or
omission on the material, they may report to the email education@ican.org.np of the Institute.

Further, printed book for the compilation of suggested answer will be available at the Institute and
the same shall be notified in website. Students willing to have the printed books may purchase
from the store of the Institute after the publication of notice.

May 2023

The Institute of Chartered Accountants of Nepal

The Institute of Chartered Accountants of Nepal 2


CAP-II Paper 7 - Income Tax and VAT
Contents
Part 1: Income Tax Act
Chapter 1: Income Tax Basic..................................................................................................................... 5
Chapter 2: Residency ................................................................................................................................ 16
Chapter 3: Income From Employment ................................................................................................... 22
Chapter 4: Deduction of expenses ........................................................................................................... 73
Chapter 5: Loss Set-off and Carry Forward .......................................................................................... 89
Chapter 6: Non-deductible expenses ....................................................................................................... 96
Chapter 7: Tax Accounting, Method and Timing ................................................................................ 100
Chapter 8: Quantification, Allocation and Characterization of Payments ....................................... 106
Chapter 9: Long Term Contract ........................................................................................................... 119
Chapter 10: Exempt Amount, Business Concession and Facilities .................................................... 122
Chapter 11: Adjusted Taxable Income ................................................................................................. 142
Chapter 12: Net Gain from Disposal of Business Assets and Liabilities ............................................ 150
Chapter 13: International Taxation ...................................................................................................... 163
Chapter 14: Withholding Tax, Installment Tax and Advance Tax .................................................... 181
Chapter 15: Income from Business ....................................................................................................... 208
Chapter 16: Income From Investment .................................................................................................. 287
Chapter 17: Computation of Tax Liability- Mixed Income ................................................................ 300
Chapter 18: Tax Administration, Documentation and Payment of Tax ............................................ 309
Chapter 19: Tax Return, Assessment and Collection of Tax .............................................................. 312
Chapter 20: Miscellaneous ..................................................................................................................... 321
Chapter 21: True False Question .......................................................................................................... 323
Chapter 22: Short Notes ......................................................................................................................... 328
Part 2: VAT Act
Chapter 1: Basic Concept of VAT ......................................................................................................... 341
Chapter 2: Registration and De-registration........................................................................................ 351
Chapter 2: Time and Place of Supply ................................................................................................... 366
Chapter 3: Taxable Value ...................................................................................................................... 368
Chapter 3: Computation of Tax Liability ............................................................................................. 384
Chapter 4: Return, Tax Payment, Fine and Assessment ..................................................................... 430
Chapter 4: Refund of VAT..................................................................................................................... 440
Chapter 5: True False Question ............................................................................................................ 457
Chapter 6: Short Notes ........................................................................................................................... 464
Chapter 7: Miscellaneous ....................................................................................................................... 478

The Institute of Chartered Accountants of Nepal 3


CAP-II Paper 7 - Income Tax and VAT

Part 1: Income Tax Act

The Institute of Chartered Accountants of Nepal 4


CAP-II Paper 7 - Income Tax and VAT

Chapter 1: Income Tax Basic

1) Distinguish between Direct Tax and Indirect Tax. (Dec 2003, 3 Marks, CA Inter)
Answer
If a taxpayer knows the actual tax obligation and makes direct tax payment to government is direct
tax, example- income tax, land registration tax, etc. Under direct taxation, a taxpayer known to tax
authority is same as the taxpayer who actually bears tax liability.

If any tax is collected using an agent and the actual taxpayer does not know how much tax he has
paid, it is called indirect tax. Example, VAT, Excise Duty, etc. Under indirect taxation, a taxpayer
defined by the law is different than the taxpayer who actually bears tax liability.

2) Define the following terms as per Income Tax Act, 2058.


a. Company (June 2017, 2.5 Marks)
Answer:
Company means a company incorporated under prevailing company law and for the purpose of
tax, the term also includes:
1. Organized institutions established under prevailing law,
2. Any unincorporated association, committee, union or society, or registered or unregistered
group of persons other than Private firm and partnership, or a trust,
3. Partnership firm comprising 20 or more partners that are registered or not registered under
prevailing law, retirement fund, cooperative, unit trust, joint venture,
4. Foreign companies, and
5. Any other foreign entity prescribed by the Director General.

b. Natural person (June 2017, 2.5 Marks; Dec 2007, 2 Marks, CA Inter)
Answer:
Natural person means an individual, and the term also includes:
1. Married spouses electing Couple assessment under Sec. 50 (1) of the Act,
2. Widower or widow deemed to be treated as Couple under Sec. 50 (3) of the Act, and
3. Private Firm of an Individual.

c. Permanent establishment (Dec 2015/Dec 2009/ June 2003)


Answer:
Permanent Establishment (PE) means any place where a person carries on its business whether
fully and partially, and the term also includes the following:
1. Any place from where a person carries on business through an agent, who is not a general agent
of independent status (Agency PE),
2. Any place where a person has, is using or is installing main equipment or machinery (Fixed
Base PE),

The Institute of Chartered Accountants of Nepal 5


CAP-II Paper 7 - Income Tax and VAT
3. one or more places of a country where a person provides technical, business or consultancy
services through employee or otherwise for more than 90 days in any 12 months period
(Service PE)
4. a place where a person is engaged in a construction, assembly, or installation project for 90
days or more, including a place where a person is conducting supervisory activities in relation
to such a project (Fixed Site PE)

d. Residential status of an individual (June 2019/Dec 2015)


An individual is resident of Nepal, if any of the following conditions is satisfied:
1. His/her habitual place of abode is in Nepal,
2. S/he stays in Nepal for 183 days or more during any period of consecutive 365 days, or
3. S/he is employee of Government of Nepal, deputed by the employer in any foreign country
during the Income Year.
Habitual Place of abode is such place where the person’s major economic activities are carried
out.
Income Tax Manual issued by Inland Revenue Department interprets a period of consecutive
365 days as Income Year, as such, if any person stays in Nepal for 183 days or more during any
Income year, the person is resident of Nepal during the Income Year.

e. Payment (July 2015, 5 Marks)


“Payment” means the following acts:
1. the transfer by one person of an asset or money to another person or the transfer by another
person of a liability to the one person;
2. the creation by one person of an asset that on creation is owned by another person or the
decrease by one person of a liability owed by another person,
3. the provision by one person of services to another person; and
4. the making available of an asset or money owned by one person for use by another person or
the granting of use of such an asset or money to another person,

f. "Business Asset'", "Depreciable Asset" and "Trading Stock" (July 2015, 5 Marks)
Business Asset
“Business Asset” means an asset used in a business, but excludes trading stock or a depreciable
asset of a business.
Depreciable Asset
“Depreciable Asset” means an asset to the extent to which it is used in the production of income
from a business or investment and that is likely to lose value because of wear and tear,
obsolescence, or the passing of time. Provided that, the term shall not include any trading stock.
Trading Stock
“Trading stock” means assets owned by a person that are intended to be sold in the ordinary course
of a business conducted by the person, work in progress on such assets, and inventories of materials
to be incorporated into such assets. The term excludes asset denominated in foreign currency.

The Institute of Chartered Accountants of Nepal 6


CAP-II Paper 7 - Income Tax and VAT
g. Final Withholding Tax (Dec 2012/June 2006)
“Final Withholding Payments” means dividend, rent, gain, interest and payments made to non-
resident person that are subject to withholding taxes as referred to in Section 92.
The taxes paid on final withholding payments are generally understood as Final withholding tax.

h. Retirement Fund (Dec 2012, 2 Marks, CA Inter)


“Retirement Fund” means an entity established only with the objective of accepting and investing
retirement fund contributions for the purpose of making retirement fund payments to the entity's
beneficiary natural persons or their dependents.

i. Resident Person (June 2011, 5 Marks, CA Inter; June 2003, 5 Marks, CA Inter)
“Resident Person” mean the following persons in respect of an income year:
1. In respect of a natural person:
(a) Place of habitual abode (normal place of abode) is in Nepal,
(b) Who is present in Nepal for 183 days or more in a period of 365 consecutive days, or
(c) who is an employee of Government of Nepal deputed by the employer in any foreign
country at any time during an income year.
2. Partnership firm.
3. In respect of a trust, such trust which:
a. Is established in Nepal,
b. The trustee of the trust is a resident person in the income year; or
c. The trust is controlled in the income year by a resident person, or by a group of persons
including him, directly or indirectly through of one or more interposed entities.
4. In respect of a company, a company:
a. Which is incorporated under the law of Nepal, or
b. the management of which is effective in Nepal in an income year
5. Government of Nepal or Province government
6. Rural Municipality, Urban Municipality, or District Coordination Committee
7. In respect of such entity of a foreign government or a political subdivision of the foreign
government, such an entity-
a. which is established under the law of Nepal, or
b. the management of which is effective in Nepal in an income year
8. An institution or entity established under a treaty or agreement, and
9. A foreign permanent establishment situated in Nepal of a non-resident person.

j. Disabled person (Dec 2010, 3Marks, CA Inter)


“Disabled person” or sometimes called as “incapacitated person” means an individual who, by
reason of mental or physical illness, is incapable of managing their affairs.

k. Non-Business Chargeable Asset (June 2004/Dec 2010)


Non-Business chargeable Asset” means land, building and interest in any entity, or securities, other
than the following assets:

The Institute of Chartered Accountants of Nepal 7


CAP-II Paper 7 - Income Tax and VAT
1. Business asset, depreciable asset or trading stock,
2. Private building of a natural person in the following conditions:
a. having owned it for ten or more years continuously, and
b. resided therein for ten or more years continuously or intermittently by such person.
Clarification: For the purpose of this Clause, “Private building” means a building, land
covered by such buildings and additional land covered by building or one ropani land
whichever is lower.
3. An interest of a beneficiary in a retirement fund,
4. Land, Land and Building and private building of a natural person that has been disposed of at
a price of less than 1 million rupees, or
5. Assets disposed of through transfer by any means within three generations, other than sale and
purchase.

l. Long Term Contract (Dec 2010 3Marks)


“Long-Term Contract” means a contract as referred to in Section 26 with term more than 12
months.
For the purpose of Section 26, “long-term contract” means a contract of following conditions
a. the term of which exceeds 12 months, and
b. that is either a contract for manufacture, installation, or construction, or, in relation to each, the
performance of related services; or a contract with a deferred return that is not an excluded
contract.

m. Entity (June 2010, 5 Marks, CA Inter)


“Entity” means the following institutions and organizations:
1. Partnership, trust, or company;
2. Rural Municipality, Municipality, or District Coordination Committee;
3. Government of Nepal, Provincial Government or local government
4. Public International Organization established under any foreign government, or the provincial
or local government functioning under such a government, or through a treaty; or
5. A permanent establishment of the institution or organization referred to in Sub-Clause (1), (2),
(3) and (4) that is not situated in the country where it is resident.

n. Exempt Entity (Dec 2011, 5 Marks, CA Inter)


“Exempt organization” means the following entities:
1. Following entities registered with the Department as an exempt organization:
a. Social, religious, educational or charitable organizations of a public character established
without profit motive,
b. An amateur sporting association formed for the purpose of promoting social or sporting related
amenities not involving the acquisition of gain by its member,
2. Political parties registered with the Election Commission.

The Institute of Chartered Accountants of Nepal 8


CAP-II Paper 7 - Income Tax and VAT
Provided that, any entity, giving benefit to any person from the assets of, and amounts derived by
the entity except in pursuit of the entity’s function as per its objectives or as payment for assets or
services rendered to the entity by the person, is not exempt from tax.

o. Lease (June 2008, 1 Mark, CA Inter; June 2006, 2 Marks, CA Inter)


“Lease” means a temporary right of a person to use the assets, other than money, of another person,
and the term includes a license, rental agreement, option, royalty agreement, or tenancy.

p. Service Fee (June 2008, 1 Mark, CA Inter)


“Service Fee” means any payment to any person, according to the market value, for the service
rendered by such a person and the term also includes commission, meeting fee (allowance),
management fee, or technical service fee.

q. Income Year (Dec 2007, 2 Marks, CA Inter)


“Income Year” means the period beginning on Shrawan 1 of a year and ending on the last day of
Ashad of the next year.

r. Turnover (Dec 2007, 2 Marks, CA Inter)


“Turnover (transaction)” means a turn-over equivalent to the total amount to be included under
Section 7, 8 or 9 for purposes of computing the income from business, employment or investment
during any income year.

s. Assessable Income (Dec 2007, 2 Marks, CA Inter)


Provision of Section 6:
Subject to Income Tax Act, the assessable income of a person for an Income year from any
employment, business, or investment, or windfall gain shall be as follows:
a. In respect of a resident person, the person’s income from business, employment, investment,
or windfall gain irrespective of the source of such income, and
b. In respect of a non-resident person, the person’s income from business, employment,
investment or windfall gain from income having source in Nepal.
Provided that, the assessable income does not include any income exempt under section 11 or
64 or both.

t. Underlying Ownership(entity) (Dec 2007, 2 Marks, CA Inter)


“Underlying Ownership” means ownership as referred to below:
a. In respect to an entity, the ownership created in the entity on the basis of an interest held in it
directly or indirectly by any natural person, or by an entity in which no natural person has any
interest, through one or more interposed entities.
b. In respect to an asset under the ownership of an entity, ownership of the asset determined on
the basis of the proportionate ownership of persons having underlying ownership of the entity.

The Institute of Chartered Accountants of Nepal 9


CAP-II Paper 7 - Income Tax and VAT
3) State the entities other than those registered under the Companies Act that is treated as
company under the Income Tax Act. (June 2009, 5 Marks)
Answer:
The following entities other than those registered under the Companies Act are treated as company
for the purpose of Income Tax Act:
a. Organized institutions established under prevailing law,
b. Any unincorporated association, committee, union or society, or registered or unregistered
group of persons other than Private firm and partnership, or a trust,
c. Partnership firm comprising 20 or more partners that are registered or not registered under
prevailing law, retirement fund, cooperative, unit trust, joint venture,
d. Foreign companies, and
e. Any other foreign entity prescribed by the Director General

4) Distinguish between: (Dec 2005, 2.5 Marks, CA Inter)


a. “Royalty” and “Payment for natural sources”
Royalty means a payment received in consideration of lease of an intangible asset and includes the
following:
a. Payment received for the use of, or a right to use a copyright, patent, design, model, plan, secret
formula or process, or trademark;
b. Payment for the supply of some know-how;
c. Payment for the use of, or a right to use a cinematography film, video tape, sound recording,
or any other such medium;
d. Payment for the supply of information regarding industrial, commercial, or scientific
experience;
e. Payment for the supply of assistance that is ancillary to the supply of the above-mentioned
matters; or
f. Payment for the total or a partial forbearance with respect to the matters prescribed above.
However, payment for natural resource is not considered as royalty.

b. “Market Value" and "Cost"


Market value is determined by the market forces and has no any connection with the cost. Cost is
the actual outflow of resources in respect of expense or asset.

c. "Underlying ownership" and "Ownership"


Ownership may be direct ownership or underlying ownership. Underlying ownership is a sub-set
of ownership.

The Institute of Chartered Accountants of Nepal 10


CAP-II Paper 7 - Income Tax and VAT
5) Define Income, Windfall gain, Assessable Income and Taxable Income. (2011 June, 5
marks)
Answer:
Income
It means the income earned by any person from employment, business, investment or windfall
gain and the term also includes total amount of such incomes calculated under this Act.

Windfall Gain
“Windfall Gain” means income from lottery, gifts, prize, tips (baksis), award for winning (Jitauri),
and any similar other payment received suddenly.

Assessable Income
Within the meaning under Sec. 6, Subject to Income Tax Act, the assessable income of a person
for an Income year from any employment, business, or investment, or windfall gain shall be as
follows:
a. In respect of a resident person, the person’s income from business, employment, investment,
or windfall gain irrespective of the source of such income, and
b. In respect of a non-resident person, the person’s income from business, employment,
investment or windfall gain from income having source in Nepal
Provided that the assessable income does not include any income exempt under section 11 or
64 or both.

Taxable Income
The taxable income of a person for an income year is the total of the person's assessable income
for the year from each of the following income heads less any reduction allowed for the year under
Sec. 12, 12Ka, 12Kha, 63 or all four sections:
a. Business,
b. Employment,
c. Investment, and
d. Windfall Gain

6) Discuss and elaborate the following quoting relevant sections of the Act :
a. Classification of taxable Income (Dec 2010, 3 Marks, CA Inter)
Answer:
The taxable income of a person for an income year is the total of the person's assessable income
for the year from each of the following income heads less any reduction allowed for the year under
Sec. 12, 12Ka, 12Kha, 63 or all four sections:
a. Business,
b. Employment,
c. Investment, and
d. Windfall Gain

The Institute of Chartered Accountants of Nepal 11


CAP-II Paper 7 - Income Tax and VAT
b. Assessable Income (Dec 2010, 3 Marks, CA Inter)
Answer:
Within the meaning under Sec. 6, Subject to Income Tax Act, the assessable income of a person
for an Income year from any employment, business, or investment, or windfall gain shall be as
follows:
i. In respect of a resident person, the person’s income from business, employment, investment,
or windfall gains irrespective of the source of such income, and
ii. In respect of a non-resident person, the person’s income from business, employment,
investment or windfall gain from income having source in Nepal

Provided that, the assessable income does not include any income exempt under section 11 or 64
or both.

7) Distinguish between "Assessable Income" and "Taxable Income" under the provisions of
Sec. 5 and 6 of the Nepal Income tax Act, 2058. (Dec 2005, 2.5 Marks, CA Inter)
Assessable Income
Within the meaning under Sec. 6, Subject to Income Tax Act, the assessable income of a person
for an Income year from any employment, business, or investment, or windfall gain shall be as
follows:
a. In respect of a resident person, the person’s income from business, employment, investment,
or windfall gain irrespective of the source of such income, and
b. In respect of a non-resident person, the person’s income from business, employment,
investment or windfall gain from income having source in Nepal
Provided that, the assessable income does not include any income exempt under section 11 or 64
or both.

Taxable Income
The taxable income of a person for an income year is the total of the person's assessable income
for the year from each of the following income heads less any reduction allowed for the year under
Sec. 12, 12Ka, 12Kha, 63 or all four sections:
a. Business,
b. Employment,
c. Investment, and
d. Windfall Gain

Difference
Taxable income is such residue of assessable income after applying reductions u/s 12, 12Ka,
12Kha and 63.

8) State under what head the following incomes will fall to be assessed? (Dec 2004, 6 Marks)
a. Interest on Deposits with banks.
b. Interest on Government securities

The Institute of Chartered Accountants of Nepal 12


CAP-II Paper 7 - Income Tax and VAT
c. Prize received in purchase of noodles
d. Scholarship received by a student from a foreign government
e. Free foreign tour provided by a distributor of motorcycle to dealer
f. Donation received by a physically handicapped person
Answer:
Particulars Income Head
Interest on Deposit with Banks
a. Bank deposit of entity Business
b. Bank deposit of natural person, conducting business Business
c. Bank deposit of a natural person, not conducting business Investment
Interest on Government securities
• Securities held by entity Business
• Securities held by natural person in connection to business Business
• Securities held by natural person, not in connection to business Investment
Prize received in purchase of noodles Windfall Gain
Scholarship received by a student from a foreign government Income Tax Free
Free foreign tour provided by a distributor of motorcycle to Business
dealer
Donation received by a physically handicapped person Windfall Gain or Beyond
Scope of Income Tax

9) Mr. B, a resident of Nepal inherits artistic pictures from his father which his father has
drawn during his lifetime. He sells the same for Rs.50 lakhs to art connoisseurs. Discuss the
taxability of this transaction. (Dec 2004, 5 Marks, CA Inter)
Answer:
The gain on disposal of following assets is chargeable to income tax as per Income Tax Act, 2058:
a. Gain on Disposal of Business Asset: as part of assessable income from business
b. Incomings from trading stock: as part of assessable income from business
c. Gain on Disposal of Depreciable asset of business: as part of assessable income from
business
d. Gain on Disposal of Depreciable asset of investment: as part of assessable income from
investment
e. Gain on Disposal of Non-business Chargeable asset: as part of assessable income from
investment
If any asset does not fall within the meaning of business asset, trading stock, depreciable asset or
non-business chargeable asset, the gain on disposal of such asset is not subject to income tax.
Meaning of assets:
Business Asset: “Business Asset” means an asset used in a business but excludes trading stock or
a depreciable asset of a business.
Depreciable Asset: “Depreciable Asset” means an asset to the extent to which it is used in the
production of income from a business or investment and that is likely to lose value because of wear

The Institute of Chartered Accountants of Nepal 13


CAP-II Paper 7 - Income Tax and VAT
and tear, obsolescence, or the passing of time. Provided that, the term shall not include any trading
stock.
Trading Stock: “Trading stock” means assets owned by a person that are intended to be sold in
the ordinary course of a business conducted by the person, work in progress on such assets, and
inventories of materials to be incorporated into such assets. The term excludes asset denominated
in foreign currency.
Non-business Chargeable Asset: Non-Business chargeable Asset” means land, building and
interest in any entity, or securities, other than the following assets:
1. Business asset, depreciable asset or trading stock,
2. Private building of a natural person in the following conditions:
a. having owned it for ten or more years continuously, and
b. resided therein for ten or more years continuously or intermittently by such person.
3. An interest of a beneficiary in a retirement fund,
4. Land, Land and Building and private building of a natural person that has been disposed of at
a price of less than 1 million rupees, or
5. Assets disposed of through transfer by any means within three generations, other than sale and
purchase.
Conclusion
When we go through the definition of these assets, the artistic pictures do not fall under any
category of assets. As such, the amount received from selling the artistic picture to art connoisseurs
is not taxable as per Income Tax Act, 2058.

10) Discuss the taxability or otherwise of the following receipts by quoting the relevant sections
of the Income Tax Act 2058. (June 2005, 10 Marks, CA Inter)
a. Sri Ram Bahadur Khadga receives a pension of Rs. 5,000 per month from Kolkata Police
department.
Answer:
The amount is exempt under Sec. 10, if the pension is paid from public fund of Indian Government
and Mr. Khadga is Nepali citizen. Otherwise, it is taxable under head employment.

b. Sri Dinesh Basnyat receives an award of Rs. 100,000 for exemplary services during his term
in Lebanon from United Nations through the Nepal Army Headquarters.
Answer:
The amount is taxable as windfall gain @ 25%.

c. Sri Man Bahadur was a student with Mr. Bhattarai, who is a teacher by profession. He gets
admitted to Harvard University. After successfully completing his Ph. D. he returns to Nepal
and in token of his respect to the teacher for his blessings and teachings before proceeding
to USA, he pays Rs. 100,000 to Mr. Bhattarai on his return from USA.
Answer
The amount is not covered by the Act and is not taxable.

The Institute of Chartered Accountants of Nepal 14


CAP-II Paper 7 - Income Tax and VAT
d. Mr. Rudra received a dividend of Rs. 50,000 from the dividend distributed by B co. Ltd. B
co. Ltd. distributed a dividend of Rs.600,000 out of Rs.950,000 received as dividend form A.
co. Ltd. who declared a dividend of Rs.2,500,000 and distributed the dividend after deducting
tax at source.
Answer:
As per Sec. 54 (3), dividend tax is not applicable on any distribution made out of such profits
where distribution tax u/s 54 (1) is already levied. Therefore, the amount received by Mr. Rudra is
not taxable.

e. Mr. S. Shrestha is the director of various companies. He receives fee for attending the Board
Meeting. The total fee earned by him for attending Board Meetings during the income year
was Rs.90, 000.
Answer
The amount is taxable, and it is a final withholding payment (assuming per meeting fee is less
than Rs. 20,000). As per Sec. 88, the TDS rate is 15%.

11) Discuss the various types of Tax Credits available under Income Tax Act, 2058.
(Dec 2016, 5 Marks)
Answer
The various types of tax credits are:
a. Female Tax credit
Female tax credit, which is equal to 10% of tax liability before all tax credits, is available to a
person when all the following conditions are satisfied:
1. The person shall be a female,
2. The person shall be resident of Nepal,
3. The person shall generate income only from employment, and
4. The source of such employment income shall be only from Nepal

b. Medical Tax Credit


Medical tax credit, which is lower of 15% of approved medical expenditure and Rs. 750 every
year, shall be available to a resident natural person.
Any excess of 15% of approved medical expenditure over Rs. 750 is carried forward to be claimed
during subsequent income year.

c. Foreign Tax Credit


Foreign tax credit is available to a resident person that generates income having source outside
Nepal and pays tax in such source country.
The maximum amount of foreign tax credit shall not exceed the amount determined by multiplying
foreign assessable income by average rate of tax in Nepal, and such maximum amount shall be
calculated separately for each country. Any excess foreign tax payment during a country is carried
forward to be claimed during subsequent income years.

The Institute of Chartered Accountants of Nepal 15


CAP-II Paper 7 - Income Tax and VAT

Chapter 2: Residency

1) Nepal Electricity Authority (NEA) hired Mr. Peter, a UK Citizen as Infrastructure Expert
with effect from September 17, 2020. He came to Nepal, 7 days before his joining date on
September 10, 2020. He left Nepal on December 20, 2020 and came back to Nepal after
Christmas leave on January 9, 2021, and continued his service for the contract period. After
the expiry of contract, he converted his visa category into Tourist Visa, and went to
Annapurna Base Camp for 17 days, and returned to his home country on April 2, 2021.
Comment about the residential status of Mr. Peter. (Dec 2021, 5 Marks)

Answer:
As per Section 2(Ka Nga), a natural person is resident for an Income Year if he stayed for 183
days or more in 365 consecutive days. As per the clarification of Income Tax Directive, 365
consecutive days should be read as Income Year.
Determination of Residential Status of Mr. Peter for an Income Year 2020-21 (2077-78)
Income Year Period of Stay Number of Days
2020-21 10.09.2020 to (30 - 10 + 1) + 31 + 30 + 20 = 102
(2077-78) 20.12.2020,
09.01.2021 to (31 - 9 + 1) + 28 + 31 + 2 = 84
02.04.2021
Total 186
Since Mr. Peter, a UK Citizen has stayed in Nepal for 186 days i.e., more than 183 days in 365
consecutive days, he is resident for income year 2020-21 (2077-78).
Note: Date of arrival and date of departure shall be inclusive.

2) A company operating in Bangladesh is engaged in production of food products. It conducted


its AGM at Dhaka and elected its board members. The majority of elected board members
represent from Nepal. During FY 2076/77, most of the board meetings took place in
Kathmandu. The company has earned Rs. 20,000,000 from Bangladesh, Rs. 20,000,000 from
India, Rs. 10,000,000 from Pakistan and Rs. 10,000,000 from Nepal during FY 2076/77.
Decide the residential status and taxable income of the Company as per the provision of
Income Tax Act, 2058. (December 2020, 5 Marks)

Answer:
As per Section 2 (Ka Nga) of IT, Act 2058, a company registered in Nepal is always resident in
Nepal. However, for company outside Nepal whose management is effective from Nepal during
the income year is also resident.
Board meeting is the supreme body of the company, where major decisions related to the
companies are taken. The control and management of a unit is usually situated at a place where
the directing powers are situated. Hence, for FY 2076/77, the Company is resident in Nepal.

The Institute of Chartered Accountants of Nepal 16


CAP-II Paper 7 - Income Tax and VAT
As per Sec. 6, worldwide income of a resident person from business, employment, investment and
windfall gain is subject to taxation and income having source in Nepal of a non-resident person
from business, employment, investment and windfall gain is subject to taxation.
In the given case, since the company is resident (or non-resident), its worldwide income, i.e. Rs.
60,000,000 is taxable income as per the provision of IT Act, 2058 (or the amount derived from
Nepal of Rs. 10,00,000 is taxable in Nepal).

3) Nepal Telecommunications Authority (NTA) hired Mr. Eric, a German Citizen as Rural
Communication Infrastructure Expert with effect from September 17, 2018. He came to
Nepal, 7 days before his joining date on September 10, 2018. He left Nepal on December 20,
2018 and came back to Nepal after Christmas leave on January 9, 2019, and continued his
service for the contract period. After the expiry of contract, he converted his visa category
into Tourist Visa, and went to Annapurna Base Camp for 17 days, and returned to his home
country on April 2, 2019. Comment about the residential status of Mr. Eric. (June 2019, 5
Marks)
Answer:
Step 1: Conditions for being Resident
A natural person is considered as resident of Nepal, if any of the following conditions is satisfied:
a. His/her habitual place of abode is in Nepal,
b. S/he stays in Nepal for 183 days or more during any period of consecutive 365 days, or
c. S/he is employee of Government of Nepal, deputed by the employer in any foreign country
during the Income Year.
Habitual Place of abode is such place where the person’s major economic activities are carried
out.
Income Tax Manual issued by Inland Revenue Department consider a natural person as
resident of Nepal if he/she stays in Nepal for 183 or more in an income year. For the purpose of
determining 183-days, ‘travel for’ period and date of entry and exit regardless of time spent in
Nepal is considered but travel through (stay due to transit) is not consider.
Step 2: Testing of Given Facts with the Condition
Mr. Eric’s habitual place of abode is not easily determinable, as such, we will test his physical
presence in Nepal to determine his residential status in Nepal.
Computation of days stay during Income Year 2018/19 (2075/76):
a. First stay, September 10 to December 20: 22 days in September plus 31 days in October
plus 30 days in November plus 20 days in December
b. Subsequent Stay: January 9 to April 2: 23 days in January plus 28 days in February plus 31
days in March plus 2 days in April
Total day-stay in Nepal: 186 days during Income Year 2075/76.
Step 3: Conclusion
Therefore, Mr. Eric is resident in Nepal during Income Year 2075/76.

4) A company operating in Bhutan is engaged in production of Jam products. It conducted its


AGM at Thimpu and elected its board members. The majority of elected board members

The Institute of Chartered Accountants of Nepal 17


CAP-II Paper 7 - Income Tax and VAT
represent from Nepal. During FY 2074/75, most of the board meetings took place in
Kathmandu. The company has earned Rs. 10,000,000 from Bhutan, Rs. 30,000,000 from
India, Rs. 20,000,000 from Bangladesh and Rs. 10,000,000 from Nepal during FY 20X4/X5.
Decide the residential status and taxable income of the Company as per the provision of
Income Tax Act, 2058? (Dec 2018, 5 Marks)
Answer:
A company is resident of Nepal, if any of the following conditions is satisfied:
i. In case it is established/incorporated under prevailing law of Nepal, or
ii. In case the management of the company has been effective in Nepal for an Income Year.

The company is not established as per the law of Nepal. To determine whether the company has
effective management in Nepal or not, we need to understand what effective management is.

As per Organization for Economic Cooperation and Development, the place of effective
management is the place where key management and commercial decisions that are necessary for
the conduct of the enterprise’s business are in substance made. The place of effective management
will ordinarily be where the most senior person or group of persons (for example a board of
directors) makes its decisions, the place where the actions to be taken by the enterprise as a whole
are determined; however, no definitive rule can be given and all relevant facts and circumstances
must be examined to determine the place of effective management. An enterprise may have more
than one place of management, but it can have only one place of effective management at any one
time.
In the given case, the fact that most of the board meetings are held in Kathmandu demonstrates
that Nepal is the place where key management and commercial decisions that are necessary for the
conduct of the enterprise’s business are in substance made. As such, the company’s effective
management is in Nepal during the Income Year.
Therefore, the company is resident.

5) Mrs. Aang Futi has stayed in Nepal in the following periods. Find out the residential status
of her for the income years 20X2/X3 and 20X3/X4. (June 2017, 5 Marks)
Period of Stay
20X1 Falgun 14 to Chaitra 11
20X2 Baishakh 13 to Ashad 26
20X2 Shrawan 3 to Bhadra 25
20X2 Marga 8 to Falgun 13
20X3 Jeshtha 18 to Bhadra 16

The number of days in the relevant months is as follows:


Year (BS) Month No. of Year (BS) Month No. of
Days Days
20X1 Falgun 30 20X2 Poush 30
Chaitra 30 Magh 29

The Institute of Chartered Accountants of Nepal 18


CAP-II Paper 7 - Income Tax and VAT
20X2 Baishakh 31 Falgun 30
Jeshtha 32 Chaitra 30
Ashad 31 20X3 Baishakh 31
Shrawan 32 Jeshtha 32
Bhadra 31 Ashad 31
Ashwin 30 Shrawan 32
Kartik 30 Bhadra 31
Marga 29

Answer:
An individual is resident of Nepal, if any of the following three conditions is satisfied:
a. His/her habitual place of abode is in Nepal,
b. S/he stays in Nepal for 183 days or more during any period of consecutive 365 days, or
c. S/he is employee of Government of Nepal, deputed by the employer in any foreign country
during the Income Year.
Habitual Place of abode is such place where the person’s major economic activities are carried
out.
Income Tax Manual issued by Inland Revenue Department consider a natural person as
resident of Nepal if he/she stays in Nepal for 183 or more in an income year. For the purpose of
determining 183-days, ‘travel for’ period and date of entry and exit regardless of time spent in
Nepal is considered but travel through (stay due to transit) is not consider.
Computations of days stay during Income Year 20X1/X2:
a. First stay, 20X1 Falgun 14 to Chaitra 11: 17 days in Falgun and 11 days in Chaitra
b. Subsequent Stay:20X2 Baishakh 13 to Ashad 26: 19 days in Baisakh, plus 32 days in Jestha
plus 26 days in Ashad

Total day-stay in Nepal: 105 days during 20X1/X2


Computations of days stay during Income Year 20X2/X3:
a. First stay, 20X2 Shrawan 3 to Bhadra 25: 30 days in Shrawan plus 25 days in Bhadra
b. Subsequent Stay: 20X2 Marga 8 to Falgun 13: 22 days in Marga, plus 30 days in Poush,
plus 29 days in Magh plus 13 days in Falgun
c. Subsequent Stay: 20X3 Jeshtha 18 to Ashad end:15 days in Jestha plus 31 days in Ashad

Total day-stay in Nepal: 190 days during 20X2/X3


Therefore, Mrs. Ang Futi is non-resident during 20X1/X2, since none of the conditions for being
resident are satisfied and is resident during 20X2/X3, since she stayed for more than 183 days.

6) Discussing the relevant provisions of Income Tax Act, 2058, Determine the residential status
and taxability of incomes in Nepal of below persons (calculation of Tax Liability is not
required): (Dec 2016; 2.5+2.5=5)
a. Mr. Shyam Khadka, a Nepali Citizen, had left Nepal on 1st Chaitra 20X2 to work as a tour
guide in United States of America. His Company in United States of America sent him Nepal

The Institute of Chartered Accountants of Nepal 19


CAP-II Paper 7 - Income Tax and VAT
1st
on Baishak 20X3 with a group of tourist. He spent 30 days in Nepal and returns back to
USA. His monthly remuneration in USA is equivalent to Nepalese Rs 5,00,000 per month.
Determine the residential status and taxability of income of Mr. Shyam Khadka in Nepal.
Answer:
An individual is resident of Nepal, if any of the following conditions is satisfied:
i. His/her habitual place of abode is in Nepal,
ii. S/he stays in Nepal for 183 days or more during any period of consecutive 365 days, or
iii. S/he is employee of Government of Nepal, deputed by the employer in any foreign country
during the Income Year.
As Mr. Shyam Khadka stayed in Nepal for more than 183 days during 20X2/X3 (full period until
Falgun and 30 days after Chaitra until Ashad), Mr. Khadka is resident of Nepal.
Mr. Khadka is liable to pay tax on his worldwide income under Sec. 1 (1) or 1 (2) of Schedule 1
of the Act.

b. Mr. Ram Bansal, an India citizen, has joined a Cement Industry in Nepal as a Chief-
Engineer Maintenance on 1st Mangsir 20X2 at a monthly gross remuneration of Rs. 1,50,000.
While in India, he has been working for a similar cement Company for a gross monthly
remuneration equivalent to Nepalese Rs. 1,00,000 per month. Determine the residential
status and taxability of income of Mr. Ram Bansal in Nepal.
Answer:
An individual is resident of Nepal, if any of the following conditions is satisfied:
i. His/her habitual place of abode is in Nepal,
ii. S/he stays in Nepal for 183 days or more during any period of consecutive 365 days, or
iii. S/he is employee of Government of Nepal, deputed by the employer in any foreign country
during the Income Year.

As Mr. Ram Bansal stayed in Nepal for more than 183 days during 20X2/X3 (full period after 1st
Mangsir 20X2, 9 Months, more than 270 days), Mr. Bansal is resident of Nepal.

Hence, Mr. Bansal is liable to pay tax on his worldwide income under Sec. 1 (1) or 1 (2) of
Schedule 1 of the Act.

7) Dr. Koirala, a senior cardiologist working in a private hospital of Kathmandu, left Nepal for
London on 1st Ashwin 20X1 of the Income Year. His visit to London was unofficial. Before
leaving Nepal, the following agreement was signed between him and the hospital.
• No salaries for non-duties
• Maximum leave period approved = one year
As per the agreement signed, the validity will remain until the end of Bhadra, 20X2.
However, he returned back to Nepal before the validity has expired as on 1 st Baisakh, 20X2
and joined the duty on the same date. He was drawing Rs. 40,000 as a gross salary per month
in Nepal during the year.

The Institute of Chartered Accountants of Nepal 20


CAP-II Paper 7 - Income Tax and VAT
As on Ashad end 20X2, Dr. Koirala received £3,000 (after TDS of £2,000) in Nepal through
western union money transfer as consideration payment made by the London hospital
against the service provided by him. The exchange rate £ 1 = NPR 153.
You are required to determine the following with reference to Income Tax Act, 2058.
(June 2016, 5 Marks)
a. Residential Status for the Income Year 20X1/X2
Answer
An individual is resident of Nepal, if any of the following conditions is satisfied:
i. His/her habitual place of abode is in Nepal,
ii. S/he stays in Nepal for 183 days or more during any period of consecutive 365 days, or
iii. S/he is employee of Government of Nepal, deputed by the employer in any foreign country
during the Income Year.
Test 1: Habitual place of abode cannot be easily determined
Test 3: Not a government employee
Test 2: He has stayed in Nepal for less than 183 days (5 Months, Shrawan 20X1, Bhadra 20X1,
Baisakh 20X2, Jestha 20X2, Ashad 20X2. Day’s Stay test also not satisfied.
Therefore, Dr. Koirala is non-resident in Nepal.

b. Tax Liability
Answer
He received Rs. 200,000 from private hospital in Nepal (income having source in Nepal), which
is taxable @ 25%, i.e. tax liability is Rs. 50,000 for the year.
The amount received for the service provided in the UK is not taxable in Nepal, since he is non-
resident and the amount are derived is not having source in Nepal [Joint reading of Sec. 6 and Sec.
67 (6)]

The Institute of Chartered Accountants of Nepal 21


CAP-II Paper 7 - Income Tax and VAT

Chapter 3: Income From Employment

1) Following are the income of Mrs. Meena Thapa who is currently employed in NGO:
i) Basic Salary 1,00,000 p.m.
ii) COLA – Allowance 20 percent of basic salary
iii) As per the requirement of SSF the NGO contributes to the Fund.
iv) One-month additional salary as festival allowance
v) Additional one-month gross salary is provided as retention allowance
vi) She uses office vehicle for official travel and pick and drop from her residence
vii) NGO provides meeting fee of Rs. 25,000 per meeting and during the year she had
attended 5 meeting during FY 77/78.
viii) Lunch is provided in office to all employees and her cost for the year is Rs. 60,000
ix) TADA of Rs. 5,000 per day is provided to her for official field visit and she was
reimbursed 1,50,000 TADA during the year.
x) During the year she was awarded as employee of the year and received scooter having
market value of Rs. 2,40,000
xi) The NGO has a policy to provide school fee up to 2 children of its all employees. During
the year, NGO had paid 4,50,000 for her children education.
xii) She has paid Rs. 50,000 for medical insurance and Rs. 1,00,000 for her Life insurance out
of which 50 percent is reimbursed by NGO.
xiii) To manage her taxes, she has deposited Rs. 100,000 in CIT.
xiv) Rs. 2,00,000 was reimbursed by the insurance company for her medical expenses.

Calculate her tax liability for FY 2077/78 (June 2022, 10 Marks)

Answer:
Calculation of Taxable Income
Particulars Note Amount
Basic Salary (100,000 x 12) 1,200,000
COLA- Allowance @ 20% of basic salary 240,000
Festival Allowance 1 120,000
Retention Allowance 1 120,000
Vehicle facility 2 -
SSF Contribution - Employer 8 240,000
Meeting fee 3 125,000
Lunch Facility 4 -
Reimbursement of TADA 5 -
Market Value of Scooter 6 240,000
School fee paid 6 450,000
Reimbursement of Insurance Claim 7 -

The Institute of Chartered Accountants of Nepal 22


CAP-II Paper 7 - Income Tax and VAT
Particulars Note Amount
Life insurance premium paid by employer 9 50,000
Medical insurance premium paid by employer 10 25,000
Assessable Income from Employment 2,810,000
Less:
SSF Contribution 8 372,000
CIT Contribution 8 100,000
- 1/3 rd. of assessable income (936,666.67)
- Actual deposit (372,000+100,000 = 472,000)
- Maximum up to 500,000
(Whichever is lower)
Taxable Income 2,338,000
Less:
Life insurance premium (max 40,000) 9 40,000
Medical insurance premium (max 20,000) 10 20,000
Net Taxable Income 2,278,000

Calculation of Tax Liability Rate Amount


First Rs. 500,000 0% -
Next 200,000 10% 20,000
Next 300,000 20% 60,000
Next -up to 1,000,000 30% 300,000
Remaining 278,000 36% 100,080
Total 480,080
Less: 10% Female Rebate 48,008
Net Tax Liability 432,072

Working Notes:
1. Regular Pay = 100000 (Basic Salary) + 20000 (Cola Allowance) = 120000
She has been getting regular pay of Rs. 120000 hench such amount is taken for the purpose
of determining Festival and retention allowance. (Alternatively, students may consider Rs.
100,000 as base for other facilities like festival allowance)
2. The vehicle facility for providing pick and drop services to the employees by the
organization is not defined as perquisite in Income Tax Act, 2058. However, if such vehicle
was provided to her personal use, then it would fall under the perquisite.
3. Calculation of Final and Advance Withholding of Meeting Fee:
Particulars Amount (P. Meeting) Final Withholding Advance
Meeting fee 25,000 20,000 5,000
Number of meetings 5 5 5
Total amount 125,000 100,000 25,000

The Institute of Chartered Accountants of Nepal 23


CAP-II Paper 7 - Income Tax and VAT
As per section 92, meeting fee of maximum 20,000 per meeting is subject to final withholding
thereby, since meeting fee per meeting is more than 20,000, all meeting fee i.e. Rs. 125,000
should be included in her taxable income. However, she can claim TDS deducted @ 15% on
such amount as advance tax.
4. Under section 8 (3) meal facility provided to all employees under equal term shall not be
included in the taxable income of employee.
5. Under section 8 (3), reimbursement of expenses that are directly related while serving the
business function of organization are not taxable income of the employee.
6. Gifts or any other facilities such as school fee paid of employee's children which are received
from employer are income from employment thus, it should be included in her income.
7. As per proviso clause (a) of Sec. 31, Compensation received against physical injury of a
natural person shall not be included in income.
8. Allowable SSF Deduction:
Particulars Amount
Basic Salary 100,000
Total number of months paid 12
Total basic salary 1,200,000
Employer Contribution @ 20% 240,000
Employee Contribution @ 11% 132,000
Sub- Total 372,000
Additional CIT Contribution 100,000
Total Contribution 472,000
9. Amount contributed by the employer for Life Insurance Premium of employee is income
from employment. Thereby, 50,000 paid is included in above calculation. As per allowable
deductions under schedule 1 of Income Tax Act, lower of actual premium or up to 40,000 is
allowed for deduction.
10. Amount contributed by the employer for Medical Insurance Premium of employee is income
from employment. Thereby, 25,000 paid is included in above calculation. As per allowable
deductions under schedule 1 of Income Tax Act, lower of actual premium or up to 20,000 is
allowed for deduction.

2) Laxmi Bank Limited declared a Voluntary Retirement Scheme (VRS) for its staffs during
2077.78. Man Bahadur Tamang 56 years old widower looking after 2 kids and working as
junior assistant decided to opt for VRS scheme announced by the bank with effective from
Chaitra end 2077 who had completed 16 years of bank service. Bank has got approval from
IRD to maintain its staff retirement fund. The following details are available for income of
Man Bahadur for the year:
• Basic Salary till Chaitra 2077 Rs. 50,000 per month.
• Monthly Allowance Rs. 30,000 per month.
• Retirement Fund contributed by employer 10% of basic salary an equal amount was
contributed by employee.

The Institute of Chartered Accountants of Nepal 24


CAP-II Paper 7 - Income Tax and VAT
• Life Insurance premium paid by Man Bahadur for himself Rs. 25,000
• Remote area allowance for working in branch situated in Category C district for 3
months Rs. 5,000 per month
• Provident Fund paid by retirement fund maintained by the bank on 15th Baishakh 2078
Rs. 1,200,000.
• Additional retirement payment paid by Bank under Voluntary Retirement Scheme was
Rs. 2000,000.
• He was nominated by the bank for 35 days banking training in Singapore and received
allowance at the rate of 75 USD per day.
• He was availing the subsidized staff housing loan from the bank for which he was paying
interest @ 3% per annum whereas the prevailing interest rate of similar loan was 9%
per annum. Average outstanding principal amount of the loan at the time of retirement
was 1,326,200.
• Mr. Man Bahadur has maintained 9% recurring FD account with the bank where he
used to deposit 15000 per month from his salary and total interest earned during the year
was 17,560.
• While going to Nepalgunj on bank duty, he suffered from road accident and treated in
Kohalpur hospital and cost of treatment amounting Rs. 156,500 was paid by the bank.

Based on the information provided above, you are required to calculate the tax liability of
Mr. Man Bahadur for the income year 2077.78. (Dec 2021, 10 Marks)

Answer:
Particulars Amount Remarks
1. Basic Salary 450,000.00 9 months @ Rs. 50,000
2. Allowances 270,000.00 9 months @ Rs. 30,000
3. Employers contribution to
45,000.00 10% of basic salary
retirement fund
4. Remote area allowances 15,000.00 3 months @ Rs. 5,000
Retirement payment from approved
5. Provident Fund paid by the bank's retirement fund, 5% final withholding on
-
retirement fund retirement payment less higher amount
of 5 lakhs or 50% of the payment
Retirement payment from non-
6. Retirement payment by the bank
- contributory retirement fund, 15% final
under VRS Scheme
withholding
7. DSA for banking training in DSA not to be included in remuneration
-
Singapore income
Included under section 27, quantification
8. Interest on subsidized housing loan 59,679.00
( 6% of Rs. 13,26,200 for 9 months)

The Institute of Chartered Accountants of Nepal 25


CAP-II Paper 7 - Income Tax and VAT

9. Interest on recurring FD - Final withholding @ 5%

Reimbursement of Medical Expenses


10. Medical Compensation - (Treatment for accident on duty) not to be
included in remuneration income
Assessable Income from employment 839,679.00
Reduction
Lower of below three:
A. 1/3rd of Assessable Income
Contribution to Approved Retirement
90,000 (839679/3), i.e. Rs. 279,893
Fund B. Max. Rs. 300,000
C. Actual Rs. 90,000
Taxable Income 749,679
Deduction for Insurance premium
25,000.00 Rs. 40,000 or actual which is lower
(Lower of below two)
Category C, maximum allowed Rs.
Deduction as the person resided in
7,500.00 30,000 per year, pro-rata basis for 3
remote area
months
Tax Applicable Income 717,179.00
Tax Calculations
Widower having dependent to look after
First 600,000 @ 1 % (SST) 6,000.00
are treated as couple under section 50
Balance 117,179 @ 10% 11,718.00
Total Tax Liability 17,718.00

3) Mr. Ram Shrestha is retired from a government company during Chaitra 2077. He is yet to
receive his retirement benefits. At the time of retirement, he is entitled to either
i) monthly pension for rest of his life from date of retirement, or
ii) lump sum pension of 10 years at the time of retirement and monthly pension for life
after 10 years.
Advise Mr. Ram Shrestha of tax implications on both the cases. (June 2021, 5 marks)

Answer:
i) Pension facility is taxable as per section 8 of ITA, 2058. However if monthly pension is
availed, deduction as per schedule 1(1) can be availed which is equal to 25% of basic slab
limit. Section 3 of the ITA, 2058 states that tax shall be levied in each income year on the
taxable of income of such person.
ii) In case of lumpsum payment also, tax needs to be deducted as a retirement payment @15% at
the time of payment, and monthly pension will be taxed as per law at the time of payment.

The Institute of Chartered Accountants of Nepal 26


CAP-II Paper 7 - Income Tax and VAT
4) Calculate the income tax liability of Mrs. Jullee Sharma for IY 2077/78 on the basis of below
detail. (June 2021, 10 Marks)
a) Basic Salary: Rs. 3,05,000 per month.
b) As per requirement of Contributed Social Security Act, 20% of basic salary is
contributed by the employer and 11% deducted from her salary and total of 31%
deposited to Social Security Fund.
c) One month basic salary is provided as festival allowance.
d) 5% of basic salary is provided as performance based allowance per month.
e) Employer has provided her one i-PAD worth Rs. 1,25,000 for her outstanding
performance.
f) Residence Allowance : Rs. 24,500 per month
g) Medical Allowance : 2% of her basic salary per month
h) Residence has one employee arranged by the employer for her domestic help with cost of
Rs. 4,500 per month.
i) Being in core management team, she has attended 21 meetings during the year and
received Rs. 4,000 per meeting.
j) The cost of education for her son and daughter costing Rs. 24,000 per month paid by her
employer.
k) The cost of life insurance policy per year is Rs. 1,10,000, 50% reimbursed by the
employer.
l) The cost of Toyota car provided to her official & personal use is Rs. 85 lacs.
m) She has donated Rs. 1,05,000 to Sahara Nepal, which is a tax exempted entity.

Answer
Calculation of Tax Liability of Mrs. Jullee Sharma for FY 2077.78
Particulars Amount Unit Total Amount (Rs.)
Basic Salary 305,000 12 3,660,000.00
Contribution to Social Security 61,000 12 732,000.00
Fund (20% of basic salary
contributed by employer)
Festival Allowance 305,000 1 305,000.00
Performance Allowance (5% of 15,250 12 183,000.00
basic salary each month)
Gift from Employer 125,000 1 125,000.00
Residential Allowance 24,500 12 294,000.00
Medical Allowance (2% of basic 6,100 12 73,200.00
salary)
Cost of Domestic Helper 4,500 12 54,000.00
Meeting Allowance (Final -
Withholding, not included in
assessable income)

The Institute of Chartered Accountants of Nepal 27


CAP-II Paper 7 - Income Tax and VAT
Cost of Education of Children paid 24,000 12 288,000.00
by employer
Cost of life policy reimbursement by 55,000 1 55,000.00
employer
Quantification of Vehicle Facility 18,300 1 18,300.00
(0.5% of 3,660,000)
Total Assessable income 5,787,500.00
Allowable Reductions:
Contribution in SSF is allowed as 500,000.00
reduction at lower of following:

Rs. 500,000 500,000


1/3rd of Assessable Income 1,929,166.67
Actual Payment (Rs. 1,134,600
305000*31%*12)
(Social Security Fund is ARF)
Donation given to exempt 100,000.00
organization is allowed as
reduction at lower of following:
5% of Adjustable Taxable Income 264,375
(5% of [5787500-500000])
Rs. 100,000 100,000
Actual Donation to Exempt 264,375
organization
Life Insurance Policy (Minimum of 40,000.00
following)
Rs. 40,000
Actual Premium Paid i.e. Rs.
110,000
Taxable Income 5,147,500.00
Assuming She claims as Couple
No social security tax for 600,000 -
10% for Next 200,000 20,000.00
20% for Next 300,000 60,000.00
30% for Next 900,000 270,000.00
36% for Balance Rs. 3,147,500 1,133,100.00
Total Tax 1,483,100.00
Less: 10% Female Tax Credit 148,310.00
Net Tax Liabilities 1,334,790.00

The Institute of Chartered Accountants of Nepal 28


CAP-II Paper 7 - Income Tax and VAT
5) Ms. Rima Khanal retired on 1st Magh, 2076 after completing 22 years of private banking
service from Kathmandu. She submits the following details with respect to her employment
for the Income Year 2076/77.
• Basic Salary: Rs. 400,000 P.M.
• Dearness Allowance: Rs. 50,000 per month.
• She gets one-month dashain allowance.
• Contribution to approved Provident fund: Rs. 40,000 P.M.
• Ms. Khanal also deposited equal amount to the same provident fund
• She received best employee award: Rs. 400,000.
• Meeting fee for 10 meetings (net) Rs. 255,000
• She avails vehicle and accommodation facility, which is owned by the bank.
• She has paid life insurance premium of Rs. 100,000 and health insurance premium of
Rs. 25,000 during the year.
• She also gets scholarship of Rs. 1,000,000 for pursuing her PHD.
• She spends Rs. 480,000 for payment of school fees of her two children.
• Her Household expenditure is Rs. 150,000 per month.

Required: (December 2020, CAP II 10 marks)


i) Calculate tax liability of Ms. Rima Khanal considering she opts for single status while
filing tax returns.
ii) State whether she has to file return under sec 97 (2).

Answer:
i)
Assessable Income from Employment
S.N. Description Amount (Rs.) Remarks
1 Basic Salary 2,400,000.00 U/S 8(2)
2 Dearness Allowance 300,000.00 U/S 8(2)
3 Dashain allowance 400,000.00 U/S 8(2)
4 Employer's contribution to PF 240,000.00 U/S 8(2)
5 Award received 400,000.00 U/S 8(2)
6 Meeting Fee (255,000/85%) 300,000.00 U/S 8(2)
7 Vehicle facility 12,000.00 WN 1
8 Accommodation facility 48,000.00 WN 2
9 Scholarship 0.00 WN 3
Assessable Income from Employment 4,100,000
Working Notes:
1. Section 27 read together with Rule13, quantification of vehicle facility is 0.5% of basic salary
and grade, i.e. 0.5% of Rs. 24 Lakhs
2. Section 27 read together with Rule13, quantification of accommodation facility is 2% of basic
salary and grade, i.e. 2% of Rs. 24 lakhs.

The Institute of Chartered Accountants of Nepal 29


CAP-II Paper 7 - Income Tax and VAT
3. Exempt income as per Section 10(f). Alternatively the student may assume that the amount is
provided by the employer and as such, forms part of employment income. The alternative
treatment shall be explicit written comment of the student. Marks shall not be awarded on
mere inclusion of amount without a note to that regard.

Statement of Total Assessable Income, Taxable Income and Balance Taxable Income
Particulars Amount Notes
Total Assessable income 41,00,000
Less: Reductions u/s 12, 12Ka, 12Kha &/or 63
Contribution to retirement fund (u/s 63) 300,0000 Note 1
Less: Deduction under Schedule 1 of the Act
Life insurance 40,000.00 Note 2
Health insurance 20,000.00 Note 3
Taxable income 3,740,000
Notes:
1. Lower of following:
Actual Contribution, 240,000*2 or,
One-third of assessable income, Rs. 40,55,000/3= 1351,666.67, or
Max. Rs. 300,000
2. For Life Insurance Facility, as per Schedule 1(1(12)), deduction allowed is Rs. 40,000 or
payment, whichever is lower.
3. For Health Insurance Facility, as per Schedule 1(1(16)), deduction allowed is Rs. 20,000 or
payment, whichever is lower.

Statement of Tax Liability


Computation of tax liability Rate
Taxable Income 3,740,000.00
Social Security tax up to Rs. 500,000 1% 5,000.00
Next Rs. 200,000 10% 20,000.00
Next Rs. 300,000 20% 60,000.00
Next Rs. 900,000 30% 270,000.00
Balance Rs. 1,740,000 36% 626,400.00
Total 981,400.00
Less: female Tax Credit 98,400.00
Less : Credit for withholding Tax on Meeting Fee 45,000.00
Net tax liability 838,260.00

Working Note:
School fee of children is domestic and personal nature expense, and is not deductible under
Section21 of the Act.

The Institute of Chartered Accountants of Nepal 30


CAP-II Paper 7 - Income Tax and VAT
ii) Conclusion on Filing of Income Return
As per Sec. 97 (1) (GA) read together with Sec. 97 (2), a resident natural person is required to file
income return if his/her taxable income exceeds Rs. 40 lakhs during the year or the return is
otherwise requested by IRD through public circular or in writing, when all the following conditions
are satisfied:
• The person’s income consists only of employment having source in Nepal,
• There is only one employer at a time and all employers are resident of Nepal,
• The person does not claim retirement contribution and medical tax credit other than that
claimed through employer, and
• The person does not claim any reduction u/s 12.

In the given case, total taxable income is less than Rs. 40 lakhs and she has employment income
only. So, she is not required to submit the income tax return.

6) Mr. Shyam is an Engineer working since 1 Jestha, 20X-75 in an Airlines Company. The
contract period is for 4 years. The Company has been charging expenses related to his
service. Calculate the tax to be deducted by the Company on his employment income for the
month of Ashwin, 20X-75. (Dec 2018, 10 Marks)

Charging Monthly Expenses:


a) Monthly remuneration Rs. 80,000 (60% is basic salary and the remaining amount is
allowance). Festival allowance Rs. 80,000 for the year.
b) The Company contributes 10% of his salary and he also contributes the same amount
and is deposited into an approved provident fund. In addition, the Company deposited
Rs. 10,000 per month to Citizen Investment Trust as per his request.
c) The Company makes a provision for gratuity @ 8.33% on his basic salary.
Facilities Provided:
a) He has been provided a car along with a driver by the Company for both official and
personal use. Actual petrol expenses Rs. 30,000 pertaining to the fiscal year for upto
Ashwin end, 20X-75 was reimbursed by the Company and no such facility shall be
provided further.
b) He has been provided a chair for physically disabled people for his convenience; annual
maintenance cost Rs. 6,000 is also borne by the Company.
c) He has got a Samsung S 9 Mobile phone from the Company on 5 Bhadra, 20X-75.
Purchase price of the phone was Rs. 90,000. Monthly total telephone expenses amounting
to Rs. 1,500 is reimbursed to him. The phone is used equivalent to 33.33 percent for
personal calls.
d) The Company is providing free lunch during office hours, the cost of which is Rs. 100 per
meal to all employees. He has been receiving Rs. 150 per meal due to his special work.
Assume 252 working days in a year.
e) Annually, the Company pays a premium of Rs. 25,000 against his health insurance.

The Institute of Chartered Accountants of Nepal 31


CAP-II Paper 7 - Income Tax and VAT
f) The Company has been providing bonus to its employees, who served at least 6 months
in the company. Such bonus is equivalent to 3 month's basic salary.
Other evidence and information submitted by him to the Company for tax purpose are:
a) He has paid premiums of Rs. 15,000 and Rs. 7,000 respectively against his and his wife's
life insurance policies.
b) He and his wife declared that they are couple in the Income Year 20X-75/X-76 and his
wife does not involve in any income generating activities.

Answer
Method of Withholding of Tax u/s 87
The employer needs to calculate the annual tax liability of the employee based on the presumptive
income every month. It deducts tax already withheld from that annual tax liability, and the result
is divided by remaining months for which the tax has to be withheld to obtain the figure of
withholding tax of the month.
Step 1: Calculation of Annual Tax obligation of the employee based on presumption
Assessable Income from Employment
Sec.
Particulars Amount Notes
Ref.
Monthly Remuneration
Basic is 60% of Remuneration, and is
Basic Salary 8 (2) 576,000
annualized
Allowances is 40% of Remuneration
Allowances 8 (2) 384,000
and is annualized
Provident fund 8 (2) 57,600 10% of his basic salary, annualized.
Employment income is computed under
cash basis of accounting. This amount
Provision for Gratuity 22 -
is not included as the amount is not
received in cash by the employee
0.5% of basic salary and grade. All
other costs related to operation of
Vehicle facility 27 2,880
vehicle is deemed to be covered by the
amount so quantified
Serves the proper business purpose of
Chair at Workplace 8 (3) -
the employer
Gift in relation to employment (Samsung
8 (2) 90,000 Quantification u/s 27.
S9)
Annualized personal phone call
Reimbursement of Personal Expense 8 (2) 6,000
expense (1,500*1/3*12)
Special Lunch 8 (2) 37,800 Lunch to all employees is exempt
Payment of his health insurance
Reimbursement of Personal Expense 8 (2) 25,000
premium
Bonus 8 (2) 144,000 3 months' basic salary
Assessable Income from Employment 1,323,280

The Institute of Chartered Accountants of Nepal 32


CAP-II Paper 7 - Income Tax and VAT

Taxable Income
Assessable Income from Employment 1,323,280
Total Assessable Income 1,323,280
Less:
Lower of following:
a. Actual Retirement Contribution
across all ARF- Rs. 235,200
(57,600*2+10,000*12)
Reduction u/s 63 235,200
b. One-third of Assessable Income- Rs.
441,093
(one third of Rs. 13,23,280)
c. Maximum Rs. 300,000
Less:
50% of Basic Exemption Limit, and
Deduction availed to disabled person 300,000
couple assessment
Payment of Life Insurance Premium 22,000 Lower of Rs. 40,000 and Rs. 22,000
Assuming the insurance company is
resident, and the limit of deduction is
Payment of Health Insurance Premium 20,000
lower of actual premium paid or Rs.
20,000
Balance Taxable Income 746,080

Presumptive Annual tax Obligation


1st Rs. 600,000 1% 6,000
Next Rs. 146,080 10% 1,461
Total Obligation for the Year 7,461

Is not possible due to absence of


information, so, we assume the
Step 2: Subtraction of taxes withheld during Shrawan and Bhadra
withholding of tax is even throughout
the year
Tax Obligation for the year divided by
Step 3: Withholding Tax for the Month of Ashwin 12 (assuming even tax withholding
621.75
throughout the year)

7) Shreedhar Kumar is appointed on 20X-66/4/1 as Accounts Officer of Nepal Telecom Limited


with salary structure of Rs. 25,000.00 - 500.00(6)-1,000.00(12). He has received interest free
home loan Rs. 1,624,000.00 from the employer. Assume market rate of interest 9% and make
other practical assumptions as necessary. Calculate monthly withholding tax to be deducted
from his salary in Income Year 20X-74/X-75. (June 2018, 5 Marks)

The Institute of Chartered Accountants of Nepal 33


CAP-II Paper 7 - Income Tax and VAT
Answer
Method of Withholding of Tax u/s 87
The employer needs to calculate the annual tax liability of the employee based on the presumptive
income every month. It deducts tax already withheld from that annual tax liability, and the result
is divided by remaining months for which the tax has to be withheld to obtain the figure of
withholding tax of the month.
Step 1: Calculation of Annual Tax obligation of the employee based on presumption
Assessable Income from Employment
Particulars Sec. Ref. Amount Notes
Monthly Remuneration
Basic Salary 8 (2) 300,000 Given basic salary, annualized
Grade 8 (2) 60,000 Refer W.N. 1, annualized
Employers' Contribution to Retirement Practical Assumption, as per Labor
8 (2) 36,000
Fund Act, 10% of basic salary and grade
Rs. 16,24,000 multiplied by (Market
Interest Rate- Charged Interest Rate),
Quantification of concessional loan 27 146,160
assuming loan is utilized throughout
the year
Assessable Income from Employment 542,160
Taxable Income
Assessable Income from Employment 542,160
Total Assessable Income 542,160
Lower of following:
a. Actual Retirement Contribution
across all ARF- Rs. 72,000
(36,000*2)
Less: Reduction u/s 63 36,000
b. One-third of Assessable Income-
Rs. 180,720
(one third of Rs. 542,160)
c. Maximum Rs. 300,000
Taxable Income 506,160
Annual Tax Liability (assuming single)
1st Rs. 500,000 1% 5,000
Next Rs. 6,160 10% 616
Total Obligation for the Year 5,616

Tax Obligation for the year divided


Step 2: Withholding Tax for a month by 12 (assuming even tax withholding
468
throughout the year)

The Institute of Chartered Accountants of Nepal 34


CAP-II Paper 7 - Income Tax and VAT
Working Note
Calculation of Monthly Grade amount for I.Y. 20X-74/X-75
Income Year No. of Grade Amount per grade Total Grade

20X-66/67 0 -
-
20X-67/68 1 500 500
20X-68/69 2 500 1,000
20X-69/70 3 500 1,500
20X-70/71 4 500 2,000
20X-71/72 5 500 2,500
20X-72/73 6 500 3,000
500 for first six years and
20X-73/74 7 4,000
1000 thereafter
500 for first six years and
20X-74/75 8 5,000
1000 thereafter

8) Mr. Tukaram Bidari joined Star General Stores, a trading firm on 1st Shrawan 20X-70. The
following information is received in relation to his employment income for the Income Year
20X-73/X-74: (Dec 2017, 10 Marks)
a. Pay scale Rs. 22,000 - 1,500 - 40,000.
b. He was paid with festival allowances equal to one months’ salary. Apart from the
allowance, the firm provided 1 set TV as a gift to the best performing staff. The market
value of the TV is Rs. 55,000.
c. The firm pays Mr. Bidari Rs. 15,000 p.m. as house rent allowance. Out of the allowance,
he pays only Rs. 12,000 p.m. as house rent and saves Rs. 3,000 p.m.
d. He is entitled to entertainment allowance of Rs. 500 p.m. and annual medical allowance
equal to 2% of salary.
e. The firm paid salary of Rs. 6,000 to the cook of Mr. Bidari in his residence. He
compensated Rs. 2,000 to the firm on this account.
f. Appointed in a senior post, he attended 12 meetings and was paid Rs. 2,000 per meeting
as meeting allowance. He also received allowance Rs. 2,000 p.m. for writing minutes of
Human Resource Committee of the firm.
g. He was provided meal in the office in an equal term to all employees of the firm. The
expenses borne by the firm for this facility was Rs. 36,000 for entire year i.e. 3,000 p.m.
h. He spent Rs. 20,000 for promotion of firm by hosting dinner to customers. Cost on this
account was reimbursed by the firm to Mr. Bidari.
i. The firm paid Rs. 1,000 p.m. as tuition fees for each of two children of Mr. Bidari. This
payment was made directly to school rather than to Mr. Bidari.
j. During the year, he was paid Rs. 10,000 on account of leave encashment. Further, at the
end of the year, his accumulated leave pay was valued at Rs. 50,000 which was payable
on retirement from the firm.

The Institute of Chartered Accountants of Nepal 35


CAP-II Paper 7 - Income Tax and VAT
k. Mr. Bidari received a sum of Rs. 30,000 from his previous employer. The payment was
made on account of target commission on sales achieved by him when he was in
employment with previous employee.
l. Mr. Bidari paid life insurance premium as follows:
• Rs. 15,000 for his policy of Rs. 5,00,000
• Rs. 10,000 for policy of Rs. 2,50,000 of his son
m. Mr. Bidari met an accident during the financial year and received Rs. 5,00,000 as
compensation from insurance company, out of which he incurred Rs. 1,00,000 for
medical treatment. Mr. Bidari had contributed only Rs. 1,50,000 as insurance premium
to insurance company so far.

You are required to calculate income from employment and net tax liability of Mr. Bidari
for income earned during FY 20X-73/X-74.

Answer:
Assessable Income from Employment
Particulars Sec. Ref. Amount Notes
Monthly Remuneration
Basic Salary 8 (2) 264,000 Given basic salary, annualized
Grade 8 (2) 54,000 Refer W.N. 1, annualized
One months' salary, which
Festive Allowance 8 (2) 26,500 includes both basic salary and
grade
Gift in relation to employment 8 (2) 55,000 Market value of gift
Total amount is included, as it is
House Rent Allowance 8 (2) 180,000
allowance
Entertainment Allowance 8 (2) 6,000
Medical Allowance 8 (2) 6,360 2% of basic salary and grade
Actual Cost borne by employer,
Cook Facility 8 (2) 4,000
which is 4,000
Final withholding, as per the
Meeting Allowance 92 - clarification by IRD through
Income Tax Directives
Minutes Writing Fee 8 (2) 24,000
Meals provided by Office 8 (3) - Not to be included under Sec. 8 (3)
Dinner Hosting Cost for Promotion of Serves proper business purpose of
8 (3) -
office the employer
Payment of tuition fee of two
Reimbursement of personal expense 8 (2) 24,000 children @ 1000 per child for 12
months
Cash basis payment received
Leave encashed 8 (2) 10,000
during the tenure of employment

The Institute of Chartered Accountants of Nepal 36


CAP-II Paper 7 - Income Tax and VAT
Cash basis, as cash is not received,
Accumulated leave payment 22 -
amount is not included
Cash basis, and employment
Compensation from previous employer 22, 8 (2) 30,000
includes past employment as well
Any compensation received
against injury caused due to
personal accident is exempt, and
Compensation from Insurance company in
31 - the related medical cost cannot be
respect of accident
treated as approved medical
expense for Medical Tax credit
purpose
Assessable Income from Employment 683,860

Taxable Income
Assessable Income from Employment 683,860
Less:
Own premium to the extent of Rs.
Payment of Investment Insurance
15,000 40,000
premium
Son's premium cannot be claimed
Balance Taxable Income 668,860

Calculation of Tax Liability


1st Rs. 500,000 1% 5,000 Considered as single
Balance Rs. 168,860 10% 16,886
Total Tax Obligation 21,886

Working Note:
Calculation of Monthly Grade amount for I.Y. 20X-74/X-75
Income Year No. of Grade Amount per grade Total Grade
20X-70/71 0 1,500 -
20X-71/72 1 1,500 1,500
20X-72/73 2 1,500 3,000
20X-73/74 3 1,500 4,500

9) Mohani Sharma was school English teacher in a reputed private school located at remote
area (classified as “A” class area by GON) between Shrawan 20X-72 to Poush 20X-72. She
joined an international organization located at Kathmandu immediately after resigning with
effect from 1st Magh 20X-72. Mrs. Sharma has received following salary and benefits during
the income year 20X-72/X-73:
a. Salary & benefit received from private school
Particulars Amount (Rs.)
Basic Salary (Gross) 50,000 p.m.

The Institute of Chartered Accountants of Nepal 37


CAP-II Paper 7 - Income Tax and VAT
Dashain Allowance 50,000
House Rent Allowance 5,000 p.m.
Remote Area Allowance 10,000 p.m.
Leave Encashment 25,000
b. Salary & benefit received from INGO in Kathmandu
Particulars Amount (Rs.)
Basic Salary (Gross) 1,00,000 p.m.
Advance Salary 1 month
House Rent Allowance 25,000 p.m.
Telephone Allowance 5,000 p.m.
Travelling & Daily Allowance (TADA) 10,000/day
School tuition fees of Mrs. Sharma children directly paid by 4,000 p.m.
INGO.

Other information:
a. Mrs. Sharma managed to deposit in retirement fund approved by IRD operated by listed
commercial bank in Kathmandu. She has deposited 4,50,000 in the retirement fund
within income year 20X-72/X-73.
b. Mrs. Sharma contributed donation of Rs. 3,00,000 to tax exempt organization.
c. Mrs. Sharma had gone outstation for 25 days to monitor INGO activities. TADA
provided to Mrs. Sharma is only to pay cost of foods & lodging charges. Mrs. Sharma
could only submit bills of Rs. 50,000 incurred by her for food and lodging charges.
d. Life insurance premium paid by Mrs. Sharma Rs. 25,000.
e. Mrs. Sharma was sick and incurred medical expenses of Rs. 5,000 to hospital run by
GON.
f. TDS deducted & deposited by school for Mrs. Sharma salary & benefits withdrawn by
her up to Poush 2072 was Rs. 63,388 and INGO had deposited 1,00,000 within the end of
Ashad 20X-73.
Required: (June 2017, 6+1+3=10)
(a) Calculate taxable income and tax liability of Mrs. Sharma for the income year 20X-72/X-73.

Answer:
Statement of Assessable Income from Employment

Sec.
Particulars Amount Note
Ref.
Rs. 50,000 per month for six
Salary from Private School 8 (2) 300,000
months
Dashain Allowance from Private School 8 (2) 50,000
House Rent Allowance from Private
8 (2) 30,000 Rs. 5,000 per month for six months
School

The Institute of Chartered Accountants of Nepal 38


CAP-II Paper 7 - Income Tax and VAT
Remote Area Allowance from Private Rs. 10,000 per month for six
8 (2) 60,000
School months
Final withholding, assuming that
the amount is received at the time
of retirement
Leave Encashment from Private School 8 (3) -
Alternatively, it can be included in
income assuming it was regular
employment payment
Rs. 100,000 per month for six
Salary from INGO 8 (2) 600,000
months
Cash basis, assumed not a loan
Advance Salary from INGO 8 (2) 100,000
from employer
Rs. 25,000 per month for six
House Rent Allowance from INGO 8 (2) 150,000
months
Telephone Allowance from INGO 8 (2) 30,000 Rs. 5,000 per month for six months
Principally Travel and daily
allowance serves proper business
Traveling and Daily allowance 8 (3) -
purpose of employer and is exempt
from tax.
Reimbursement of Personal Expenses School fee of children, Rs. 4,000
8 (2) 24,000
from INGO per month for six months
Assessable Income from Employment 1,344,000

Statement of Taxable Income


Total Assessable Income 1,344,000
Less: Reductions
Lower of following:
a. One third of Assessable Income- Rs. 448,000
Under Section 63 300,000
b. Actual Contribution among all ARF- Rs. 450,000
c. Maximum Rs. 300,000
Lower of following:
a. 5% of Adjusted Taxable Income- Rs. 52,200
(Assessable income – Allowable Contribution to
ARF)
Under Section 12 52,200
=1,344,000-300,000
b. Actual Gift or Donation to Exempt Organization-
Rs. 300,000
c. Maximum Rs. 100,000
Less: Deductions under
Schedule 1
Residence in Remote Area 25,000 Six months in "A" Category Remote Area
Payment of Investment
25,000 Lower of actual premium 25,000 or Rs. 40,000
Insurance Premium
Balance Taxable Income 941,800

The Institute of Chartered Accountants of Nepal 39


CAP-II Paper 7 - Income Tax and VAT

Statement of Tax Liability


1st Rs. 500,000 1% 5,000 Assuming Individual Assessment
Next Rs. 200,000 10% 20,000
Balance Rs. 241,800 20% 48,360
Total before Tax Credits 73,360
Less: Female Tax Credit 7,336 10% of computed tax liability before all tax credits
Lower of Following:
a. 15% of approved Medical expense- Rs. 750
Less: Medical Tax Credit 750 (Cost incurred in Government hospital for her own
treatment is Rs. 5,000)
b. Maximum Rs. 750
Tax Liability for the Year 65,274

(b) Does Mrs. Sharma require to file income tax return as per the provision of Income Tax
Act, 2058.
Answer:
As per Sec. 97 (1), a natural person satisfying all the following conditions, is not required to file
income return unless IRD requires so in writing or through public circular or the natural person’s
income exceeds Rs. 40 Lakhs during the Income Year:
i. The natural person earns income only from employment having source in Nepal,
ii. There is only one employer at a time, and all employers are resident of Nepal, and
iii. The person claims medical tax credit and contribution to approved retirement fund to the extent
paid through employer and does not claim donation

In the given case, first two conditions are satisfied and not the third one as the employee has
claimed donation and contribution to approved retirement fund by way of own contribution, the
person is not exempt from filing income return due to this criterion.
Further, the person has payable tax obligation for the year, does not derive all income from final
withholding payments and does not satisfy conditions of Sec. 97 (1) (Gha) and (Nga), therefore,
the person shall file income return.

(c) Compute amount of TDS need to be deposited as last installment for the salary & benefit
withdrawn by Mrs. Sharma during the income year 20X-72/X-73.
Answer
The tax obligation is less than the amount already paid, i.e. there seems no additional withholding
of taxes.

10) Mr. Robin is a retired person from Nepal Bank Limited, now he is receiving pension @ Rs.
30,000 per month and residing in Kathmandu with his family. For tax purpose, he wants to
select a single person. You are required to make tax assessment for the Income Year 20X-
72/X-73 (Dec 2016, 2.5 Marks)

The Institute of Chartered Accountants of Nepal 40


CAP-II Paper 7 - Income Tax and VAT

Answer
Total income is Rs. 390,000 (including Dashain expense), which is below the basic exemption
limit of Rs. 500,000 (in case of individual assessment).
Therefore, there is no tax obligation for Mr. Robin during the year.

11) Mr. Donald is an employee of American Embassy located at Kathmandu. He is working since
15 Falgun, 20X-72. He has received amounting of Rs. 10 lakhs from his part-time consultancy
services provided to an educational consultancy Pvt. Ltd. located at Kathmandu. You are
required to make a tax assessment for the Income Year 20X-72/X-73. (Dec 2016, 2.5 Marks)

Answer
The amount from consultancy is not an exempt income, but business income of Mr. Donald. Mr.
Donald is resident of Nepal, as such, the tax liability (assuming single) is as follows:
1st Rs. 500,000 0% 0
(business income)
Next Rs. 200,000 10% 20,000
Balance Rs. 300,000 20% 60,000
Total 120,000

12) Laxmi Khadka, a married female, took up an employment in Shrawan 20X-72 with an NGO
in Khalanga of Jumla District. The following are the particulars of her income for the
financial year 20X-72/X-73:
a. Total salary Rs. 720,000
b. Total allowances Rs. 200,000

NGO provided her an accommodation free of cost in Khalanga for which NGO has to pay
the rental charges of Rs. 10,000 per month. The above allowances include total Daily
Allowance of Rs. 100,000 paid to her during the year for her official travel. She is also entitled
for Dashain Festival Allowance equivalent to 1 month’s salary.

Compute her taxable income and tax liability for the financial year 20X-72/X-73. (June 2016,
5 Marks)

Answer
Statement of Assessable Income from Employment
Particulars Sec. Ref. Amount Note
Salary 8 (2) 720,000
Allowances for official travel serves
Allowances 8 (2) 100,000 proper business purpose of employer,
and thus, exempt as per Sec. 8 (3)

The Institute of Chartered Accountants of Nepal 41


CAP-II Paper 7 - Income Tax and VAT
2% of Khai paai aaeko talab, i.e. 2%
Accommodation Facility 8 (2) 14,400
of basic salary and grade
Assumed to be one month Salary
Festive Allowance 8 (2) 60,000
(excluding allowances)
Assessable Income from Employment 894,400

Statement of Taxable Income


There is no reduction u/s 12, 12Kha
Total Assessable Income 894,400
and 63
Less: Deductions under Schedule 1
Residence in Remote Area 40,000 Jumla is Class B Remote Area
Balance Taxable Income 854,400

Statement of Tax Liability


1st Rs. 500,000 1% 5,000 Assuming Individual Assessment
Next Rs. 200,000 10% 20,000
Balance Rs. 154,400 20% 30,880
Tax Liability 55,880

13) Mrs. Indira Dahal was retired on 1st Magh, 20X-71 after completing 22 years of private
banking service from Kathmandu. She submits the following details with respect to her
employment for the Income Year 20X-71/X-72.
a. Monthly salary Rs. 35,000
b. Two months' salary is provided for dress and festival, one month for each.
c. She has obtained overtime allowance of Rs. 20,000
d. The bank deducts 10 % from her salary and contributes 20 % of her salary to an
approved retirement fund.
e. She has car facility from the office for all purpose. Expenses Rs. 1,000 per month and
salary of the driver of Rs. 12,000 per month were paid by the bank.
f. Compensation received as per Employee Rule Rs. 100,000
g. Insurance premium paid by Employer on her behalf, Rs. 25,000 that include Rs. 12,000
for her policy and Rs. 13,000 for her son's policy.
h. The bank makes payment equivalent one-month salary to the employees who work for
full 12 months during the years.
i. The bank has provided her with accommodation facility on leased house for which 2 %
of salary was deducted. Actual rent paid by the bank was Rs. 35,000 per year.
j. On the retirement, she has entitled to receive gratuity amount equivalent to one month
salary for each year services.
k. The company paid bonus to her equivalent to two month's salary related to the Income
Year 20X-71/X-72 on 20th Aswin 20X-72.
She has only income from this employment and opted for single. You are required to assess
income from employment and her tax liability. (Dec 2015, 10 Marks)

The Institute of Chartered Accountants of Nepal 42


CAP-II Paper 7 - Income Tax and VAT

Answer:
Statement of Assessable Income from Employment
Sec.
Particulars Amount Note
Ref.
Salary 8 (2) 210,000 Rs. 35,000 per month for six months
Dashain Allowance 8 (2) 35,000
Dress Allowance 8 (2) 35,000
Overtime Allowance 8 (2) 20,000
Employers' contribution to Retirement
8 (2) 21,000
Fund
0.5% of salary, i.e. 0.5% of Rs. 210,000
Vehicle Facility 27 1,050 Car operating cost including driver's salary is
covered by the amount so calculated
Compensation related to employment 31 10,000
Insurance premium paid by employer in
Reimbursement of personal expense 8 (2) 25,000 addition to salary and other benefits, hence
included in income from employment
Additional pay by bank 8 (2) 17,500 Point (h), pro-rata on the basis of time served
Accommodation Facility 27 4,200 2% of Salary
Final withholding, since it is retirement
Gratuity 92 -
payment
Bonus 8 (2) 70,000 Cash basis and it is received this year
Assessable Income from Employment 448,750

Statement of Taxable Income


Total Assessable Income 448,750
Less: Reduction under Sec. 63
1/3rd of Assessable Income or actual
Actual Contribution is Rs. 42,000 (20% of
contribution or Rs. 300,000; whichever 42,000
salary)
is lower
Payment of Investment Insurance Her own premium only
12,000
Premium Minimum of actual payment or Rs. 25,000
Balance Taxable Income 394,750

Statement of Tax Liability


1st Rs. 394,750 1% 3,975 Individual Assessment
Tax Liability 3,975

14) Smriti works in a private school at monthly salary of Rs. 30,000 for financial year 20X-72/X-
73. She is paid Dashain allowance of Rs. 20,000. Her contribution to approved retirement
fund is Rs. 40,000 per annum. She has already paid medical insurance of Rs. 25,000 and life
insurance premium of Rs. 15,000 for the year. She incurs Rs. 3,000 for the purpose of

The Institute of Chartered Accountants of Nepal 43


CAP-II Paper 7 - Income Tax and VAT
transportation to and fro school from her residence, and also pays Rs. 5,000 for baby care of
her young child so that she could work in school. The school pays Rs. 6,000 school fee for her
elder child per month. Calculate the tax liability assume individual. (Dec 2015, 5 Marks)

Answer
Statement of Assessable Income from Employment
Particulars Sec. Ref. Amount Note
Rs. 30,000 per month for 12
Salary 8 (2) 360,000
months
Dashain Allowance 8 (2) 20,000
No deduction of expenses for
Transportation expense 21 -
employment income
No deduction of expenses for
Baby care expense 21 -
employment income
Rs. 6,000 per month for her
Reimbursement of personal expense 8 (2) 72,000
child's education
Assessable Income from Employment 452,000
Statement of Taxable Income
Total Assessable Income 452,000
Less: Reduction under Sec. 63
Actual or 1/3rd of Assessable Income or Rs.
40,000
300,000; whichever is lower
Less: Deduction under Schedule 1
Minimum of actual payment or
Payment of Investment Insurance Premium 15,000
Rs. 40,000
Minimum of actual payment or
Payment of Health Insurance Premium 20,000
Rs. 20,000
Balance Taxable Income 377,000

Statement of Tax Liability


1st Rs. 377,000 1% 3,770 Individual Assessment
Tax Liability 3,770

15) Mr. Robert Wooster, a citizen of USA, is employed in research department of Mokshya Pvt.
Ltd., Nepal with effect from 1st Poush 20X-71. Due to nature of his job requirements Mr.
Wooster has to reside in Nepal. His employer furnishes him pay slip showing deduction of
Rs. 265,000 as withholding Tax under sec. 87 of Income Tax Act, 2058 for financial year 20X-
71/X-72.
Mr. Wooster has obtained following information about remuneration from Mokshya Pvt.
Ltd.
Particulars Amount (Rs.)
Basic salary per month 50,000

The Institute of Chartered Accountants of Nepal 44


CAP-II Paper 7 - Income Tax and VAT
Uniform allowance 7,000
Technical allowance per month 60,000
Flat rent of Mr. Wooster paid by employer 60,000
Telephone facility per month 500
Free vehicle facility provided
Compensation paid by current employer to previous employer of Mr. Wooster 200,000
which had to be paid by Mr. Wooster
Advance salary for Shrawan 20X-72 and Bhadra 20X-72 100,000
Leave pay 50,000
Value of accumulated leave provision payable at the end of service period 10,000
Tuition fees of son of Mr. Wooster for learning Nepali language paid directly 40,000
by employer to private tutor
Air fare of Mr. Wooster from USA to Nepal paid by employer 100,000
Canteen bill paid by employer 40,000
Drivers salary of vehicle provided by employer 100,000

Required: (July 2015, 7+2+1=10)


a. Mr. Wooster now seeks your advice whether withholding Tax deducted by his employer
under sec. 87 of Income Tax Act, 2058 is correct.
Answer
Statement of Assessable Income from Employment
Sec.
Particulars Amount Note
Ref.
Rs. 50,000 per month for 7 months
Basic Salary 8 (2) 350,000
(Poush to Ashad)
Uniform Allowance 8 (2) 7,000
Rs. 60,000 per month for 7 months
Technical Allowance 8 (2) 420,000
(Poush to Ashad)
2% of salary, assuming that flat is
leased by employer.
If students assume that the rental
Flat Rent of Wooster 8 (2) 7,000
payment was made directly to
Wooster in cash, the whole amount is
included in income
Rs. 500 per month for 7 months
Telephone facility 8 (2) 3,500
(Poush to Ashad)
0.5% of Salary, i.e. 0.5% of Rs.
350,000
Vehicle Facility 8 (2) 1,750 The driver's salary in last row is
deemed to be covered by the amount
so quantified

The Institute of Chartered Accountants of Nepal 45


CAP-II Paper 7 - Income Tax and VAT
Compensation to be paid by Wooster
Reimbursement of personal expense 8 (2) 200,000 to previous employee paid by current
employer
Cash basis, and the cash is received
Advance Salary 22 100,000
this year
Cash basis, and the cash is received
Leave pay 22 50,000
this year
Not yet paid in cash, as such, not
Accumulated leave provision 22 -
included
Tuition fee of Mr. Wooster
Reimbursement of personal expense 8 (2) 40,000
reimbursed
Reimbursement of personal expense 8 (2) 100,000 Airfare from USA to Nepal
Assumed that food is provided on
similar terms to all employees.
Canteen Bill paid by employer 8 (3) - Alternatively, if it is assumed that food
is provided to him specifically, the
amount shall be included in income
Driver's Salary 27 - Refer Notes on Vehicle facility above
Assessable Income from Employment 1,279,250

Statement of Taxable Income and Balance Taxable Income


Assessable Income from Employment 1,279,250
Total Assessable Income 1,279,250
Less: Reduction under Sec. 63, 12 or 12Ka -
Taxable Income 1,279,250
Less: Deduction under Schedule 1 -
Balance Taxable Income 1,279,250

Statement of Tax Liability


1st Rs. 500,000 1% 5,000 Individual Assessment
Next Rs. 200,000 10% 20,000
Next Rs. 300,000 20% 60,000
Balance Rs. 279,250 30% 83,775
Tax Liability 168,775

As computed above, the total tax obligation of Mr. Wooster is Rs. 168,775; which is less than that
deducted by the accountant.

b. Whether Mr. Wooster is required to file income Tax Return for financial year 2071-72 in
Nepal as per Income Tax Act, 2058?
Answer

The Institute of Chartered Accountants of Nepal 46


CAP-II Paper 7 - Income Tax and VAT
As per Sec. 97 (1) jointly read with Sec. 97 (2), a person is not required to file income return in
the following conditions, unless otherwise required by IRD in writing or through public circular
or where the natural person’s income exceeds Rs. 40 Lakhs during the Income Year:
a. If the person does not have any payable tax liability,
b. If the person generates income only from final withholding income,
c. If the person satisfies conditions of Sec. 4 (3),
d. If the person is a natural person not operating vehicle paying tax under Sec. 1 (13) of Schedule
1 of the Act through a private firm, or
e. If a person derives income only from disposal of non-business chargeable asset, desirous of
not filing income return
Since condition (c) above is satisfied, Mr. Wooster is not required to file income return.

c. Mr. Wooster contends that as he is a citizen of USA, he is required to pay tax in USA and
not in Nepal. Is Mr. Wooster Correct? If not explain why is he required to pay Taxes in
Nepal.

Answer
Mr. Wooster is resident of Nepal, as he has stayed in Nepal for more than 183 days during the
Income Year.
As per Sec. 6, a resident person has to pay tax on worldwide income (global income) and a non-
resident person on income having source in Nepal.
Though he is citizen of USA, however, he is resident of Nepal, as such, he has to pay tax in Nepal
on his worldwide Income. Further, if he were not resident of Nepal and since his employment is
in Nepal (which means employment income has source in Nepal), he is still required to pay tax in
Nepal @25%.

16) Saru Pvt. Ltd., Kathmandu declared a Voluntary Retirement Scheme (VRS) for its staffs
during FY 20X-71/X-72. Provident funds are deposited in CIT, an approved retirement fund.
Man Bahadur decided to take VRS with effect from 1st Baishakh 20X-72. The following
details are available for income of Man Bahadur for the year.
• Basic Salary till Chaitra 20X-71 Rs. 50,000 per month.
• Monthly Allowance Rs. 30,000 per month.
• Provident Fund contributed by employer 10% of basic salary
An equal amount was contributed by employee.
• Life Insurance premium paid by Man Bahadur for himself Rs. 25,000
• Remote area allowance for working in Category C district for two months Rs. 5,000 per
month
• Provident Fund paid by CIT on 15th Baishakh 20X-72 Rs. 12,00,000.
• Retirement payment paid by Saru Pvt. Ltd. as per Voluntary Retirement Scheme was
Rs. 20,00,000, paid to Man Bahadur directly by the company.

The Institute of Chartered Accountants of Nepal 47


CAP-II Paper 7 - Income Tax and VAT
Required: (Dec 2014, 7+3)
a. Calculate taxable income and tax liability of Man Bahadur for F.Y. 20X-71/X-72 as per
Income from Employment. Assume individual.
Answer
Statement of Assessable Income from Employment
Particulars Sec. Ref. Amount Note
Rs. 50,000 per month
Basic Salary 8 (2) 450,000 for 9 months (until
Chaitra)
Rs. 30,000 per month
Monthly Allowance 8 (2) 270,000 for 9 months (until
Chaitra)
Employer's Contribution to Retirement Fund 8 (2) 45,000 10% of Basic Salary
Rs. 5,000 per month for
Remote Area Allowance 8 (2) 10,000
2 months (until Chaitra)
Assessable Income from Employment 775,000

Statement of Taxable Income


Total Assessable Income 775,000
Less: Reduction under Sec. 63 -
Actual is Rs. 90,000;
assuming the
(One-third of Assessable Income, or Actual, or
90,000 contribution is made at
Rs. 300,000; whichever is lower)
Approved Retirement
Fund
Less: Deduction under Schedule 1
C' Category is entitled
to deduct Rs. 30,000 for
the year; as the
Residence in Remote Area 5,000 residence is for two
months, pro-rated for
two months
(30,000*2/12)
Lower of actual
Investment Insurance Premium Paid 25,000
payment or Rs. 40,000
Balance Taxable Income 655,000

Statement of Tax Liability


1st Rs. 500,000 1% 5,000 Individual Assessment
Next Rs. 155,000 10% 15,500
Tax Liability 20,500

The Institute of Chartered Accountants of Nepal 48


CAP-II Paper 7 - Income Tax and VAT
b. Identify any payments subject to final withholding Tax (Final TDS) and applicable TDS rate
and Tax amount.
Answer
Identification of and amount of Final withholding Payments and Tax
Provident Fund by CIT 1,200,000
Gain is calculated by deducting
higher of 50% of Rs. 1,200,000 or
Taxation on Gain @ 5% 30,000
Rs. 500,000 from Rs. 1,200,000;
i.e. Gain is Rs. 600,000
Retirement Payment under VRS 2,000,000
Taxation @ 15% 300,000
Total Final Withholding Taxes 330,000

17) Mr. Sailesh, a disabled person, is working in Dailekh branch of Nabil Bank Ltd. In Fiscal
Year 20X-69/X-70, following are the transactions;
a. Salary and allowances Rs. 50,000 per month; Dashain allowance Rs. 20,000; Bonus Rs.
40,000.
b. Bank has managed to contribute to Provident Fund (approved) Rs. 24,000 from employer
as well as employee side.
c. In addition to the remuneration received from Bank, Mr. Sailesh has also received Rs.
150,000 from Nepal Government in the form of Pension.
d. He has covered the insurance and paid the insurance premium of Rs. 25,000 during Fiscal
Year 20X-69/X-70.
e. He has declared the couple under section 50, in Fiscal Year 20X-69/X-70.
Compute Assessable Income, Taxable Income, Tax liability of Mr. Sailesh for the Fiscal Year
20X-69/X-70 stating relevant provisions of Income Tax Act, 2058. (June 2014, 10 Marks)

Answer
Statement of Assessable Income from Employment
Particulars Sec. Ref. Amount Note
Rs. 50,000 per month for
Salary and allowances 8 (2) 600,000
a year
Dashain Allowance 8 (2) 20,000
Bonus 8 (2) 40,000
Assuming 24,000 is total
Employer's Contribution to Retirement Fund 8 (2) 12,000 contribution, 50% is
employer's part
Cash basis, employment
Pension from Government of Nepal 8 (2) 150,000 includes past
employment as well
Assessable Income from Employment 822,000

The Institute of Chartered Accountants of Nepal 49


CAP-II Paper 7 - Income Tax and VAT
Statement of Taxable Income
Total Assessable Income 822,000
Less: Reduction under Sec. 63 -
(One-third of Assessable Income, or Actual, or Rs.
24,000
300,000; whichever is lower)
Dailekh is in Category
"C", deduction of Rs.
Residence in Remote Area 30,000 30,000 assuming he
resides there throughout
the year
Lower of actual payment
Investment Insurance Premium Paid 25,000
or Rs. 40,000
50% of Basic exemption
Incapacitated Person 300,000 limit of Rs. 600,000 for
couple assessment
Lower of actual pension
income or 25% of Basic
Pension Income 150,000 exemption limit of Rs.
600,000 for couple
assessment
Taxable Income 293,000

Statement of Tax Liability


Couple Assessment, and
other employment
1st Rs. 293,000 1% 2,930
income is above basic
exemption limit
Tax Liability 2,930

18) Mr. Sinha is an employee of Surya Moon Corporation Ltd. His financial activities F.Y. 20X-
69/X-70 includes the following:
Basic Salary Rs. 40,000 P.M.
Traveling Allowance (From Home to Office and Rs. 4,000 P.M.
Back)
Contribution to approved Provident fund Rs. 4,000 P.M.
Dividend income from Surya Moon Corporation Rs. 10,000
Ltd.
Mr. Sinha also deposited equal amount to the same provident fund.

On the occasion of 25th Anniversary of the corporation, the corporation organized a National
1 Km. race competition in association with the Nation Game Association. Mr. Sinha
participated in the race and won the first prize worth Rs. 50,000. Mr. Sinha has a Life
Insurance Policy worth Rs. 2,00,000 and annual premium paid is Rs. 15,000.

The Institute of Chartered Accountants of Nepal 50


CAP-II Paper 7 - Income Tax and VAT

Calculate the total income and tax liability from the employment for the fiscal year 20X-
69/X-70, if he selects to be assessed as couple. (Dec 2013, 2+5=7)

Answer
Statement of Assessable Income from Employment
Particulars Sec. Ref. Amount Note
Rs. 40,000 per month for a
Basic Salary 8 (2) 480,000
year
Travel from home to office
Reimbursement of Personal Expenses 8 (2) 48,000
and back
Final withholding assuming
Dividend Income 92 - distributing company is
resident
Employer's Contribution to Retirement
8 (2) 48,000
Fund
Windfall gain, and final
withholding
If the competition were
Prize from National 1 KM race 92 -
among the staffs of the
corporation, the amount
would be included in income
Assessable Income from Employment 576,000

Statement of Taxable Income and Balance Taxable Income


Total Assessable Income 576,000
Less: Reduction under Sec. 63 -
(One-third of Assessable Income, or Actual, Total contribution is Rs.
96,000
or Rs. 300,000; whichever is lower) 96,000
Less: Deduction under Schedule 1
Lower of actual payment or
Investment Insurance Premium Paid 15,000
Rs. 40,000
Balance Taxable Income 465,000
Statement of Tax Liability
1st Rs. 465,000 1% 4,650 Couple Assessment
Tax Liability 4,650

19) Green Development Bank Ltd. has appointed Mr. Hira Paudel as CEO for the effective
operation of the bank. As per the TOR with the CEO, annual salary and allowances Rs.
5,000,000 (net of tax) shall be paid to the CEO by the bank. All tax related to the employment
of the CEO shall be borne by the company and paid to Inland Revenue Office. Mr. Hira is
to be assessed as couple and he claimed Rs. 20,000 of insurance premium. Should the bank
deduct the tax amount from his salary? If yes, then mention the relevant provisions of the

The Institute of Chartered Accountants of Nepal 51


CAP-II Paper 7 - Income Tax and VAT
Act; compute the tax liability of Mr. Hira and deductible salary and allowances for this
payment on the tax assessment of the bank. (June 2013, 5 Marks)

Answer:
As per section 87 of the Act, the bank should deduct the TDS from salary of Mr. Hira. Total tax to
be deducted from salary of Mr. Hira is Rs. 1,876,500 and his deductible salary and allowances to
the bank is Rs. 7,212,500 (W.N. 1).

Working Note 1:
Gross Amount = Net Amount divided by (1- Tax Rate)
Net Amount is Rs. 4,980,000 (net of insurance premium paid)
The balancing figure is subject to 36% tax
The balance of net amount is 49,80,000 minus net amounts of first, second, third and fourth slab

Gross Amount Tax Rate Tax amount Net Amount


Particulars Note
A B C D= A- C
1st Rs. 600,000 1% 6,000 594,000
Next Rs. 200,000 10% 20,000 180,000
Next Rs. 300,000 20% 60,000 240,000
Next Rs. 900,000 30% 270,000 630,000
(3,336,000/64%) (b.f.) Reverse
Balance Rs. 5,212,500 36% 1,876,500 3,336,000 calculation
Total 7,212,500 2,232,500 4,980,000
Insurance Premium Paid 20,000
Total Gross Salary 7,232,500

20) List out the payments not included in the income from employment under Income Tax Act,
2058. (June 2013, 5 Marks)
OR
What are the payments that are received or deemed to have been received by an employee
from the present or past employer or his associates which are not includible in the income
from “Employment” under the Income Tax Act, 2058? (June 2005, 7 Marks, CA Inter)

Answer:
The following amounts are not included in the income from employment:
i. Exempt amounts
ii. Final withholding payments
iii. Any reimbursements or discharge of expenses satisfying any of the following two
conditions:
• That serves proper business purpose of the employer
• That would or otherwise be exempt from inclusion in business or investment income
iv. Lunch or tiffin to all employees under similar terms

The Institute of Chartered Accountants of Nepal 52


CAP-II Paper 7 - Income Tax and VAT
v. The amount of the costs is so small as to make it unreasonable to require or
administratively impracticable for the person to account for them.

21) Mr. Z, retired person from Nepal Government on 1st Sharwan, 20X-68 after 30 years of
Service. After retirement, he joined Kathmandu Bank at Kathmandu on the same day,.i.e.
Sharwan, 20X-68 and remained in service till Ashad end 20X-69. Information about his
income are as follows:
S. N. Particulars Nepal government Kathmandu bank
1. Salary Rs. 3,00,000 Per month
2. House rent Rs. 20,000 Per month
allowance
3. Other allowance Rs. 10,000 Per month
4. Provident fund Additional 10 % of basic
contribution salary & grade
5. Pension received Rs. 144,000 during ----------------
the year

Other information:
a. He has been provided a vehicle for personal as well as official use by the Kathmandu
bank.
b. Both employers have deducted 10% of salary for provident fund and deposited 20% to
the approved retirement fund.
c. He had received all retirement payment in Ashad 20X-68.
d. He has not been provided Dashain allowances during the period.
e. His wife does not have any income in this Income year and they declared they are couple.
Calculate the assessable income, taxable income and tax amount of Mr. Z for the income
year. (Dec 2012, 6 Marks)

Answer
Statement of Assessable Income from Employment
Particulars Sec. Ref. Amount Note
Salary from Kathmandu Bank 8 (2) 3,600,000
House Rent Allowance from Bank 8 (2) 240,000
Other Allowance 8 (2) 120,000
There cannot be
contribution to Fund by
Employer's Contribution to Retirement Fund 8 (2) 360,000
GoN while the person is
obtaining pension income
Pension from Government of Nepal 8 (2) 144,000
0.5% of basic salary and
Vehicle Facility 27 18,000
grade
Assessable Income from Employment 4,482,000

The Institute of Chartered Accountants of Nepal 53


CAP-II Paper 7 - Income Tax and VAT

Statement of Taxable Income and Balance Taxable Income


Total Assessable Income 4,482,000
Less: Reduction under Sec. 63 -
(One-third of Assessable Income, or Actual, or Rs. Total contribution is Rs.
300,000
300,000; whichever is lower) 720,000
Lower of pension income
i.e. Rs. 144,000 or 25% of
Less: Deduction for Pension Income 144,000 basic exemption limit i.e.
600,000*25%= Rs.
150,000
Balance Taxable Income 4,038,000

Statement of Tax Liability


Couple Assessment, and other employment income is
1st Rs. 600,000 1% 6,000
above basic exemption limit
Next Rs. 200,000 10% 20,000
Next Rs. 300,000 20% 60,000
Next Rs. 900,000 30% 270,000
Balance Rs. 2,038,000 36% 733,680
Tax Liability 1,083,680

22) Mr. Ram Prasad Thapa, a Joint secretary in Nepal Government had worked since 1/4/20X-
68 to Poush end 20X-68 as acting ambassador in USA. Due to his unsatisfactory performance
GoN has called him back to Nepal and transferred to Solukhumbu as Chief District Officer.
During the income year 20X-68/X-69 he had taken following amount of salary and
perquisites:
Particulars Amount in Rs.
Salary per month 25,000
Dashain allowance 25,000
Contribution to EPF 30,000
(an equal amount of deduction has been made from his salary as his own contribution)
Foreign allowance monthly 50,000
Calculate his taxable income for income year 20X-68/X-69. (Dec 2012, 5 Marks, CA Inter)

Answer
Statement of Assessable Income from Employment
Particulars Sec. Ref. Amount Note
Salary 8 (2) 300,000
Dashain Allowance 8 (2) 25,000
Employer's Contribution to Retirement Fund 8 (2) 30,000

The Institute of Chartered Accountants of Nepal 54


CAP-II Paper 7 - Income Tax and VAT
Six months from
Foreign Allowance 8 (2) 300,000
Shrawan to Poush
Assessable Income from Employment 655,000

Statement of Taxable Income


Total Assessable Income 655,000
Less: Reduction under Sec. 63 -
(One-third of Assessable Income, or Actual, or Rs. Actual Contribution is
60,000
300,000; whichever is lower) Rs. 60,000
Salleri of
Solukhumbu district
is in Category "D",
Residence in Remote Area 10,000
deduction of Rs.
20,000; pro-rata for
six months
75% of foreign
75% of Foreign Allowance 225,000
allowance
Balance Taxable Income 360,000

23) Mrs. Sita Nepal is an employee at Manang Branch of ABC Bank Ltd. For income year 20X-
68/X-69 she earns Rs. 25,000 per month as salary and allowances. She received Rs. 20,000 as
Dashain expenses and Rs. 40,000 against bonus. Bank has contributed Rs. 24,000 during the
year towards contribution to approved retirement fund and by deducting same amount from
her salary, deposited the amount in approved retirement fund. Calculate the taxable income
and tax liability of Mrs. Sita for income year 20X-68/X-69. (Dec 2012, 4+3=7, CA Inter)
Note:
a. Mrs. Sita Nepal opts to be assessed as individual and has no other source of income.
b. Manang falls under class A as per classification given by Income Tax Act.

Answer
Statement of Assessable Income from Employment
Particulars Sec. Ref. Amount Note
Salary and Allowance 8 (2) 300,000
Dashain Allowance 8 (2) 20,000
Bonus 8 (2) 40,000
Employer's Contribution to Retirement Fund 8 (2) 24,000
Assessable Income from Employment 384,000
Statement of Taxable Income
Total Assessable Income 384,000
Less: Reduction under Sec. 63 -
(One-third of Assessable Income, or Actual, or Rs. Actual Contribution
48,000
300,000; whichever is lower) is Rs. 48000

The Institute of Chartered Accountants of Nepal 55


CAP-II Paper 7 - Income Tax and VAT
‘A' category
Residence in Remote Area 50,000
Remote Area
Balance Taxable Income 286,000

Tax Liability (1%) 2,860

24) Mr. Bhai Raja is a person working in a Financial Institution as Chief Manager. Besides
working in Financial Institution, he is a professor of Account in Shankar Dev Campus. He
generates the following income from the employers in Fiscal Year 20X-67/X-68.
a. Monthly Remuneration NPR 40,000; Dashain Allowance NPR 40,000; Education
Allowance NPR 2,000 per month; Expenditure Allowance NPR 2,000 per month; Bonus
NPR 60,000; Remuneration from teaching in Shankar Dev Campus NPR 6,000 per
month.
b. He is provided a car along with driver for his official and personal purpose. Monthly
remuneration and allowance of driver is NPR 8,000.
c. Petrol Expenditure is provided for his car which is NPR 10,000 per month and
maintenance expenditure of such car is NPR 25,000 in Fiscal Year 20X-67/X-68.
d. He is provided a housing facility by the employer.
e. Monthly expenditure of Telephone connected in his resident is NPR 2,000 per month.
Out of this 50% is his personal telephone expenditure.
f. A security guard is being provided in his resident and it is being paid NPR 6,000 per
month by the employer. Because of being used the security guard in his resident, NPR
3,000 is being deducted from his monthly salary.
g. He has used the loan amounting to NPR 1,000,000 under housing loan facility at the
rate of 5% interest. 8% interest rate is being charged by the institution under similar
loan to other borrowers.
h. He has received one-month salary against his house leave during Mangsir, 20X-67.
i. The employer contributes 10% of his drawn salary and the employee same amount
from his salary, to approved provident fund. Also, the employer contributes NPR
20,000 per month to Citizen Investment Fund up on request of employee.
j. He has covered the insurance of NPR 250,000 and NPR 150,000 of his wife in Rastriya
Beema Sansthan. He pays the premium of NPR 19,000 and NPR 14,000 respectively in
that Fiscal Year. Also, he has contributed to tax exempted entity approved from
department as donation amounting to NPR 60,000.
k. He has expensed out NPR 20,000 for his treatment in Teaching Hospital.
l. He and his wife declared that they are couple in the Fiscal Year 20X-67/X-68 and his
wife does not have any income in that Fiscal Year.

Compute Assessable Income, Taxable Income and Tax liability of Mr. Bhai Raja for the
Fiscal Year 20X-67/X-68. (Dec 2011, 12 Marks)

The Institute of Chartered Accountants of Nepal 56


CAP-II Paper 7 - Income Tax and VAT
Answer
Statement of Assessable Income from Employment
Particulars Sec. Ref. Amount Note
Monthly Remuneration 8 (2) 480,000
Dashain Allowance 8 (2) 40,000
Education Allowance 8 (2) 24,000
Expenditure Allowance 8 (2) 24,000
Bonus 8 (2) 60,000
Remuneration from Shankar Dev Campus 8 (2) 72,000
0.5% of basic salary and grade
derived by the employee from
the. All other vehicle operation
Vehicle Facility 27 2,400 expenses including driver's
salary, fuel, repair, etc. is
deemed to be included in amount
so quantified.
Driver's Salary 27 - Refer Notes on Vehicle Facility
Accommodation Facility 27 9,600 2% of basic salary and grade
Personal Telephone Expense 8 (2) 12,000
Actual cost borne by employer
Security Guard Expenses 8 (2) 36,000
per month is Rs. 3,000
Loan amount multiplied by
differential interest rate, 3% is
Loan at Concessional Interest Rate 27 30,000 differential interest rate, Loan is
assumed to be used throughout
the year
Leave Encashment 22 40,000 Cash basis amount
10% of monthly remuneration
Employer's Contribution to Retirement from bank, assuming Shankar
8 (2) 48,000
Fund Dev Campus does not provide
such amount
Assessable Income from Employment 878,000

Statement of Taxable Income


Total Assessable Income 878,000
Less: Reduction under Sec. 63
Actual Contribution is Rs.
(One-third of Assessable Income, or
292,667 48000+ Rs. 48,000+ Rs.
Actual, or Rs. 300,000; whichever is
240,000=336,000
lower)
Lower of: 5% of ATI or Actual
or Rs. 100,000
Less: Donation to Exempt Entity 12 29,267 ATI, in this case = Total
Assessable Income minus
Eligible Contribution to ARF

The Institute of Chartered Accountants of Nepal 57


CAP-II Paper 7 - Income Tax and VAT
Actual premium paid Rs. 33,000
Less: Payment of Investment Insurance
33,000 or Rs. 40,000; whichever is
Premium
lower
Balance Taxable Income 523,066
Statement of Tax Liability
1st Rs. 523,066 1% 5,231 Couple Assessment
Tax Liability 5,231

25) Mr. Yubaraj Thapa, a bank officer, retired from his job in the year 20X-65/X-66. He received
Rs. 450,000 as retirement payment. The payment was made through an approved retirement
fund. From the Shrawan 1, 20X-66 he is entitled to get a pension of Rs. 21,000 per month.
He also entitled to get one month's pension as dashain bonus. Mr. Yubaraj received interest
of Rs. 50,000 from a fixed deposit from Rastriya Banijya Bank. He has also received a
dividend of Rs. 95,000 from Nabil Bank. He started a consultancy business in the month of
Shrawan 20X-66 and earned net profit of Rs. 25,000 in the year 20X-66/X-67.

His wife Mrs. Lolita is a school teacher in a government school and getting Rs. 10,000
monthly salary. She is also operating a boutique since last five years and the net profit and
the annual sales for the year 20X-66/X-67 are Rs. 1,50,000 and Rs. 12,45,000 respectively.
Mrs. Lolita owns a house located at Putalisadak and the rental income of the house is Rs.
50,000 per month. The house was rented to a commercial bank. She also does the assignment
of question paper setting and evaluation of answers and she received Rs. 1,05,000 during the
year 20X-66/X-67.

Mr. Yubaraj and his wife have not selected the option as a couple for tax returns purpose.
Calculate the tax liability of Mr. Yubaraj and Mrs. Lolita and also state their responsibility
to file the income tax return for the year 20X-65/X-66 and 20X-66/X-67. (June 2011, 12
Marks)

Answer
Calculation of Tax Liability of Mr. Yubaraj
Sec.
Particulars Amount Note
Ref.
Pension Income 8 (2) 252,000
Dashain Bonus 8 (2) 21,000
Retirement payment - Considered in Previous Income Year
Final withholding, Considering
Interest 92 - Rastriya Banijya Bank a resident
company
Final withholding, Considering Nabil
Dividend 92 -
Bank a resident company
Assessable Income from Employment 273,000

The Institute of Chartered Accountants of Nepal 58


CAP-II Paper 7 - Income Tax and VAT
Statement of Taxable Income
Consultancy Business profit, assuming
Assessable Income from Business 25,000
computed as per Income Tax Law
Assessable Income from Employment 273,000
Taxable Income 298,000

There is no tax liability, since the amount of balance taxable income falls within the range of basic
exemption limit and income is generated through pension and business.

Calculation of Tax Liability of Mrs. Lolita


Sec.
Particulars Amount Note
Ref.
Assessable Income from
8 (2) 120,000 Salary from School
Employment
Profit from Boutique, assuming computed
as per Income Tax Law
Assessable Income from Business 7 150,000
She cannot pay presumptive tax, as she has
employment income as well
House Rent Income - Exempt, refer definition of RENT
Income from setting question paper
92 - Final withholding
and checking answer sheets
Taxable Income 270,000

Statement of Tax Liability


1st Rs. 120,000 1% 1,200
1st Rs. 150,000 0% - There is no 1% tax on business income
Total 1,200

26) Mr. Ram Singh is a salaried employee having taxable salary income amounting to Rs.
1,50,000 during the fiscal year 20X-66/X-67. Calculate the tax liability of Mr. Ram Singh
(opting as couple) as per the provisions of Income Tax Act, 2058 for the fiscal year 20X-66/X-
67. Will your answer be different if his earning was from proprietorship business not from
salary? (Dec 2009, 5 Marks)

Answer
The tax liability is 1% of Rs. 150,000 (i.e. Rs. 1,500)
There is no 1% tax on basic exemption limit in case of business income, investment income,
pension income or when a natural person contributes to Contribution based social security Fund
or Contribution based Pension Fund.
Therefore, if it were business income, the tax would be nil.

The Institute of Chartered Accountants of Nepal 59


CAP-II Paper 7 - Income Tax and VAT
27) What are the deductions, reduction and exemptions permitted under the Income Tax Act,
2058 (including Schedule 1) for the income year 20X-65/X-66, in the computation of income
under the head "employment”? Discuss. (June 2009, 5 Marks, CA Inter & June 2006, 6
Marks, CA Inter)

Answer
Reductions- There are no specific deductions in respect of employment income. However, a
natural person is eligible to reduction for donation or gift to exempt organization, contribution to
Prime Minister Relief Fund or National Reconstruction Fund of Government of Nepal, and
Contribution to Approved Retirement Fund while arriving at taxable income from Total
Assessable Income.

Deductions- A natural person is entitled to following deductions under Sec. 1 of Schedule 2 of the
Act. Please note that this is available for all income and not just employment income only:
a. Residence in Remote Area: The deduction is as follows, if a natural person stays at such remote
area throughout the year:
Category for Remote Area Amount
A 50,000
B 40,000
C 30,000
D 20,000
E 10,000
b. Facility to an employee of GON deputed in Foreign Diplomatic Mission [Sch. 1- Sec. 1 (6)]
In case a natural person who is an employee of Government of Nepal is deputed in Diplomatic
Mission of Nepal situated at foreign country derives Allowance in the head “Foreign Allowance”,
75% of amount derived in the head “Foreign Allowance” is deductible from taxable income.

c. Deduction to an incapacitated resident natural person [Sch. 1- Sec. 1 (10)]


In case a resident natural person is a disabled individual, 50% of Basic Exemption Limit is
deductible from taxable income:

d. Deduction for payment Investment Insurance Premium (Sch. 1- Sec. 1- 12)


In case a resident natural person makes payment of investment insurance premium during any
Income Year, the lower of following amount can be deductible from taxable income:
i. Actual amount of Investment Insurance Premium paid, or
ii. Rs. 40,000

e. Deduction for payment of Health Insurance premium [Sec. 1 (16) of Schedule 1]


In case a Resident Natural Person pays Medical/health Insurance Premium to a Resident Insurance
Company, the lower of the following amount can be deducted from the taxable income of that
individual:

The Institute of Chartered Accountants of Nepal 60


CAP-II Paper 7 - Income Tax and VAT
i. Annual Medical Insurance Premium paid, or
ii. Rs. 20,000

f. Deduction for payment of Building Insurance premium [Sec. 1 (16ka) of Schedule 1]


In case a Resident Natural Person pays building Insurance Premium to a Resident Insurance
Company to insure building under his/her ownership, the lower of the following amount can be
deducted from the taxable income of that individual:
i. Annual Insurance Premium paid, or
ii. Rs. 5,000

28) Mr. Sudip Sharma, Manager of M/s Jeevan Nirman Bank Ltd. furnishes the following
information related to Income Year 20X-63/X-64 (20X-63/4/1 to 20X-64/3/32).
Particulars Rs.
Monthly Basic Salary 55,000
Dashain Allowance 35,000
Bonus 50,000
Children Education Allowance 10,000
Medical Expenses Reimbursement 20,000
Free Meal (Lunch) 10,000
Dress Allowance 15,000

Other information:
a. He resides at the flat provided by the company for which company paid Rs. 5,000 p.m.
b. A car having market value of Rs. 2,000,000 is provided to him by the company. Petrol
and driver expenses for running such car is Rs. 300,000 for the year 20X-63/X-64 is
borne by the company.
c. Company provides meal (lunch) to all staff at office, ordered from Bakery Cafe, a
restaurant nearby the office at free of cost in equal basis.
d. His approved medical expenses are Rs. 1,000.
e. He has fixed deposit of Rs. 1,000,000 on Nepal Bank Ltd. for complete year. Interest
rate on deposit is 7% p.a. He has not yet received and bank has not credited interest on
his account too.
f. The Bank provides him Housing Loan of Rs. 500,000 on 20X-63/10/1 with interest of
6% p.a. Regular interest rate of such loan to outsiders is 10% p.a.
g. Mr. Sudip has life insurance policy of Rs. 100,000 and premium Rs. 12,000 is paid by
the bank.
h. Unadjusted Medical Tax Credit Rs. 850 carried forward from F.Y. 20X-61/X-62.
Calculate the Tax Payable if he opted as couple. (June 2008, 8 Marks, CA Inter)

The Institute of Chartered Accountants of Nepal 61


CAP-II Paper 7 - Income Tax and VAT
Answer
Statement of Assessable Income from Employment
Particulars Sec. Ref. Amount Note
Monthly Basic Salary 8 (2) 660,000
Dashain Allowance 8 (2) 35,000
Bonus 8 (2) 50,000
Children Education Allowance 8 (2) 10,000
Medical expense reimbursement 8 (2) 20,000
Provided to all employees under similar
Free Lunch 8 (3) -
terms, hence not included
Dress Allowance 8 (2) 15,000
Accommodation Facility 27 13,200 2% of basic salary and grade
0.5% of basic salary and grade, assuming
car is not gifted to him but is available for
his use
Vehicle Facility 27 3,300 All other vehicle operation expenses
including driver's salary, fuel, repair, etc.
is deemed to be included in amount so
quantified
Cash basis, as such, the amount is not
income as it is not paid.
Interest on Fixed Deposit 22 -
Even if it were income, it would have
been final withholding payment.
Loan amount multiplied by differential
Loan at Concessional Interest Rate 27 10,000 interest rate, 4% is differential interest
rate, Loan used for six months

Insurance Premium Paid by Bank 8 (2) 12,000

Assessable Income from


828,500
Employment

Statement of Taxable Income and Balance Taxable Income


Assessable Income from
828,500
Employment
Total Assessable Income 828,500
Less: Reduction under Sec. 63 -
Taxable Income 828,500
Payment of Investment Insurance Actual premium paid Rs. 12,000 or Rs.
12,000
Premium 40,000; whichever is lower
Balance Taxable Income 816,500

Statement of Tax Liability


1st Rs. 600,000 1% 6,000 Couple Assessment

The Institute of Chartered Accountants of Nepal 62


CAP-II Paper 7 - Income Tax and VAT
Next Rs. 200,000 10% 20,000
Balance Rs. 16,500 20% 3,300
Tax Liability before Tax Credit 29,300
Lower of:
a. 15% of Approved Medical Expense+
Carried over from Previous Year i.e. 15%
Less: Medical Tax Credit 750 of Rs. 1,000+850= Rs. 1,000
b. Rs. 750
c. Actual tax liability before any tax
credit.
Tax Liability for the Year 28,550

29) Mr. Neelmani Shakya, who is going to retire from 1st Shrawan 20X-65, submits the following
details with respect to his employment for the income year 20X-64/X-65.
Net Salary received Rs. 2,44,800
Later on, TDS Rs. 43,200 reimbursed by employer
Contribution by employer to retirement fund (approved) Rs. 80,000
His Contribution Rs. 50,000
Compensation received Rs. 40,000
Life Insurance Premium paid by Employer on his behalf, Rs. 20,000
Insured sum of Rs. 2,00,000
Encashment of un-availed leave upto Chaitra 18, 2058 Rs. 50000, after Chaitra 18, 2058 Rs.
70,000
Emergency Medical Treatment paid by Employer Rs. 10,000. But he didn’t claim.
Reimbursement of Tour Expenses during Official visit to Germany Rs. 2,40,000
Gratuity received
Up to 18 Chaitra, 2058 Rs. 500,000
After 18 Chaitra, 2058 Rs. 600,000 (approved)
Provident Fund received
Up to Chaitra 18, 2058 Rs. 400,000 (unapproved)
After Chaitra 18, 2058 Rs. 500,000 (approved)
He also provides following further information not related to his employment:
a. Gain Rs. 1,00,000 on Sale of non-business chargeable asset.
b. Donation to a work fully allowed for deduction Rs. 50,000.
c. Interest of Rs. 20,000 from deposit account from Everest Bank.
Mrs. Shakya works in a school and the couple has elected to assess the income separately.

Required: (Dec 2007, 15 Marks, CA Inter)


a. Assessable Income from Employment
b. Total Assessable Income
c. Tax Payable

The Institute of Chartered Accountants of Nepal 63


CAP-II Paper 7 - Income Tax and VAT
Answer
Statement of Assessable Income from Employment
Sec.
Particulars Amount Note
Ref.
Net Salary Received 8 (2) 244,800
TDS Reimbursed 8 (2) 43,200 Reimbursement of Personal expense
Employer's Contribution to Retirement Fund 8 (2) 80,000
Compensation received 8 (2) 40,000 Assuming related to employment
Life Insurance Premium Paid by Employer 8 (2) 20,000
Final withholding, assuming it is paid at
Leave Encashment 92 -
the time of retirement
Emergency Medical Treatment paid by
8 (2) 10,000
employer
Serves proper business purpose of
Reimbursement of Tour Expenses 8 (3) -
employer
Gratuity Received 92 - Retirement payment, final withholding
Provident Fund Received 92 - Retirement payment, final withholding
Final withholding payment, assuming
Interest on Deposit 92 -
Everest Bank is resident
Assessable Income from Employment 438,000

Statement of Taxable Income and Balance Taxable Income


Assessable Income from Business -
Assessable Income from Employment 438,000
Assessable Income from Investment 100,000
Total Assessable Income 538,000
Lower of:
a. one-third of total assessable income,
Less: Reduction under Sec. 63 130,000 b. Actual Contribution to ARF- Rs.
130,000
c. Maximum Rs. 300,000
Lower of:
a. 5% of ATI, ATI = 538,000- 130,000
Less: Reduction under Sec. 12 20,400 b. Actual Donation to Exempt Entity-
Rs. 50,000
c. Maximum Rs. 100,000
Actual premium paid Rs. 20,000 or Rs.
Payment of Investment Insurance Premium 20,000
40,000; whichever is lower
Balance Taxable Income 367,600
Statement of Tax Liability
Assessable income from employment is
1st Rs. 367,600 1% 3,676
more than Rs. 367,600

The Institute of Chartered Accountants of Nepal 64


CAP-II Paper 7 - Income Tax and VAT
Tax on Payment of Gratuity:
It is assumed that gratuity and provident fund is paid by same Approved Retirement Fund (ARF).
Students may assume them as two separate approved retirement fund.

Total Payment to be Made from ARF 2,000,000


Less: Payment accrued till 2058.12.18 900,000
Remaining Amount 1,100,000
Less: Higher of Following:
i. Rs. 500,000
ii. 50% of Remaining Amount i.e. Rs. 550,000 550,000
Taxable Amount 550,000
Rate of Tax 5%
Tax Payable (Final Withholding u/s 92) 27,500

Payment of leave accrued till 2058.12.18 is exempted from tax. However, payment of leave of Rs.
70,000 accrued after 2058.12. 18 is subject to TDS @ 15% u/s 88.

30) Calculate taxable income and tax amount for the financial year 20X-61/X-62 of Mrs. X, a
widow, with no children, employed by a bank. Explain the tax implication on the followings:
(June 2006, 8 Marks, CA Inter)
a. Provident fund as on the date 18 Chaitra 2058 was Rs. 500 thousand, which was approved
retirement fund and total provident amount of Rs. 1.2 million is received at her
retirement on Jestha 30, 20X-62.
b. She gets Rs. 400 thousand from unapproved retirement fund in which her contribution
was 300 thousand.
c. She also gets 100 thousand in total in total as medical benefits related F.Y. 20X-55/X-56.
d. She received 10 years pension amount Rs, one million in advances. She has no other
income.
e. She also received Rs. 5,000 as widow allowances from government.
Answer
Particulars Tax Implication
Provident fund as on the date 18 Chaitra 2058 was
Provident fund as on the date 18 Chaitra 2058 Rs. 500,000 is exempt
was Rs. 500 thousand, which was approved Out of remaining Rs. 700,000; gain is taxable
retirement fund and total provident amount of @5%
Rs. 1.2 million is received at her retirement on Gain= 700,000 - Higher of (500,000 or 50% of
Jestha 30, 20X-62 700,000)= 200,000
Tax = 10,000 and it is final withholding.
She gets Rs. 400 thousand from unapproved
Gain of Rs. 100,000 is taxable @ 5%
retirement fund in which her contribution was
Tax = Rs. 5,000 and it is final withholding
300 thousand

The Institute of Chartered Accountants of Nepal 65


CAP-II Paper 7 - Income Tax and VAT
She also gets 100 thousand in total in total as Medical Benefit up to Rs. 180,000 accrued until
medical benefits related F.Y. 20X-55/X-56 2058 Chaitra 18 is exempt, as such, this is exempt
The pension income is taxable as income from
She received 10 years pension amount Rs. one
employment during the year of cash receipt. The
million in advance. She has no other income
tax liability is computed in Rs. 10 Lakhs
She also received Rs. 5,000 as widow
It is exempt as per Sec. 10 of the Act
allowances from government

31) What are the types of incomes that are included under the head of Income from Employment
under Sec.8 of the Income Tax Act, 2058? (June 2005, 8 Marks, CA Inter)
Answer
The amounts to be included are as follows:
a. Payment of Salary, Wages, Leave Encashment, overtime pay, fees, commission, prizes, gifts,
bonus and other benefits
b. Payment of Personal Allowances including Dearness allowance, subsistence allowance, rent,
entertainment or Transportation allowance
c. Payments providing any discharge or reimbursement of costs incurred by the individual or an
associate of the individual
d. Payments for the individual's agreement to any conditions of the employments
e. Payments for redundancy or loss or termination from employment
f. Retirement Payments & Employer’s Contribution to the Retirement Fund
g. Other payments in respect of employment
h. Payments to be included as per Chapter 6 or 7 of the Act
This includes: Quantification of Perquisites, Indirect Payments, Amount of Compensation in
relation to Employment, Amount to be included as per the order of IRD in case where income
splitting arrangement is proved

32) Mr. S. Rana received the following amounts during the income year 20X-60/X-61. Calculate
his taxable income explaining with reasons for including or for not including any particular
item in the taxable income.
(June 2005, 15 Marks, CA Inter)
Salary Rs.25,000 per month
Dashain Allowance Rs.15,000
Dearness Allowance Rs.10,000 per month
Employer’s Contribution to approved Provident Fund 10% of salary
a. He contributed similar amount to Provident Fund.
b. He retired from service from 1st Mangsir 20X-60. He received a pension of Rs. 5,000/- per
month from the date of his retirement. He joined service on 1st Mangsir, 2020. He also
received gratuity of Rs. 600,000 from the employer.
c. He was allowed free accommodation by the employer, who allowed him to stay there till
Ashad 20X-61.

The Institute of Chartered Accountants of Nepal 66


CAP-II Paper 7 - Income Tax and VAT
d. He received his Provident Fund amount of Rs. 800,000 from the approved Provident
Fund on 1st Poush 20X-60. The balance of Provident Fund amount in his account as on
19th chaitra 2058 was Rs. 600,000.
e. He was also paid a sum of Rs. 10,000 by the Managing Director as his personal token in
appreciation of his sincere services to the employer.
f. He received a sum of Rs. 50,000 as unutilized leave salary as per the rules on 1st Magh,
20X-60.
g. He was provided one car for both official and personal purpose. Employee allowed him
to retain the car with him till end of Ashad 20X-61.
h. During Shrawan 20X-60, he was admitted to a private hospital for a week and the
employer spent Rs.75, 000 on his medical treatment to the hospital.
i. Other employees presented him with a silver casket engraving costing Rs. 25,000
j. He received a dividend of Rs. 95,000 from various companies.

Answer
Assessable Income from Employment
Sec. Reason for inclusion or
Particulars Amount Note
Ref non-inclusion
For four months from
Salary 8 (2) 100,000 Included as per Sec. 8 (2)
Shrawan to Kartik
Dashain Allowance 8 (2) 15,000 Included as per Sec. 8 (2)
For four months from
Dearness Allowance 8 (2) 40,000 Included as per Sec. 8 (2)
Shrawan to Kartik
Employer's Contribution to RF 8 (2) 10,000 10% of salary Included as per Sec. 8 (2)
For eight months from
Pension Income 8 (2) 40,000 Included as per Sec. 8 (2)
Mangsir to Ashad
Final withholding tax
Gratuity 92 - @15%, it is retirement Not included u/s 8 (3)
payment
There is no clear provision
for quantification of
accommodation facility,
when the same person uses
the facility in two different
capacity. Therefore, 2% of
salary has been taken as
Accommodation Facility 27 2,000 Included as per Sec. 8 (2)
benefit.
The sum of 0.5% of salary
and 25% of Market or
actual rent for eight
months would also be
appropriate for
quantification

The Institute of Chartered Accountants of Nepal 67


CAP-II Paper 7 - Income Tax and VAT
Payment from Approved
Retirement Fund
Until Chaitra 18, 2058 - Tax Exempt Not included u/s 8 (3)
Final withholding, it is
After Chaitra 18, 2058 92 - Not included u/s 8 (3)
retirement payment
It is not employment,
business or investment
Gift from Managing Director - income Not included u/s 5 and 6
Can be treated as windfall
gain income
Final withholding, it is
Sum of unutilized leave 92 - Not included u/s 8 (3)
retirement payment.
There is no clear provision
for quantification of
vehicle facility, when the
same person uses the
facility in two different
capacity. Therefore, 0.5%
of salary has been taken as
benefit, in absence of
Vehicle Facility 27 500 Included as per Sec. 8 (2)
information regarding
market value of vehicle
Ideally, the sum of 0.5% of
salary and 1% of Market
value of vehicle for eight
months would be
appropriate for
quantification
Medical Treatment Reimbursement of
8 (2) 75,000 Included as per Sec. 8 (2)
Reimbursement personal expenses
it is not employment,
business or investment
Gift from Colleague - income Not included u/s 5 and 6
Can be treated as windfall
gain income
Dividend Income 92 - Final withholding, assuming all companies are resident.
Total Assessable Income 282,500
Lower of:
a. one-third of total assessable income,
Contribution to ARF 63 20,000
b. Actual Contribution to ARF- Rs. 20,000
c. Maximum Rs. 300,000
Taxable Income or Balance
262,500
Taxable Income

The Institute of Chartered Accountants of Nepal 68


CAP-II Paper 7 - Income Tax and VAT
33) Mr. D is employed in an undertaking and gets the following remuneration:
Basic Salary Rs. 36,000
Dearness Allowance 50% of salary
House Rent Allowance Rs. 10,000
The employer takes a house on rent for Mr. D paying a rent of Rs. 20,000/- per month. Out
of which he recovers Rs. 10,000 from Mr. D. The employer pays the electricity charges, phone
bills and water charges. Personal phone calls are recovered from Mr. D. Mr. D has taken a
Life Insurance Policy for Rs. 5 lakhs and pays a premium at the rate of Rs. 1,250 p.m. He is
entitled to Provident Fund calculated at 10% of his salary and a similar amount is
contributed by the employer and paid into the Citizen Investment Trust. In addition, the
employer deposits Rs. 30,000 towards gratuity to the Citizen Investment Trust. Mr. D owns
a car. But the car maintenance expenses are paid by the Company.
Mr. D's wife is employed as a teacher drawing a salary of Rs. 10,000 p.m. She is also entitled
to 10% Provident Fund which is deposited with the Employee Provident Fund. She has also
taken an Insurance Policy for Rs. 5,00,000 paying Rs. 12,000 premium per annum.
The school bus takes her to school in the morning and drops her at home in the evening. The
school provides free transport on the school bus. The school collects Rs. 500 per month from
the students for the bus transport.

Advise whether they should submit separate returns and work-out the tax payable. (June
2004, 10 Marks, CA Inter)

Answer:
Assessable Income of Mr. D from employment
Particulars Sec. Ref Amount Note
Basic Salary 8 (2) 432,000 Assumed per month
Dearness Allowance 8 (2) 216,000
House Rent Allowance 8 (2) 120,000 Assumed per month
Accommodation Facility 27 8,640 2% of Basic Salary
Not quantifiable due to missing
Electricity Charges 27 -
information
Not quantifiable due to missing
Water charges 27 -
information
Expense for Personal call not
Telephone charges -
reimbursed
Assuming salary includes basic salary
Employers' Contribution to PF 8 (2) 43,200
only
Gratuity deposited in personal
Employers' Contribution to PF 8 (2) 30,000
account of employee
Not quantifiable due to missing
Car maintenance expense 27 -
information
Assessable Income from employment 849,840

The Institute of Chartered Accountants of Nepal 69


CAP-II Paper 7 - Income Tax and VAT
Less:
Lower of:
a. one-third of total assessable
income,
Contribution to Approved RF 63 116,400
b. Actual Contribution to ARF=Rs.
43,200*2+30,000
c. Maximum Rs. 300,000
Lower of actual premium Rs. 15,000
Payment of Investment Insurance Premium Sch. 1 15,000
or Rs. 40,000
Taxable Income 718,440
Computation of Tax Liability Assuming single
1st Rs. 500,000 1% 5,000
2nd Rs. 200,000 10% 20,000
Balance Rs. 18,440 20% 3,688
Total Tax Payable 28,688

Assessable Income of Mrs. D from employment


Particulars Sec. Ref Amount Note
Salary 8 (2) 120,000
Employers' Contribution to PF 8 (2) 12,000
Not included in income of Mrs.
Use of School bus 21 -
D.
Assessable Income from employment 132,000
Less:
Lower of:
a. one-third of total assessable
income,
Contribution to ARF 63 24,000
b. Actual Contribution to ARF-
Rs. 12,000*2
c. Maximum Rs. 300,000
Lower of actual premium Rs.
Payment of Investment Insurance Premium Sch. 1 12,000
12,000 of Rs. 40,000
Taxable Income 96,000

Tax Liability 960 1% of taxable income.

Assuming all conditions of Sec. 4 (3) are satisfied neither of Mr. D and Mrs. D are required to
submit income returns.

34) Mr. Ram, a chartered accountant, is working as a finance manager in a company. The
company pays him Rs. 50,000 per month as salary and Rs. 5,000 as helper allowance of which
he actually pays Rs. 4,500 to the helper for his duties.

The Institute of Chartered Accountants of Nepal 70


CAP-II Paper 7 - Income Tax and VAT
He has made an application for 3 months advance salary in the month of Falgun 20X-59 and
the company approved and paid him 2 months advance salary (excluding allowances) to be
deducted from his salary within 6 months starting from 1st Shrawan, 20X-60.
The company has provided a quarter to Mr. Ram. The company has also provided a car
facility for private use of Mr. Ram.
Mr. Ram has paid Rs. 10,500 for his medical expenses during the year. He has contributed
Rs. 35,000 to various institutions which include Rs. 25,000 given to a Public charitable
institution approved as exempt organization by the Inland Revenue Department.
Mr. Ram has received Rs. 17,000 (net of TDS) as meeting fee during the year. Due to the
poor performance of the company, it faces severe problem and did not pay any salary to its
staff in the month of Baisakh 20X-60.
Work out the taxable income from remuneration and the Tax liability of Mr. Ram for the
financial year 20X-59/X-60, assuming he has no other income. (Dec 2003, 6 Marks, CA
Inter)

Answer
Assessable Income from Employment
Sec.
Particulars Amount Note
Ref
Cash basis, as such, amount not received for the
month of Baisakh is not included in income. It
Salary 8 (2) 550,000
will be included when there is inflow of cash to
Mr. Ram
Assumed that the helper is deployed for
employers’ work. Income = (5,000-4,500)*11; 11
Helper Allowance 8 (2) 5,500 months as no payment for the month of Baisakh.
Students may assume helper as domestic helper.
In this case income would be Rs. 5,000*11.
It is not salary income. It is actually a loan, and a
Advance Salary -
loan is not an income.
Accommodation Facility 27 11,000 2% of salary
Vehicle Facility 27 2,750 0.5% of salary
Final withholding, assuming per meeting fee is
Meeting Fee 92 -
less than Rs. 20,000
Dividend Income 92 -
Total Assessable Income 569,250
Less:
Lower of:
a. 5% of ATI i.e. 569,250*5%
Donation to Exempt Organization 63 25,000
b. Actual - Rs. 25,000
c. Maximum Rs. 100,000
Taxable Income 544,250

The Institute of Chartered Accountants of Nepal 71


CAP-II Paper 7 - Income Tax and VAT
Tax Liability
1st Rs. 500,000 1% 5,000 Assuming Individual Assessment
Balance Rs. 44,250 10% 4,425
Tax Liability before tax credit 9,425
Min of:
a. 15% of Approved Medical expense, 15% of Rs.
10,500 (assuming Rs. 10,500 is approved)
Les: Medical tax Credit 750
b. Max. Rs. 750
c. Actual tax liability before any tax credit i.e., Rs.
9,425.
Net Tax Liability for the year 8,675

35) Mr. Ram is a retired Civil Servant and Pension holder. His wife Mrs. Mohani is a housewife.
Ram has received pension Rs. 10,500 per month for 13 months including Dashain allowance.
Calculated the tax liability for the income year 20X-59/X-60 while Mr. Ram is residing in
class (D) location. (June 2003, 6 Marks, CA Inter)

Answer
The total income without considering deduction for having stayed at “D” Category Remote Area
is less than the basic exemption limit. As such, there is no tax obligation of Mr. Ram, in both the
assessment whether it is couple assessment or individual assessment.
Total Balance Taxable Income= Rs. 136,500 minus Rs. 20,000= Rs. 116,500

The Institute of Chartered Accountants of Nepal 72


CAP-II Paper 7 - Income Tax and VAT

Chapter 4: Deduction of expenses

1) Simple Innovations Pvt. Ltd., a manufacturing company received a tax assessment order for
fiscal year 20X-66/X-67 on 20X-69/8/20 from Inland Revenue Office. Total tax assessed by
tax authority is Rs. 75,00,000 which includes disputed tax of Rs. 54,43,500. Some of the issues
stated in the assessment order are as follows: (Dec 2013)
a. Company has capitalized interest cost worth Rs. 19,34,569 (interest till X67.01.10, on loan
taken for machinery) to plant and machinery ready to /put to use in 20X-67.01.10. As per
Section 14(1), interest on loan which is utilized in the same fiscal year or loan taken for
purchase of assets which is utilized in same fiscal year is allowed as deduction. Thus,
interest on assets put to use in fiscal year 20X-66/X-67 is not allowed to be capitalized and
need to be removed from depreciation base.
b. Value of opening stock includes Repair & Maintenance Expense of Rs. 7,56,760 and
Closing Stock includes Repair Expense Rs. 6,54, 650. Same is not allowed and need to be
written back.
Management is not satisfied with the assessment order and seeks your consultancy.

a. Comment on the above issues in light of provisions of Income Tax Act, 2058. (3+3=6)

Answer
My comments to management are as follows:
With regards to capitalization of interest expense
The argument of the tax officer is correct. As per Sec. 14 (1), interest on loan which is utilized in
the same fiscal year or loan taken for purchase of assets which is utilized in same fiscal year is
allowed as deduction. When an asset is put to use for a moment during any Income Year, the
interest expense of whole year is deductible and cannot be capitalized.

With regards to deduction of repair cost as part of Cost of Trading Stock:


The question is not clear about the nature of repair and maintenance expenses. So, we have the
following alternatives to this question:
• If it is related to depreciable asset or business asset, the argument of tax officer is correct; as
Sec. 16 deals with repair cost of depreciable asset and repair cost of business asset is treated as
outgoings of such asset
• If it is related to trading stock, the argument of tax officer is incorrect.

b. Company is planning to apply for administrative review. Answer the following in these
regards. (21=2)
i. Time limit to file application for administrative review.
Within 30 days of receipt of Assessment Order u/s 102. Further, 30 days can be extended by IRD
if an application with genuine reason is filed within 7 days of elapse of first 30 days.

The Institute of Chartered Accountants of Nepal 73


CAP-II Paper 7 - Income Tax and VAT
(Not applicable to CAP II under current syllabus)
ii. Amount of deposit to be kept for administrative review.
The amount is 100% of undisputed tax amount and one-fourth of disputed tax amount.
(Not applicable to CAP II under current syllabus)

2) A company engaged in garment manufacturing, debited to its Profit and Loss Account a sum
of Rs. 90,000, being the interest on loan of Rs. 9,00,000 taken for financing its expansion
scheme. The plant and machinery purchased for the project with the loan was not received
during the year and those were still in transit at the end of the year.
Discuss the admissibility or otherwise of the interest on borrowing. (June 2006, 5 Marks, CA
Inter)

Answer
As per Sec. 14 (1), interest on loan which is utilized in the same fiscal year or loan taken for
purchase of assets which is utilized in same fiscal year is allowed as deduction. When an asset is
put to use for a moment during any Income Year, the interest expense of whole year is deductible.
However, if the asset is not put to use for whole year, the expense is not deductible.
As such, the interest of Rs. 90,000 on plant and machinery not put to use during the year is
ineligible for deduction. It forms part of outgoings of plant and machinery.

3) Explain in brief the procedure to be followed for the calculation of closing stock as per
Income Tax Act, 2058. Is it in line with the Nepal Accounting Standard? (Dec 2008, 5 Marks,
CA Inter)

Answer
Procedure followed for calculation of value of Closing Stock
Closing stock is valued at lower of cost or Market value.

Cost of Closing stock:


The cost of closing stock is determined using Specific Identification Method, however, when this
method cannot be used, it is determined using First-In-First-Out Cost Formula or Weighted
Average Cost formula.

Cost of Inventory for manufacturing industries


With regards to calculation of cost of inventory in production process, a person following cash
basis may use prime costing or absorption costing; however, the person following accrual basis
must use absorption costing.

In absorption and prime costing, both, the depreciation and repair and improvement cost of
depreciable asset cannot be allocated as cost of inventory.

The Institute of Chartered Accountants of Nepal 74


CAP-II Paper 7 - Income Tax and VAT
Resemblance with Nepal Financial Reporting Standard
As per NAS 02- Inventories, inventories are valued at Cost or Net Realizable Value (NRV). NRV
differs from Market value, since NRV is market value less cost to sales.
Further, if NFRS has to be followed, the person can use only absorption costing and not prime
costing while determining the cost of production, and depreciation of production equipment and
other cost may be allocated to cost of inventory.
Apart from the above, valuation of closing stock confirms the valuation principle under prevailing
accounting standard.

4) How do you treat “a Company receives Rs. 10 lacs from Insurance Company for the damage
of its Stock in Transit” for tax purpose? (June 2007, 1 Mark, CA Inter)
Answer
The amount is included in income as per Sec. 31 during the year when the compensation is
received.

5) Explain the methods of stock valuation under the Income Tax Act, 2058. (Dec 2003, 3 Marks,
CA Inter)
Answer
Valuation of Opening Stock
The value of opening stock is same as the value of closing stock of previous year.

Valuation of Closing Stock


Closing stock is valued at lower of cost or Market value.
The cost of closing stock is determined using Specific Identification Method, however, when this
method cannot be used, it is determined using First-In-First-Out Cost Formula or Weighted
Average Cost formula.

With regards to calculation of cost of inventory in production process, a person following cash
basis may use prime costing or absorption costing; however, the person following accrual basis
must use absorption costing.

In absorption and prime costing, both, the depreciation and repair and improvement cost of
depreciable asset cannot be allocated as cost of inventory.

6) The status of property, plant and equipment and repair and improvement Expenditure XYZ
& Co. during Income Year 20X-68/X-69 is as follows:
Block of Assets Repair and Improvement Expenditure (Rs.) Depreciation basis (Rs.)
Building 20,00,000 10,00,00,000
Computer 100,000 8,00,000
Automobile 16,00,000 1,60,00,000
Total 37,00,000 11,68,00,000
Answer the followings mentioning the relevant provisions of Income Tax Act, 2058:

The Institute of Chartered Accountants of Nepal 75


CAP-II Paper 7 - Income Tax and VAT
a. Can repair and improvement expenditure of Rs. 3,700,000 be claimed under Income Tax
Act, 2058. If not, compute the amount of repair and improvement expenditure that can
be claimed by XYZ & Co. during Income Year 20X-68/X-69.
b. What is the implication of the amount of repair and improvement expenditure, if any,
which cannot be claimed during Income Year 20X-68/X-69?
(Dec 2012, 8 Marks)
Answer
Calculation of Eligible Repair and Improvement Cost
Block A Block B Block C Total
Depreciable Basis
Building 10,000,000
Computer 800,000
Automobile 16,000,000
Depreciable Basis 10,000,000 800,000 16,000,000
7% of Depreciable Basis (A) 700,000 56,000 1,120,000
Actual Repair and Improvement Cost
(B) 2,000,000 100,000 1,600,000
Eligible (C=Lower of "A" or "B") 700,000 56,000 1,120,000 1,876,000
Excess Cost (B - C) 1,300,000 44,000 480,000 1,824,000

Conclusion
As calculated above, only Rs. 1,876,000 is eligible for deduction u/s 16 of the Act.
Treatment of Excess amount:
The excess cost of Block A, B and C of Rs. 1,300,000, 44,000 and 480,000 respectively are
treated as outgoings for respective block and added to the block to calculate opening
depreciation base for 20X-69/X-70.

7) An Airlines company operating in Nepal undertakes full repairs and maintenance of


aircrafts as per norms and standards prescribed by civil Aviation Authority of Nepal. The
total annual expenditure for such repairs & maintenance stood at Rs. 6.40 crore which is
equivalent to 8% of depreciation base at year end of that Assets Pool. Suggest the maximum
allowable repairs and maintenance as per the Act, quoting the relevant section. (June 2011,
CA Inter, 4 Marks)

Answer
As per proviso to Sec. 16 (2), the overhauling expense of aircraft as per the standard prescribed by
Civil Aviation Authority of Nepal can be claimed without invoking the limit of 7% of depreciable
basis.
As such, the expense of Rs. 6.40 can be claimed irrespective of the limit.

The Institute of Chartered Accountants of Nepal 76


CAP-II Paper 7 - Income Tax and VAT
8) Horizon Pvt. Ltd. entered into a publication copyright contract on 1 st Chaitra, 20X-72 with
Florish Pvt. Ltd. to publish a book. Royalty is fixed Rs. 1,200,000 for 2 years and 10 months
with maximum copies of 10,000 per year. Find the deductible expenses for three years for
tax return. (Dec 2018, 5 Marks)

Answer
Life (converted to nearest half year) = 3 years
Depreciation Method is Straight Line
Block E
Particulars 20X- 20X- 20X- 20X-
72/X-73 73/X-74 74/X-75 75/X-76
Opening Depreciation Base - 933,333 533,333 133,333
Absorbed Additions
800,000 -
(2/3rd of Rs. 12 Lakhs)
Disposal Proceeds - -
Depreciable Basis (A) 800,000 933,333 533,333 133,333
Depreciation Rate (B)
33.33% 33.33% 33.33% 33.33%
(100/3%)- SLM basis
Depreciation Amount (C=A*B) 266,667 400,000 400,000 133,333
Unabsorbed Additions (D) 400,000 - -
Opening Depreciation Base for next year
933,333 533,333 133,333 -
(E=A-C+D)

Note 1:
a. Depreciation for Year 2 and 3 is one-third of Rs. 12 Lakhs
b. Balance amount is depreciation for Year 4

9) As a tax consultant, you have been enquired by the clients on the depreciation facility for the
purpose of income tax on following cases: (June 2011, 2×2.5=5)

a. Assets, required for power generation for its industry, capitalized by the production-
oriented industries.

Answer
If a person purchases asset for the purpose of power generation, 50% of the cost of such asset shall
be deductible during the year when such assets were capitalized.

b. If a person who wants to issue the tax invoices using fiscal printer and cash machine.

Answer
In case where fiscal printer or cash machine is purchase, the cost of such fiscal printer and cash
machine shall be deductible during the year of purchase.

The Institute of Chartered Accountants of Nepal 77


CAP-II Paper 7 - Income Tax and VAT

10) XYZ Co. Ltd purchased an Accounting Software for Rs. 50,00,000 with the life span of 10
yrs and 5 months. It was installed on 1st Shrawan 20X-65. Also, the Company purchased
Inventory Management Software for Rs. 31,50,000 with the working life of 10 yrs and 6
months. This was put in use in the month of Baisakh 20X-66.
Compute the allowable depreciation for Tax purpose for Financial Year 20X-65/X-66.
(June 2011, 5 Marks)

Answer
Calculation of Depreciation for Income Year 20X-65/X-66
Inventory
Accounting
Management
Particulars Software Total Note
Software
Block E1 Block E2
Opening Depreciation Base - -
100% of Cost in case of
Absorbed Additions 5,000,000 1,050,000 "E1" and one-third of cost
in case of "E2"
Disposal Proceeds - -
Depreciable Basis (A) 5,000,000 1,050,000
Depreciation Rate (B) 9.52% 9.52%
Depreciation Amount
476,190 100,000 576,190
(C=A*B)

Working Note
1. Depreciation Rate for Block E1

The life is 10 years 5 months, and when converted into nearest half year (using mathematical
rounding off formula), it is 10.5 years. Depreciation Rate is 100/10.5=9.52%

2. Depreciation Rate for Block E2

The life is 10 years 6 months, and when converted into nearest half year, it is 10.5 years.
Depreciation Rate is 100/10.5 =9.52%

11) M/s Everest Distilleries Ltd. (Liquor manufacturing unit) gives you the following
information pertaining to F.Y. 20X-66/X-67.
Land and Building 1,75,50,000
Machineries 2,70,80,000
Motorcycle 2,50,000
Truck 24,00,000
Computer and Laptop 8,70,000
Office Furniture 2,50,000

The Institute of Chartered Accountants of Nepal 78


CAP-II Paper 7 - Income Tax and VAT
Air Conditioner 3,60,000

The following Assets were added during the year.


Addition date
Car Rs. 28,00,000 Ashad 25, 20X-67
Computer & Laptop Rs. 3,00,000 Falgun 12, 20X-67
Computer & Laptop Rs.1,70,000 Baishak 03, 20X-67
Air Conditioner Rs.1,20,000 Ashad 05, 20X-67

During the year Computer for Rs. 2,10,000 (book value Rs. 2,60,000) and Air conditioner for
Rs. 70,000 (book value Rs. 60,000) were also sold.

Note: The cost of acquisition of land is Rs. 35,50,000. (June 2011, 15 Marks, CA Inter)
a. Calculate allowable pool-wise depreciation as per Income Tax Act, 2058.
Answer
Calculation of Depreciation Blocks
A B C D
Opening Depreciation Base 14,000,000 1,480,000 2,650,000 27,080,000
Add: Absorbed Additions - 296,667 933,333 -
Less: Disposal Proceeds - (280,000) - -
Depreciable Basis (A) 14,000,000 1,496,667 3,583,333 27,080,000
Depreciation Rate (B) 5% 25% 20% 15%
Depreciation (C=A*B) 700,000 374,167 716,667 4,062,000

Working Notes
1. Opening Depreciation Base
Blocks
Particulars
A B C D
Land and Building 17,550,000
Less: Value of Land (3,550,000)
Machineries 27,080,000
Motorcycle 250,000
Truck 2,400,000
Computer and Laptop 870,000
Office Furniture 250,000
Air Conditioner 360,000
Opening Depreciation Base 14,000,000 1,480,000 2,650,000 27,080,000

The Institute of Chartered Accountants of Nepal 79


CAP-II Paper 7 - Income Tax and VAT

2. Absorbed Additions
Blocks
A B C D
Until Poush - - - -
From Magh to Chaitra (2/3rd of
Cost)
Computer & Laptop 200,000
From Baisakh to Ashad (one-
third of cost)
Car 933,333
Computer & Laptop 56,667
Air Conditioner 40,000
Absorbed Additions - 296,667 933,333 -

b. Can this company claim 1/3rd additional depreciation being a manufacturing unit?
Answer
This company cannot claim additional depreciation, as it is not a special industry.

12) KMC Pvt. Ltd. had these depreciated balances in ‘Block B’ of assets as on Ashad 31, 20X-
66:
Amount (Rs.)
Furniture and Fixtures 50,000
Computers 25,000
Other office Equipment 10,000
Total of Block B 85,000

The company on Shrawan 10, 20X-66 disposed of the computer for the following
considerations:
Condition I: Sales consideration received was Rs. 100,000.
Condition II: Sales consideration received was Rs. 30,000.

Calculate the taxable gain from the disposal of Computer and balances in the Block for the
purpose of depreciation for the year 20X-66/X-67 in each of the above conditions. (June
2010, 5 Marks)

Answer
Calculation of Depreciation Block B
Condition I Condition II
Opening Depreciation Base 85,000 85,000
Add: Absorbed Additions - -

The Institute of Chartered Accountants of Nepal 80


CAP-II Paper 7 - Income Tax and VAT
Less: Disposal Proceeds (100,000) 30,000
Depreciable Basis (A) - 115,000
Balance Charge (Inclusion) 15,000
Depreciation Rate (B) 25% 25%
Depreciation (C=A*B) - 28,750

Note: When depreciable basis is negative, it is the case of deemed disposal. The balance
of Rs. 15,000 is treated as gain on disposal of depreciable asset and the Pool is continued
at Zero Value

13) On 20X-65 Magh 10, M/s Stunning Traders Ltd. has purchased a license for a trademark at
a cost of Rs. 6,00,000. The useful life of the license is 5 years 2 months. The stunning traders
Ltd. wants to claim the entire amount as an expense in the current year. As a tax consultant
give your opinion on the admissibility of this transaction by quoting the relevant provision
of the Act. (June 2010, 5 Marks, CA Inter)

Answer
M/s Stunning Traders Ltd. cannot claim the entire amount as expense during the year, as the
purchase of trademark is purchase of intangible asset, which is subject to depreciation u/s 19 and
Schedule 2 of Income Tax Act, 2058.
The useful life of the asset is 5 years (when rounded off to nearest half year), which gives
depreciation rate of 20% on Straight Line Method.
The depreciation for the year is two-third of Rs. 120,000 [20% of 600,000 multiplied by two-third
as added in Magh], i.e. Rs. 80,000.
The company has to claim depreciation of Rs. 120,000 each for next four years and remaining Rs.
40,000 will be claimed as depreciation in sixth income year from now.

14) If a person has constructed building on leased land, shall depreciation on that building is
allowable as per the Act? (Dec 2009, 2 marks, CA Inter)
Answer
Yes, the depreciation shall be claimed over the lease period.

15) M/s Mechi Cigarette Industries Ltd. (producer of cigarettes) furnished the following
particulars to you pertaining to the income year 20X-64/X-65.
a. Opening balance (WDV) of depreciable assets as on 20X-64/4/1:
Particulars NRs.
Building 10,00,000
Car 6,00,000
Computers 70,000
Office Equipment 1,20,000

The Institute of Chartered Accountants of Nepal 81


CAP-II Paper 7 - Income Tax and VAT
Plant & Machinery 8,00,000
Tools 30,000
Repair & Improvement cost capitalized ( block D) 10,000
b. The company has purchased a plant & machinery as on 20X-65/3/15 for NRs.6,00,000.
The company has also purchased a mini bus as on 20X-64/6/25 for NRs.3,50,000.
c. During the year one old computer having written down value of NRs. 12,500 is sold for
NRs. 7,500. One printer having written down value of NRs. 3,500 became unusable and
the company recovered nothing from it.
d. Repair and improvement expenses of the company during the year are:
Particulars NRs.
Building 90,000
Office Equipment 10,000
Car 75,000
Plant & Machinery 80,000
e. During the year the company has incurred NRs. 5,00,000 on research and development.
However, only NRs. 3,75,000 is allowable deduction for research and development cost
for the income year 20X-64/X-65.

Required: (June 2009, 5 X 3= 15)


a. Classify the assets as per schedule 2 of Income Tax Act, 2058.
b. Amount of depreciation for the income year 20X64/X-65 as per schedule 2 of income Tax
Act 2058.
c. Amount of opening depreciable basis for the Income Year 20X-65/X-66.

Answer
Calculation of Depreciation Blocks
A B C D
Opening Depreciation Base (A) 1,000,000 190,000 600,000 840,000
Add: Absorbed Additions (B) - - 350,000 200,000
Less: Disposal Proceeds ( C) - (7,500) - -
Depreciable Basis (D = A+B-C) 1,000,000 182,500 950,000 1,040,000
Depreciation Rate (E) 5% 25% 20% 15%
Depreciation (F= D X E) 50,000 45,625 190,000 156,000
Unabsorbed Additions (G) - - - 400,000
Excess Repair and Improvement Cost (H) 20,000 - 8,500 7,200
Excess PCC and R&D Cost (I) 125,000
Opening Depreciation Base for next Year (D-
F+G+H+I) 970,000 136,875 768,500 1,416,200

The Institute of Chartered Accountants of Nepal 82


CAP-II Paper 7 - Income Tax and VAT

Working Notes
1. Opening Depreciation Base & Classification of Asset
Blocks
A B C D
Building 1,000,000
Car 600,000
Computers 70,000
Office Equipment 120,000
Plant & Machinery 800,000
Tools 30,000
Repair & Improvement cost capitalized ( block D) 10,000
Opening Depreciation Base 1,000,000 190,000 600,000 840,000

2. Absorbed Additions
Blocks
A B C D
Until Poush
Minibus 350,000
From Magh to Chaitra (2/3rd of Cost) - - - -
From Baisakh to Ashad (one-third of cost)
Machinery - 600,000
Absorbed Additions - - 350,000 600,000

3. Calculation of Excess Repair and Improvement Cost


A B C D
7% of Depreciable Basis (A) 70,000 12,775 66,500 72,800
Actual Repair and Improvement Cost (B) 90,000 10,000 75,000 80,000
Eligible (C= Lower of "A" or "B") 70,000 10,000 66,500 72,800
Excess (B-C) 20,000 - 8,500 7,200

16) GTC Pvt. Ltd. had the following balances in block B of depreciable assets (after charging
depreciation for income year 20X-63/X-64)) as on the end of income year 20X-63/X-64.
Office equipment Rs. 1,50,000
Furniture Rs. 95,000
Repair & improvement exp. to be capitalized in income year 20X- Rs. 25,000
64/X-65
The Company has purchased furniture for Rs. 45,000 on Falgun 15, 20X-64. The entire block
B has been destroyed by a fire occurred as on Chaitra 1, 20X-64 and the Company has
received Rs. 325,000 from an insurance company. Calculate the gain from the disposal.
(Dec 2008, 5 Marks, CA Inter)

The Institute of Chartered Accountants of Nepal 83


CAP-II Paper 7 - Income Tax and VAT

Answer:
This is the case of disposal of a pool, as such, balancing charge or terminal depreciation
is calculated without considering absorption during addition of assets
Block B Note
Opening Depreciation Base 270,000 Sum of given figures in Table
Additions during the year 45,000
Disposal Proceeds (325,000)
Terminal Depreciation / (Balancing Charge) (10,000) To be included in income
The amount of balancing charge is treated as gain on disposal of depreciable asset.

17) State the businesses on which additional depreciation (one third of the basic rate) is allowed
as per Income Tax Act, 2058. (Dec 2008, 5 Marks, CA Inter)

Answer
One-third additional depreciation is allowed for Pool A, B, C and D (not E) for the following entity
or projects:
a. Cooperatives that are not dealing in tax exempt business under Sec. 11,
b. Projects (of entity) involving construction of power houses, generation and transmission of
electricity and where these are carried out by an entity,
c. Public Infrastructure projects of entity that are ultimately be handed over to Government of
Nepal and where these are carried out by an entity,
d. Special industries operated by entity,
e. Entities operating trolley or tram bus,
f. Entities involved in building and operation of ropeway, cable car or overhead bridge, and
g. Entities involved in building and operation of road, bridge, tunnel way, subway, railway and
airport.

18) Geeta Air (P) Ltd., an airlines company has purchased a mini air craft. What is the rate of
depreciation applicable for such crafts? (June 2008, 1 Mark, CA Inter)
Answer: 15%

19) How will you deal with the following for the purpose of Income Tax Act, 2058? (June 2007,
2 Marks, CA Inter)
a. “Purchase of Patent right for Rs. 20 lacs for using the special process for the next 10
years”
Answer:
Depreciation is charged during next 20 years @ 10% in Straight Line Method.

The Institute of Chartered Accountants of Nepal 84


CAP-II Paper 7 - Income Tax and VAT
b. Charges paid to technical engineer of Rs. 5 lacs for locating a proper place for plantation
of Machinery.

Answer:
The amount is treated as part of outgoings of plant and machinery.

20) Mr. Ram is the proprietor of HHH Manufacturing, a special Industry. He is an expert in the
field of his business but lacks the knowledge of the provisions of Income Tax Act, 2058
relating to depreciation. He asks you to calculate the amount allowable for depreciation or
the income year 20X-62/X-63 from his manufacturing industry.
Also, advise him if he is eligible for any additional depreciation as per the Income Tax Act.
Following information is provided to you:
Opening WDV as on 1 Shrawan 20X-62:
Building 15,00,000
Machinery 3,50,000
Office Equipment 80,000
Transactions during the year:
a. Office equipment worth Rs. 50,000 purchased on Chaitra 23, 20X-62.
b. Shipping expenses of Rs. 10,000 paid for the above equipment.
c. Machine, the WDV of which was 53,000 was sold for Rs. 45,000 on Baishak 1, 20X-
63.
(June 2006, 5 Marks, CA Inter)

Answer
Advice on additional Depreciation: Additional depreciation is provided to an entity and not to a
natural person. As such, private firm of Mr. Ram, HHH Manufacturing, is not entitled to additional
depreciation.
Calculation of Depreciation Blocks
A B C D
Opening Depreciation Base (A) 1,500,000 80,000 - 350,000
Add: Absorbed Additions (B) - 40,000 - -
Office Equipment (two-third of cost., added in
Chaitra)
Less: Disposal Proceeds ( C) - - - (45,000)
Depreciable Basis (D = A+B-C) 1,500,000 120,000 - 305,000
Depreciation Rate (E) 5% 25% 20% 15%
Depreciation (F= D X E) 75,000 30,000 - 45,750

The Institute of Chartered Accountants of Nepal 85


CAP-II Paper 7 - Income Tax and VAT
21) M/s Gandaki Brewery Industries Ltd. furnished the following particulars to you pertaining
to the income year 20X-61/X-62.
a. Opening balance (WDV) of depreciable assets as on 20X-61-4-1.
Particulars Rs.
Building 20,00,000
Car 12,00,000
Computers 1,40,000
Office equipment 2,40,000
Plant & Machinery 16,00,000
Tools 60,000
Repair & improvement cost capitalized (block D) 20,000
b. The Company has purchased a plant & machinery as on 20X-62/3/15 for Rs. 12,00,000.
The Company has also purchased a mini bus as on 20X-61/6/25 for Rs. 7,00,000.
c. During the year one old computer having written down value of Rs. 25,000 is sold for Rs.
15,000. One printer having written down value of Rs. 7,000 became unusable and the
Company recovered nothing from it.
d. Repair & improvement expenses of the Company during the year are:
Particulars Rs.
Building 1,80,000
Office equipment 20,000
Car 1,50,000
Plant & Machinery 1,60,000
e. During the year the Company has incurred Rs. 10,00,000 on research and development.
However, only Rs. 7,50,000 is allowable deduction for research and development cost for
the income year 20X-61/X-62.

Required: (Dec 2005, 15 Marks, CA Inter)


a. Classify the assets as per schedule 2 of Income Tax Act 2058.
b. Amount of depreciation for the income year 20X-61/X-62 as per schedule 2 of Income
Tax Act 2058.
c. Amount of depreciable basis for the Income Year 20X-62/X-63.

Answer
Calculation of Depreciation Blocks
A B C D
Opening Depreciation Base (A) 2,000,000 380,000 1,200,000 1,680,000
Add: Absorbed Additions (B) - - 700,000 400,000
Less: Disposal Proceeds ( C) - (15,000) - -
Depreciable Basis (D = A+B-C) 2,000,000 365,000 1,900,000 2,080,000
Depreciation Rate (E) 5% 25% 20% 15%
Depreciation (F= D X E) 100,000 91,250 380,000 312,000

The Institute of Chartered Accountants of Nepal 86


CAP-II Paper 7 - Income Tax and VAT
Unabsorbed Additions (G) - - - 800,000
Excess Repair and Improvement Cost (H) 40,000 - 17,000 14,400
Excess PCC and R&D Cost (I) 250,000
Opening Depreciation Base for next Year (D-
F+G+H+I) 1,940,000 273,750 1,537,000 2,834,200

Working Notes
1. Opening Depreciation Base & Classification of Asset
Blocks
A B C D
Building 2,000,000
Car 1,200,000
Computers 140,000
Office Equipment 240,000
Plant & Machinery 1,600,000
Tools 60,000
Repair & Improvement cost capitalized ( block D) 20,000
Opening Depreciation Base 2,000,000 380,000 1,200,000 1,680,000

2. Absorbed Additions
Blocks
A B C D
Until Poush
Minibus 700,000
From Magh to Chaitra (2/3rd of Cost) - - - -
From Baisakh to Ashad (one-third of cost)
Machinery - 400,000
Absorbed Additions - - 700,000 400,000

3. Calculation of Excess Repair and Improvement Cost


A B C D
7% of Depreciable Basis (A) 140,000 25,550 133,000 145,600
Actual Repair and Improvement Cost (B) 180,000 20,000 150,000 160,000
Eligible (C= Lower of "A" or "B") 140,000 20,000 133,000 145,600
Excess (B-C) 40,000 - 17,000 14,400

22) How will you treat the following in books of accounts and for tax purpose? (Dec 2005, 3
Marks, CA Inter)
a. Purchase of Patent rights for Rs.25 lacs for using the special process for the next 10 years.
Answer:
Depreciation is charged during next 20 years @ 10% in Straight Line Method.

The Institute of Chartered Accountants of Nepal 87


CAP-II Paper 7 - Income Tax and VAT

b. Survey expenses incurred Rs.50 lacs for locating a proper place for putting up a dam for
generation of electricity.
Answer:
The expense is treated as outgoings of dam, which will be added to Pool “A” asset when put to
use.

c. A generator is purchased for Rs.25 lacs as standby to be used when the existing generator
has to be stopped for overhauling. But during the year, there was no opportunity to put the
newly purchased generator to use.
Answer
Generators are purchased to use during power stoppage and such assets are always in passive use
from the date when it is available for use. Therefore, depreciation can be claimed on such asset.

23) The ABC Co. Pvt. Ltd. has assets of B category (computer group) worth Rs. 3,20,000 as on
32 Asadh, 20X-59. On Poush 15, the Company purchased a branded computer-set costing
Rs. 150,000 and the company also paid for repair of old computer Rs. 42,000. During the
year one set computer was disposed at Rs. 60,000. Compute the eligible depreciation for this
group of assets and depreciation base for the year 20X-59/X-60 according to provision of
Income Tax Act 2058. (June 2003, 4 Marks, CA Inter)
Answer
Particulars Block B Note
Assumed to be after depreciation for the year 20X-
Opening Depreciation Base (A) 320,000
58/59
Add: Absorbed Additions (B) 150,000 Purchased in Poush, 100% of cost is absorbed
Less: Disposal Proceeds ( C) (60,000)
Depreciable Basis (D = A+B-C) 410,000
Depreciation Rate (E) 25%
Depreciation (F= D X E) 102,500

The Institute of Chartered Accountants of Nepal 88


CAP-II Paper 7 - Income Tax and VAT

Chapter 5: Loss Set-off and Carry Forward

1) X & Co., a sole trading firm owned by Mr. X, dealing with the business of selling male
garments had a profit of Rs. 1,200,000 in FY 20X-71/X-72. During the same financial year,
X & Co. has opened up another shop at different location for selling exclusively female
garments. The outlet being a new, it has suffered a loss of Rs. 200,000. Mr. X had registered
another proprietorship X & Sons for furniture business. It incurred a loss of Rs. 300,000.
Also, he has a partnership firm with his spouse that too incurred a loss of Rs. 200,000. For
income Tax purpose, Mr. X prepared the financial statement by clubbing income statements
of all firms showing profit of Rs. 500,000. Is the clubbing done by Mr. X justified? (June
2016, 5 Marks)

Answer:
“Person” means natural person and entity. The sole trading firm (private firm) of a natural person
is also a natural person. The income derived by a sole trading firm is deemed to be derived by the
owner of the firm himself. As such, if a natural person has multiple sole trading firms, income
derived from each of such sole trading firm is consolidated and income tax liability is computed
on such consolidated income.
In the given case, Mr. X has two shops of under the sole trading firm X &Co and another, X&
Sons, a furniture business. These two sole trading firms do not have legal status, but are part of
business of Mr. X and income or loss derived by each of these firms is deemed to be derived by
Mr. X himself.
A partnership is an entity as per the definition of Income Tax Act. Owner of the entity and the
entity are separate for the purpose of taxation (Sec. 52).
When Sec. 11 applies, income derived for each of different businesses is deemed to be derived by
a separate person.
As per Sec. 20 (1), while determining assessable income from business, one can set off the business
loss of any other business of the same person sustained during the same Income Year.

Conclusion:
Based on the facts above, profit from selling male garments of Rs. 1.2 Million can be utilized to
set off loss from selling female garment of Rs. 200,000 and from furniture business of Rs. 300,000;
as these are trading business and are not subject to concessions under Sec. 11 and is the business
income or loss of same person.
Since partnership firm is an entity, a separate person from Mr. X, the loss sustained by the
partnership firm cannot be set off against the income of Mr. X.

The Institute of Chartered Accountants of Nepal 89


CAP-II Paper 7 - Income Tax and VAT
2) Discussing the relevant provisions of setoff and carry forward of losses as per Income Tax
Act, 2058, advise regarding setoff and carry forward of losses in following cases for the
income year 2077/78 of a domestic Company: (June 2021, 5 marks)
Domestic business Foreign Foreign
i
loss 25,000 business gain 10,000 investment gain 2,000
Foreign
ii Domestic business business loss in Foreign business
gain 25,000 Country A 25,000 gain in Country B 2,000

Answer:
i) Unrelieved domestic business loss of a person can be set-off with any domestic or foreign
business or investment income.
ii) Unrelieved foreign business loss (of a country) can be set-off with foreign business income
and foreign investment income (of the same country) only.
iii) Unrelieved domestic investment loss can be se- off with any investment income
iv) Unrelieved foreign investment loss (of a country) can be set-off with foreign investment
income (of the same country) only.
Loss from business or investment of a tax-exempt entity shall not be allowed to be carried forward
for Set-off of loss:

Source of Loss Amount of Loss Remarks


Foreign Country (25,000) This loss can’t be set-off during the year as there is no
Business Loss other business gain or investment gain in country A. This
loss can be carried forward for next 7 years.
Domestic (25,000) This loss can be set-off with domestic business gain.
Business Loss Alternatively, this loss can be set-off with foreign source
business or investment gain and unrelieved portion can be
set-off with domestic business gain.

3) Mr. Sonu, a sole shareholder of M/s Sonu Industries Pvt. Ltd., was worried about the
performance of the Company as it incurred losses of Rs. 5 crores during the last 4 financial
years ending F.Y. 20X-73/X-74. Mr. Rakesh, an expert acquired 60% stake in the Company
on Ashad 31, 20X-74. The company earned Rs. 1.5 crore as profit in F.Y. 20X-74/X-75. The
Company has submitted the income tax return by assessing a taxable loss of Rs. 3.5 crore for
F.Y. 20X-74/X-75 under self-assessment by adjusting the carry forward losses of Rs. 5 crores
up to F.Y. 20X-73/X-74 u/s 20 of Income Tax Act, 2058. The Chief Tax Officer issued an
order to pay income tax on Rs. 1.5 crore along with interest thereon. The management of the
Company seeks your advice on the said order of Inland Revenue Office. (June 2019, 5
Marks)

The Institute of Chartered Accountants of Nepal 90


CAP-II Paper 7 - Income Tax and VAT
Answer:
As per Sec. 20 (1), where a person has assessable income from any business or investment during
any Income Year, such income may be used to set off losses sustained by the same person from
any other business during the same income year or any unrelieved portion of such losses sustained
by the same person in the same or any other business in last seven income year(s).

As per Sec. 57 (2) (Kha), where there is change in ownership by 50% of any entity while compared
with the ownership three years previously, the income generated by the entity after the change of
control cannot be used to set off any unrelieved loss sustained during the period before such change
in control.

As per Sec. 57 (1Ka), where the changes in ownership of such shareholders holding 1% or more
shares shall be counted to determine the threshold of 50%, and in case of persons holding less than
1% shares, changes in respect of such holders who are associated to holders holding 1% or more
of ownership shall only be counted.

Conclusion:
In the given case, there is change in 60% ownership of the entity, which means, the income
generated by the entity after the change of control cannot be used to set off any unrelieved loss
sustained during the period before such change in ownership.
As such, the company’s act of setting of losses of the period before such change from the income
derived after such change is not justifiable as per Sec. 57 (2) (Kha). The tax officer’s contention is
correct in this regard.

4) Mr. Shyam, a sole shareholder of M/s Ganpati Industries Pvt. Ltd., was worried about the
performance of the Company as it incurred losses to the tune of Rs. 5 crores during last 4
financial years ending the F.Y. 20X-68/X-69. Mr. Mohan, an expert acquired 60% stake in
the Company on Ashad 31, 20X-69. Miracally, the company has managed to earn Rs. 1.5
crore as profit in F.Y. 20X-69/X-70. The Company has submitted the Income Tax Return by
assessing a taxable loss of Rs. 3.5 crore for F/Y 2069/70 under self-assessment by adjusting
the carry forward losses of Rs. 5 crores up to F/Y 2068/69 u/s 20 of Income Tax Act, 2058.
The Chief Tax Officer issued an order to pay income tax on Rs. 1.5 crore along with interest
thereon. The management of the Company seeks your advice on the said order of Inland
Revenue Office. (June 2014, 4 Marks)

Answer:
As per Sec. 20 (1), where a person has assessable income from any business or investment during
any Income Year, such income may be used to set off losses sustained by the same person from
any other business during the same income year or any unrelieved portion of such losses sustained
by the same person in the same or any other business in last seven income year(s).

The Institute of Chartered Accountants of Nepal 91


CAP-II Paper 7 - Income Tax and VAT
As per Sec. 57 (2) (Kha), where there is change in ownership by 50% of any entity while compared
with the ownership three years previously, the income generated by the entity after the change of
control cannot be used to set off any unrelieved loss sustained during the period before such change
in control.

As per Sec. 57 (1Ka), where the changes in ownership of such shareholders holding 1% or more
shares shall be counted to determine the threshold of 50%, and in case of persons holding less than
1% shares, changes in respect of such holders who are associated to holders holding 1% or more
of ownership shall only be counted.

Conclusion:
In the given case, there is change in 60% ownership of the entity, which means, the income
generated by the entity after the change of control cannot be used to set off any unrelieved loss
sustained during the period before such change in ownership.
As such, the company’s act of setting of losses of the period before such change from the income
derived after such change is not justifiable as per Sec. 57 (2) (Kha). The tax officer’s contention is
correct in this regard.

5) Singha Road Ltd. located at Hetauda is a public infrastructure project. The project will be
completed on Chaitra end 20X-75 as per estimation, and then it will be handover to
Government of Nepal. The company has incurred loss continuously from Income Year 20X-
61/X-62 to 20X-68/X-69 as follows:
Income Year Loss amount Rs.
20X-61/X-62 1,000,000
20X-62/X-63 500,000
20X-63/X-64 600,000
20X-64/X-65 200,000
20X-65/X-66 150,000
20X-66/X-67 100,000
20X-67/X-68 75,000
20X-68/X-69 10,000
During income year 20X-69/X-70, it has incurred the profit of Rs. 1,500,000. Compute the
taxable income for the income year 20X-69/X-70 and carry forward losses for the income
year 20X-70/X-71 for set off with references to the provisions of Income Tax Act, 2058
relevant to this project. (June 2014, 5 Marks)

Answer:
As per Sec. 20 (1), where a person has assessable income from any business or investment during
any Income Year, such income may be used to set off losses sustained by the same person from
any other business during the same income year or any unrelieved portion of such losses sustained
by the same person in the same or any other business in last seven income year(s).

The Institute of Chartered Accountants of Nepal 92


CAP-II Paper 7 - Income Tax and VAT
As per the proviso of Sec. 20 (1), losses sustained by public infrastructure projects that are
ultimately handed over to Government of Nepal can be carried forward until 12 Years, if built and
operated by the same person.

Testing the case with the provisions of law


The question is not clear whether Singha Road Ltd. is a construction company who works as
contractor or it is undertaking the project under Build and Operate modality. We assume the latter
is the case.

In such scenario, the income of income year 20X-69/X-70 can be used to set off any unrelieved
losses from the same projects since last 12 income years.

Conclusion
Assessable income of IY 20X-69/X-70 before set off of losses from Previous Years 1,500,000
Less: Losses of the project of since IY 20X-57/X-58 (2,635,000)
Losses for the year (1,135,000)
This loss is in relation to Income Year 20X-63/X-64 and onwards. The carried forward loss eligible
for set off from income of IY 20X-70/X-71 is Rs. 1,135,000.

Alternatively:
If we assume that the company is a construction company and is not undertaking the project under
build and operate modality, losses of last seven income years can only be set off from income of
income year 20X-69/X-70, which means losses since IY 20X-62/X-63 can only be set off. Total
losses since IY 20X-62/X-63 is Rs. 1,635,000; and available income is Rs. 1,500,000, this can
only set off all losses of IY 20X-62/X-63 to IY 20X-65/X-66 and Rs. 50,000 of IY 20X-66/X-67.
The carried forward loss eligible for set off from income of IY 20X-70/X-71 is Rs. 1,135,000,
belonging to IY 20X-66/X-67 and onwards.

6) Answer the followings given facts based on the provisions of the Income Tax Act, 2058. The
answer must also consist of implication as per the Act and opinion/ recommendation.
a. The BB Company has made a loss of Rs. one million from the sale of investment made in
shares of another resident company and claims such loss as deductible expenses under
section 13 of the Act as normal business expenses. (June 2007, 1.67 marks, CA Inter)
Answer
Such loss is not deductible expense under Sec. 13, since it is loss on disposal of business asset. As
per Sec. 36, such loss can be set off from gain on disposal of any other business asset or business
liability of M/s BB Company.

The Institute of Chartered Accountants of Nepal 93


CAP-II Paper 7 - Income Tax and VAT
b. Mr. A. Sharma purchased equity shares in Dreamland Hotel Ltd. on Ashad 15, 20X-61.
He sold the share on Ashad 21, 20X-62 at a loss of Rs. 65,000. He wants to set off the loss
against his business income during the year. Examine whether such set off is permissible.
(June 2006, 5 marks, CA Inter)

Answer
Such loss can’t be allowed to set-off with gain derived from business. Only the business loss can
be set off from business income [Sec. 20 (1) jointly read with sec. 20 (2)].

Similarly, investment losses can be set off from investment income, with maximum carry forward
to next seven income years [Sec. 20 (2)]
Equity shares of Dreamland Hotel Ltd. for Mr. Sharma is “non business chargeable asset”. Loss
on disposal of NBCA can be set off from gain on disposal of NBCA [Sec. 36 (2)]
Conclusion
Based on the above facts, Mr. Sharma cannot set off loss on sales of shares with business income
or investment income, but can only set it off from gain on disposal of any other Non-Business
Chargeable asset.

7) Ms. S. Sharma had placed to fixed deposit of Rs. 15 lakhs in her personal capacity in a bank
on which she received interest of Rs. 90,000. She had also borrowed Rs. 5 lakhs from the
same bank on the security of fixed deposit to conduct her business and was liable to pay Rs.
45,000 by way of interest to the bank. She therefore, offered the difference between two
amounts of Rs. 45,000 as income from investment. Is this correct? (June 2006, 5 marks, CA
Inter)

Answer
The interest income on fixed deposit (not on business capacity) in bank (assuming resident) is final
withholding. The tax withheld for final withholding payments cannot be treated as creditable TDS
[Sec. 93 (2) jointly read with Sec. 92 (2)]

The interest expense paid by her is business expense. As per Sec. 14, in case an interest expenses
is incurred on loan used for business purpose, the expense is deductible while calculating
assessable income from business.

Therefore, Ms. Sharma’s intent to set off interest expense from interest income is not correct.

8) Explain the provision and procedure of carry back of losses. (Dec 2004, 5 Marks, CA Inter)
Answer:
As per Sec. 20 (4), if all the following conditions are satisfied, any loss sustained during latter
income year can be set off from the income of previous income years:
a. The loss shall be related to “Long term Contract”,

The Institute of Chartered Accountants of Nepal 94


CAP-II Paper 7 - Income Tax and VAT
b. The long-term contract shall be awarded to the contractor on global bidding or International
Competitive Bidding (ICB),
c. The loss shall be sustained by the contractor at the time of completion of contract, or at such
time when the contract is otherwise disposed,
d. There shall be sufficient taxable income from the same contract in previous income year; i.e.
set off is allowed from previous years’ taxable income of same contract;
There is written permission from Inland Revenue Department to set off such losses.

The Institute of Chartered Accountants of Nepal 95


CAP-II Paper 7 - Income Tax and VAT

Chapter 6: Non-deductible expenses

1) Smart Co. Ltd. had contributed Rs. 350,000 to a non-approved Provident Fund on behalf of
the employees and has claimed it as expenditure. The Tax Office proposes to disallow the
claim and add to the income. The company argues that since it has already spent the money,
it is a business expenditure spent for the operation of normal business and it should be fully
allowed under Section 13. What should be the correct action under the provisions of Income
Tax Act, 2058? (Dec 2009, 5 Marks)

Answer:
The language used in the question is confusing. Therefore, we have two assumptions while
interpreting the question.
Assumption 1: The amount contributed to unapproved retirement fund is additional
contribution by the employer
In this case, the amount is added to the income of the employee while calculating withholding tax
u/s 87 of the Act. The amount so contributed is normal business expense and is deductible u/s 13
of the Act. The company’s argument is valid in this regard.

Assumption 2: The amount is deducted from the salary paid by the employer and is
contributed as per the request of the employee
In this case, the amount does not qualify as expense of the company, and tax officer’s argument is
valid.

2) Mr. Anup pays an advance amounting to Rs. 10 lakhs in cash to Mr. Binod, who required it
urgently to clear his goods from customs. The same was returned by Mr. Binod in cash after
six months. Discuss the tax consequence of the above transaction quoting the relevant
provisions of Income Tax Act, 2058. (Dec 2009, 5 Marks)

Answer:
a. Anup has made cash payment to Binod. It was required for customs clearance. While making
advances to Binod, the outgoings related to asset of Mr. Anup Rs. 10 lakhs as outgoings, though
made in cash, is not not-deductible u/s 21 (2) of the Act, since it is “Cash payment under
unavoidable circumstances”.
b. While discharging the liability by Mr. Binod to Mr. Anup, the conditions mentioned in Sec. 21
(2) is not satisfied. Therefore, if Mr. Binod’s turnover is more than Rs. 20 Lakhs for the year,
the outgoings on settlement of liability is Nil (cash payment above Rs. 50,000 at a time) and
as such, there is gain on disposal of liability.

The Institute of Chartered Accountants of Nepal 96


CAP-II Paper 7 - Income Tax and VAT
3) How will you deal with the followings for the purpose of Income Tax Act, 2058?
(June 2007, 2×1=2, CA Inter)
i. A Company distributes goods worth Rs. 50,000 as free sample.
Answer
The amount is deductible as business expense for distributing company.

ii. Entertainment expenses of Rs. 40,000 incurred during the Previous Year
Answer
The amount is not deductible as it does not meet “incurred during the year” criteria under Sec. 13
of the Act.

4) Advise whether the following expenditure are allowable: (Dec 2005, 1×3=3, CA Inter)
a. A. Co. Ltd. purchased cane from an agriculturist and paid Rs. 1,20,000 in cash
Answer:
Any amount paid in cash to producer of primary agro products even when the produces are
primarily processed is not not-deductible merely because of cash payment. Therefore, subject to
fulfillment of other conditions of Income Tax Act, the amount is deductible.

b. B. Co. Ltd. had a fire accident and in order to douse the fire and recover whatever assets
could be recovered and employed the people from the nearby villages who were called to
help and for the purpose paid cash Rs. 1,50,000 to Mr. A for distribution to the various
villagers who helped the Company by payment not exceeding Rs. 5000 to each.

Answer:
Any amount paid in cash under unavoidable circumstances is not not-deductible merely because
of cash payment. Further, cash payment to a person at a time exceeding Rs. 50,000 is not
deductible. In this case, there is no such case. Therefore, and subject to fulfillment of other
conditions of Income Tax Act, the amount is deductible.

c. C. Company Ltd. incurred an expenditure of Rs.10 lacs on repairs to Machinery whose


Depreciable value was Rs.200 lacs. But Rs.10 Lacs included purchase of a new machinery
costing Rs. 50,000.
Answer
The repair expense is Rs. 950,000 as the cost of new machinery has to be capitalized in Pool D, at
the time which is earlier of purchase date or use date.
As per Sec. 16, repair cost for a pool to the extent 7% of depreciable basis (i.e. 14 lakhs, 7% of Rs.
200 Lakhs) is deductible during the year. Therefore, repair cost is deductible.

The Institute of Chartered Accountants of Nepal 97


CAP-II Paper 7 - Income Tax and VAT
5) How will you treat the in the accounts and for the purpose of income tax? (Dec 2005, 3 Marks,
CA Inter)
a. Advertising and Promotional expenses incurred Rs.15 lacs whose benefit is expected to
be available for 5 years
Answer
It is charged as expense during the year when it is incurred.

b. A Company has received a notice for payment of additional excise duty, and penalty and
interest totaling Rs.5 crores. The Company intends to appeal to the authorities.
Answer
Till date, no payment is made. There is no effect of this payment for tax purpose. When finally
settled, the amount is treated as penalty (if needs to be paid) and hence, not deductible u/s 21 of
the Act.

c. A Company receives Rs.50 lacs from Insurance Company for damages to its stock of raw
material and finished goods.
Answer
The amount is included in income during the year of receipt of compensation u/s 31 of the Act.

6) What are the expenses considered as not allowable as deductible business expenses under
section 21 of the Income Tax Act, 2058? (June 2004, 7 Marks, CA Inter)

Answer
The following expenses are not deductible under Sec. 21 of the Act:
a. Expense of Domestic and Personal Nature
b. Penalty paid against the breaches of law of any country
c. Income Tax paid under Income Tax Act, 2058
d. Expenses incurred to the extent to earn exempt amounts
e. Expense incurred to the extent to collect or earn final withholding payments
f. Cash payment exceeding Rs. 50,000 at a time; when certain conditions are satisfied
g. Distribution of Profit
h. Payment to labor or employee, who has not obtained Permanent Account Number
i. Payment of any expenses without invoice bearing Permanent Account Number
j. Expense of Capital Nature
Expense of Domestic and Personal Nature and Expense of Capital nature bear meaning under Sec.
21, which must be written in examination by students considering the marks allocated.
Further, the matters as to what is considered cash payment, condition when cash payment makes
an expenditure not deductible and exceptions when cash payment is allowed without any limit
should also be written for Examination purpose, considering the marks allocated.

The Institute of Chartered Accountants of Nepal 98


CAP-II Paper 7 - Income Tax and VAT
7) What are considered as "Personal Expenses" not allowable as business expenses under the
Income Tax Act and what are the exceptions to the same? (June 2011, 5 Marks; June 2004,
5 Marks, CA Inter)

Answer
For the purpose of Sec. 21, Expense of Personal Nature means the following:
a. Interest incurred on loan utilized for (siphoned out loan) personal purpose
b. Private expenses of a natural person that includes:
i. The cost incurred in maintaining the natural person, including in providing shelter as well
as meals, refreshment, entertainment, or other leisure activities
ii. The cost incurred with respect to the natural person’s commuting, other than commuting
in the course of conducting a business or investment that does not involve commuting
between the individual's home and a place at which the business or investment is conducted
iii. The cost incurred in acquiring clothing for the individual, other than clothing that is not
suitable for wearing outside of work
iv. The cost incurred in educating the individual, other than education that is directly relevant
to a business or investment conducted by the person and that does not lead to a degree or
diploma

Exceptions to the above are as follows:


Where a person makes a payment to an individual, or the costs are incurred in making the payment
to the individual, including costs incurred in favour of a third person, except when any of the
following conditions is satisfied:
a. The payment is included in calculating the income of the individual,
b. The individual makes a return payment of an equal market value to the person as a
consideration for the payment, or
c. The amount of the costs is so small as to make it unreasonable to require or administratively
impracticable for the person to account for them (Rule 6 prescribes the maximum ceiling
for such amounts of Rs. 500 at a time. The amounts include expenses for or reimbursement
of tea expenses, stationery expenses, prizes, gifts, emergency medical facility, or other such
payments as specified by IRD).

The Institute of Chartered Accountants of Nepal 99


CAP-II Paper 7 - Income Tax and VAT

Chapter 7: Tax Accounting, Method and Timing

1) Mr. X is engaged in legal consultancy services. Mr. X has maintained his books of accounts
on cash basis till income year 2077/78. From income year 2078/79, he wants to changeover to
accrual basis of accounting. Following information from records of his books of accounts of
income year 2077/78 is abstracted as follows:
▪ Service provided but fee not received Rs. 5,00,000 which was not included in t h e
income
▪ Advance received from customers but service not rendered Rs. 3,00,000. The amountwas
included in the income in the income year 2077/78.
▪ Rent for the period 2078 Shrawan to 2078 Poush paid in Jestha 2078. Theamount is
included in the expense in the income year 2077/78.
You are required to advise Mr. X. (June 2022, 5 Marks)

Answer:
Particulars Treatment under Prior Treatment Conclusion
Accrual Method
Service provided but Since right to receive Since cash basis was Since it has not been
fee not received Rs. the payment was followed, the included in income, as
5,00,000 which was established during IY amount could not be such, to avoid omission,
not included in the 2077/78, the amount included in income the amount has to be
income had to be included in during I.Y. 2077/78. included in income during
income. I.Y. 2078/79.
Advance received Since right to receive Since cash basis was Since it has already been
from customers but the payment was not followed, the amount included in income, as
service not rendered established during IY was included in such, to avoid taking into
Rs. 3,00,000 2077/78, the amount income during I.Y. account more than once,
has to be presented as 2077/78. the amount shall be
liability. ignored in IY 2078/79
Rent for the period Rent for 2078 Shrawan Since cash basis was Since it has already been
2078 Shrawan to to 2079 Ashad is the followed, the deducted, as such, to avoid
2078 Poush paid in expense of I.Y. amount was taking into account more
Jestha 2078 2078/79 deducted as expense than once, the amount
during I.Y. 2077/78 shall be ignored in income
during I.Y. 2077/78

2) Munal Trade Link is a various goods supplier. The trade link is using accrual basis of
accounting for the income generated from the business. He gave an application to the
department to change his basis of accounting from accrual to cash in Income Year 20X-70/X-
71. In accordance with the generally accepted accounting principles, Inland Revenue

The Institute of Chartered Accountants of Nepal 100


CAP-II Paper 7 - Income Tax and VAT
Department has given the approval to account for his income on cash basis in Income Year
20X-70/X-71. Some accounting information of the trade link at the end of 20X-69/X-70 before
changing the basis of accounting, was as below:
▪ Goods worth of Rs. 75,000 has already been supplied to a retailer in Income Year 20X-
69/X-70 but amount has not been received. Advance Rs. 45,000 has been received from
another retailer.
▪ Goods worth of Rs. 60,000 has been purchased from a vendor in Income Year 20X-
69/X-70 but amount has not been paid. Advance Rs. 35,000 has been paid to another
vendor.
▪ Communications and electricity expenses of Rs. 3,500 were paid on Bhadra, 20X-71.
These expenses were incurred in Jestha and Ashad, 20X-70.

State how you adjust of income and expense in FY 20X-70/X-71 in above conditions? (Dec
2014, 5 marks)

Answer:
Particulars Treatment under Conclusion
Accrual Method
Goods worth of Rs. 75,000 has The sales is To avoid double inclusion, no adjustments
already been supplied to a included in need to be done at the time of change. The
retailer in Income Year 20X- income u/s 7 firm shall not recognize Rs. 75,000 as
69/X-70 but amount has not during I.Y. 20X- income when cash is originally received after
been received 69/X-70 change in control.
Advance Rs. 45,000 has been The amount is The amount shall be included in income at
received from another retailer recorded as the time of change in basis of accounting to
in relation to sales liability. cash.
Goods worth of Rs. 60,000 has The amount is No treatment during I.Y. 20X-70/X-71
been purchased from a vendor recorded as cost
in Income Year 20X-69/X-70 during 20X-69/X-
but amount has not been paid 70, as per Sec. 15
Advance Rs. 45,000 has been The amount is The amount shall be included in income at
received from another retailer recorded as the time of change in basis of accounting to
in relation to purchase liability. cash.
Communications and The amount is No treatment during I.Y. 20X-70/X-71
electricity expenses of Rs. recorded as cost
3,500 were paid on Bhadra, during 20X-69/X-
20X-71. These expenses were 70, as per Sec. 13
incurred in Jestha and Ashad,
20X-70

The Institute of Chartered Accountants of Nepal 101


CAP-II Paper 7 - Income Tax and VAT
3) Standard Chartered Bank has appointed AX Consultancy Pvt. Ltd. as Tax Consultant on
Magh 20, 20X-70 and the effective date of contract is Shrawan 1, 20X-71. Under the contract,
the bank is liable to pay an annual fee of Rs. 500,000. As per the contract, an advance amount
of Rs. 100,000 is to be paid on appointment date (i. e. on Magh 20, 20X-70). AX Consultancy
Pvt. Ltd. is in confusion whether the amount of Rs. 100,000 to be included in its income of
Income Year 20X-70/X-71 as the appointment letter is already received during the Income
Year. Advice. (Dec 2014, 5 marks)

Answer
As per Sec. 22 (3), a company is required to record its transaction under accrual basis for tax
purpose.
As per Sec. 2, a company incorporated under Companies Act is a company for tax purpose.
Therefore, AX Consultancy Pvt. Ltd., which is a company for tax law purpose as well, shall use
accrual basis of accounting.
As per Sec. 24, income under accrual basis has to be recognized as and when the right to receive
the payment is established and value of payment can be measured accurately.
In the given case, right to receive payment will be established only when the service is rendered,
i.e., only after Shrawan 1, 20X-71.
Therefore, a liability is recorded when the advance is received on Magh 20, 20X-70, and the
amount becomes taxable only when the service is rendered.

4) Mr. X is engaged in legal consultancy services. Mr. X has maintained his books of accounts
on cash basis till income year 20X-68/X-69. From income year 20X-69/X-70, he wants to
changeover to accrual basis of accounting. Following information from records of his books
of accounts of income year 20X-68/X-69 is abstracted:
▪ Service provided but fee not received Rs. 5,00,000 which was not included in the
income.
▪ Advance received from customers but service not rendered Rs. 3,00,000. The amount
was included in the income in the income year 20X-68/X-69.
▪ Rent for the period 20X-69 Shrawan to 20X-69 Poush paid in Ashad 20X-69. The
amount is included in the expense in the income year 20X-68/X-69.
You are required to advise Mr. X (Dec 2013, 5 Marks)

Answer:
Particulars Treatment Prior Conclusion
under Accrual Treatment
Method
Service provided but Since right to Since cash basis Since it has not been
fee not received Rs. receive the was followed, included in income, as such,
5,00,000 which was not payment was the amount to avoid omission, the
included in the income established could not be amount has to be included in
during IY 20X- included in

The Institute of Chartered Accountants of Nepal 102


CAP-II Paper 7 - Income Tax and VAT
68/X-69, the income in IY income during IY 20X-
amount had to be 20X-68/X-69 69/X-70
included during
same IY
Advance received from Right to receive Since cash basis Since it has already been
customers but service the payment was was followed, included in income, as such,
not rendered Rs. established when the amount was to avoid taking into account
3,00,000 the service is included in more than once, the amount
provided. income during shall be ignored in income
IY 20X-68/X- during IY 20X-69/X-70
69
Rent for the period Rent for 20X-69 Since cash basis Since it has already been
20X-69 Shrawan to Shrawan to 20X- was followed, deducted, as such, to avoid
20X-69 Poush paid in 69 Ashad is the the amount was taking into account more
Ashad 20X-69 expense of I.Y. deducted as than once, the amount shall
20X-69/X-70 expense during be ignored in income during
I.Y. 20X-68/X- I.Y. 20X-69/X-70
69

5) Mr. Devan Mahara is lawyer. He is using cash basis of accounting for the income generated
from his profession. He gave an application to the department to change his basis of
accounting from cash to accrual in Fiscal Year 20X-67/X-68. In accordance with the
generally accepted accounting principles, Income Tax Department has given the approval to
account for his income on accrual basis in Fiscal Year 20X-67/X-68. The accounts of Mr.
Mahara immediately before changing the basis of accounting, was as below:
▪ Service has already been rendered in FY 20X-66/X-67 but amount has not been
received and hence no income is included in that FY NPR 80,000.
▪ In FY 20X-66/X-67 advance has been taken (service has been rendered on FY 20X-
67/X-68) but amount is included in that FY on cash basis though service is still to be
rendered NPR 30,000.
▪ In FY 20X-66/X-67 House Rent from Magh 20X-66 to Poush 20X-67 has been paid and
expense has been claimed in FY 20X-66/X-67 on cash basis of accounting NPR 48,000.
State how you make adjustments of income and expense in FY 20X-67/X-68 in above
conditions? (Dec 2011, 5 Marks)

Answer
Particulars Treatment under Prior Conclusion
Accrual Method Treatment
Service has already Since right to receive the Since cash basis Since it has not been
been rendered in FY payment was established was followed, included in income, as such,
20X-66/X-67 but during I.Y. 20X-67/X-68, the amount to avoid omission, the
amount has not been the amount had to be could not be amount has to be included in

The Institute of Chartered Accountants of Nepal 103


CAP-II Paper 7 - Income Tax and VAT
received and hence no included during same included in income during I.Y. 20X-
income is included in I.Y. income during 67/X-68
that FY NPR 80,000 I.Y. 20X-66/X-
67
Advance received from Right to receive the Since cash basis Since it has already been
customers but service payment was established was followed, included in income, as such,
not rendered Rs. when the service is the amount was to avoid taking into account
30,000 provided. included in more than once, the amount
income during shall be ignored in income
I.Y. 20X-66/X- during I.Y. 20X-67/X-68
67
Rent for the period Rent for Since cash basis Since it has already been
20X-66 Magh to 20X-a. Poush 20X-66 to Ashad was followed, deducted, as such, to avoid
67 Poush paid during 20X-66 is expense of I.Y. the amount was taking into account more
20X-66/X-67 20X-66/X-67 deducted as than once, the amount shall
b. 20X-67 Shrawan to 20X- expense during be ignored in income during
67 Poush is the expense I.Y. 20X-66/X- I.Y. 20X-67/X-68
of I.Y. 20X-67/X-68 67

6) What do you understand by method of accounting? How far method of accounting is relevant
in computing the income for tax purpose? (June 2009, 6+4=10, CA Inter)
Answer
Tax accounting deals with the question at what time amounts derived and costs incurred are
recognized for taxation purposes, i.e. when these amounts are to be included in the calculation of
the taxable income.
Mainly there are two types of accounting methods-
i) Cash Basis of Accounting and
ii) Accrual Basis of Accounting.

Under the cash system of accounting, revenue and expenses are recorded only when received or
paid irrespective of the fact whether it relates to the previous year or not. Income under cash system
of accounting is, therefore, excess of receipts over disbursements during the previous year.
Under the Accrual (Mercantile) system of accounting, income and expenditure are recorded at the
time of their occurrence during the previous year. The profit calculated under the accrual system
is profit actually earned during the previous year, though not necessarily realized in cash. In other
words, where accounts are kept on accrual basis, the profits or gains are credited though they are
not actually realized and entries, thus, made really show nothing more than an accrual or arising
or the said profits at the material time.
Income Tax Act, 2058 prescribes the following methods of accounting for tax purposes:
1) An individual has to account on a cash basis in calculating his income from employment and
investment, i.e. the cash method is to be used by employees for their employment and
investment income (s 22 (2)).

The Institute of Chartered Accountants of Nepal 104


CAP-II Paper 7 - Income Tax and VAT
2) A company has to account on an accrual basis (s 22 (3)).
3) Any other taxpayer may choose between the cash or the accrual method for calculating his
income from business or investment unless the IRD prescribes otherwise by notice in writing
(s 22 (4)).
4) Taxpayers can apply in writing to the IRD for a change in his basis of accounting. The IRD
may then in exceptional cases allow the change if it is necessary to clearly reflect the taxpayer’s
income (u/s 22).

7) Distinguish between Cash basis of Accounting and Accrual Basis of Accounting. (Dec 2003,
3 Marks, CA Inter)

Answer:
Under the cash system of accounting, revenue and expenses are recorded only when received or
paid irrespective of the fact whether it relates to the previous year or not. Income under cash system
of accounting is, therefore, excess of receipts over disbursements during the previous year.

Under the Accrual (Mercantile) system of accounting, income and expenditure are recorded at the
time of their occurrence during the previous year. The profit calculated under the accrual system
is profit actually earned during the previous year, though not necessarily realized in cash. In other
words, where accounts are kept on accrual basis, the profits or gains are credited though they are
not actually realized and entries, thus, made really show nothing more than an accrual or arising
or the said profits at the material time.

The Institute of Chartered Accountants of Nepal 105


CAP-II Paper 7 - Income Tax and VAT

Chapter 8: Quantification, Allocation and Characterization of Payments

1) Mr. Sitaram Keshari is a consultant appointed for an assignment by a reputed banking


company for Rs. 50,000. For execution of the assignment the bank has provided Mr. Sitaram
Keshari a vehicle for 3 months which he can use for both official as well as personal purposes.
Current market price of the vehicle is Rs. 20,00,000. Compute his income from business with
respect to this transaction. Give your answer by quoting relevant provisions of the Income
Tax Act/Rules. (June 2010, 5 Marks, CA Inter)

Answer
When a person provides vehicle facility to any other person who is not employee of the first person,
1% per annum of market value of such vehicle forms part of income of the person enjoying the
facility pursuant to Sec. 27 of the Act; therefore, the amount to be quantified as vehicle facility is:
Rs. 20,00,000 * 1% * 3/12= Rs. 5,000
The amount to be included with respect to the transaction is Rs. 5,000.

2) City Computers Pvt. Ltd. is dealer of Acer Laptops. It imports the laptops from abroad and
sells it to the local computer shops all over Nepal. The company had imported laptops worth
USD 1,00,000 which reached in Birgunj Custom Office on Magh 25, 2078, however the
company had received invoice issued by foreign company on Magh 1, 2078. Due to
disturbance in transportation, the company had to pay 50,000 as demurrage charge and
custom duty 12,50,000. The company made payment to the foreign company on Chaitra 20,
2078. Following are the exchange rates:
Magh 1, 2078: 1USD = Rs. 120
Magh 25, 2078: 1USD = Rs. 123
Chaitra 20, 2078: 1USD = Rs. 118
Calculate the cost of purchase to be considered under section 15 and gain/loss in foreign
exchange transaction citing relevant provisions of Income Tax Act, 2058. (June 2022, 5
Marks)

Answer:
Calculation of Inclusion and Deduction:
Particulars USD Exchange Rate Amount (Rs.)
Purchase Price 100,000 120 12,000,000
Custom Duty 1,250,000
Demurrage Charge 50,000
Total Purchase Cost 13,300,000
Exchange rate prevailing at the date of receiving invoice should be considered while account for
purchase as the company has to maintain its books of account on accrual basis.

The Institute of Chartered Accountants of Nepal 106


CAP-II Paper 7 - Income Tax and VAT
The company had to pay only Rs. 11,800,000 for settlement of its liability on Chaitra 20, 2078
thereby, it will account for foreign exchange gain of Rs. 200,000.

Working Notes:
i) The company should maintain its books of accounting in accrual basis under section 22 of
the Act.
As per section 24 (4) while calculating a person's income earned from a business or investment on
the accrual basis, where the person includes any payment to which the person is entitled or deducts
any payment that the person is obliged to make and a difference in the payment received or made
by the person occurs as a result, inter alia, of a change in exchange rate, the amount of difference
shall be adjusted at the time when payment is received or made.

3) Oppo International Pvt. Ltd., Kathmandu is a dealer of Oppo Electronics. The Oppo
Electronics are manufactured in China by Oppo International. The company imported the
electronics goods worth of US $ 50,000 on 23 Baishakh, 20X-74, the payment was done on 23
Shrawan, 20X-74. The exchange rate of the transaction was as follows:
Baishakh 23: Rs. 104 per US $
Closing of Ashad: Rs. 107 per US $
Shrawan 23: Rs. 106 per US $
You are required to compute the amount for inclusions or deductions for the tax assessment
of this transaction mentioning the relevant provisions of Income Tax Act, 2058. (Dec 2017, 5
Marks)

Answer:
As per Sec. 28 of Income Tax Act, 2058; the accounting of foreign currency transaction shall be
done on spot rate, but when allowed by IRD, average rate prescribed by IRD may be used to
translate any foreign currency transaction into Nepalese currency.
In the given case, the company would pass the following entry as on Baisakh 23, 20X-74 to record
the purchase transaction for tax purpose:
Purchase A/c Dr. [ $50,000 X Rs. 104/$) 5,200,000
To Payable A/c [Payable in dollar in shrawan 23] 5,200,000

There will be no entries for tax purpose on Ashad end, as the foreign currency is translated using
spot rate.

The following entry will be passed on Shrawan 23, 20X-74 to record the payment to suppliers:
Payable A/c Dr. 5,200,000
Foreign Exchange Loss Dr. [(106-104)*50,000] 100,000
(Difference between actually paid and payable)
To Bank A/c [$50,000 X 106 Rs./$] 5,300,000

The Institute of Chartered Accountants of Nepal 107


CAP-II Paper 7 - Income Tax and VAT
Conclusion:
The resulting gain/loss on settlement of foreign currency asset or liability is claimed as deduction
(can only be set off against any gain on disposal of any other business asset or business liability)
during the year of settlement.

4) Quantify the value of perquisite as per the provision of Income Tax Act, 2058 provided by
RST Ltd. to employees and others as under:
i) RST Ltd. provides luxury car with market value of Rs. 1.26 crore to Mrs. Rosy Poudel,
manager of production for both personal and official use. During FY 2077/78, RST Ltd.
incurred expenditure of Rs. 300,000 being cost of driver, fuel, repairs etc. in relation to
car provided to Mrs. Poudel. RST Ltd. pays Basic salary of Rs. 50,000/month, Grade of
Rs. 3,000/month and other allowances of Rs. 30,000/month to Mrs. Poudel.
ii) Mrs. Rupa Gurung, manager- sales of RST Ltd. owns personal car. The company
provides fuel as per employee benefit policy. During FY 2077/78, RST Ltd. Provided 165
Ltr. fuel for car owned by Mrs. Gurung and its cost was Rs. 190,000.
iii) RST Ltd. provides luxury car with market value of Rs. 1 crore to Mrs. Rojina Rungta,
marketing consultant for both personal and official use. During FY 2077/78, RST Ltd.
incurred expenditure of Rs. 300,000.00 being cost of driver, fuel, repairs etc in relation
to car provided to Mrs. Rungta. RST Ltd. pays consulting fees of Rs. 8 Lakh to Mrs.
Rungta during FY 2078/79.
iv) RST Ltd. sold luxury car, depreciable assets owned by the company to its employee Mr.
Deepak Pradhan, manager of finance at price of Rs. 50 Lakh. The depreciated value of
the car is Rs. 70 Lakh whereas its market value is Rs. 60 Lakh.
v) Mrs. Rama Rungta, manager- administration of RST Ltd. owns personal car. The
company provides driver for vehicle owned by Mrs. Rungta. Cost to company on account
of driver provided to Mrs. Rungta is Rs. 9 Lakh which includes basic salary of Rs.
50,000/month and remaining Rs. 300,000 being other allowances. (Dec 2021, 5 Marks)

Answer:
Quantification of perquisite/benefits as per the provision of Income Tax Act, 2058 are as
follows:
i) Rs. 3,180 being 0.5% of basic salary and grade of Mrs. Poudel [((50000+3000)*12)*0.5%)]
ii) Rs. 190,000 being cost of fuel provided.
iii) Rs. 100,000 being 1% p.a. of market value of car
iv) Rs. 10 Lakh being market value of car less consideration paid
v) Rs. 9 Lakh being cost incurred by company

The Institute of Chartered Accountants of Nepal 108


CAP-II Paper 7 - Income Tax and VAT
5) Quantify the perquisites provided by a company in the following cases:
i) Vehicle facility is provided to Mr. A, an employee having annual salary of Rs. 12,00,000
for official purpose. Cost of the vehicle was Rs. 40 lacs.
ii) Vehicle facility is provided to Mr. B, an employee having annual salary of Rs. 18,00,000.
Additionally, a driver is hired by the company exclusively for driving Mr. B's car for
annual salary of Rs. 360,000. Cost of the vehicle was Rs. 40 lacs.
iii) Vehicle facility is provided to Mrs. C, a consultant, hired for consultancy charge of Rs.
5,00,000 (lump sum) for three months. Cost of the vehicle was Rs. 40 lacs
iv) Housing facility is provided to Mr. D, an employee having annual salary of Rs. 12,00,000.
The company paid rent of Rs. 1,00,000 per month for the building.
v) Housing facility is provided to Mr. E, an consultant hired for lump sum of Rs. 5,00,000
for three months. The building was in the name of the company and rent of similar
building is Rs. 100,000, per month. (Dec 2020 5 Marks)

Answer:
i) As per section 27(1)(b)(i) and Rule 13(1)(a), vehicle facility provided to an employee is
quantified at rate of 0.5% of the remuneration paid to the employee if the vehicle is used for
fully or partly personal purpose. In the given case vehicle is used for the Official Purpose and
hence it will not be taxable.
Alternatively:
If students assume that in absence of lack of supporting evidence of official purpose they can
consider the same for valuation. If so,
So value of perquisite is 6,000

ii) As per 27(1)(b)(i)and Rule 13(1)(a), vehicle facility provided to an employee is quantified at
rate of 0.5% of the remuneration paid to the employee. However, if vehicle facility is
quantified, then fuel and driver facility need not be clubbed in the annual income of the
employee.
So value of perquisite is 9,000.

iii) As per 27(1)(b)(i)and Rule 13(1)(b), vehicle facility provided to person other than an employee
is quantified at rate of 1% per annum of the market value of the vehicle. In the given case, the
amount of market value of vehicle is not given. Therefore, there cannot be quantification at all
with the available information.

iv) As per 27(1)(b)(ii)and Rule 13(2)(a), housing facility provided to an employee is quantified at
rate of 2% of the remuneration paid to the employee
So value of perquisite is 24,000

v) As per 27(1)(b)(ii)and Rule 13(2)(c), housing facility provided to a person other than an
employee is quantified at rate of 25% of the market (deemed) value of rent for similar facility.
So value of perquisite is 75,000

The Institute of Chartered Accountants of Nepal 109


CAP-II Paper 7 - Income Tax and VAT

6) Quantify the value of consideration as per Section 27 of Income Tax Act, 2058 under the
following circumstances: (Dec 2018, 5×1=5)
a. Mr. Ram gave his smart TV for settlement of payables to a shopkeeper. Mr. Ram had
purchased the TV for Rs. 45,000 three years back, and the market value at the time of
handing over to the shop was Rs. 25,000.
Answer
As per Sec. 27 (1) (Ka), where consideration is payable in kind, the market value of such kind is
taken as the value of consideration.
As such, the value of consideration paid by Mr. Ram is Rs. 25,000.

b. XYZ limited provides free motor vehicle costing Rs. 15,000,000 to Mr. Ganpath, a
government official.
Answer
As per Sec. 27 (1) (Kha) (1) jointly read with Rule 13 (1) (Kha), the amount be quantified is 1%
p.a. of market value of asset.
In this case, market value of motor vehicle is not given, therefore, quantification cannot be done.
Alternatively:
Students may assume that the Company gives motor vehicle to Mr. Ganpath at free of cost (transfer
of ownership). So, value of vehicle i.e. Rs. 15,000,000 is considered as payment.

c. PQR Limited provides two security personnel free of cost to Mr. Mohan, a marketing
manager, for which the company pays Rs. 240,000 for a year.
Answer
As per Sec. 27 (1) Nga), the amount to be quantified is the value of benefit received by the person.
As such, Rs. 240,000 should form part of income of Mr. Mohan.

d. K&K Ltd. provides free accommodation to a public director of the Company, for which
the Company pays annual rent Rs. 500,000.
Answer
As per Sec. 27 (1) (Kha) (1) jointly read with Rule 13 (2) (Kha), the amount be quantified is 25%
p.a. of rent paid.
Payment = 500,000*25%= Rs. 125,000

e. K&K Ltd. provides its own bungalow costing Rs. 20,000,000 as free accommodation to a
local leader, normal rent for the bungalow is Rs. 100,000 per month.
Answer
As per Sec. 27 (1) (Kha) (2) read together with Rule 13 (2) (Ga), the amount to be quantified in
such case is 25% of actual rent paid. As such, the local leader is deemed to receive income of Rs.
25,000 per month against such facility.

The Institute of Chartered Accountants of Nepal 110


CAP-II Paper 7 - Income Tax and VAT
7) Quantify the value of perquisite as per the provision of Income Tax Act, 2058 provided by
PQR Ltd. to the employee and others as under: (Dec 2017, 5 Marks)
a. The company pays Rs. 35,000/month for a furnished accommodation facility to the
Executive Officer (Mr. Lama). The value of the furniture and equipment is Rs. 1,50,000.
Mr. Lama’s basic salary and grade was Rs. 1,00,000 p.m. during the financial year 20X-
73/X-74.
Answer
As per Sec. 27 (1) (Kha) (2) read together with Rule 13 (2) (Ka), the amount to be quantified in
such case 2% of Basic Salary and Grade of the employee.
As such the amount to be quantified as income of Mr. Lama is 2% of Rs. 1,200,000 (assuming he
drew salary throughout the year), i.e. Rs. 24,000.

b. The company pays Rs. 15,000/month for a flat provided to Program Officer, Mr.
Tamang. Mr. Tamang’s basic salary and grade was Rs. 50,000 per month during the
financial year 20X-73/X-74.
Answer
As per Sec. 27 (1) (Kha) (2) read together with Rule 13 (2) (Ka), the amount to be quantified in
such case 2% of Basic Salary and Grade of the employee.
As such the amount to be quantified as income of Mr. Tamang is 2% of Rs. 600,000 (assuming he
drew salary throughout the year), i.e. Rs. 12,000.

c. The company pays Rs. 1,00,000/month for a flat provided to Mr. Thapa, a board director.
The value of the furniture and equipment provided by the company in the rented flat was
Rs. 5,00,000.
Answer
As per Sec. 27 (1) (Kha) (2) read together with Rule 13 (2) (Ga), the amount to be quantified in
such case 25% of rent paid or payable.
As such the amount to be quantified as payment to Mr. Thapa is 25% of Rs. 1,200,000 i.e., Rs.
25,000.

d. Free four-wheeler facility is provided to the Executive Officer (Mr. Lama). The company
meets the cost of fuel and maintenance charge of the vehicle provided to Mr. Lama. The
cost of fuel and maintenance charges paid by the company was Rs. 2,50,000 and the value
of four-wheeler was Rs. 75,00,000. Mr. Lama’s basic salary and grade was Rs. 100,000
per month during the financial year 20X-73/X-74.
Answer
As per Sec. 27 (1) (Kha) (1) read together with Rule 13 (1) (Ka), the amount to be quantified in
such case 0.5% of Salary of the employee.
As such the amount to be quantified as income of Mr. Lama is 0.5% of Rs. 1,200,000 (assuming
he drew salary throughout the year), i.e. Rs. 6,000

The Institute of Chartered Accountants of Nepal 111


CAP-II Paper 7 - Income Tax and VAT
e. The company meets the cost of fuel and maintenance charge of vehicle provided to Mr.
Thapa, the board director. The cost of fuel and maintenance charges paid by the
company was Rs. 3,00,000 and the value of four-wheeler was Rs. 1,00,00,000.
Answer
As per Sec. 27 (1) (Kha) (2) read together with Rule 13 (1) (Kha), the amount to be quantified in
such case 1% of market value of Vehicle.
As such the amount to be quantified as payment to Mr. Thapa is Rs. 100,000.

8) Mr. A has been working in the capacity of senior manager for XYZ Co. Ltd. The company
provided him a Volkswagon car for official as well as personal use. The current market price
of the car is Rs. 5,700,000. Details of Mr. A’s remuneration during the Income year 20X-
66/X-67 are as follows:
Salary Rs. 1,50,000 per month
Grade Rs. 75,000 per annum
Allowance Rs. 7,000 per month
Other Allowances Rs. 9,000 per month
Quantify the amount for the vehicle facility provided for computing Mr. A’s taxable income
from remuneration for the F.Y. 20X-66/X-67.
Will the answer be different in case XYZ Co. Ltd. hired Mr. A for four months in the capacity
of consultant for a fee of Rs. 700,000 in total? (Dec 2010, 5 Marks)

Answer
As per Sec. 27 (1) (Kha) (1) read together with Rule 13 (1) (Ka), the amount to be quantified in
such case is 0.5% of salary of the employee.
As such the amount to be quantified as income of Mr. Lama is 0.5% of salary i.e. Rs. 13,500 (0.5%
of (150,000+75,000)*12).
If he were a consultant:
As per Sec. 27 (1) (Kha) (1) read together with Rule 13 (1) (Kha), the amount to be quantified in
such case is 1% p.a. of market value of motor vehicle.
Therefore, amount to be treated as part of his service fee is:
Market Value 5,700,000
1% of above 57,000
To be quantified (for four months) [57,000÷12 X4] 19,000

9) Mr. A and Mr. B contributed Rs. 4 million and Rs. 6 million respectively and deposited into
a common bank account to be used to purchase a plot of land and to resale the same in the
future. After 6 months of depositing the money into the bank account, they purchased a plot
of land for Rs. 9 million and got the land registered in their joint name. Expenses on
registration and commission at the time of purchasing the land are amounted to Rs. 1 million.
Later, they sold the plot of land for Rs. 17.2 million. Also, they paid Rs. 1.2 million as sales
commission and other incidental expenses pertinent to the disposal. The bank has given
interest @ 12% per annum on such deposit.

The Institute of Chartered Accountants of Nepal 112


CAP-II Paper 7 - Income Tax and VAT
Compute the amount to be included as income in the hand of Mr. A and Mr. B. (June 2022,
5 Marks)
Answer:
Computation of amount to be included in the hands of Mr. A and Mr. B
Particulars Amount (Rs. In Million)
Amount derived from disposal of Land 17.2
Less: Outgoings
Purchase price of Land 9
Registration and Commission Expenses 1
Commission on Sales 1.2
Net Gain on Disposal of Land 6
Income of Mr. A U/s 30 2.4
Income of Mr. B U/s 30 3.6
Alternatively, if students include interest income @ 12% mentioning them as final
withholding, full marks is to be allotted.

10) Saina and Safina decided to jointly invest Rs. 2,000,000 and Rs. 3,000,000 respectively. Out
of the collected amount, they bought land for Rs. 4,500,000 with additional registration and
other expenses Rs. 500,000. They sold the land for Rs. 8,600,000, out of which, they spent Rs.
600,000 for commission and other expenses. Calculate their taxable income as per Income
Tax Act, 2058? (Dec 2018, 5 Marks)
Answer
As per Sec. 30 of Income Tax Act, 2058; where two or more natural persons jointly invest in
owning an investment asset, the incomings and outgoings in relation to such an asset shall be
apportioned between the co-investors in their investment ratio.

Investment ratio in the given case is 2:3.


Allocation of Incomings and Outgoings and Gain accrued to each individual
Particulars Total Saina Safina
Incomings from Sales of Land 8,600,000 3,440,000 5,160,000
Outgoings of Land
Purchase cost 4,500,000
Add: Registration and Other Expenses 500,000
Add: Commission and Other Exp. On Sales 600,000
5,600,000 2,240,000 3,360,000
Gain on Disposal of Land (to be included 3,000,000 1,200,000 1,800,000
as income in each other’s hand)

11) Mr. A and Mr. B contributed Rs. 4 million and Rs. 6 million respectively and deposited into
a common bank account to be used to purchase a plot of land and to resale the same in the

The Institute of Chartered Accountants of Nepal 113


CAP-II Paper 7 - Income Tax and VAT
future. After 6 months of depositing the money into the bank account, they purchased a plot
of land for Rs. 9 million and got the land registered in their joint name. Expenses on
registration and commission at the time of purchasing the land are amounted to Rs. 1 million.
Later, they sold the plot of land for Rs. 17.2 million. Also, they paid Rs. 1.2 million as sales
commission and other incidental expenses pertinent to the disposal. The bank has given
interest @ 12% per annum on such deposit. (June 2018, 5 Marks)

Answer
As per Sec. 30 of Income Tax Act, 2058; where two or more natural persons jointly invest in
owning an investment asset, the incomings and outgoings in relation to such an asset shall be
apportioned between the co-investors in their investment ratio.

Investment ratio in the given case is 2:3.


Allocation of Incomings and Outgoings and Gain accrued to each individual
Particulars Total Mr. A Mr. B
Incomings from Sales of Land 17,200,000 6,880,000 10,320,000
Outgoings of Land
Purchase cost 9,000,000
Add: Registration and Other Expenses 1,000,000
Add: Commission and Other Exp. on 1,200,000
Sales 11,200,000 4,480,000 6,720,000
Gain on Disposal of Land (to be included 6,000,000 2,400,000 3,600,000
as income in each other’s hand)

Treatment of Interest income from Bank


Applying the standard set forth by Sec. 30 of Income Tax Act, 2058; the interest income from
saving account of bank is treated as received by Mr. A and Mr. B from jointly held property in
their investment ratio. As such, the interest income is final withholding in the hand of Mr. A and
Mr. B.

12) Ram and Krishna had contributed Rs. 20 lakhs & Rs 30 lakhs respectively and deposited the
amount into a common bank account to be used to purchase a plot of land to resale the same
later on. After 6 months of depositing the money into the bank account, they purchased a
plot of land for Rs. 45 lakhs and got the land registered in their joint name (i.e. in the name
of both Ram & Krishna jointly). Expenses on registration and commission at the time of
purchase of land amounted to Rs. 500,000. They later on sold that plot of land at Rs. 86 lakhs
and they have paid Rs. 6 lakhs as sales commission and other incidental expenses related
with sales. The bank has given interest @18% per annum on such bank deposit.
Calculate the amount to be incorporated as “income” in the hand of Ram and Krishna
respectively from above mentioned transaction. How much would be the interest income and
how it is treated in the hand of Ram & Krishna? (June 2012, 5 Marks)

The Institute of Chartered Accountants of Nepal 114


CAP-II Paper 7 - Income Tax and VAT
Answer
As per Sec. 30 of Income Tax Act, 2058; where two or more natural persons jointly invest in
owning an investment asset, the incomings and outgoings in relation to such an asset shall be
apportioned between the co-investors in their investment ratio.

Investment ratio in the given case is 2:3.


Allocation of Incomings and Outgoings and Gain accrued to each individual
Particulars Total Ram Krishna
Incomings from Sales of Land 8,600,000 3,440,000 5,160,000
Outgoings of Land
Purchase cost 4,500,000
Add: Registration and Other Expenses 500,000
Add: Commission and Other Exp. on 600,000
Sales 5,600,000 2,240,000 3,360,000
Gain on Disposal of Land (to be included 3,000,000 1,200,000 1,800,000
as income in each other’s hand)

Treatment of Interest income


Applying the standard set forth by Sec. 30 of Income Tax Act, 2058; the interest income from
saving account of bank is treated as received by Mr. Ram and Mr. Krishna from jointly held
property in their investment ratio. As such, the interest income is final withholding in the hand of
Ram and Krishna.

13) Mr. Ram & Mr. Shyam jointly owned a house property costing Rs. 50,000,000. At the time
of acquisition of the said house property, Mr. Ram had invested Rs. 1 crore and rest of the
amount invested by Mr. Shyam. This house property has been sold out for Rs. 64,000,000,
and incurred Rs. 700,000 on account of brokerage commission plus other incidental charges.
You are required to appropriate between Mr. Ram & Mr. Shyam- as per sec 30 of Income
Tax Act 2058- for the net income that originated from the jointly owned investment. (Dec
2010, 5 Marks)

Answer:
As per Sec. 30 of Income Tax Act, 2058; where two or more natural persons jointly invest in
owning an investment asset, the incomings and outgoings in relation to such an asset shall be
apportioned between the co-investors in their investment ratio.

Investment ratio in the given case is 1:4 for Ram and Shyam.
Allocation of Incomings and Outgoings and Gain accrued to each individual.
Particulars Total Ram Shyam
Incomings from Sales of house property 64,000,000 12,800,000 51,200,000
Outgoings of House Property

The Institute of Chartered Accountants of Nepal 115


CAP-II Paper 7 - Income Tax and VAT
Purchase cost 50,000,000
Add: Commission and Other Exp. on 700,000
Sales 50,700,000 10,140,000 40,560,000
Gain on Disposal of Land (to be included 13,300,000 2,660,000 10,640,000
as income in each other’s hand)

14) AB Pvt. Ltd. has made an agreement with ABC Pvt. Ltd. to arrange the cash flow for its
transactions. Chairman of AB Pvt. Ltd. is also a representative board member of ABC Pvt.
Ltd. In Shrawan 1, 20X-72, loan amounting Rs. 50 million had been provided by the ABC to
the AB. Interest rate mutually agreed was 10 percent. Both the companies are located at
Kathmandu. Tax Officer denied the interest expenses of Rs. 5 million claimed by AB Pvt. Ltd. on the
ground that this transaction is a related party's transaction. Answer whether the Tax Officer's
assessment is correct. (June 2018, 5 Marks)
Answer
As per Sec. 33 of Income Tax Act, 2058; in any arrangement between persons who are associates,
the Department may, by notice in writing, distribute, apportion, or allocate amounts to be included
or deducted in calculating income and foreign income tax paid between the persons as is necessary
to reflect the taxable income or tax payable that would have arisen for them if the arrangement had
been conducted at arm’s length.

This means, where the transactions are conducted in arm’s length, IRD cannot use the power under
Sec. 33 to allocate, apportion or distribute any amounts to be included or deducted; even if the
transaction is between related parties or associated persons.

Deduction of interest expense is dealt by Sec. 14 of Income Tax Act, 2058; and so far the loan on
which interest expense is incurred satisfies conditions under Sec. 14 (1); the total interest expense
is deductible subject to Sec. 14 (2).

Sec. 14 (2) limits the amount of interest expenses to be deductible where a resident entity
controlled by exempt organization pays interest to its holder of shares and that are treated as
exempt entities under the clarification clause of Sec. 14. Provisions of Sec. 14 (2) are in deferral
in nature, not in restriction of allowances of interest expenses.

As such, if conditions of Sec. 14 (1) are satisfied and subject to provisions of Sec. 14 (2), IRD
cannot deny the deduction of interest expenses in the name of “related party transaction”.

15) What do you mean by Income Splitting? Mention the right available to Inland Revenue
Department to prevent any reduction in tax payable as a result of the splitting of income.
(June 2010, 5 marks)

The Institute of Chartered Accountants of Nepal 116


CAP-II Paper 7 - Income Tax and VAT
Answer
Meaning of Income Splitting:
As per Sec. 34 (2); a person having attempted to split income include a reference to a transfer of
the following amounts, either directly or indirectly through one or more interposed entities,
between the person and the associate so as to lower the total tax payable by the person or an
associate:
a. Amounts to be received or costs to be incurred, or
b. An amount received or enjoyed by the transferee of an asset that is derived from the asset; or
an amount paid or expenses incurred in owning the asset.
In determining whether a person is seeking to split income, the Department shall consider the
market value of any payment made for the transfer.

Power of Inland Revenue Department


Where a person attempts to split income with another person, and in case that will cause a reduction
in the amount of tax payable, the Department may, by notice in writing, adjust amounts to be
included or deducted in calculating the income of each person to prevent any such reduction in the
amount of tax payable.

16) What is the Tax Avoidance Scheme? Mention the power available to Inland Revenue
Department under General Anti Avoidance Rule for determining tax liabilities. (Dec 2009, 5
Marks)

Answer
Meaning of Tax Avoidance Scheme
For the purpose Section 35, “tax avoidance scheme” means any arrangement, one of the main
purposes of which is the avoidance or reduction of liability to tax of any person.

Power of Inland Revenue Department


For the purposes of determining tax liability under this Act, the Department may:
a. Re-characterize an arrangement or a part of an arrangement that is entered into or sought to be
entered into as part of a tax avoidance scheme,
b. Disregard an arrangement or a part of an arrangement that does not have substantial economic
effect, or
c. Re-characterize an arrangement or a part of an arrangement that does not reflect its substance

17) State the conditions for finance lease. (June 2006, 3 Marks, CA Inter)
Answer:
Conditions of Finance Lease: If any of the following conditions is satisfied, a lease is treated as
finance lease for tax purpose:
a. Ownership Transfer test: In case the lease agreement includes a clause that the ownership of
the asset is transferred to lessee after the end of the lease term.

The Institute of Chartered Accountants of Nepal 117


CAP-II Paper 7 - Income Tax and VAT
b. Option to purchase asset Test: The lessee has the option to purchase the asset at a
predetermined or fixed price at the expiry of lease term.
c. Term of lease test: The term of lease is more than 75% of the useful life of the asset.
d. Consumption of value test: The expected market price at the end of lease term is less than
20% of the market price prevailing at the inception of the lease.
e. Present value of lease rental test: In case of a lease that is started before the last 25% of the
useful life of the asset, the present value of minimum lease rental is 90% or more of the market
value of the asset prevailing at the inception of the lease term.
f. Specific nature of asset test: The leased assets are of such a specialized nature that only the
lessee can use them without major modifications.

The Institute of Chartered Accountants of Nepal 118


CAP-II Paper 7 - Income Tax and VAT

Chapter 9: Long Term Contract

1) ABC Ltd. has entered into a construction contract with PQR Ltd. with a term of 3 years. The
contract price and the total estimated cost are Rs.60 Billion and Rs. 40 Billion respectively
in Year 1 of the contract, cost amounting to Rs. 10 Billion was incurred and up to year 2
Rs.25 Billion has been expensed. Calculate the income of ABC Ltd. to be included in the
concerned head of income with respect to long term contract in Year 1 and Year 2 by also
showing the calculation of Cumulative Inclusions and Cumulative Deductions. Briefly
explain on Long Term Contract, Excluded Contract and Deferred Return. (Dec 2012,
5+5=10)

Answer:
S.N. Particulars Year I Year II
A Initial Total Revenue for the Contract 60,000,000,000 60,000,000,000
B Variation Order - -
C Escalation Clause - -
D Total Revenue for the Contract(A+B+C) 60,000,000,000 60,000,000,000
E Initial Total Estimated Cost of the Contract 40,000,000,000 40,000,000,000
F Increase in estimated cost due to variation order - -
Increase in estimated cost due to cost escalation, not
G
considered elsewhere (G) - -
Total Estimated Cost to complete the Contract
H (E+F+G) 40,000,000,000 40,000,000,000
Actual Cost incurred to Date (Cumulative Cost till
I this year) 10,000,000,000 25,000,000,000
J Percentage Completion (I/H *100) 25.00% 62.50%
K Cumulative Revenue till this year (D * J) 15,000,000,000 37,500,000,000
L Cumulative Profit/(Loss) till this Year (K - I) 5,000,000,000 12,500,000,000
M Profit/(Loss) up to Previous Year - 5,000,000,000
N Profit/(Loss) for the Year (L - M) 5,000,000,000 7,500,000,000

Long term Contract


Long term contract includes the following contract having term more than twelve months:
a. A contract for manufacture, installation, or construction, or, in relation to each, the
performance of related services, or
b. A contract with a deferred return that is not an “excluded contract”.

Excluded Contract
The following deferred return contracts are excluded from the scope of Sec. 26:
a. Any contract consisting of an interest in any entity,

The Institute of Chartered Accountants of Nepal 119


CAP-II Paper 7 - Income Tax and VAT
b. Any contract consisting of membership in a retirement fund, or
c. Any contract of investment insurance

Deferred Return Contract


A contract is a “Deferred Return Contract”, in case party to the contract fails to show the matters
as prescribed by IRD in relation to the estimated profit and estimated loss in every six months after
the commencement of the contract.

2) Torrent Co. Ltd. is a large publicly listed company whose main activity involves construction
contracts. Torrent won a global contract to build a new football stadium owned by Star
Youth Club at a contracted price of Rs. 40,00,000 thousand. This construction contract was
to be completed over the period of three years starting from financial year 20X-65/X-66.
Initially, the total cost of completing the said stadium was estimated at Rs. 36,00,000
thousand.
Star Youth Club had placed additional variation order in 2nd and 3rd financial years with
the details as given below:
20X-66/X-67 20X-67/X-68
Variation order Rs. 70,000 thousand Rs. 80,000 thousand
Additional Estimated Cost due to Rs. 50,000 thousand Rs. 4,20,000
variation Order thousand

Details of actual cumulative costs over the period of 3 years to build above football stadium
were as follows:
(Rs. In ‘000)
20X-65/X-66 20X-66/X-67 20X-67/X-68
Actual accumulated costs to date 18,00,000 26,28,000 39,86,000
Rectification costs - - 34,000
The rectification costs are the costs incurred in widening access roads to the stadium. This
was the result of an error by Torrent’s architect when he made his initial drawings.

Compute the extracts of taxable income over those three years ending F.Y. 20X-67/X-68 and
amount of losses carried back. (Dec 2011, 15 Marks)

Answer:
Assumption: Contract started from Shrawan 1
Particulars 20X-65/X-66 20X-66/X-67 20X-67/X-68
Initial Total Revenue for the Contract (A) 4,000,000,000 4,000,000,000 4,000,000,000
Variation Order (B) - 70,000,000 150,000,000
Escalation Clause (C ) - - -
Total Revenue for the Contract(D=
A+B+C) 4,000,000,000 4,070,000,000 4,150,000,000

The Institute of Chartered Accountants of Nepal 120


CAP-II Paper 7 - Income Tax and VAT
Initial Total Estimated Cost of the
Contract (E) 3,600,000,000 3,600,000,000 3,600,000,000
Increase in estimated cost due to variation
order (F) - 50,000,000 470,000,000
Increase in estimated cost due to cost
escalation, not considered elsewhere (G) - - -
Total Estimated Cost to complete the
Contract (H= E+F+G) 3,600,000,000 3,650,000,000 4,070,000,000
Actual Cost incurred to Date (Cumulative
Cost till this year) (I) 1,800,000,000 2,628,000,000 4,020,000,000
Percentage Completion (J= I/H *100) 50.00% 72.00% 100.00%
Cumulative Revenue till this year (K= D *
J) 2,000,000,000 2,930,400,000 4,150,000,000
Cumulative Profit/(Loss) till this Year (L=
K - I) 200,000,000 302,400,000 130,000,000
Profit/(Loss) up to Previous Year (M) - 200,000,000 302,400,000
Profit/(Loss) for the Year (N= L - M) 200,000,000 102,400,000 (172,400,000)

As per Sec. 20 (4), the loss incurred in a long-term contract won through global bidding at the
end or disposal of contract can be set off against the income from same contract pertaining to
previous Income Year(s), subject to the approval from IRD. Therefore, Torrent Co. can carry
back its loss of Year III to set it off against income of Year I and Year II as instructed by IRD.
The taxes paid for income of previous Income Year(s) are subject to refund u/s 113.

The Institute of Chartered Accountants of Nepal 121


CAP-II Paper 7 - Income Tax and VAT

Chapter 10: Exempt Amount, Business Concession and Facilities

1) What are the entities considered as tax exempted entities? Whether Sections 87, 88, 88ka and
89 of Income Tax Act with regard to TDS are attracted to such entities? Explain. (June 2012,
5 Marks)
Answer:
Meaning of Tax-Exempt Entities:
“Exempt organization” means the following entities:
a. Following entities registered with the Department as an exempt organization:
i. Social, religious, educational or charitable organizations of a public character established
without profit motive,
ii. An amateur sporting association formed for the purpose of promoting social or sporting
related amenities not involving the acquisition of gain by its member,
b. Political parties registered with the Election Commission.

Provided that, any entity, giving benefit to any person from the assets of, and amounts derived by
the entity except in pursuit of the entity’s function as per its objectives or as payment for assets or
services rendered to the entity by the person, is not exempt from tax.

Exempt Organization as Withholding Agent:


Tax exempt organization shall have obligation to withhold tax pursuant to Sec. 87, 88, 88Ka and
89 of the Act as a Withholding Agent.

Exempt Organization as Withholdee:


Exempt Organization derives two types of income, i.e. exempt amounts (u/s 10 (chha)) and those
amounts that are taxable. Exempts amounts of those organizations are relieved from withholding
tax u/s 88 (4) and 89 (4), as such, while any payer makes payments that are exempt for tax exempt
entities, they must not withhold tax. However, the payer must withhold tax on making payment of
any such amounts that are taxable for tax exempt entities, and when such payments are listed in
Sec. 88, 88Ka or 89 of the Act.

2) Write short notes on Exempt organizations under Income Tax Act, 2058. (Dec 2003/June
2003)
Answer:
“Exempt organization” means the following entities:
a. Following entities registered with the Department as an exempt organization:
i. Social, religious, educational or charitable organizations of a public character established
without profit motive,
ii. An amateur sporting association formed for the purpose of promoting social or sporting
related amenities not involving the acquisition of gain by its member,

The Institute of Chartered Accountants of Nepal 122


CAP-II Paper 7 - Income Tax and VAT
b. Political parties registered with the Election Commission.
Provided that, any entity, giving benefit to any person from the assets of, and amounts derived by
the entity except in pursuit of the entity’s function as per its objectives or as payment for assets or
services rendered to the entity by the person, is not exempt from tax.

3) Mr. ‘Z’ a Canadian citizen is deputed by the Canadian government to work on a Canadian
aided project and he stays in Nepal for 94 days. He receives his salary in Canada and he is
provided a free accommodation and a daily allowance of Rs. 2,000 to meet his boarding and
other expenses. (Dec 2012, 4 Marks)
a. Examine the liability to tax under Income Tax Act, 2058.
b. Will your answer be different if Mr. ‘Z’ comes to Nepal under the services of a private
contractor instead of the Canadian government in the above case?

Answer:
a. As per Sec. 10 (Kha) of Income Tax Act, 2058; any amounts received by an individual as a
result of employment of foreign government is exempt from tax, if both the following
conditions are satisfied:
i. The person is resident of Nepal only because of the employment is non-resident of Nepal,
and
ii. The person receives the amount from the public fund of foreign government.
Conclusion
Since, Mr. Z is deputed by Canadian Government and is receiving income from public fund of
such government and is non-resident in Nepal, his income is exempt from tax in Nepal.
b. If he were hired by a private contractor, his taxable income and tax liability would be as
follows:
Salary Rs. X
Free Accommodation Nil
(Since it is official visit and he is not permanently relocated)
Daily allowance Nil
(Since it is official visit)
Taxable Income would be taxed @ 25%.

Only amount of salary would be tax here in Nepal @25%.

4) Nirdhan Samaj Sewa is a not-for-profit organization registered in Kathmandu. It has


obtained tax exemption certificate from IRD. During the year 20X-67/X-68 the organization
received the following incomes:
Membership Fees Rs. 275,000
Interest Rs. 170,000
Donation Rs. 450,000
Dividend income Rs. 1,200,000
Net profit from running a school Rs. 120,000

The Institute of Chartered Accountants of Nepal 123


CAP-II Paper 7 - Income Tax and VAT

Nirdhan Samaj Sewa has claimed that it is a tax exempted organization and so it is not
obliged to pay tax and has filed a refund application for TDS on interest of Rs. 30,000. Justify
its claim on the basis of the provisions of Income Tax Act, 2058.
Also state the legal provisions regarding filing of income tax return and renewal of tax
exemption certificate by Nirdhan Samaj Sewa. (Dec 2011, 5 Marks)

Answer:
As per Sec. 10 (chha), the following amount of an exempt organization is exempt from tax:
a. Donation or Gift,
b. Any contribution directly related to the objective of the organization, and where the
contribution does not obtain or expect any consideration against such contribution
Conclusion
a. Membership fee: Condition (b) above satisfied, exempt from tax
b. Interest: both the above conditions are not satisfied, as such the interest amount is taxable in
the hand of Nirdhan Sewa Samaj. If it is received from resident bank, financial institutions,
cooperatives, entity issuing bonds or listed entities, it is final withholding.
c. Donation: Condition (a) above satisfied, exempt from tax
d. Dividend income: both the above conditions are not satisfied, as such the interest amount is
taxable in the hand of Nirdhan Sewa Samaj. If it is paid by resident company, it is final
withholding.
e. Net Profit from running a school: both the above conditions are not satisfied, as such the
interest amount is taxable in the hand of Nirdhan Sewa Samaj.

Therefore, Nirdhan Sewa Samaj may not claim refund of TDS on Interest due to following reasons:
a. If it is paid by resident banks, financial institutions, entity issuing bonds, listed entities or
cooperatives, interest income is final withholding.
b. If it is not paid by persons as above, the interest income form part of assessable income from
business, along with inclusion of profit from running school in assessable income from
business and if dividend is received from non-resident entity, including the same amount as
well. Then the organization should compute taxable income and tax liability, and adjust the
TDS on interest as advance tax, and pay the remaining due, if any. In case of excess, it may
claim refund of such excess TDS amount.

Filing of Income Return and Renewal of Exemption Certificate


Nirdhan Sewa Samaj shall file income return within three months of the end of Income Year. If
such time is not adequate, it may extend return filing date by three months with permission of IRD
after filing an application in that regard within due date of filing income return.
Tax exemption certificate must be renewed every year. An application to renew tax exemption
certificate must be filed within one year from the end of Income year, along with following
documents:
a. Proof of filing income return,

The Institute of Chartered Accountants of Nepal 124


CAP-II Paper 7 - Income Tax and VAT
b. Proof of payment of withholding taxes to be withheld by Exempt Organizations, and
c. Proof of filing withholding tax returns.

5) As a tax consultant, you have been enquired of the implication on income tax on Income of
the Securities Board of Nepal. (June 2011, 1 Mark)
Answer:
The income of Securities Board of Nepal is taxable. It must compute assessable income from
business, taxable income and tax liability and pay tax at the rate 25%.

6) Mr. Clinton is working in an embassy representing USA and come to Nepal on 1 st Ashwin,
20X-65. His salary and other emoluments are paid by the country he represents. He declared
the following sources of income during the year 20X-65/X-66.
a. Salary of Rs. 280,000 per month.
b. Dearness allowance Rs. 150,000 per month.
c. Foreign allowance Rs. 80,000 per month.
d. He runs a handicraft business in Nepal and earned a net profit of Rs. 175,000 in the
income year 20X-65/X-66.
e. He was a member of Royal Golf Club of Nepal. He won a prize of Rs. 150,000 from the
game.
However, he claimed the following expenses:
a. Renewal of membership and subscription Rs. 50,000
b. Golf Expenses Rs. 32,000
c. Donation to a school of remote district Rs. 50,000
(School is Tax Exempt organization recognized by IRD)
d. Tax deducted at source in Nepal Rs. 2,000

You are required to find Mr. Clinton’s residential status and taxable income for the income
year 20X-65/X-66. (June 2010, 10 Marks)
Answer:
Residential Status of Mr. Clinton
An individual is resident of Nepal, if any of the following conditions is satisfied:
a. His/her habitual place of abode is in Nepal,
b. S/he stays in Nepal for 183 days or more during any period of consecutive 365 days, or
c. S/he is employee of Government of Nepal, deputed by the employer in any foreign country
during the Income Year.
Since Mr. Clinton has stayed in Nepal for more than 183 days (from Ashoj 20X-65 till Ashad
end 20X-66), he is resident of Nepal, as he satisfies Condition (b) above.

The Institute of Chartered Accountants of Nepal 125


CAP-II Paper 7 - Income Tax and VAT
Calculation of Assessable Income, Taxable Income and Tax Liability of Mr. Clinton
For Income Year 20X-65/X-66
Particulars Sec. Ref Amount Working Notes
Inclusions
Salary Income 10 (Kha) - This is exempt amount as sec 10 states
Dearness Allowance 10 (Ga) - following:
Foreign Allowance 10 (Ga) - Amounts derived by a natural person
from employment in the public service of
government of a foreign country,
provided:
a. The person is resident solely by reason of
performing the employment or is a non-
resident person,
b. The amounts are payable from the public
funds of the country
Assessable Income from Employment -
Income from Business 175,000 Assuming inclusion and deductions are
as per Income Tax Act, 2058
Prize from Playing Golf 88Ka/92 - Windfall Gain
Renewal of Membership & 21 - Expense of Domestic and Personal
Subscription Nature
Golf Expenses 21 - Expense in relation to Final Withholding
Payments
Assessable Income from Business 175,000
Total Assessable Income 175,000
Less: 8,750 Actual: Rs. 50,000;
Reduction u/s 12- Donation or Gift to 5% of Rs. 175,000; or
Exempt Organization Rs. 100,000: which ever is lower
Taxable Income 166,250 Since the taxable income from business
Tax Liability 0 is within the first slab, tax is applicable at
0%.
Tax Deducted at Source In Nepal May be refundable, if not related to prize.

7) What will be the income tax implication of pension received from British Army by Mr. Ram
Gurung, a Nepali citizen? (Dec 2009, 2 Marks, CA Inter)

Answer:
As per Sec. 10 (ja) of Income Tax Act, 2058, pension income derived by a Nepali Citizen from
public fund of foreign government as a result of retirement after service the police or army of such
country is exempt from tax.

The Institute of Chartered Accountants of Nepal 126


CAP-II Paper 7 - Income Tax and VAT
In the given case, Ram Gurung is a Nepali Citizen. He was retired from the service of British Army
(foreign government). He derives pension income. So far the income is received from public fund
of British Government; the pension income of Mr. Ram is exempt.
In case it is received from any other fund than public fund of British government, the amount is
taxable in Nepal.

8) What are the exempt amounts? Describe the provisions of the Income Tax Act in this regard.
(June 2009, 10 Marks; Dec 2004, 5 Marks, CA Inter)

Answer:
The following amounts shall be exempt from tax as per section 10 of Income Tax Act, 2058:
a. Tax exempt amounts derived by a person entitled to tax exemption facilities pursuant to
bilateral or multilateral treaty or agreement between/among Government of Nepal and any
foreign country or international organization or institution.
b. Amounts derived by a natural person from employment in the public service of government
of a foreign country, provided:
(1) The person is resident solely by reason of performing the employment or is a non-resident
person,
(2) The amounts are payable from the public funds of the country
c. Amounts derived by a natural person who is not a Nepali natural citizen and who is referred
to in clause (b) or his/her immediate family member,
d. Amounts derived by natural person who not a Nepali citizen and appointed in the service of
Government of Nepal with the tax exemption term,
e. All types of allowances in the nature of social security payments made by Government of
Nepal, provincial government or local government
f. Amounts derived by way of gift, bequest, inheritance or scholarship, except as required to be
included in income pursuant to Sec. 7, 8, or 9
g. Following amounts derived by exempt organization:
(1) Donation, gift,
(2) Any other contributions directly related to the activities of the exempt organization in the
nature specified in Sec. 2 (dha), where the contributor does not receive or expect any
consideration
h. Any amount of pension income derived by a Nepali Citizen, who is retired after serving the
army or police of any foreign government, from the public fund of such foreign government,
i. Any income of government of Nepal, provincial government or local government
j. Amounts generated by Nepal Rastra Bank as per its objective
k. Any income of generated by Drinking Water and Sanitation Consumer Society established
under Water Resources Act, 2049
l. Income generated as per own objectives by mutual fund approved by SEBON.
m. Income generated by educational institute with memorandum of understanding with GoN or
entity related with GoN, whose objective is not to earn profit or not to distribute profit.

The Institute of Chartered Accountants of Nepal 127


CAP-II Paper 7 - Income Tax and VAT
9) Mr. Ford is working in an embassy representing UK and come to Nepal on 1st Ashwin, 20X-
63. His salary and other emoluments are paid by the country he represents. He declared the
following sources of income during the year 20X-63/X-64. (June 2008, 10 Marks, CA Inter)
a. Salary of Rs. 180,000 per month.
b. Dearness allowance Rs. 15,000 per month.
c. Foreign allowance Rs. 80,000 per month.
d. He runs a handicraft business in Nepal and earned a net profit of Rs. 150,000 in the
income year 20X-63/X-64.
e. He was a member of Royal Golf Club of Nepal. He won a prize of Rs. 50,000 from the
game.

He claimed the following expenses:


a. Renewal of membership and subscription Rs. 10,000
b. Golf Expenses Rs. 12.000
c. Donation to a school of remote district Rs. 50,000
(School is the organization recognized as tax exempted by IRD)
d. Tax Deducted at source in Nepal

You are required to find Mr. Ford’s residential status and taxable income for the income
year 20X-63/X-64.

Answer:
Residential Status of Mr. Ford
An individual is resident of Nepal, if any of the following conditions is satisfied:
a. His/her habitual place of abode is in Nepal,
b. S/he stays in Nepal for 183 days or more during any period of consecutive 365 days, or
c. S/he is employee of Government of Nepal, deputed by the employer in any foreign country
during the Income Year.
Since Mr. Ford has stayed in Nepal for more than 183 days (from Ashoj 20X-63 till Ashad end
20X-64) during IY 20X-63/X-64, he is resident of Nepal, as he satisfies Condition (b) above.

Calculation of Assessable Income, Taxable Income and Tax Liability of Mr. Ford
For Income Year 20X-65/X-66
Particulars Sec. Ref Amount Working Notes
Inclusions
Salary Income 10 (Kha) - This is exempt amount and sec 10 states
Dearness Allowance 10 (Ga) - following:
Foreign Allowance 10 (Ga) - Amounts derived by a natural person from
employment in the public service of
government of a foreign country, provided:

The Institute of Chartered Accountants of Nepal 128


CAP-II Paper 7 - Income Tax and VAT
a. The person is resident solely by reason of
performing the employment or is a non-
resident person,
b.The amounts are payable from the public
funds of the country
Assessable Income from Employment -
Income from Business 150,000 Assuming inclusion and deductions are as per
Income Tax Act, 2058
Prize from Playing Golf 88Ka/92 - Windfall Gain
Renewal of Membership & 21 - Expense of Domestic and Personal Nature
Subscription
Golf Expenses 21 - Expense in relation to Final Withholding
Payments
Assessable Income from Business 150,000
Total Assessable Income 150,000
Less: 7,500 Actual: Rs. 50,000;
Reduction u/s 12- Donation or Gift to 5% of Rs. 150,000; or
Exempt Organization Rs. 100,000: w/e is lower
Taxable Income 142,500 Since the taxable income from business is
Tax Liability 0 within the first slab, tax is applicable at 0%.
Tax Deducted at Source In Nepal May be refundable, if not related to prize.

10) Any kind of Income earned by Nepal Government is exempt from Tax. Is it true? (June 2008,
1 Mark, CA Inter)
Answer:
Yes, as per Sec. 10 (Jha), any amount earned by Nepal Government, Provincial Government or
Local Level is exempt from tax.

11) Kanyam Tea Estate Ltd. is a co-operative society established under the Co-operative Society
Act, 2074. In that society, 40 farmers of Kanyam region are involved in tea gardening and
processing. In the financial year 20X-67/X-68, Kanyam Tea Estate has earned Rs. 1 Crore.
State with reason on the taxability of such income earned. Further, what will be the impact
on income tax if dividends are declared by that society? (June 2011, 2 Marks)
Answer:
Assuming that Kanyam Tea Estate Ltd. is involved in tea farming and processing, as per Sec. 11
(2), income of Kanyam Tea Estate Ltd. is exempt from tax.

However, if it acts as sales agent between the tea farmers and purchases tea leaves from farmers
and processes it and sales it, the income of Rs. 1 crore is taxable for the Cooperative.

12) Mr. Ram Kumar has opened a bank account with Grameen Biaks Bank in Parbat with a
deposit of Rs. 50,000 with interest at the rate of 8 percent per annum. During the year, the

The Institute of Chartered Accountants of Nepal 129


CAP-II Paper 7 - Income Tax and VAT
bank has to pay Mr. Ram Kumar Rs. 4,000 as Bank Interest. The bank seeks for your advice
regarding the applicability of withholding tax.
If for same deposit of Rs. 50,000 with the interest rate of 24%, the bank has to pay Rs. 12,000
for that year, then will your answer differ? (Dec 2010, 5 Marks)

Answer:
Assuming that the Grameen Bikas Bank is based on Rural Municipality in Parbat, the interest
received by Mr. Ram Kumar of Rs. 4,000 is exempt from tax as per Sec. 11 (2Ka). As per Sec. 88
(4), a withholding agent is not required to withhold tax on any amounts exempt from tax. As such,
the bank is not required to withhold tax on payment of interest of Rs. 4,000.

As per Sec. 11 (2Ka), the interest from Grameen Bikas Bank based on Rural Municipality up to
Rs. 25,000 is exempt from tax. Since, the interest income of Mr. Ram Kumar does not exceed the
threshold of Rs. 25,000; no TDS shall be deducted.

13) What are the incomes exempted from tax under the Income Tax Act, 2058? (June 2009, 5
Marks)
Answer:
The following incomes are exempt under Income Tax Act, 2058:
a. Income of non-resident not having source in Nepal (Sec. 6)
b. Incomes that fall under the lists in Sec. 10, which are exempt amounts
c. Agriculture income of an individual subject to provisions of Sec. 11 (1)
d. Income derived by cooperatives subject to provisions of Sec. 11 (2)
e. Dividend income of members of cooperatives subject to provisions of Sec. 11 (2)
f. Interest income of individual subject to provisions of Sec. 11 (2Ka)
g. Income derived by Special Industry subject to provisions of Sec. 11 (3) (Gha) for five-year
time period
h. Income derived by industries established in Special Economic Zones for five years or ten years,
subject to provisions of Sec. 11 (3Ka)
i. Dividend distributed by industries established at Special Economic Zone for five years from
the date of commercial operation
j. Income derived by a person involved in exploration and extraction of minerals, petroleum or
natural gas for five years from commercial operation date, if commences commercial operation
until 2080 Chaitra end
k. Income derived by a person involved in production, transmission or distribution of electricity
for ten years from commercial operation date, if commences commercial operation until 2083
Chaitra end
l. Income derived by a person involved in production, transmission or distribution of electricity
for fifteen years from commercial operation date, if the financial closure is completed within
2085 Chaitra.
m. Income derived by tourism industry or international flight operators for five years subject to
provisions of Sec. 11 (3Ta)

The Institute of Chartered Accountants of Nepal 130


CAP-II Paper 7 - Income Tax and VAT
n. Dividend distributed by Special Industry or Tourism industry by capitalizing profit
o. Income derived by micro-industry for up to 7 years subject to provisions of Sec. 11 (3ta)
p. Income of Approved Retirement Fund
q. Compensation received by a resident natural person against physical injury sustained in
personal accident
r. Compensation against death of a natural person

14) Citizen Saving & Credit Co-operative Ltd, established under Co-operative Society Act, 2074,
running its business at Lalitpur Sub-Metropolitan City earns Taxable Profit Rs. 500,000 &
distributes dividend Rs. 200,000. Calculate dividend tax. (June 2008, 1 Mark, CA Inter)
Answer:
The dividend is taxed at the rate 5% (Joint reading of Sec. 54, 88 (2) and Sec. 92). Therefore, the
amount of dividend tax is Rs. 10,000.

15) Explain the applicability of tax in following entities: (Dec 2006, 4×2=8 Marks, CA Inter)
a. Community-based saving & credit co-operative society
b. Agriculture income
Answer:
a. Community-based Saving and Credit Cooperative Society: Income of Cooperatives based
on rural municipality including saving and credit is exempt from tax. Income of co-operative
(except rural based and dealing in agro and forest based product) is subject to 5% corporate
tax when it is operated in municipality, 7 % when in sub-metropolitan city and 10% when it is
operated in sub-metropolitan or metropolitan city.
b. Agriculture Income: Income from agriculture business (i.e. crop farming or leasing the land
for farming of crops) is exempt from tax, except in the following circumstances:
i. Agriculture business is operated by registering firm, partnership or organized institutions, or
ii. Agriculture business is operated in such land which is held for the purpose of agro-industrial
purpose or industrial purpose u/s 12 (gha) and (Nga) of Land Related Act, 2021.

16) Anupama Herbal Industries Private Limited is a manufacturer of herbal soaps established
in remote area since 2069. After 7 years of operation in Baisakh 2076 Anupama Herbal
Industries Private Limited sold the entire plant to Jumla Cottage Industries which was
established in 2075. Management of the Jumla Cottage Industries is of the view that they will
receive the concession under section 11(7) for 10 years to come with effective from Baisakh
2075. Please advise the management with appropriate provisions of the Income Tax Act.
(December 2020, 5 Marks)

Answer:
In cases when the industries are entitled to time-bound exemptions/concessions under Sec 11, and
the industries are operated after acquiring assets of industries that have already entertained such
facilities; then the period of such time-bound concessions shall be considered after including the
period of use of such assets by previous industries.

The Institute of Chartered Accountants of Nepal 131


CAP-II Paper 7 - Income Tax and VAT
In this case Jumla Cottage Industries is entitled to effective applicable tax rate till IY 2078/79 up-
to 10 years only. Since the concession period shall be counted based on the utilization of assets of
the industry, not on the status of the industry.

17) Himali Garments Pvt. Ltd. deals with export and local sales of readymade garments. Based
on the following information, determine the taxable income and tax liability from local sales
and export business. (June 2019, 5 Marks)
Particulars Amount (Rs.)
Total Sales 20 Million
Local Sales 5 Million
Opening Stock of Garments 3 Million
Cost of Production 17 Million
Closing Stock 4 Million
Export Expenses 0.5 Million
Promotional Expenses in Nepal 0.9 Million
Depreciation 1.6 Million

Answer:
Calculation of Tax Liability
Particulars Sec. Domestic Export Working Note
Ref Sales Sales
Sales income 7 (2) 5,000,000 15,000,000
Less:
Cost of Trading Stock 15 4,000,000 12,000,000 (Opening stock plus cost of production
(3,000,000+17,000,000- minus closing stock) in (sales Ratio)
4,000,000) in 1:3 ratio
Export Expense 13 500,000
Promotional Expense 13 900,000 In Nepal, so domestic expense
Depreciation 19 400,000 1,200,000 In sales ratio, i.e. 1:3
Assessable Income from (300,000) 1,300,000
Business
Less: Horizontal Loss
Set-off u/s 20 (300,000)
Taxable Income 1,000,000
Tax rate 25% 8% Export of goods produced in Nepal by
special industry. (income of a
company, applying Sec. 11 (3Kha) and
11(3nga).
Tax Liability 80,000

The Institute of Chartered Accountants of Nepal 132


CAP-II Paper 7 - Income Tax and VAT
18) Fine Distillery Pvt. Ltd. in Mustang District started its business of producing apple cider
with 11% alcohol by volume since 20X-69/X-70. For the first time during the income year
20X-72/X-73, company made taxable profit of Rs. 1,150,080 prior to the adjustment of
previous years assessed loss of Rs. 6,55,000. Calculate Tax Liability. (June 2017, 2.5 Marks)

Answer:
As per Sec. 11 (3Ja), any industry producing brandy, cider and wine based on fruits in very
undeveloped area are subject to 40% income tax exemption for a period of ten years from the date
of commercial operation. So applicable tax rate is 30*60%= 18%

Mustang is very undeveloped area as per Industrial Enterprises Act, 2076; therefore, income from
production of Apple Cider by Fine Distillery Pvt. Ltd. in Mustang is subject to 40% tax concession
for 10 years from 1st Shrawan 20X-69 (assuming commercial production started that date). As IY
20X-72/X-73 is within such ten year’s period, the exemption is applicable to taxable income of IY
20X-72/X-73.

Computation of CIT Liability:


Assessable income for the year 1,150,080
Less: Adjustment of Previous Year Loss (can be set-off as it’s not tax holiday) (655,000)
Assessable Income 495,080
(Taxable income as there is no reduction u/s 12, 12Ka or 12Kha)
Tax Liability @ 18% 89,114.40

19) Far-western food production Pvt. Ltd is a food manufacturing industry operating since 20X-
66 located at Attariya, Kailali district, it is categorized as a special industry. It has net taxable
income amounting to Rs. 20 lakhs in Income Year 20X-72/X-73. You are required to make
tax assessment for the Income Year 20X-72/X-73. (Dec 2016, 2.5 Marks)

Answer:
Attariya, Kailali is located at under developed area as per Indsutrial Enterprises Act, 2076.

Under Sec. 11 (3) (Kha), special industries located at under developed areas are subject to tax rate
at the rate 30% of applicable tax rate for first ten Years from the date of commencement of
commercial operation. As per Sec. 11 (2Kha) (Ga), in case of a special industry, the rates for
further concessions under Sec. 11 shall be taken as the reduced tax rate obtained after the
application of Sec. 11 (2Kha), as such the effective tax rate for special industry at Kailali is 6% for
first ten years from the date of commencement of commercial operation.

The commercial operation begun from 20X-66, as the question is silent about when the commercial
operation started, we assume that it started on 1st Shrawan 20X-66. The effective tax rate of 6% is
applicable for ten years until 31st Ashad 20X-76.

The Institute of Chartered Accountants of Nepal 133


CAP-II Paper 7 - Income Tax and VAT
For IY 20X-73/X-73, the effective tax rate is 6%.
Taxable Income 2,000,000
Tax rate 6%
Tax Liability 120,000

20) Ramhari Rice Mill, a complete manufacturing industry has the following transaction during
the Income Year 20X-69/X-70. The Mill is a special industry under section 11 of the Income
Tax Act, 2058. The Mill is the sole proprietorship firm and the owner of the Mill has not
selected the Section 50 of the income Tax Act, 2058. You are required to calculate the
applicable tax rate and compute the tax liability based on the following information. (June
2014, 6 Marks)
Particulars Amount (Rs.)
Local sales 11,250,000
Export 3,750,000
Cost of sales 9,000,000
Indirect cost 500,000
Answer:
Calculation of Balance taxable Income (including Assessable Income & Taxable Income)
Particulars Sec. Domestic Export Working Notes
Ref Sales Sales
Sales Income 7 (2) 11,250,000 3,750,000 Given
Cost of Sales 15 6,750,000 2,250,000 Assuming as per Sec. 15 and apportioned in
Sales ratio (i.e., 3:1)
Indirect Cost 375,000 125,000 Assuming used for both, and apportioned in
sales ratio
Taxable Income 4,125,000 1,375,000 No reductions u/s 12, 12Kha & 63 and
deductions under Sec. 1 of Schedule 1

Applicable Tax Rate and Tax Liability


Taxable Rate of Tax Remarks
Slab
Income Tax Liability
st
1 500,000 0% 0
nd
2 200,000 10% 20,000
rd
3 300,000 20% 60,000
th
4 1,000,000 20% 200,000
th
5 2,125,000 25% 510,000

5th 1,375,000 9% 123,750 Export of goods produced in Nepal by special


industry. (income of a resident natural person,
applying Sec. 11 (3Kha) and 11(3nga).
Total 5,500,000 913,750

The Institute of Chartered Accountants of Nepal 134


CAP-II Paper 7 - Income Tax and VAT

21) Worthy Jute Industries Limited, a 100% export-oriented entity, is engaged in the
manufacturing of jute products. There are 1501 Nepali citizens working in the company
throughout the year. The accountant of the company computed the taxable income
amounting to Rs. 12,550,000 for F.Y. 20X-70/X-71 and income tax of Rs. 2,008,000. Being a
tax auditor, whether you agree with the tax amount calculated by the accountant of the
Company? Will your answer be different if there are 1,499 Nepali citizens working
throughout the year instead of 1,501? (June 2014, 5 Marks)

Answer
Worthy Jute Industry is Special Industry and its income is from export. The reduced tax rate for
Special industries is 20%.
As per Sec. 11 (3) (Ka), where a special industry provides direct employment to 1,000 or more
Nepali nationals throughout the income year, the effective tax rate of such special industry shall
be 70% of reduced tax rate, i.e. the effective tax rate for IY 20X-70/X-71 is 14% due to this reason.
Similarly, when Sec. 11 (2kha) (Ga) is jointly read with Sec. 11 (3Nga) (Kha) and (Ga), the
effective tax rate by a manufacturing special industry is 8%.
As per Sec. 11 (5), where multiple exemptions under Sec. 11 are applicable, the exemption as
chosen by the taxpayer is applicable, and a rational taxpayer always chooses lower effective tax
rate.
The corporate tax liability of Worthy Jute Industries Ltd.:
Taxable Income 12,550,000
Tax Rate 8%
Tax Liability 1,004,000

Conclusion:
The tax amount computed by the company’s accountant is not correct.

Even if the employee’s number was 1,499; the answer would not be different, as the effective tax
rate would be 8%.

22) Calculate the applicable tax rate on ABC & Co. based on following information: (June 2012,
5 Marks)
a. ABC & Co. is registered as a special industry as provided in section 11 of Income Tax
Act, 2058.
b. The industry is situated in an under developed area with an employment to 350 people.

Answer
The reduced tax rate for Special Industry operated by an entity, is 20% (Sec. 11 (2Kha) (kha)-
assuming ABC & Co. is an entity.

The Institute of Chartered Accountants of Nepal 135


CAP-II Paper 7 - Income Tax and VAT
A special industry providing direct employment to 300 or more Nepali nationals throughout the
income year is entitled to effective tax rate of 80% of reduced tax rate, i.e. the effective tax rate, if
300 or more employees out of 350 employees are Nepali national, is 16%.
A special industry located at under developed area is entitled to effective tax rate of 20% of reduced
tax rate for first ten years from the date of commencement of commercial operation. This means,
if the special industry is being operated within 10 Years from the date of commencement of
commercial operation, the effective tax rate is 4%.
As per Sec. 11 (5), where multiple exemptions under Sec. 11 are applicable, the exemption as
chosen by the taxpayer is applicable, and a rational taxpayer always choose lower effective tax
rate.

Conclusion
If the special industry is being operated within first 10 Years of commencement of commercial
operation, ABC & Co., being an entity will pay tax @ 4% on taxable income.
If first ten years have been elapsed:
a. It will pay tax @ 16%, assuming 300 or more employees are Nepali National.
b. But if out of 350 employees, less than 300 employees but 100 or more employees are Nepali
nationals, it will pay tax at the rate 18%. A special industry providing direct employment to
100 or more Nepali nationals throughout the income year is entitled to effective tax rate of
90% of reduced tax rate, i.e. the effective tax rate, if 100 or more employees out of 350
employees are Nepali national, is 16%
c. If less than 100 employees are Nepali national, it will pay tax @ 20%.

23) A Company incorporated as a Limited Company and listed in SEBON having involved in
business of tourism sector approached you for getting your advice on tax implication. Advise
the Company giving consideration to the provisions of Income Tax Act, 2058. (Dec 2011, 3
Marks, CA Inter)

Answer:
As per Sec. 11 (3Chha), an entity involved in tourism service, and listed in recognized stock
exchange is entitled to concessions in tax @ 15% of applicable tax rate. It means, as a result of
listing of tourism sector entity, effective tax rate is 85% of applicable tax rate.

The applicable tax rate for tourism industry is 25%.

Conclusion:
The company should pay tax @ 21.25% on its taxable income.

24) Foreign investors have earned Rs. 50,00,000 as foreign technology and management service
fee from the industry established in the Special Economic Zone. (June 2011, 1 Mark)

The Institute of Chartered Accountants of Nepal 136


CAP-II Paper 7 - Income Tax and VAT
Answer
As per Sec. 11 (3Ka) (Gha), the tax applicable on foreign technology and management service fee
from the industry established in the Special Economic Zone (SEZ) earned by foreign investors is
subject to concessions at the rate 50%. So, effective rate of TDS u/s 88(1) is 7.5% (50% of 15%)

If the form of income is passive (without active involvement of human resources in Nepal); the
industry at SEZ shall withhold tax at the rate 7.5%.
Amount earned by Foreign Investor 5,000,000
Effective tax to be withheld @7.5% 375,000

But, if the involvement is active, it is deemed that permanent establishment of foreign investors
exist in Nepal, and the foreign permanent establishment is subject to 50% concessions on
applicable tax on its income derived as foreign technology and management service fee from SEZ.
The tax exemption is given on look-through approach.

25) Income earned by the Entities listed in the securities market and engaged in the business of
production, Tourism sector. (June 2011, 1 Mark)
Answer
As per Sec. 11 (3Chha), an entity involved in tourism service, and listed in recognized stock
exchange is entitled to concessions in tax @ 15% of applicable tax rate. It means, as a result of
listing of tourism sector entity, effective tax rate is 85% of applicable tax rate.

26) As a tax consultant, you are required to advice your client with regards to concessions and
exemptions available in case of following: (June 2011, 2.5 Marks, CA Inter)
a. Person engaged in the business of exploration and excavation of Petroleum and Natural
gas.
b. Person receiving royalty income from the export of Intellectual Property and the income
from the sale/ transfer of Intellectual Property.
Answer:
a. Where a person is involved in extraction and exploration of minerals, petroleum or natural
gases commences its commercial operation until B.S. 2080 Chaitra end, there shall be 100%
tax exemption for seven years from the date of commencement of commercial operation and
50% tax exemption for next three years [Sec. 11 (3Kha)]

b. Where a person exports intellectual property, such export income is subject to tax exemption
@ 25% [Sec. 11 (3Jha)], and where a person disposes intellectual property by way of sales,
such sales income is subject to tax exemption @ 50% [Sec. 11 (3Yna)].

27) A firm having three Partners, Mr. Ram, his wife and his adult son is engaged in construction
and operation Rope Way in Nepal. State the applicable tax rate. (June 2008, 1 Mark, CA
Inter)

The Institute of Chartered Accountants of Nepal 137


CAP-II Paper 7 - Income Tax and VAT
Answer:
As per Sec. 11 (3Cha), in case an entity builds and operates ropeway in Nepal, the entity is entitled
to 40% concession on tax rate for first 10 years from date of commercial operation. As such, the
partnership firm is required to pay tax @ 15% (25%-25% X 40%) on taxable income.
After that the firm needs to pay tax @ 25%.

28) Explain the applicability of tax in Special Industry. (Dec 2006, 4 Marks, CA Inter)
Answer:
Sec. 11 (2Kha)
a. In case of a resident natural person: Marginal tax rate (tax rate of 4th slab) shall be 20% instead
of 30%.
b. In case of an entity: The income from special industry is subject to tax concession at the rate
20% on applicable tax rate.
c. The reduced tax rate must be applied while determining other concessions under other
provisions of Sec. 11.
Sec. 11 (3), 11 (3Nga), 11 (3Chha), 11 (3Dha) (Any one out of the following)
Effective
Basis Facilities
Tax rate
Direct employment to 100 or more Nepali Tax to be payable, which is 90% 18%
Nationals throughout the income year of reduced tax rate
Direct employment to 100 or more Nepali Tax to be payable, which is 81% 16.2%
Nationals throughout the income year, where of reduced tax rate
one-third of work force is women, oppressed
and disabled persons
Direct employment to 300 or more Nepali Tax to be payable, which is 80% 16%
Nationals throughout the income year of reduced tax rate
Direct employment to 300 or more Nepali Tax to be payable, which is 72% 14.4%
Nationals throughout the income year, where of reduced tax rate
one-third of work force is women, oppressed
and disabled persons
Direct employment to 500 or more Nepali Tax to be payable, which is 75% 15%
Nationals throughout the income year of reduced tax rate
Direct employment to 100 or more Nepali Tax to be payable, which is 13.5%
Nationals throughout the income year, where 67.5% of reduced tax rate
one-third of work force is women, oppressed
and disabled persons
Direct employment to 1000 or more Nepali Tax to be payable, which is 70% 14%
Nationals throughout the income year of reduced tax rate
Direct employment to 1000 or more Nepali Tax to be payable, which is 63% 12.6%
Nationals throughout the income year, where of reduced tax rate

The Institute of Chartered Accountants of Nepal 138


CAP-II Paper 7 - Income Tax and VAT
one-third of work force is women, oppressed
and disabled persons
Operated at very undeveloped area, and the rate Tax to be payable, which is 10% 2%
is applicable for first ten years from the date of of reduced tax rate
commencement of commercial operation
Operated at undeveloped area, and the rate is Tax to be payable, which is 20% 4%
applicable for first ten years from the date of of reduced tax rate
commencement of commercial operation
Operated at least developed area, and the rate is Tax to be payable, which is 30% 6%
applicable for first ten years from the date of of reduced tax rate
commencement of commercial operation
New Industry with capital investment of more Tax exemption for first five years 0%
than Rs. 1 Billion and direct employment to from the date of commercial
more than 500 individuals operation
Concessions @50% for next 10%
three years thereafter
Existing industry with capacity enhancement by Tax exemption for first five years 0%
25%, increment of capital investment to Rs. 2 from the date of commercial
Billion and direct employment to more than 300 operation in the income from
individuals enhanced capacity
Concessions @50% for next 10%
three years thereafter on income
from enhanced capacity
Export by Special Industry Concession @ 20% 16%
Export of Goods Produced in Nepal Additional Concession of 50% on 8% for
rates derived after applying entity and
concession @ 20% 7.5% for
Natural
Person
Listing of Manufacturing Special Industry in Concessions @ 15% 17%
recognized stock exchange
Special industry involved in production and Concession @ 50% 10%
processing of domestic tea, clothes

If Multiple provisions as explained in the table applies simultaneously for a special industry, such
rate as per its choice should be applied for tax computation.

29) Calculate the tax liability of XYZ Co. Ltd. (An Industry situated in an under-developed are
with the employment of 650 people) on the basis of following information: (Dec 2003, 4
Marks)

The Institute of Chartered Accountants of Nepal 139


CAP-II Paper 7 - Income Tax and VAT
XYZ Co. is registered as a special industry as provided in Sec. 11 of the Income Tax Act,
2058.
a. The income is Rs. 2,000,000
b. The eligible deduction is Rs. 1,500,000.
c. There is special fee of 3% of profit.

Answer:
Assumption: First ten years from date of commercial operation not elapsed, so the effective tax
rate is 6% (since the effective tax rate derived by applying concessions as a result of employment
to 500 or more Nepali nationals throughout the income year is 15%, which is higher than 6%)
Computation of Taxable Income
Inclusions u/s 7 2,000,000
Deductions (eligible under Income Tax Act) (1,500,000)
Special Fee Expense (assumed to be expense- 500,000* 3%) (15,000)
(Alternatively, the figure can be taken as 500,000÷103 X 3)
Assessable Income from Business 485,000
(= Taxable income, as there is no reduction u/s 12, 12Ka, or 12Kha)
Tax liability @ 6% 29,100

30) Comfort Airways Pvt. Ltd. has been established in FY 20X-68/X-69 with total paid up capital
of Rs. 1.5 billion. Currently, as the domestic airline, it has been operating all flights by 4
Beach Crafts to different tourist destinations. These Beach Crafts are acquired directly from
the manufacturer on Operating Lease.
Of late, the management of the company is very much certain to earn additional profit Rs.
7.5 million at the end of F.Y. 20X-73/X-74. To achieve this target, the company plans to
acquire one more 77-seater ATR aircraft on finance lease for 15 years period to expand its
wings to India. This is possible subsequent to the intake of additional paid-up capital of Rs.
75 million for which all the shareholders are ready to invest.
As a tax consultant, the management wants to know any tax implication to the company by
adding the aircraft. (Dec 2017, 5 Marks)

Answer:
As per Sec. 11 (3Ta), an airline operating international flights, if commences it commercial
operation with capital investment of more than Rs. 1 Billion, the tax on such income is exempt for
five years from the date of commencement of commercial operation and there shall be 50%
exemptions in income tax for three years thereafter.

So, the company shall pay tax at 12.5% (50% of 25%) FY 20X-73/X-74 to 20X-75/X-76.

31) A Client incorporated as a Limited Liability Company wants to establish industries in


proposed Special Economic Zone (SEZ) in different districts approached you for the

The Institute of Chartered Accountants of Nepal 140


CAP-II Paper 7 - Income Tax and VAT
suggestion or guidelines about tax benefits. What do you suggest him about tax benefits
available to him? (June 2012, 3 Marks, CA Inter)

Answer:
As per Sec. 11 (3Ka), Industries established at Special Economic Zone can enjoy following tax
benefits:
a. 100% tax exemption for first ten years from the date of commercial operation and 50% tax
concession thereafter, if the industry is set up at SEZ located at Himali District or Hilly District
prescribed by Government of Nepal,
b. 100% tax exemption for first five years from the date of commercial operation and 50% tax
exemption thereafter, if the industry is set up at SEZ located at other areas as described above
c. 100% tax exemption on distribution of profit for first five years from the date of commercial
operation and 50% tax exemption thereafter for next three years
d. 50% tax exemption on tax applicable on royalty and foreign investment or management fee
derived by foreign investor investing in industries located at SEZ
The above-mentioned concessions/exemptions are not available to any industry that is set up
using the machineries/equipment of another similar or any other industry [Sec. 11 (7)].

The Institute of Chartered Accountants of Nepal 141


CAP-II Paper 7 - Income Tax and VAT

Chapter 11: Adjusted Taxable Income

1) Nepal Soft Ltd. is established as a wholly owned subsidiary of India Soft Ltd. which is 50%
subsidiary of UK Soft Ltd. The following particulars are furnished for your information for
FY 2076/77.
i) Net Profit Before Tax As per books Rs. 2,91,000.
ii) Interest Income from FD Rs.80,000.
iii) Donation Paid to political party registered in election Commission Rs. 25,000.
iv) Interest Paid to UK Soft Ltd. for loan availed of Rs.35 lakh @ 16%. The loan has been
taken on Kartik 01, 2076.
(Dec 2020, 5 marks)
Calculate the Interest allowable for deduction as per provisions of Income Tax Act, 2058.

Answer:
Since interest is paid to UK Soft Ltd., which is also an associated company. (50% owner of India
Soft Ltd., which is 100% owner of Nepal Soft Ltd.), Section 14(2) of Income Tax Act 2058 will
be attracted. According to Section 14(2), interest paid to a tax-exempt controlling entity is
deductible to the extent of the amount of interest income and 50% of adjusted taxable income of
any in come year. Adjusted taxable income shall be computed without including any interest
income and without deducting any interest expenses.
Particulars Amount (Rs.) Note
Assuming that the net profit is
Profit Before tax as per Books 291,000 determined as per tax law after
charging donation
Add: Interest Expenses Paid to
Controlling Entity
(35 lakh @ 16% for 9 months =
420,000) 420,000
Add: Donation to Tax Exempt
Organization u/s 12 25,000
Less: Interest Income (80,000)
Adjusted Taxable Income (ATI) for
section 14 (2) 656,000
50% of ATI 328,000
Interest Income 80,000
Total Allowable Limit (A) 408,000
Actual Interest paid (B) 420,000
Lower of A & B 408,000
Hence, the allowable interest is Rs. 4,08,000. Unclaimed interest expenses Rs. 12000 shall be
carried forward for next fiscal year.

The Institute of Chartered Accountants of Nepal 142


CAP-II Paper 7 - Income Tax and VAT

2) The Profit and Loss Account of XYZ Limited for the year ended Ashad 32, 20X-64 shows a
net profit of Rs. 1,000,000 and other information is given here-under:
a. The expenses include a donation of Rs. 50,000 out of which Rs. 20,000 is provided to the
organization recognized as tax exempted by IRD.
b. During the year the company has incurred Rs. 550,000 on pollution control but the
expenses are not included in the profit.
c. During the year the company has incurred Rs. 300,000 on research and development but
the expenses are not included in the profit.
Calculate the taxable income of XYZ Limited for the income year 20X-63/X-64. (Dec 2007,
5 marks)
Answer:
Adjusted Taxable income means, taxable income derived by ignoring:
a. Reductions u/s 12 (donation), and
b. Deductions u/s 14 (2) [interest expense and interest income], u/s 17 (Pollution control cost) or
u/s 18 (Research and Development cost)
Calculation of Adjusted Taxable Income for the purpose of Sec. 17, 18 and 12:
Particulars For Sec. 17 For Sec. 18 For Sec. 12
Net profit before deduction/reduction u/s 17, 18 1,000,000 1,000,000 1,000,000
& 12
Add: Donation already reduced 50,000 50,000 50,000
Less: Pollution Control Cost (Actual) - 550,000 550,000
Less: Research and Development Cost (Actual) 300,000 - 300,000
Adjusted Taxable Income (ATI) 750,000 500,000 200,000

Eligible Pollution Control Cost u/s 17


Lower of:
a. Actual Cost 550,000
b. 50% of ATI (50% of 700,000) 375,000
So, Rs. 375,000 is allowed as deduction.
Eligible Research & Development Cost u/s 18:
Lower of:
a. Actual Cost 300,000
b. 50% of ATI (50% of 450,000) 250,000
So, Rs. 250,000 is allowed as deduction.

Eligible Donation u/s 12:


Lower of:
a. Actual donation or gift to Exempt Organization 20,000
b. 5% of ATI (5% of 150,000) 10,000
c. Maximum 100,000
So, Rs. 10,000 is allowed as reduction.

The Institute of Chartered Accountants of Nepal 143


CAP-II Paper 7 - Income Tax and VAT

Calculation of Taxable Income:


Taxable Income before deductions/reductions u/s 17, 18 & 12 1,050,000
Less: Deductions u/s 17 375,000
Less: Deduction u/s 18 250,000
Less: Reduction u/s 12 10,000
Taxable Income 415,000

3) Fewa Pvt. Ltd. has been established by Nepalese promoters under foreign direct investment.
The eighty percent of the total capital has been held by non-resident persons. In addition,
The Pvt. Ltd. has borrowed the amount from the foreign investors. For the year ended Ashad
end, 20X-70, the profit and loss account of the company has the following transactions. (June
2014, 5 Marks)
Particulars Amount in Rs.
Sales 5,000,000
Interest income 75,000
Total income 5,075,000
Expenses
Cost of sales 2,000,000
Administrative expenses 1,100,000
Interest expenses 1,500,000
Pollution control expense 300,000
Total Expenses 4,900,000
Net profit 175,000

a. Interest expense Rs. 1,400,000 out of total has been charged on the borrowed amount
from foreign investors. Answer whether all the above expenses can be claimed under
Income Tax Act, 2058?
b. If these expenses cannot be claimed during the year, what would be the implication?

Answer
Calculation of Eligible Interest Expense
Step 1: Calculation of ATI for the purpose of Sec. 14 (2)
Particulars Amount Note
Sales 5,000,000
Cannot be included as per Sec.
Interest Income - 14 (2), ATI must be calculated
without interest component
Income to be considered 5,000,000
Deductions
Cost of Sales 2,000,000 Eligible under Income Tax Act

The Institute of Chartered Accountants of Nepal 144


CAP-II Paper 7 - Income Tax and VAT
Administrative Expense 1,100,000 Eligible under Income Tax Act
cannot be included as per Sec.
Interest Expense - 14 (2), ATI must be calculated
without interest component
Actually incurred during the
Pollution Control Cost 300,000 year, as clarified by Income
Tax Directive
Total expense to consider 3,400,000
Adjusted Taxable Income 1,600,000

Step 2: Calculation of Maximum Limit


Maximum limit is sum of interest income and 50% of ATI calculated above
Interest Income 75,000
50% of ATI Computed above 800,000
Maximum Limit 875,000

Step 3: Calculation of Eligible Interest Expense u/s 14 (2)


Lower of actually paid to controlling person or maximum limit as per Step 2
Actually, paid to controlling person 1,400,000
Maximum Limit 875,000
Eligible u/s 14 (2) 875,000

Step 4: Total eligible interest expense


Eligible interest, where Sec. 14 (2) is not Total interest less paid to
100,000
applicable controlling person
Eligible interest, where Sec. 14 (2) is applicable 875,000
Total Eligible Interest expense 975,000
Ineligible Portion 525,000

Treatment of Ineligible Portion


As per Sec. 14 (3), the amount can be carried forward for claim during next Income Year(s)
and is treated as being paid with regards to next Income Year(s). The carry forward continue
unless there is application of Sec. 57

4) From the following Income Statement of a resident entity controlled by exempt


Organization, compute the taxable income for the financial year 20X-66/X-67. (June 2010, 5
Marks)
Income
Sales Rs. 7,50,000
Interest Income Rs. 5,000

The Institute of Chartered Accountants of Nepal 145


CAP-II Paper 7 - Income Tax and VAT
Total Income Rs. 7,55,000
Expenditure
Cost of sales Rs. 5,00,000
Administrative Expenses Rs. 1,20,000
Interest expenses to controlling (exempt) entity Rs. 75,000
Depreciation expenses Rs. 20,000
Rs. 7,15,000
Net Profit Rs. 40,000

Answer:
Statement of Assessable Income from Business

Particulars Note
Amount
Inclusions
Sales 750,000
Interest Income 5,000
Total Inclusion 755,000
Deduction
Cost of Sales 500,000
Administrative Expenses 120,000
Interest Expense 60,000 Refer W.N. 1
Depreciation 20,000
Total Deductions 700,000
Assessable Income from It is taxable income, as there is no reduction
55,000
Business u/s 12, 12Ka or 12Kha

W.N. 1. Calculation of Eligible Interest Expense


Step 1: Calculation of ATI for the purpose of Sec. 14 (2)
Particulars Amount Note
Sales 750,000
cannot be included as per Sec. 14 (2), ATI
Interest Income - must be calculated without including interest
expense and interest income
Income to be considered 750,000
Deductions
Cost of Sales 500,000 Eligible under Income Tax Act
Administrative Expense 120,000 Eligible under Income Tax Act
cannot be included as per Sec. 14 (2), ATI
Interest Expense - must be calculated without including interest
expense and interest income

The Institute of Chartered Accountants of Nepal 146


CAP-II Paper 7 - Income Tax and VAT
Actually incurred during the year, as clarified
Depreciation 20,000
by Income Tax Directive
Total expense to consider 640,000
Adjusted Taxable Income 110,000

Step 2: Calculation of Maximum Limit


Maximum limit is sum of interest income and 50% of ATI calculated above
Interest Income 5,000
50% of ATI Computed above 55,000
Maximum Limit 60,000

Step 3: Calculation of Eligible Interest Expense u/s 14 (2)


Lower of actually paid to controlling person or maximum limit as per Step 2
Actually paid to controlling person 75,000
Maximum Limit 60,000
Eligible u/s 14 (2) 60,000

Step 4: Total eligible interest expense


Eligible interest, where Sec. 14
- Total interest less paid to controlling person
(2) is not applicable
Eligible interest, where Sec. 14
60,000
(2) is applicable
Total Eligible Interest expense 60,000

5) The income statement of Mishra Pvt. Ltd. for the year 2077/78 shows a net profit of NPR
750,000. Additional information on the income statement are as follows:
i) The expenses include donations of NPR 85,000 out of which NPR 40,000 is provided to a
tax-exempt organisation.
ii) The expenses include pollution control costs of NPR 400,000
iii) The expenses include research and development costs of NPR 900,000
Calculate the taxable income of the company for the year. (June 2022, 5 Marks)

Answer:
Calculation of net profit before deductions u/s
12, 17 & 18
Particulars Amount
Net profit for the year 750,000
Add: Donation 85,000
Add: Pollution Control Costs (PCC) 400,000
Add: Research and Development Costs (R&D) 900,000

The Institute of Chartered Accountants of Nepal 147


CAP-II Paper 7 - Income Tax and VAT
Net profit before deductions u/s 12, 17 and 18 2,135,000

Calculation of adjusted taxable income for


section 12, 17 & 18
Particulars For sec 12 For sec 17 For sec 18
Net profit before deductions u/s 12, 17 and 18 2,135,000 2,135,000 2,135,000
Less: Actual Pollution Control Costs (400,000) - (400,000)
Less: Actual Research and Development Costs (900,000) (900,000) -
Adjusted Taxable Income 835,000 1,235,000 1,735,000

Calculation of eligible Donation u/s 12


Lower of
a. Actual Cost 40,000
b. 5% of ATI (5% of NPR 835,000) 41,750
c. NPR 100,000 100,000
NPR 40,000 shall be deductible

Calculation of eligible PCC u/s 17


Lower of
a. Actual Cost 400,000
b. 50% of ATI (50% of NPR 1,235,000) 617,500
NPR 400,000 shall be deductible

Calculation of eligible R&D u/s 18


Lower of
a. Actual Cost 900,000
b. 50% of ATI (50% of NPR 1,735,000) 867,500
NPR 867,500 shall be deductible

Calculation of Taxable Income


Net profit before deductions u/s 12, 17 and 18 2,135,000
Less: Donation u/s 12 40,000
Less: PCC u/s 17 400,000
Less: R&D u/s 18 867,500
Taxable Income 827,500

6) The Profit and Loss Account of CBG & Co. for the financial year 20X-59/X-60 shows the
following details: (Dec 2003, 2 Marks, CA Inter)
Sales 500,000.00
Cost of Sales 300,000.00
Donation 20,000.00
Pollution Control Cost 300,000.00

The Institute of Chartered Accountants of Nepal 148


CAP-II Paper 7 - Income Tax and VAT
Repairs & Maintenance 10,000.00

How much amount is deductible as Pollution Control Cost in financial year 20X-59/X-60?

Answer:
Calculation of Eligible Pollution Control Cost
Step 1: Calculation of ATI for the purpose of Sec. 17
Particulars Amount Note
Inclusions
Sales 500,000 Eligible under Income Tax Act
Total Inclusions 500,000
Deductions
Cost of Sales 300,000 Eligible under Income Tax Act
Donation - Not considerable, as per the definition of ATI
Pollution Control Cost - Not considerable, as per Income Tax Directive
Repairs and Maintenance 100,000 Eligible under Income Tax Act
Total Deductions 400,000
Adjusted Taxable Income 100,000

Step 2: Calculation of Maximum Limit


Maximum Limit 50,000 50% of ATI from all businesses

Step 3: Eligible Amount


Lower of:
Maximum Limit 50,000
Actually Incurred 300,000
Eligible 50,000

Hence, Rs. 50,000 is deductible as Pollution Control Cost in financial year 20X-59/X-60.

The Institute of Chartered Accountants of Nepal 149


CAP-II Paper 7 - Income Tax and VAT

Chapter 12: Net Gain from Disposal of Business Assets and Liabilities

1) Mr. Ramesh had purchased land and private building on 2070/5/6 for Rs. 2.8 million. He
sold such property for Rs. 5.2 million on 2077/9/10. Calculate the total tax liability of Mr.
Ramesh on above transaction. Can Ramesh claim such tax as advance tax? (June 2021, 5
marks)

Answer:
Incomings from disposal of NBCA= Rs.5.2 Million and Outgoings of the NBCA = Rs.2.8 Million
Gain on Disposal of NBCA = Incomings – Outgoings = Rs. 2.4 Million

The tax rate @ 5% shall be levied on gain from NBCA under section of schedule 1 relating to
section 4 of Income Tax Act 2058 as the NBCA has been sold after holding more than 5 years of
ownership. Hence, the tax amount shall be Rs. 120,000

As per Sec. 95Ka, the gain on disposal of land and building is subject to collection of advance tax
by Land Revenue Office (Malpot Karyalaya) at the rate 5% of gain, if the ownership of such land
and building is 5 years or more. In the given case, the collection of advance tax by Land Revenue
Office (Malpot Karyalaya) is Rs. 120,000.
The amount can be claimed as advance tax.

2) Mr. Dangol has incurred a net loss of Rs. 3 lakhs during income year 2076/77 from sale of
shares. In the same year, he incurred a net loss of Rs. 3 lakhs from his stationary business.
During the year 2076/77, he sold his personal land (non-business chargeable asset) for Rs. 50
lakhs net of brokerage and other expenses. He purchased the land at Rs. 10 lakhs before 4
years. During the year 2075/76, he incurred a taxable loss of Rs. 1 lakhs from the business.
Calculate net taxable income of Mr. Dangol mentioning the relevant provisions of Income
Tax Act, 2058 for the year 2076/77. (Dec. 2020, 5 Marks)

Answer:
Section 36(2) of Income Tax Act, 2058 prescribes the following provisions;
Net gains from the disposal of non-business chargeable assets of an investment of a person for an
income-year are calculated as the total of all gains from the disposal of non-business chargeable
assets of the investment during the year reduced by the following losses-
• The total of all losses suffered from the disposal of non-business chargeable assets of the
investment during the year;
• Any unrelieved net loss out of any losses of business or investment of the person for the year;
and

The Institute of Chartered Accountants of Nepal 150


CAP-II Paper 7 - Income Tax and VAT

• Any unrelieved net loss for a previous income-year out of the losses of investment, any
business, or other investment of the person.
Accordingly.
Computation of net gain on disposal of NBCA of Mr. Dangol
Particulars Amount ('000 Rs.)
Gain from land
Net incomings 5,000
Net outgoings 1,000 4000
Less: Loss from non-business chargeable assets (From
share transaction) 300
Loss from business during the year 300
Unrelieved loss from previous year 100 (700)
Net Gain on Disposal of Non-business chargeable asset
(to be included in income as per Clause (Kha) of Sec. 9
(2)) 3,300

3) Discuss the applicability of withholding Tax in the following cases:


i) “A telecommunication company Resident in Nepal is providing international
communication services to local customers by using facilities of optical fiber and satellite
service providers situated outside Nepal. The Company pays fixed charge plus fee based
on transmission of information/data periodically against these services.”

ii) Mr. Ferguson, a European resident wanted to do business in Nepal and accordingly he
brought foreign currency amounting to US$ 50,000 and promoted a one-man company
and purchased agricultural land in the name of the Company equivalent to US$ 50,000
at that time. The total amount invested in the Company was Rs. 2,500,000 at the
prevailing exchange rate. But, due to various reasons, he could not start his operation for
more than 5 years, and so he lost interest in the project and wanted to sell the Company
at an agreed price of Rs. 7,500,000 and get the money remitted back to his country.
(Dec 2020 CAP II 5 marks)
Answer:
i) As per sec.67(6) (ja) of the Act, payments received by a person who conducts a business of
transmitting messages by cable , radio, optical fiber, or satellite communications in respect of
the transmission of messages by apparatus established in Nepal , whether or not such messages
originate in Nepal is deemed to have source in Nepal. Since in the question the optical and
satellite service provider are situated outside Nepal and naturally the equipment is also
expected to be situated outside Nepal, their income has no source in Nepal and is not liable to
tax in Nepal.

The Institute of Chartered Accountants of Nepal 151


CAP-II Paper 7 - Income Tax and VAT

If the equipment were situated in Nepal, then the provider of service shall be liable to payment of
tax of 5% as per Schedule 1-Section 2(7) by itself under the provisions of Sec. 70. Therefore, there
is no requirement of withholding of tax by the service recipient in both the cases.

ii) Section 95ka (2) states that a company shall collect advance tax (and not TDS) @ 25% attract on
gain on sale of shares of a company not listed in NEPSE, if the seller is non-resident. The
responsibility of collection of advance tax and deposition of such amount rests with such non-
listed company. In this case, when Mr. Ferguson, a non-resident person, sells his stake, in which
his investment was Rs.25 lakhs for Rs.75 lakhs, there is gain of Rs. 50 lakhs on which the new
management such non-listed company, has to collect 25% tax at the time of registration Sales
Purchase Agreement with the company, i.e. Rs.12,50,000 and deposit it with the Tax Department.

4) Mr. Rich has purchased a private building amounting to Rs. 2 crores on Kartik 23, 20X-60
at Kathmandu. He sold such building on Rs. 3 crores on Poush 26, 20X-70. During such
period, Mr. Rich was gone abroad, intermittently for a period of 120 days. Will such building
be considered as "Non-Business Chargeable Assets"? What will be your answer if such
building was sold after two months (i.e. on Falgun 26, 20X-70)? (Dec. 2014, 5 Marks)

Answer:
A building of a natural person is non-business chargeable asset, when it does not satisfy any of the
following conditions:
a. Ownership and Residence Test: It is private building, which the owner has continuously
owned for 10 years and more and has resided for 10 years or more, either continuously.
b. Disposal Value Test: The disposal value of building is less than Rs. 1 Million, and
c. Three Generation Test: The building is transferred within three generation without involving
commercial transactions.

Testing the given building against the conditions above:


a. Ownership & Residence Test: The building is owned for more than 10 years (23rd Kartik
20X-60 to 26th Poush 20X-70), but the residence is for less than 10 years (just 120 days), this
test is also not satisfied.
b. Disposal Value Test: The building is disposed off for value more than 1 Million, i.e. 30
Million
c. Three Generation Test: It is not the case of transfer within three generation without involving
commercial transactions.

Conclusion
None of the above conditions is satisfied, that brings us to the conclusion that the building in
concern is a “non-business chargeable asset”.

The Institute of Chartered Accountants of Nepal 152


CAP-II Paper 7 - Income Tax and VAT

Even if the building was sold after two months, none of the above conditions would be satisfied;
and there would be no change in our conclusion, i.e., the building remains “non-business
chargeable asset” within the definition of Income Tax Act, 2058.

5) Mr. Ramesh has sold a building with one ropani land at a consideration of Rs. 15,000,000 on
Chaitra 31, 20X-66. This building was purchased by him on Falgun 20, 20X-61 at a
consideration of Rs. 5,000,000 and also incurred an expenditure of Rs. 1,000,000 for
extension of building. Calculate the amount of tax to be deducted at the time of the sale of
land.
Will your answer be different if Mr. Ramesh had purchased that building on Falgun 20, 20X-
55? (June 2010, 5 Marks)

Answer:
A building of a natural person is non-business chargeable asset, when it does not satisfy any of the
following conditions:
a. Ownership and Residence Test: It is private building, which the owner has continuously
owned for 10 years and more and has resided for 10 years or more, either continuously.
b. Disposal Value Test: The disposal value of building is less than Rs. 1 Million, and
c. Three Generation Test: The building is transferred within three generation without involving
commercial transactions.

Testing the given building against the conditions above:


a. Ownership & Residence Test: The building is owned for more less than 10 years (20th Falgun
20X-61 to 31st Chaitra 20X-66), also the residence is for less than 10 years (assuming resided
from the date of purchase to date of sales), this test is also not satisfied.
b. Disposal Value Test: The building is disposed off for value more than 1 Million, i.e. 15
Million
c. Three Generation Test: It is not the case of transfer within three generation without involving
commercial transactions.

Conclusion
None of the above conditions is satisfied, that brings us to the conclusion that the building in
concern is a “non-business chargeable asset”.

Gain on Disposal of Non-business Chargeable Asset


Incomings Rs. 15,000,000
Less: Outgoings Rs. 6,000,000
Gain on Disposal Rs. 9,000,000
Advance tax to be collected by Malpot Karyalaya (@5%) Rs. 450,000
(Since the ownership is more than 5 Years, the rate is 5% as per Sec. 95Ka)

The Institute of Chartered Accountants of Nepal 153


CAP-II Paper 7 - Income Tax and VAT

Alternative Question:
If the purchase date was 20th Falgun 20X-55 and assuming that he has resided since then,
Ownership and Residence Test is satisfied, which makes us to conclude that the asset in concern
is not a non-business chargeable asset. There would be no taxation as per Income Tax Act, 2058
in such asset.

6) Mr. C owns a house property in Delhi, inherited from father. Since Mr. C is resident of Nepal
and has no intention of staying in India and sells the property for IRs.1.25 Crores. Discuss
the taxability of this transaction. Will it make a difference if the property is sold for IRs.60
lakhs only? (Dec 2004, 5 Marks, CA Inter)

Answer:
A building of a natural person is non-business chargeable asset, when it does not satisfy any of the
following conditions:
a. Ownership and Residence Test: It is private building, which the owner has continuously
owned for 10 years and more and has resided for 10 years or more, either continuously.
b. Disposal Value Test: The disposal value of building is less than Rs. 1 Million, and
c. Three Generation Test: The building is transferred within three generation without
involving commercial transactions.

Testing the given building against the conditions above:


a. Nature of Asset Test: The asset is not business asset, trading stock or depreciable asset.
b. Ownership & Residence Test: As the ownership information is not given, we assume that it
is owned for more less than 10 years and also the residence is for less than 10 years, this test
is also not satisfied.
c. Disposal Value Test: The building is disposed off for value more than 1 Million, i.e. 20
Million
d. Three Generation Test: It is not the case of transfer within three generation without involving
commercial transactions.

Conclusion
None of the above conditions is satisfied, that brings us to the conclusion that the building in
concern is a “non-business chargeable asset”. As such, the gain on disposal of such property is
taxable in Nepal as “Foreign Sourced Income”. Even if the property is sold for Rs. 9.6 Million, the
gain on disposal would be taxable in Nepal as “Foreign Sourced Income”.

7) M/s Hanuman Pvt. Ltd. having registered office at Kathmandu, exclusively deals in
computer trading wholesale business. It recently extended its branches to all 77 districts of
Nepal. The Company imports computers for wholesale business @ Rs. 85,000 per computer.

The Institute of Chartered Accountants of Nepal 154


CAP-II Paper 7 - Income Tax and VAT

After opening the branches, the Company sent the imported computers for office use one
piece to each branch. Is there any tax implication for the above transaction as per the
provision of Income Tax Act, 2058? (Dec 2021/2018, 5 Marks)

Answer:
When the trading stock of M/s Hanuman Pvt. Ltd. is changed to depreciable assets, it is deemed
as disposal as per Section 40 (3) (Gha) of IT Act where the person begins to use the assets in such
a way that it ceases to be an asset of the type it was immediately before that use.
So, the company shall consider the disposal of trading stock and ensure that there is inclusion u/s
7.
Further, the cost of computers shall also be treated as part of cost of trading stock u/s 15.
In addition to this, the company shall treat the computer as Pool B depreciable asset and compute
depreciation based on date of put to use of computer as per Schedule 2 of Income Tax Act, 2058.

8) Oriental Pvt. Ltd is dealing with buying, constructing and selling of land and building. It has
the following transactions for one of the projects during the year 20X-73/X-74. (Dec 2017, 5
Marks)
Land purchase for the project Rs. 50 million
Land purchase for office Rs. 10 million
Housing construction cost (10 houses) Rs. 150 million
Building construction cost for office Rs. 10 million
Land in Stock Rs. 20 million
Housing in Stock Rs. 60 million
Sales of housing (8 houses) Rs. 300 million

Based on the above information, answer the following questions:


a. Calculate gain and loss in this transaction
Answer:
Particulars Trading Stock Depreciable Asset
Incomings
Sales of Housing 300,000,000 -
Outgoings
Cost of Land 50,000,000 10,000,000
Cost of Construction of 10 houses 150,000,000 -
Building Construction for office - 10,000,000
Less: Closing Stock
Land in Stock (20,000,000)
Housing in Stock (60,000,000)
Total Outgoings 120,000,000
Gain on Disposal 180,000,000

The Institute of Chartered Accountants of Nepal 155


CAP-II Paper 7 - Income Tax and VAT

b. Whether the company should pay capital gain tax or not?


Answer
There is no concept of capital gain tax in Nepal. However, as per Sec. 95Ka, where an entity sale
its land and building, 1.5% of sales value shall be paid as advance tax at the time of transfer of
ownership at land revenue office. The amount of such advance tax would be Rs. 4,500,000.

c. If the company should pay capital gain tax, how much and where to pay?
Answer
Refer explanation to part (b) above.

d. If the company does not require to pay the capital gain tax, then what and where to pay
tax on the gain of this transaction?
Answer
The company needs to pay advance tax at the rate 1.5% of sales value at the time of transfer of
ownership at land revenue office. Further, it needs to pay corporate tax on its annual income within
three months of the end of Income Year.

9) Mr. Ram has been holding shares of various listed companies of Nepal from primary and
secondary markets. Total cost of the shares is Rs. 2 million. He left for USA for a year on
Aswin 30, 20X-70 but he returned back on Kartik 15, 20X-72. The market value of the shares
was Rs. 3.5 million at the time he became non-resident.
He sold all the shares for Rs. 3.6 million on Kartik end, 20X-73. He has received his payment
from share broker after deducting broker commission Rs. 20,000. Advance tax is deducted
by NEPSE as per Section 95Ka (2). Compute gain and loss under Section 40 (3), gain and
loss calculation on Kartik end 20X-73 by NEPSE and tax withholding amount in these
situations. (Dec 2017, 5 Marks)

Answer
Mr. Ram became non-resident on Ashad end, 20X-70; when the market value of shares was Rs.
3.5 Million. The outgoings related to such shares was Rs. 2 Million.

As per Sec. 40 (3) (Cha), when a resident natural person becomes non-resident, he is deemed to
dispose all his asset and liabilities except land and building just before he becomes non-resident.

(Students are not to compute gain/(loss) on disposal as this part is not covered in syllabus of
CAP II)
Remaining answer will be:
As per Sec. 41, such disposal is at market value, that means, Mr. Ram has gain on deemed disposal
of shares of Rs. 1.5 Million just before he becomes non-resident.

The Institute of Chartered Accountants of Nepal 156


CAP-II Paper 7 - Income Tax and VAT

Further, when he disposes off his shares as per Sec. 40 (1), the incomings is Rs. 3.6 Million and
the outgoings is Rs. 3.52 Million (3.5 Million as cost for subsequent disposal of shares as per Sec.
41 and 20,000 as cost to broker), where the gain is Rs. 0.48 Million, on which he has to pay tax on
actual disposal of assets.

Conclusion:
He pays tax on Rs. 1.5 Million as per the application of Sec. 40 (3) (Cha) and he pays tax on Rs.
0.48 Million at the time of actual disposal of shares.

10) Mr. Apurva Das has 250 shares of Nabil Bank Ltd. from IPO. Bonus shares has been
distributed 300% (100% before Income Tax Act and remaining thereafter) till now. He
became a non-resident for income year 20X-72/X-73. The market value at the time of
introduction of the Act was Rs. 1,800 and at the time of becoming non-resident was Rs. 2,600.
Quoting the related criteria for deemed disposal of shares. Advise him on, if any, tax liability
arises on those shares for income year 20X-72/X-73. (June 2017, 5 Marks)

Answer
It is assumed that bonus share’s percentage is expressed in terms of original number of shares, i.e.,
total additional bonus share is 750 kittas (250 Kitta before the implementation of prevailing Act
and 500 kittas thereafter)
Where a practical assumption is made, total numbers of shares become more than 1000, as per
different assumptions.

As per Sec. 40 (3) (Cha), when a resident natural person becomes non-resident, he is deemed to
dispose all his asset and liabilities except land and building just before he becomes non-resident.

As per Sec. 41 (remember syllabus of CAP II does not cover Sec. 41, and this part may be skipped
in examination), such disposal is at market value, i.e. shares are deemed to be disposed at Rs. 2,600
per share (per share assumed). Total incomings from deemed disposal of shares is Rs. 2.6 Million
(1,000 shares @ Rs. 2,600 each)

Outgoings of Shares:
Number of Shares as on implementation of the Act- 250 kittas plus 100% bonus shares= 500 Kittas
Outgoings = 500 * Rs. 1,800= Rs. 900,000 for 500 Kitta
Additional bonus shares, 500 Kittas (200% of 250 Kitta) at Rs. 100 each = Rs. 50,000
Therefore, total outgoings of 1000 kitta shares is Rs. 950,000
Gain on Disposal= Incomings – Outgoings = Rs. 1.65 Million

Tax liability, in respect of this gain, would be 5% of Gain

The Institute of Chartered Accountants of Nepal 157


CAP-II Paper 7 - Income Tax and VAT

Note to CAP II Students:


Students may answer this question in theoretical aspect, since a part of the question is out of
syllabus.

11) Mr. Govinda has incurred a net loss of Rs. 2 lakhs during income year 20X-71/X-72 from
sale of shares. In the same year he incurred a net loss of Rs. 3 lakhs from his stationary
business. During the year 20X-71/X-72, he sold his personal land (non-business chargeable
asset) for Rs. 60 lakhs net of brokerage and other expenses. He purchased the land at Rs. 20
lakhs before 4 years. During the year 20X-70/X-71, he incurred a taxable loss of Rs. 2 lakhs
from the business.

Calculate net taxable income from disposal of the land mentioning the relevant provisions of
Income Tax Act, 2058. (June 2016, 5 Marks)

Answer
Gain on Disposal of Non-Business Chargeable Asset = Rs.40 Lakhs

As per Sec. 20 (2), any loss on one investment during an Income Year can be offset against the
gain from another investment during the same Income Year. Further, Sec. 11 is not applicable in
respect of Gain on Disposal of Non-Business Chargeable asset, as such, the loss on sale of shares
of Rs. 2 lakhs can be offset against the gain on disposal of NBCA, i.e. net gain after set off of
investment loss is Rs. 38 Lakhs.

As per Sec. 20 (1), business loss can be offset against investment income, as such, business loss
of Rs. 3 lakhs can be offset against gain of Rs. 38 Lakhs, this gives us Rs. 35 Lakhs as income
from investment during the Income year.

12) Mr. Ram had purchased land and private building on 20X-65/7/4 for Rs. 2.8 million. He sold
such property for Rs. 5.2 million on 20X-71/8/15. Answer the followings mentioning the
relevant provisions of Income Tax Act/Rules. (Dec 2015, 5 Marks)
a. Calculate capital gain and tax thereon. Can Ram claim such tax as advance tax?
Answer
Incomings from disposal of NBCA = Rs. 5.2 Million
Outgoings of the NBCA = Rs. 2.8 Million
Gain on Disposal of NBCA = Incomings – Outgoings = Rs. 2.4 Million

There is no such concept of capital gain and capital gain tax in Income Tax law, since the gain on
disposal is treated part of investment income.

The Institute of Chartered Accountants of Nepal 158


CAP-II Paper 7 - Income Tax and VAT

As per Sec. 95Ka, the gain on disposal of land and building is subject to collection of advance tax
by Land Revenue Office at the rate 5% of gain, if the ownership of such land and building is 5
years or more. In the given case, the collection of advance tax by Land Revenue Office is Rs.
120,000.

The amount can be claimed as advance tax.

b. What shall be your answer if such property was purchased on 20X-60/4/15 and Mr. Ram
has been residing continuously?
Answer
As per Sec. 2, the definition of NBCA excludes private building having owned continuously for
10 years or more and having resided for 10 years or more, either continuously or intermittently.

Assuming, the building occupies total land area and the land area is within 1 ropani, if such
property was bought on 20X-60/4/15 and having been resided since then, the private building is
not within the scope of definition of NBCA.

Further, the private building cannot be treated as business asset, trading stock or depreciable asset
as per the definition provided in the Act, as such, the gain on disposal is exempt from Income Tax.

c. Will your answer be different if such property was purchased on 20X-69/4/15 and sold
for Rs. 0.9 million?
Answer
As per Sec. 2, the definition of NBCA excludes land and/or private building disposed off for less
than Rs. 1 Million.

In case the property is sold for Rs. 0.9 Million, the property does not meet the definition of NBCA.

Further, the property cannot be treated as business asset, trading stock or depreciable asset as per
the definition provided in the Act, as such, the gain on disposal is exempt from Income Tax.

13) JKL Limited purchased 100 shares of MNO Bank Limited for each Rs. 200 each including
brokerage and other expenses, as on Chaitra 19, 2058, the company was holding 250 shares
of MNO including the shares purchased and bonus shares paid by MNO. The quoted closing
price on Chaitra 18th, 2058 on MNO shares was Rs. 800 each.

Calculate the gain from the disposal of the shares under the circumstances given below: (Dec
2014, 10 Marks)

The Institute of Chartered Accountants of Nepal 159


CAP-II Paper 7 - Income Tax and VAT

Assumption: 100 shares purchased before Chaitra 19, 2058 and total 250 shares were held on
Chaitra 18, 2059

a. On Jestha 15, 20X-71 the company had disposed of the total shares at the rate of Rs. 1,000
each. The brokerage paid was 1.2% of the sales amount.
Answer
Incomings from the disposal = 250 shares * Rs. 1,000 per share= Rs. 250,000
Outgoings =
Shares held on 2058-12-18 = 250 shares * Rs. 800 per share = Rs. 200,000
Brokerage charge on sales = Rs. 250,000 * 1.2% = Rs. 3,000
Total Outgoings = Rs. 203,000
Gain = Incomings – Outgoings = Rs. 47,000

b. On Jestha 15, 20X-71 the company had disposed the total shares at the rate of Rs. 750
each. The brokerage paid was 1.2% of the sales amount.
Answer
Incomings from the disposal = 250 shares * Rs. 750 per share= Rs. 187,500
Outgoings =
Shares held on 2058-12-18 = 250 shares * Rs. 800 per share = Rs. 200,00
Brokerage charge on sales = Rs. 187,500 * 1.2% = Rs. 2,250
Total Outgoings = Rs. 202,250
Gain = Incomings – Outgoings = Rs. 14,750 (loss)

c. On Falgun 10, 20X-70 the company had received 125 bonus shares for the 250 shares held
by it. On Jestha 15, 20X-71 the company had disposed 250 shares at the rate of Rs. 750
each. The brokerage paid was 1.2% of the sales amount.
Answer
Per share
Particulars Amount
value
Incomings from Sale of 250 shares (A) 187,500.00 750
Outgoings
Value of 250 shares 200,000.00 800
Value of 125 bonus shares 12,500.00 100
Total Cost 212,500.00
Weighted average cost per share 566.67
Outgoings of sold shares 141,667
Brokerage charge to sale shares 2,250.00
Total Outgoings of 250 shares (B) 143,917

Gain on disposal (A-B) 43,583

The Institute of Chartered Accountants of Nepal 160


CAP-II Paper 7 - Income Tax and VAT

14) Mr. Anil has held 500 shares, at the rate of Rs. 100 per share (par value), of Standard
Chartered Bank Nepal Ltd (listed on Nepal Stock Exchange Ltd). He has requested to the
stockbroker to sell those shares held by him. Mr. Anil has purchased those shares at the rate
of Rs. 1,000 per share. He has paid Rs. 4,000 as brokerage commission at the time of purchase
of shares. He sold those shares at the rate of Rs. 7,000 per share and paid the brokerage
commission of Rs. 17,500 to the broker on sale.
a. Compute the gain on shares disposal.
Answer

No. of Per share


Particulars Total
shares value

Incomings from sale 500 7000 3,500,000


Outgoings
Initial Purchase cost 500 1000 500,000
Brokerage cost during
purchase 4,000
Brokerage cost during sales 17,500
Outgoings related to sold
shares 521,500
Gain on Disposal 2,978,500

b. Is there any implication of withholding taxes? If yes, calculate the payment to be made
to Mr. Anil stating relevant provisions of the Income Tax Act, 2058.
Answer
There is no implication of withholding taxes. However, advance tax is collected by Nepal Stock
Exchange Ltd. from the gain on disposal at the rate 5% of gain, as per Sec. 95ka of the Act.
(Assuming the shares are held for more than 365 days)

The amount of advance tax to be paid to Nepal Stock Exchange Ltd. is Rs. 148,925.

c. What will be the implication if the above-mentioned shares were held and sold by an
entity instead of Mr. Anil? (June 2012, 6 Marks)
Answer
If those shares were held and sold by entity:
i. The rate of advance tax collection by Nepal Stock Exchange Ltd. would be 10% of gain,
and the amount of advance tax would be Rs. 297,850.
ii. The amount of gain would be included in income from business of the entity and would be
taxed at corporate tax rate.

The Institute of Chartered Accountants of Nepal 161


CAP-II Paper 7 - Income Tax and VAT

15) Smart Pvt. Ltd. purchased a piece of land on Ashwin 25, 20X-64 for Rs. 1,000,000 and
incurred an expenditure of Rs. 150,000 on registration and brokerage. It constructed a
building on the land costing Rs. 2,500,000. The building was ready for use on Shrawan 25,
20X-65. The depreciated value in Block A was Rs. 1,000,000 at the end of year 20X-64/X-65
excluding the cost of the building under construction. During the year 20X-65/X-66 the
company capitalized Rs. 60,000 as repair and improvement cost in Block A.

On Shrawan 1, 20X-66, the company sold the land and newly constructed building for Rs.
4,000,000. The market value of land on that date was Rs. 1,500,000 and that of the building
was Rs. 2,500,000. You are required to calculate taxable gain for the disposal of land and
building for the year 20X-66/X-67. (June 2011, 5 Marks)

Answer
The land is business asset and the building is depreciable asset. Since, all the building in Pool ‘A’
are not disposed, there is existence of Pool ‘A’ at the end of 20X-66/X-67.

Calculation of Gain on Disposal of Land:


Particulars Amount Notes
Incomings 1,500,000 Market value as on the date of sales

Outgoings
Purchase cost 1,000,000
Registration Charges 150,000
Total Outgoings 1,150,000
Gain on Disposal 350,000

Calculation of effect of disposal of Building


Particulars 20X-65/X-66 20X-66/X-67 Notes
Opening Depreciation Base 1,000,000 3,385,000
Absorbed Additions 2,500,000 - Added in Ashoj
Disposal Proceeds - (2,500,000)
Depreciable Basis 3,500,000 885,000
Depreciation Rate 5% 5%
Depreciation Amount 175,000 44,250
Unabsorbed Additions - -
Capitalized Repair Cost 60,000 - Given
Opening Depreciation Base for next Year 3,385,000 840,750

Since, the depreciable basis for I.Y. 20X-66/X-67 is positive and there is existence of assets in the
pool, there is no gain on disposal (balance charge) of building

The Institute of Chartered Accountants of Nepal 162


CAP-II Paper 7 - Income Tax and VAT

Chapter 13: International Taxation

1) Bottlers Nepal Ltd. Kathmandu has paid consulting fees Rs. 30 million in the year 20X-74/X-
75 to International Consultancy Group, New York (ICGN). As per the agreement, the
consultant should deploy at least 2 employees at the project site located at Butwal, Province
No. 5 throughout the year. Can we define ICGN as a permanent establishment in Nepal?
What about the tax withholding implications in the following situations? (June 2018, 5
marks)

Answer:
Definition of Permanent Establishment:
Permanent Establishment (PE) means any place where a person carries on its business whether
fully and partially, and the term also includes the following:
a. Any place from where a person carries on business through an agent, who is not a general agent
of independent status (Agency PE),
b. Any place where a person has, is using or is installing main equipment or machinery (Fixed
Base PE),
c. one or more places of a country where a person provides technical, business or consultancy
services through employee or otherwise for more than 90 days in any 12 months period
(Service PE)
d. a place where a person is engaged in a construction, assembly, or installation project for 90
days or more, including a place where a person is conducting supervisory activities in relation
to such a project (Fixed Site PE)

Testing the facts and Conclusion


The consultant deploys at least two employees at the project site located at Butwal, Province No.
5 throughout the year; or even if it does not deploy staff; the company provides service for more
than 90 days in last twelve-month period.

Therefore, the company is deemed to have permanent establishment in Nepal.

When a non-resident person has a permanent establishment in Nepal, it must register itself as
Permanent Establishment (PE), obtain Permanent Account Number, raise invoice from PE in
Nepal and collect the service fee in the bank account of Nepal. The PE should pay Corporate Tax
and repatriation tax in Nepal, and repatriate the profit from Nepal after due approval from Nepal
Rastra Bank by paying Repatriation Tax @5% sec 1(2) of Schedule 1.

a. If the company is registered in Nepal through its local agent and payment was made to
local agent.

The Institute of Chartered Accountants of Nepal 163


CAP-II Paper 7 - Income Tax and VAT

Answer:
If the company is registered in Nepal through its local agent, it would issue tax invoices (VAT) in
Nepal. The withholding tax rate should be 1.5%.

b. If the company is not registered in Nepal and payment was transferred directly to their
bank maintained in New York.
Answer:
This cannot be the case; the company must register as Permanent Establishment in Nepal. It is the
duty of Bottlers Nepal to advise its vendor regarding possible taxation implications in Nepal. When
it is registered, the implication would be as explained in part(a) of this question.

2) Discuss on the "Permanent Establishment" in the following cases mentioning the relevant
provisions of Income Tax Act/Rules. (June 2016, 10 Marks)
Answer:
Definition of Permanent Establishment:
Permanent Establishment (PE) means any place where a person carries on its business, and the
term also includes the following:
i. Any place from where a person carries on business through an agent, who is not a general agent
of independent status (Agency PE),
ii. Any place where a person has, is using or is installing main equipment or machinery (Fixed
Base PE),
iii. one or more places of a country where a person provides technical, business or consultancy
services through employee or otherwise for more than 90 days in any 12 months period
(Service PE)
iv. a place where a person is engaged in a construction, assembly, or installation project for 90
days or more, including a place where a person is conducting supervisory activities in relation
to such a project (Fixed Site PE)

a. American Sport Inc. has appointed the distributor – Amco Sports Pvt Ltd in Nepal in
order to make a sale of its products. Amco Sport Pvt Ltd is importing the products from
such American Sports Inc. and selling the products determining the price by its own.
There is no ownership of such products, no any restriction or special assistance from
American company on sale price or the prospective customers. The American company
exports its products through Amco Pvt Ltd. Is Amco Pvt Ltd is a 'Permanent
Establishment' of American Sport Inc.?

Answer:
In the given case, since Amco Sports Pvt. Ltd. is general agent of independent status, Permanent
Establishment of American Sport Inc. is not formed in Nepal.

The Institute of Chartered Accountants of Nepal 164


CAP-II Paper 7 - Income Tax and VAT

b. American Sport Inc. has been awarded as task of making research of potential
opportunities of sports in Nepal. To perform the task, such company sent its employees
on January 1 for 35 days. Again, on July and December of the same year, the company
sent 2 employees for 30 days and 60 days respectively. The employees of American Sports
Inc. worked in two Income Years of Nepal. Is there any consequence of in Nepal on
'Permanent Establishment" in this case?

Answer:
Total days of service in Nepal during consecutive 12 months period: 35 days in January, plus 30
days in July and 60 days in December; this is 125 days in 12 months period.

This makes us to conclude that a service Permanent establishment is formed in Nepal as a result
of service more than 90 days through employee or otherwise in Nepal during any 12-month period.

c. Singapore Construction Company has been awarded 100 man-days for drying the soil in
Melamchi project. Such company has sent 5 employees in order to perform such task and
the employees have completed such task within 90 days. Is there any consequence in
Nepal on 'Permanent Establishment" in this case?

Answer:
Total days of service in Nepal during consecutive 12 months period: less than 90 days in any 12-
month period.
This makes us to conclude that Permanent establishment is not formed in Nepal as service period
is not more than 90 days through employee or otherwise in Nepal during any 12 month period.

3) Specify the country where the following sources of income are situated according to Income
Tax Act, 2058.
i) Income from a property situated in Japan
ii) Freight received by an airlines company in Nepal for a ticket issued from Colombo to
Jakarta
iii) Dividend paid by a commercial bank in Nepal
iv) Income from an export from Nepal to Germany
v) Business service provided in India for which Government of Nepal makes the payment.
(June 2022, 5 Marks)
Answer:
As per section 67 of Income Tax Act, 2058,
i) Source of Income is Japan as the fixed property is in Japan.
ii) Source of Income is Sri Lanka as the cargo embarks from Sri Lanka.
iii) Source of Income is Nepal as the bank is in Nepal (sec. 67(6)).
iv) Source of Income is Nepal as the business activity is in Nepal (sec. 67(1)).

The Institute of Chartered Accountants of Nepal 165


CAP-II Paper 7 - Income Tax and VAT

v) Source of Income is Nepal as the payment is made by Government of Nepal (sec. 67(6)).

4) Point out the country where the following sources of income are situated according to Income
Tax Act, 2058. (Dec 2004, 4 Marks, CA Inter)
a. Income from an export from Nepal to Bangladesh.
b. Service (otherwise than employment) provided in India for which Government of Nepal
makes payment.
c. Freight received by an air company in Nepal for a ticket issued for Delhi to U.A.E.
d. Income from a property situated in Pakistan.

Answer:
a. Source of Income is Nepal, as the business activity is conducted in Nepal [Sec. 67 (6) (Tha)
(2)]
b. Source of Income is India, as the service is not performed in Nepal and payment is not made
by GoN for employment [Sec. 67 (6) (Jha)]
c. Source of Income is India, as the cargo embarks the aircraft from India [Sec. 67 (6) (Chha)]
d. Source of Income is Pakistan, as the fixed property is situated in Pakistan [Sec. 67 (6) (Gha)]

5) Gulf Bank, a Dubai based banking company has an office in Kathmandu for its wholesale
banking service to local banks. During the year it made Rs. 7,500,000 profits from the
operation in Nepal of which Rs. 5,000,000 was repatriated to head office Dubai. Calculate
Tax Liability. (June 2017, 2.5 Marks)

Answer:
Corporate Tax Liability (assuming profit is before tax)
CIT (Corporate Income Tax)= 25% of Rs. 7,500,000 = Rs. 1,875,000

Tax on Repatriation Income


Repatriated Amount (net): Rs. 5,000,000
Amount before repatriation tax= Rs. 5,000,000 ÷ 0.95 = Rs. 5,263,158
Repatriation Tax = 5% of Rs. 5,263,158 = Rs. 263,158

(Alternatively, students may assume Rs. 5,000,000 as gross amount of repatriation. In that
case, tax on repatriation would be Rs. 250,000)

6) AMEXO Bank is registered in the USA and operating its liaison office in Kathmandu. During
the financial year 20X-67/X-68, it has following summarized transactions.
Particulars Rs.
Income recognized 5,000,000
Expenses 4,000,000 (before Income Tax)

The Institute of Chartered Accountants of Nepal 166


CAP-II Paper 7 - Income Tax and VAT

The liaison office has policy to repatriate all the remaining profits to its corporate office.
Compute maximum amount that can be repatriated from Kathmandu liaison office. (Dec
2011, 5 Marks)

Answer:
As per Sec. 3 (Ka) and (Kha), a foreign permanent establishment (person) is required to pay
corporate income tax in Nepal in its taxable income and also while repatriating the after-tax profit.
Corporate Tax
In the given case, the taxable income is Rs. 10 Lakhs, corporate tax rate is 25% (Sec. 2 of Schedule
1) and the tax liability is Rs. 2.5 Lakhs.
Repatriation Tax
AMEXO bank can repatriate Rs. 7.5 lakhs to its head office after paying repatriation tax at the rate
5% (Sec. 2 (6) of Schedule 1).

Repatriated Amount
There are two schools of thoughts regarding computation of repatriation tax:

Alternative 1: 5% of Rs. 7.5 Lakhs is repatriation tax, which is Rs. 37,500 and the amount that
can be repatriated is Rs. 712,500. This is backed by interpretation of Inland Revenue Department
through Income Tax Directives. (Students are suggest to follow Alternative 1 in exam.)

Alternative 2: 5% of (Rs. 7.5 Lakhs ÷ 1.05), which is Rs. 35,714.29 and the amount that can be
repatriated is Rs. 714,285.71. This is logical in the sense that the taxing person is FPE in Nepal
and the repatriation tax is levied on repatriated amount.

7) Air Asia is a non-resident person having branch in Kathmandu, it has recorded Rs. 50 lakhs
as income from sale of the tickets related to the passengers departed from TIA, Kathmandu
and it has Rs. 30 lakhs branch expenses including salary expenses Rs. 20 lakhs. You are
required to make tax assessment for the Income Year 20X-72/X-73. (Dec 2016, 2.5 Marks)

Answer:
As per Sec. 70 of Income Tax Act, 2058; The taxable income of non-resident air transport operator,
water transport operator or chartered service provide for a particular Income Year shall be the
amount received for following activities, except as a result of transshipment:
• Amount received as a result of transport of passengers who embark from Nepal
• Amount received for the transportation of mail, livestock or other tangible assets that embark
from Nepal
The expenses in relation to the generation of income from air transport operation, water transport
operation or chartered service from Nepal with destination outside Nepal is not deductible. As

The Institute of Chartered Accountants of Nepal 167


CAP-II Paper 7 - Income Tax and VAT

such, tax is applicable in Rs. 50 Lakhs @ 5%. The expense is not deductible. The tax liability is
Rs. 2.5 Lakhs.

8) Explain on the taxability and the implication thereon, of the following transactions as per
the Income Tax Act, 2058. (June 2013, 6+4=10 )
a. A Den airline registered in Denmark, having contact office in Nepal and is operating its
airlines business. During Income Year 20X-68/X-69, it has sold the tickets in Nepal as
follows:
i. Sale of tickets from the passengers departing from Nepal - Rs. 50 crores.
ii. Sale of tickets in Nepal, for the passengers departing from country other than Nepal
– Rs. 10 crores.
Answer
As per Sec. 70 of Income Tax Act, 2058; The taxable income of nonresident air transport operator,
water transport operator or chartered service provide for a particular Income Year shall be the
amount received for following activities, except as a result of transshipment:
a) Amount received as a result of transport of passengers who embark from Nepal.
b) Amount received for the transportation of mail, livestock or other tangible assets that embark
from Nepal.

The expenses in relation to the generation of income from air transport operation, water transport
operation or chartered service from Nepal with destination outside Nepal is not deductible.
i. The amount is taxable, and the tax rate is 5%.
ii. Principally, the amount is not taxable. However, proviso to Sec. 2 (7) of Schedule 1 of the
Act requires payment of tax @ 2%.

b. Singtel Ltd. is a company registered in Singapore. The company, with its objective to
transmit information and storing data, has a communication hub in Nepal (without any
office in Nepal). Through such system, the companies in Europe and America are storing
data and transmitting information. Singtel has received USD 1 million for such services.

Answer
The taxable income of nonresident telecommunication operator (who conducts a business of
transmitting messages by cable, radio, optical fibre, or satellite communication) shall include
any amount received by a person in respect of the transmission of messages by apparatus
established in Nepal, whether or not such messages originate in Nepal.
Therefore, the amount of USD 1 Million is taxable in Nepal, as Sec. 70 is applicable to Singtel.

9) Rolls Airways is conducting the airplane services between Kathmandu and Saudi Arabia.
Rolls Airways has a branch office at Kathmandu for the purpose of operation of its services.

The Institute of Chartered Accountants of Nepal 168


CAP-II Paper 7 - Income Tax and VAT

The income of Rolls Airways for the income year 20X-67/X-68 while conducting the airplane
services and other operations are as below:
SN Particulars Amount (Rs.)
1 Amount received from the passengers departed from 10,000,000
Kathmandu
2 Amount received from the transportation of the goods having 5,000,000
first point entry at Kathmandu
3 Income from the travels from Bhutan and Lhasa having 5,000,000
transit point at Kathmandu
4 Amount received from the rentals of the goods carried from 5,000,000
Kathmandu which were transported from Bhutan
5 Amount received from the services of restaurant operated for 2,500,000
the facility of passengers at Kathmandu Airport
6 Amount received from the services of Ground Handling for 2,500,000
another airlines’ operation
Total Income 30,000,000

The followings are the details of expenditures of Rolls Airways:


SN Particulars Amount (Rs)
7 Expenditures from Air tickets 5,000,000
8 Expenditures against the accommodation and fooding for 5,000,000
crew members
9 Expenditures against ground handling services 3,000,000
10 Expenditure for the operation of restaurant 2,000,000
11 Expenditure for the transportation of passengers from 5,000,000
Bhutan, Lhasa
12 Expenditure for the re-transportation of the goods 5,000,000
transported from Bhutan
Total Expenditure 25,000,000

Note: Rolls Airways has managed the ground handling against 100 flights of its own and 100
flights of other airlines during Fiscal Year 20X-67/X-68.
Explain about the inclusion of above incomes and allowances of expenditures of a non-
resident person, for the purpose of section 70 of Income Tax Act, 2058 stating the relevant
provisions. (June 2012, 10 Marks)

The Institute of Chartered Accountants of Nepal 169


CAP-II Paper 7 - Income Tax and VAT

Answer:
Explanation on Taxability of the given amounts
Particulars Amount Taxability
The amount is taxable under Sec. 70, as it
Amount received from the is received by a non-resident air transport
1 passengers departed from 10,000,000 operator in respect of passengers
Kathmandu embarking from Nepal, not being
transshipment
Amount received from the The amount is taxable under Sec. 70, as it
transportation of the goods is received by a non-resident air transport
2 5,000,000
having first point entry at operator in respect of goods embarking
Kathmandu from Nepal, not being transshipment
Income from the travels from The amount is not taxable, as the amount
3 Bhutan and Lhasa having 5,000,000 does not have source in Nepal; since it is
transit point at Kathmandu for the passengers in transit
Amount received from the
The amount is not taxable, as the amount
rentals of the goods carried
4 5,000,000 does not have source in Nepal; since it is
from Kathmandu which were
for the goods in transit
transported from Bhutan
Amount received from the
This amount is not within purview of Sec.
services of restaurant
70; but within purview of Sec. 7; i.e.
5 operated for the facility of 2,500,000
business income of Permanent
passengers at Kathmandu
Establishment
Airport
This amount is not within purview of Sec.
Amount received from the
70; but within purview of Sec. 7; i.e.
6 services of Ground Handling 2,500,000
business income of Permanent
for other airlines’ operation
Establishment
Expenditures from Air
7 5,000,000 Not deductible under Sec. 70
tickets
Expenditures against the
8 accommodation and fooding 5,000,000 Not deductible under Sec. 70
for crew members
Deductible while calculating assessable
Expenditures against ground income of Permanent Establishment.
9 3,000,000
handling services However, not deductible to the extent it is
incurred for flight operation

Expenditure for the operation Deductible while calculating assessable


10
of restaurant 2,000,000 income of Permanent Establishment.

The Institute of Chartered Accountants of Nepal 170


CAP-II Paper 7 - Income Tax and VAT

However, not deductible to the extent it is


incurred for flight operation

Expenditure for the


Not deductible under Matching concept,
11 transportation of passengers
5,000,000 as the income is also not taxable in Nepal
from Bhutan, Lhasa
Expenditure for the re-
Not deductible under Matching concept,
12 transportation of the goods
5,000,000 as the income is also not taxable in Nepal
transported from Bhutan

Calculation of Taxable Income under Sec. 70


Amount received from the passengers departed from Kathmandu 10,000,000
Amount received from the transportation of the goods having first
5,000,000
point entry at Kathmandu
Total Taxable Income 15,000,000
Tax Liability @ 5% 750,000

Calculation of Assessable Income for Permanent Establishment on Other business


Income
Particulars Amount
Inclusions
Amount received from the services of restaurant
operated for the facility of passengers at Kathmandu 2,500,000
Airport
Amount received from the services of Ground
2,500,000
Handling for other airlines’ operation
Total Inclusions 5,000,000
Deductions
Cost for 200 flights.
However, 100 flights
Expenditures against ground handling services 1,500,000 are own, therefore
50% expense is not
deductible u/s 70
Expenditure for the operation of restaurant 2,000,000
Total Deductions 3,500,000
Assessable Income from Business 1,500,000

10) Mr. Basant Bohara resident in Nepal during FY 2077/78 provides following details of his
income during FY 2077/78:
i) Assessable income from Nepal 2,500,000 Advance tax paid in Nepal was Rs. 400,000
The Institute of Chartered Accountants of Nepal 171
CAP-II Paper 7 - Income Tax and VAT

ii) Income from Germany Rs. 500,000 tax paid in Germany was Rs. 150,000
iii) Income from Switzerland Rs. 500,000 tax paid in Switzerland was Rs. 50,000

Required:
Calculate amount of Income tax liability and amount of any unrecovered foreign tax credit
of Mr. Bohara for FY 2077/78 assuming that he opted to be assessed as couple and adopted
Credit Method for availing foreign tax credit. (Dec 2021, 5 Marks)

Answer:
Computation of Income tax liability of Mr. Bohara (assuming him as couple) after claiming foreign
tax credit under Credit Method for 2077/78:
Tax
Taxable Income Rate Tax (Rs)
Up to Rs. 600,000 0% 0.00
Next Rs. 200,000 10% 20,000.00
Next Rs. 300,000 20% 60,000.00
Next Rs. 900,000 30% 270,000.00
Balance Rs. 1,500,000 36% 540,000.00
Total Tax Liability before Any Tax Credit 890,000.00
Less: Advance tax paid in Nepal (400,000.00)
Less: Foreign tax Credit available (W.N. 3) (177,150.00)
Net tax Liability after claiming foreign tax credit 312,850.00

Working Note: 1
Calculation of Tax Liability before Foreign Tax Credit
Assessable Income from Nepal 2500,000
Assessable Income from Germany 500,000
Assessable Income from Switzerland 500,000
Total Assessable Income
(Taxable/Balance Taxable Income) 3,500,000
Working Note: 2
Average Income Tax Rate:
890,000 x100
Nepal Average Income tax rate = = 25.43%
3,500,000

Working Note-2:
Computation of average IT Rate and eligible foreign tax credit available to Mr. Bohara by using
Credit Method for the FY 2077/78:

The Institute of Chartered Accountants of Nepal 172


CAP-II Paper 7 - Income Tax and VAT

Available Foreign
Taxable Tax Paid Average tax tax Credit (3) min
Country Income (1) (2) of 1 or 2
i) Germany 500,000 150,000 127,150 127,150
ii)Switzerland 500,000 50,000 127,150 50,000
Total 1,000,000 200,000 254,300 177,150

Mr. Bohara can avail foreign tax credit of Rs. 177,150 and his unrecovered tax credit from
Germany is Rs. 22,850 (Rs. 150,000-Rs. 127,150), and it will be available for set off from his
future income of Germany.

11) Mr. Udit, a resident natural person, is a musician deriving income from concerts performed
from various countries outside Nepal. During financial year 2077/78, he performed concerts
in India. Details of his assessable income and tax paid in the country as aforesaid where the
concerts were given are:
Income Tax
India Rs. 1,000,000 Rs. 300,000
Also, he earned Rs. 500,000 in Nepal during the financial year 2077/78.
Assuming that he has chosen to be couple, find his tax liability on his total income and
amount for foreign tax credit available to him under section 71(1) for the income year
2077/78. (June 2021/Dec 2021 5 marks)

Answer:
Step 1: Calculation of Tax Liability before Any Tax Credit
Assessable Income from Nepal 500,000
Assessable Income from India 1,000,000
Total Assessable Income
(Taxable/Balance Taxable Income) 1,500,000
Tax Liability
1st Rs. 600,000 0% -
Next Rs. 200,000 10% 20,000
Next Rs. 300,000 20% 60,000
Next Rs. 400,000 30% 120,000
Tax Liability before Tax Credit 200,000

Step 2: Calculation of Average Tax rate in Nepal


Tax Liability before Foreign Tax Credit (A) 200,000
Taxable Income in Nepal (B) 1,500,000
Average Tax Rate in Nepal [A/B * 100] 13.33%

The Institute of Chartered Accountants of Nepal 173


CAP-II Paper 7 - Income Tax and VAT

Step 3: Calculation of Eligible Foreign Tax Credit and excess, if any


India
A Foreign Assessable Income 1,000,000
B Average Rate of Tax in Nepal 13.33%
C Average Tax liabilities in Nepal (A * B) 133,333
D Actual Tax Paid in Foreign Country 300,000
E Excess Taxes of Previous Income Year (s) -
F Sum of D & E 300000
G Eligible Amount [Lower of "C" or "F"] 133,333
H Excess to be carried forward [F- G] 166,666

Step 4: Calculation of Tax Liability after


Tax Credit
Tax Liability before Foreign Tax Credit 200,000
Less: Eligible Credit from Step 3 (133,333)
Net Tax Liability after Foreign Tax Credit 66,667

12) Mr. Ram Prasad, Nepali resident with single status for taxation, had submitted the following
details of income from business and investment as self-assessment of his tax liability for the
Income Year 20X-73/X-74:
Amount (Rs.)
Country Assessable Income Tax paid
Nepal 500,000.00 25,000.00
USA 200,000.00 65,000.00
UK 300,000.00 -
Canada 500,000.00 80,000.00
Total 1,500,000.00 170,000.00
Compute any unabsorbed foreign tax credit of Mr. Ram Prasad during the assessment. (June
2018, 5 Marks)
Answer
Step 1: Calculation of Tax Liability before Foreign Tax Credit

Assessable Income from Nepal 500,000


Assessable Income from USA 200,000
Assessable Income from UK 300,000
Assessable Income from Canada 500,000
Total Assessable Income
(Taxable Income) 1,500,000

The Institute of Chartered Accountants of Nepal 174


CAP-II Paper 7 - Income Tax and VAT

Tax Liability Note


Assuming there is no
1st Rs. 500,000 0% -
employment income
Next Rs. 200,000 10% 20,000
Next Rs. 300,000 20% 60,000
Balance Rs. 500,000 30% 150,000
Tax Liability before FTC 230,000

Step 2: Calculation of Average Tax rate in Nepal


Tax Liability before Foreign Tax
Credit (A) 230,000
Taxable Income in Nepal (B) 1,500,000
Average Tax Rate in Nepal [A/B
* 100] 15.33%

Step 3: Calculation of Eligible Foreign Tax Credit for each country and excess, if any

Particulars USA UK Canada Total


A Foreign Assessable Income 200,000 300,000 500,000
B Average Rate of Tax in Nepal 15.33% 15.33% 15.33%
C Average Tax Paid in Nepal (A * B) 30,660 45,990 76,650
D Actual Tax Paid in Foreign Country 65000 - 80000
Excess Taxes of Previous Income Year
E (s) 0 - 0
F Sum of D & E 65,000 0 80,000
G Eligible Amount [Lower of "C" or "F"] 30,660 - 76,650 107,310
H Excess to be carried forward [F- G] 34,340 - 3,350 37,690

13) What are the possible methods of taking foreign tax credit to avoid double taxation of
resident person? (June 2017, 5 Marks)

Answer:
There are two methods:
a. Credit method:
Foreign tax credit is available to a resident person that generates income having source outside
Nepal and pays tax in such source country.
The maximum amount of foreign tax credit shall not exceed the amount determined by multiplying
foreign assessable income by average rate of tax in Nepal, and such maximum amount shall be
calculated separately for each country. Any excess foreign tax payment during a country is carried
forward to be claimed during subsequent income years.

The Institute of Chartered Accountants of Nepal 175


CAP-II Paper 7 - Income Tax and VAT

b. Deduction Method: The tax paid in foreign country can be claimed as deduction, if the
taxpayer so wishes to relinquish its right to use credit method.

14) Mr. Z has a source of income in Nepal and also in more than one foreign country. During
the Fiscal Year 20X-67/X-68, income and tax paid in each foreign country is given below:
Name of the Country Income (Rs.) Tax Paid (Rs.)
USA 200,000 60,000
Australia 150,000 30,000
UAE 100,000 5,000
Nepal 250,000 -
He is a resident natural person and selected for the couple as tax payers during the year.
Calculate his tax liability during the Fiscal Year 20X-67/X-68. (June 2012, 5 Marks)

Answer
Step 1: Calculation of Tax Liability before Foreign Tax Credit

Assessable Income from Nepal 250,000


Assessable Income from USA 200,000
Assessable Income from Australia 150,000
Assessable Income from Canada 100,000
Total Assessable Income/Taxable Income 700,000

Tax Liability
1st Rs. 600,000 0% - Assuming no employment income
Next Rs. 100,000 10% 10,000
Tax Liability before Any Tax Credit 20,000

Step 2: Calculation of Average Tax rate in Nepal


Tax Liability before Foreign Tax Credit (A) 10,000
Taxable Income in Nepal (B) 700,000
Average Tax Rate in Nepal [A/B * 100] 1.43%

Step 3: Calculation of Eligible Foreign Tax Credit for each country and excess, if any

Particulars USA Australia UAE Total


A Foreign Assessable Income 200,000 150,000 100,000
B Average Rate of Tax in Nepal 1.43% 1.43% 1.43%
C Average Tax Paid in Nepal (A * B) 2,860 2,145 1,430

The Institute of Chartered Accountants of Nepal 176


CAP-II Paper 7 - Income Tax and VAT

D Actual Tax Paid in Foreign Country 60,000 30,000 5,000


Excess Taxes of Previous Income Year
E (s) - - -
F Sum of D & E 60,000 30,000 5,000
G Eligible Amount [Lower of "C" or "F"] 2,860 2,145 1,430 6,435
H Excess to be carried forward [F- G] 57,140 27,855 3,570 88,565

Step 4: Calculation of Tax Liability after Tax Credit


Tax Liability before Any Tax Credit 10,000
Less: Eligible Credit from Step 3 (6,435)
Tax Liability after Foreign Tax Credit 3,565

15) Mr. A, a resident natural person, is a musician deriving income from concerts performed
from various countries outside Nepal. During Financial Year 20X-66/X-67, he performed
concerts in India. Details of his assessable income and tax paid in the country as aforesaid
where the concerts were given are:
Income Tax
India Rs. 1,000,000 Rs. 300,000
Also, he earned Rs. 5,00,000 in Nepal during the financial year 2066/67.
Assuming that he has chosen to be couple, find his tax liability on his total income and
amount for foreign tax credit available to him under section 71(1) for the income year 20X-
66/X-67. (Dec 2010, 10 Marks)

Answer
Assuming there is no employment income
Step 1: Calculation of Tax Liability before Any Tax Credit

Assessable Income from Nepal 500,000


Assessable Income from USA 1,000,000
Total Assessable Income
(Taxable Income) 1,500,000

Tax Liability
1st Rs. 600,000 0% -
Next Rs. 200,000 10% 20,000
Next Rs. 300,000 20% 60,000
Next Rs. 400,000 30% 120,000
Tax Liability before Any Tax Credit 200,000

Step 2: Calculation of Average Tax rate in Nepal

The Institute of Chartered Accountants of Nepal 177


CAP-II Paper 7 - Income Tax and VAT

Tax Liability before Foreign Tax Credit


(A) 200,000
Taxable Income in Nepal (B) 1,500,000
Average Tax Rate in Nepal [A/B * 100] 13.33%

Step 3: Calculation of Eligible Foreign Tax Credit and excess, if any

India
A Foreign Assessable Income 1,000,000
B Average Rate of Tax in Nepal 13.33%
C Average Tax Paid in Nepal (A * B) 133,333
D Actual Tax Paid in Foreign Country 300,000
Excess Taxes of Previous Income Year
E (s) -
F Sum of D & E 300000
G Eligible Amount [Lower of "C" or "F"] 133,333
H Excess to be carried forward [F- G] 166,667
Step 4: Calculation of Tax Liability after Tax Credit
Tax Liability before Any Tax Credit 200,000
Less: Eligible Credit from Step 3 (133,333)
Tax Liability after Foreign Tax Credit 66,667

16) In what context “Average Rate of Nepal Tax” is relevant under the Income Tax Act? Define
the same. (Dec 2008, 5 Marks, CA Inter)
Answer:
As per Explanation of Section 71 of Income Tax Act, 2058, “Average Rate of Nepal Tax” is
relevant in the context of determining maximum limit of foreign tax credit available to a resident
person paying tax in foreign country in income generated from such country.
It is calculated by dividing tax liability of a person before foreign tax credit by taxable income for
the year of such person.

17) Mr. Bimal Shrestha has a source of income in Nepal as well as foreign countries. His net
income and tax paid in each foreign country during the income year 20X-62/X-63 is given
below:
Name of the Country Net Income (Rs.) Tax Paid (Rs.)
UAE 2,00,000 10,000
USA 4,00,000 1,20,000
Australia 3,00,000 60,000
Nepal 5,00,000 -

The Institute of Chartered Accountants of Nepal 178


CAP-II Paper 7 - Income Tax and VAT

Mr. Bimal Shrestha is a resident natural person and opted couple status for income tax
purpose. He decided to elect to get benefit of foreign tax credit. Calculate his tax liability for
the income year 20X-62/X-63. (Dec 2006, 8 Marks, CA Inter)

Answer
Step 1: Calculation of Tax Liability before Foreign Tax Credit
Assessable Income from Nepal 500,000
Assessable Income from UAE 200,000
Assessable Income from USA 400,000
Assessable Income from Australia 300,000

Total Assessable Income/ Taxable Income 1,400,000


Assuming there is no employment income
Tax Liability
1st Rs. 600,000 0% -
Next Rs. 200,000 10% 20,000
Next Rs. 300,000 20% 60,000
Next Rs. 300,000 30% 90,000
Tax Liability before FTC 170,000

Step 2: Calculation of Average Tax rate in Nepal


Tax Liability before Foreign Tax Credit (A) 170,000
Taxable Income in Nepal (B) 1,400,000
Average Tax Rate in Nepal [A/B * 100] 12.14%

Step 3: Calculation of Eligible Foreign Tax Credit for each country and excess, if any
UAE USA Australia Total
A Foreign Assessable Income 200,000 400,000 300,000
B Average Rate of Tax in Nepal 12.14% 12.14% 12.14%
C Average Tax Paid in Nepal (A * B) 24,280 48,560 36,420
D Actual Tax Paid in Foreign Country 10,000 120,000 60,000
E Excess Taxes of Previous Income Year (s) - - -
F Sum of D & E 10,000 120,000 60,000
G Eligible Amount [Lower of “C” or “F”] 10,000 48,560 36,420 94,980
H Excess to be carried forward [F- G] - 71,440 23,580 95,020
Computation of Net Tax Liabilities
Tax Liability before FTC 170,000
Less: Foreign Tax Credit 94,980
Net Tax Liabilities 75,020

The Institute of Chartered Accountants of Nepal 179


CAP-II Paper 7 - Income Tax and VAT

18) Mr. A receives Royalty Rs. 25,00,000 from Nepal on which 15% taxes deducted at source.
Mr. A occasionally visits Nepal and during the year 20X-59/X-060 he stays in Nepal for 100
days. He is a resident of USA and he has to pay tax in USA. Can he claim the tax he has to
pay in USA to be set off against the tax deductible in Nepal and claim refund as that is the
only income that he receives in Nepal? He has to pay a tax at 35% in the USA in the Royalty
income earned. (Dec 2004, 5 Marks, CA Inter)

Answer:
Mr. A is non-resident of Nepal since his habitual place is in the USA, he stays in Nepal for less
than 183 days during the Income Year and he is not employee of GoN deputed by GoN in foreign
country, and as per sec. 6, a non-resident has to pay tax on Nepal sourced Income.
As per Sec. 71, a resident person is entitled to foreign tax credit in case the person has paid tax in
foreign county on its foreign sourced income.

As such, Mr. A cannot claim foreign tax credit in Nepal, and he has to pay tax @15% on royalty
income having source in Nepal, which is treated as final withholding u/s 92 of the Act.

19) Write short notes on “Foreign Tax Credit”. (Dec 2003, 3 Marks, CA Inter)
Answer:
Foreign tax credit is available to a resident person that generates income having source outside
Nepal and pays tax in such source country.
The maximum amount of foreign tax credit shall not exceed the amount determined by multiplying
foreign assessable income by average rate of tax in Nepal, and such maximum amount shall be
calculated separately for each country.
Any excess foreign tax payment during a country is carried forward to be claimed during
subsequent income years.

The Institute of Chartered Accountants of Nepal 180


CAP-II Paper 7 - Income Tax and VAT

Chapter 14: Withholding Tax, Installment Tax and Advance Tax

1) Mr. Komal purchased a piece of land at Rs. 30 lakhs. He sold the land at Rs. 45 lakhs. He
paid registration expenses Rs. 2 lakhs for this land. In this case, what would be the tax
implications on the following situations? (Dec 2014, 5 Marks)

Answer:
The gain is Rs. 13 Lakhs (i.e. 45 lakhs minus 30 Lakhs minus 2 Lakhs)

a. The land was purchased on Chitra 20X-64 and sold it on Magh 20X-70.
Answer:
The land is owned for more than 5 years, as such, the tax on gain is 5%, to be paid at Land Revenue
Office at the time of ownership transfer.

b. The land was purchased on Magh 20X-67 and sold it on Magh 20X-70.
Answer:
The land is owned for less than 5 years, as such, the tax on gain is 7.5%, to be paid at Land Revenue
Office at the time of ownership transfer.

c. The land was purchased on Chitra 20X-59 and sold it on Baishak 20X-71.
Answer:
The land is owned for more than 5 years, as such, the tax on gain is 5%, to be paid at Land Revenue
Office at the time of ownership transfer.

d. If selling and buying of the land were completed through a sole shareholder of a Pvt. Ltd.
The shareholder is Mr. Komal.
Answer:
The company would have paid advance tax at Land Revenue Office @ 1.5% of sales value of land,
which is then treated as advance tax when determining the corporate tax liability of the company.

2) A telecommunication company has the following transactions with its VAT registered
recharge card dealers:
i) The company sold its recharge (recharge card and MPOS) at Discount on MRP to the
authorized recharge dealers instead of commission for sale to ultimate subscribers.
Additional commission is also provided for the recharge dealers on the basis of
yearly/monthly target.
ii) In addition, the company also sold Sim/Ruim card through the dealers on based of service
fee and sales commission. The company provides Rs.50 for each sim card sold and terms
of payment are:

The Institute of Chartered Accountants of Nepal 181


CAP-II Paper 7 - Income Tax and VAT

Rs. 40 on submitting the registration form of subscribers


Rs. 5 on verification of registration form of subscribers
Rs. 5 on first top up done by subscriber

Discuss the applicability of withholding taxes in the above case with suitable
characterization. (June 2022, 5 Marks)

Answer:
As per the sec 88 of Income Tax Act, 2058 where a resident person pay commission, sales bonus
having source in Nepal , the person shall withhold tax on the gross amount of the payment at the
rate of 15%.
Provided that tax deduction shall be made at the rate of 1.5% on the payment made to service
provider person registered in VAT.
i) As the telecommunication company sold its recharge (recharge card and MPOS) at Discount
on MRP to the authorized recharge dealers instead of commission for sale to ultimate
subscribers. TDS shall not be applicable on trade discount. So the company shall not deduct
tax on such discount provided to the registered recharge card dealers.
Additional commission provided for the recharge dealers on the basis of yearly/monthly target
shall be characterized as sales bonus and the 15% tax shall be deducted on the same.
The same treatment has been confirmed by circulars has been given by IRD dated 2075.11.04
as well.

ii) As the company also sold Sim/Ruim card through the dealers on based of service fee and sales
commission, tax shall be deducted @1.5% on service fee portion. tax shall be deducted @15%
sales commission .
- Rs.40 per sim/ruim card sales shall be characterized as service fee, as the recharge card Dealers
provide necessary service to make the sales.
- Remaining Rs.10 characterized as sales commission, as the recharge card dealers does nothing on
that payment portion.

3) As per provisions of Income Tax Act, 2058, mention the applicable withholding tax rate and
classify them as final or advance withholding tax in case of the following transaction of the
financial year 2077/78 as per the provision of Income Tax Act 2058. Assume that
transactions are between person resident in Nepal unless clearly mentioned in the question:
i) RST Limited paid Rs. 100,000 as sales commission to PQR Ltd., a company not resident
in Nepal.
ii) PQR a commercial bank paid interest of Rs. 1,000,000 on deposit to a Life Insurance
Company.
iii) ABC college of Kathmandu paid Rs. 900,000 to University of Philippines for registration,
education fee and exam fee of 9 students studying course offered by that university.

The Institute of Chartered Accountants of Nepal 182


CAP-II Paper 7 - Income Tax and VAT

iv) Tax incentive amount for consumer who paid their bill through electronic payment
instruments like payment card, e-money (wallet), mobile banking on their purchase.
v) CP Pvt. Ltd. paid annual office space rent Rs. 1,500,000 to PT Ltd.
(Dec 2021, 5 Marks)
Answer:
As per provisions of Income Tax Act, 2058, applicable withholding tax rate and its classification
as final or advance withholding tax in case of the given transactions are as follows:
TDS or Not Rate Final withholding or not
i) Yes 15% Final Withholding
ii) Yes 15% Not Final Withholding
iii) Yes 5% Final Withholding
iv) No - -
v) Yes 10% Creditable

4) Discussing the relevant provisions of Income Tax Act, 2058, and Rules 2059, advise whether
the following transactions attract withholding of tax or not? If yes, mention the withholding
tax rates and whether it is final or adjustable. (June 2021, 5 marks)
i) Commission paid by the resident employment company to non-resident person.
ii) Stall charge paid to Trade Fair India by Nepal Craft Paper Ltd. to participate in Indian
National Trade Fair 2020.
iii) Payment for carriage service to vehicle provider in rent for carriage service.
iv) Interest paid by bank to tax exempt NGO.
v) Premium paid by resident person to non-resident insurance company for life insurance.

Answer:
i) Advance tax withholding at the rate of 5% is required for the commission paid by the
resident employment company to non-resident person and this is final withholding.
ii) No tax withholding required for payment made to book stall in foreign trade fairs. (IRD
advance ruling to Laxmi Bank Limited dated 2072.06.28)
iii) Tax withholding at the rate of 1.5% if the payment is against VAT invoice otherwise 2.5%.
Final withholding to natural person and adjustable to others.
iv) At the rate of 15% which is final withholding.
v) At the rate of 1.5% which is final withholding.

5) What is the applicable withholding tax amount on the following: (June 2019, 5 Marks)
a. Service fees of Rs. 50,000 paid for VAT exempted services.
Answer- 1.5% of Service fee amount if payment is made to resident entity dealing in VAT
exempted services. Otherwise, TDS is at 15%.
Withholding tax Amount= 1.5% of Rs. 50,000 = Rs. 750

The Institute of Chartered Accountants of Nepal 183


CAP-II Paper 7 - Income Tax and VAT

b. ABC Limited paid house rent Rs. 100,000 to landlord Mr. Sharma.
Answer: As per definition of "Rent", natural person's house rent income that is not generated
through a private firm does not fall under the definition of rent, which means such income is
exempt from income tax.
As per Sec. 88 (4), there is no withholding tax requirement on exempt income.

c. MNO Ltd. paid Rs. 50,000 as freight charges to Sulabh Transport Co.
Answer: 2.5%
Withholding tax Amount= 2.5% of Rs. 50,000 = Rs. 1,250

d. Bank of Kathmandu paid Rs. 500,000 as interest in fixed deposit account maintained by
Sagarmatha Mutual Funds.
Answer: As per Sec. 88 (4(ga)), there shall be no withholding of taxes in case of income is
exempted u/s 10. Income of mutual fund is exempted u/s 10.

e. ABC Ltd. provides car worth Rs. 2,500,000 to the winner in the lottery scheme of the Co.
Answer: As per Sec. 88Ka, the withholding tax rate is 25%, and the amount is 25% of Rs. 25
Lakhs, i.e Rs. 6.25 lakhs.

6) Write the tax implications on the following cases as per Income Tax Act, 2058: (Dec 2018, 5
Marks)
a. Smooth Telecom Pvt. Ltd. deposits all gratuity amounts into an Approved Retirement
Fund. On 30 Bhadra, 20X-75, the Fund paid for gratuity Rs. 30 lakhs to Mr. Ram upon
termination of his job.
Answer:
It is assumed that the employers operate employers-single-account in Approved Retirement Fund
in order to deposit gratuity, in which case, TDS is applicable on payment of gratuity to Ram.
Smooth Telecom is withholding agent and it needs to withhold tax @ 15% of gratuity paid, i.e.
Rs. 450,000.

For Ram, it is a final withholding payment. This is the case of retirement payment from non-
contributory fund.

b. Mr. Shayam owns a truck for providing transportation services. He is paying annual tax
as per Income Tax Act, 2058. During Ashwin 20X-75, he has not issued any invoice but
has earned transportation services fee amounting to Rs. 2 lakhs.

The Institute of Chartered Accountants of Nepal 184


CAP-II Paper 7 - Income Tax and VAT

Answer:
The income from truck operation is subject to income taxes under the head income from business.
Person (if entity or business of natural person) making payment of transportation fee to Mr. Shyam
shall deduct TDS @2.5%.

c. Kathmandu Metropolitan City (KMC) has an agreement with Kalimati Users Committee
for drain construction; the agreement was made on 30 Baishakh, 20X-75. KMC paid Rs.
70 lakhs to the Committee on 10 Ashwin, 20X-75 after completion of the works.
Answer:
As per Sec. 89 (3Ka), when any work is operated through Consumer (User) Committee and where
the payment exceeds Rs. 50 Lakhs, tax shall be withheld by payer @ 1.5% of payment, i.e., Rs.
105,000.

d. An NGO having tax exempt certificate, has booked income from house rent Rs. 500,000
during the year 20X-74/X-75.
Answer
The house rent income is subject to corporate tax of NGO, as it is not an exempt amount under the
conditions of Sec. 10 (Chha). Person (if entity or business of natural person) making payment of
rent to NGO shall deduct TDS @10%.

e. A Pvt. Ltd. has booked as income from sale of Car Rs. 300,000 on Ashwin, 20X-75. The
Company is registered with the objectives of providing transportation services.
Answer
The car is a depreciable asset of A Pvt. Ltd., the amount from sale of car is deducted while
calculating depreciable basis of Block C Assets.

7) Mr. Ramji Paneru retired in 2075 Baishakh end from 30 years of his service with Nepal
Timber Corporation and received lump sum retirement payments as follows: (June 2018, 5
Marks)
a. Payment from approved retirement fund Rs. 1,675,852.00.
b. Payment from unapproved retirement fund Rs. 675,800.00 to which his contribution was
Rs. 511,256.00. This payment is related only with the period after Income Tax Act, 2058.
You are required to calculate the tax withholding on these payments along with reason there
for.

Answer:
a. To end up with 30 years of service, Mr. Paneru had joined the service from 2055 Jestha 1. That
means, he earned retirement benefit from 2055 Jestha 1 to 2058 Chaitra 18 and thereafter. Out
of 360 months, payment in relation to 46.5 months is tax exempt.
Exempt payment = 1,675,852/360*46.5 = 216,464

The Institute of Chartered Accountants of Nepal 185


CAP-II Paper 7 - Income Tax and VAT

Taxable Portion= Rs. 1,459,388


Deduction of 50% of taxable portion or Rs. 500,000 whichever is higher
Gain = Rs. 729,694
Tax @ 5% on Gain = Rs. 36,484

b. Amount of Gain from Unapproved Retirement Fund is Rs. 164,544 (675,800-511,256). Tax is
5% of Gain, i.e. Rs. 8,227.20.

8) Calculate the TDS to be deducted on following cases: – (June 2017, 5 Marks)


a. Mr. Santosh Sharma has bought a house on 20th Asoj 20X-67 for Rs. 9,000,000 including
registration expenses and other expenses. Again, he has purchased a plot of land on 3rd
Chaitra 20X-70 for Rs. 15,000,000 including registration expenses and other expenses.
He sold both of the property on 22nd Mangsir 20X-73 to Mr. Binod Sharma for Rs.
3,00,00,000 and Rs. 3,50,00,000 respectively.
Stating the relevant provisions of the Income Tax Act, 2058, advise whether TDS needs
to be deducted or not in the said transaction, and if yes, advise who will deduct the TDS
and amount of TDS to be deducted on the above transaction.

Answer
Withholding of tax is not applicable on disposal of Non-Business Chargeable asset, as tax
withholding is dealt by Sec. 87- 89 of the Act and such sections do not cover gain on disposal of
non-business chargeable asset.

However, Sec. 95Ka, Collection of Advance Tax by Agent, is applicable in such circumstances,
in which case if a natural person disposes his land and building qualifying as non-business
chargeable asset, advance tax must be paid at Land Revenue Office at the time of property
registration. The rate of advance tax is 7.5% in case of ownership period less than 5 years and 5%
in all other cases.

In the given case, property bought on 20th Ashoj 20X-67 is sold after 5 years, which means gain
on disposal of such property is subject to payment of advance tax @ 5%. Gain amount is Rs.
21,000,000 and payment of advance tax is Rs. 1,050,000.

Further, property bought on 3rd Chaitra 20X-70 is sold before 5 years, which means gain on
disposal of such property is subject to payment of advance tax @ 7.5%. Gain amount is Rs.
20,000,000 and payment of advance tax is Rs. 1,500,000.

b. Mr. Shyam Silwal and Everest Holdings Pvt. Ltd. are shareholders of Shree Holdings
Pvt. Ltd. Mr. Shyam Silwal holds 5,000 shares and Everest Holdings Pvt. Ltd. holds

The Institute of Chartered Accountants of Nepal 186


CAP-II Paper 7 - Income Tax and VAT

10,000 shares of Shree Holdings Pvt. Ltd. which is 10% and 20% shares of total shares
of Shree Holdings Pvt. Ltd. respectively.
Mr. Shyam Silwal has purchased said share from Mr. Naresh Rana on 25 Mangsir 20X-
65 for Rs. 22,50,000. He sold the said share on 25 Poush 20X-73 for Rs. 30,00,000. He has
paid interest of Rs. 5,00,000 against loan taken for purchase of the said share.
Everest Holdings Pvt. Ltd. is a promoter shareholder of the Shree Holdings Pvt. Ltd.,
and has paid Rs. 10,00,000. Everest Holdings Pvt. Ltd. sold the said share on 25 Poush
20X-73 for Rs. 60,00,000. Everest Holdings Pvt. Ltd. has purchased the said share from
their own capital.
Stating the relevant provisions of the Income Tax Act, 2058, advise Shree Holdings Pvt.
Ltd. whether TDS needs to be deducted or not in the said transaction, and if yes, compute
TDS amount to be deducted by Shree Holdings Pvt. Ltd. on the above transaction.

Answer
Withholding of tax is not applicable on disposal of Non-Business Chargeable asset, as tax
withholding is dealt by Sec. 87- 89 of the Act and such sections do not cover gain on disposal of
non-business chargeable asset.

However, as per Sec. 95Ka, advance tax is applicable on transaction of securities. The rate of
advance tax is 10% of gain in case securities of non-listed entities are held by resident natural
person; 15% of gain in case securities of non-listed entities are held by resident entity and 25% of
gain for others (including a non-resident person).

Applicability of Tax
Gain on Disposal for Mr. Shyam Silwal= Incomings – Outgoings = Rs. 30 Lakhs – Rs. 27.50 Lakhs
= Rs. 2.5 Lakhs (Interest expense on loan for purchase of shares form part of outgoings)
Advance tax to be collected by Shree Holdings Pvt. Ltd. = 10% of Gain = Rs. 25,000 (assuming
Mr. Shyam is resident, otherwise it is 25% of gain, i.e. Rs. 62,500)

Gain on Disposal for M/s Everest Holdings Pvt. Ltd.= Incomings – Outgoings = Rs. 60 Lakhs –
Rs. 10 Lakhs = Rs. 50 Lakhs
Advance tax to be collected by Shree Holdings Pvt. Ltd. = 15% of Gain = Rs. 750,000 (assuming
the company is resident, otherwise it is 25% of gain, i.e. Rs. 1,250,000)

9) Calculate tax on source (TDS) amount, if applicable, for 20X-72/X-73 as per Income Tax Act,
2058. (Dec 2016, 5 Marks)
a. Manhood Pvt. Ltd. has borrowed loan amounting of Rs. 1 crore from Boston
International Pvt. Ltd (Non-resident) and paid interest Rs. 10 lakhs.
Answer
The TDS rate is 15% and the TDS amount is Rs. 150,000 (Sec. 88 (1))

The Institute of Chartered Accountants of Nepal 187


CAP-II Paper 7 - Income Tax and VAT

b. Kalika Mohan JV has distributed its profit amounting of Rs. 10 lakhs.


Answer
A joint venture is a company under the definition of Income Tax Act, and when a resident company
distributes dividend, it has to withhold tax at the rate 5% of distributed amount as per Sec. 88 (2).
Therefore, TDS amount is Rs. 50,000.

c. UNICEF Nepal (Tax Exempt Entity) has paid vehicle rent amounting of Rs. 200,000 to
the Flexible transport Private Ltd.
Answer
Assuming Flexible Transport is not registered for VAT, UNICEF shall withhold tax @ 10% as per
Sec. 88 (1). Therefore, UNICEF Nepal shall withhold Rs. 20,000 as tax.

Assuming the vehicle was hired for the purpose of transportation of goods, the withholding tax
would be 2.5% (not registered for VAT) of Rs. 200,000.

Note that, Tax exemption is only related to relief from payment of income taxes, not from the
obligation to withhold taxes.

d. Amount paid by an unapproved retirement fund Rs. 15,00,000 to it’s a contributor. Total
amounting Rs. 12,00,000 has been deposited by the contributor to the retirement fund
during the period of 12 years.
Answer
The gain is Rs. 300,000 (15 lakhs – 12 lakhs) and it is subject to Withholding tax @5%, i.e. Rs.
15,000 shall be withheld by Unapproved Retirement Fund u/s 88 (2).

e. Complete Private Ltd. has paid rental charges amounting of Rs. 1,200,000 to the
Employee Provident Fund in 20X-72/X-73.
Answer
Income of an approved retirement fund is exempt from tax. As per Sec. 88 (4), there shall be no
withholding of taxes while making payments of amounts exempt from tax. Therefore, there shall
be no withholding of taxes.

10) Mr. Pravan has got retirement on Ashad end, 20X-72 from government service after
completing 12 years. The approved and unapproved retirement fund has providing 10
percent interest rate on its contribution. The interest is providing monthly basis.
He has received statements from these funds, that statements show the following balances as
on first Shrawan, 20X-72.
• Approved fund: 1,500,000 including total interest of Rs. 300,000
• Unapproved fund: Rs. 800,000 including total interest of Rs. 150,000

The Institute of Chartered Accountants of Nepal 188


CAP-II Paper 7 - Income Tax and VAT

He came with you for the payment of the balance amount on 30th Ashoj, 20X-72. Assume you
are the finance officer; calculate the TDS on these payments and mention why have you
deducted the TDS? (Dec 2015, 5 Marks)

Answer
a. Payment from Approved Retirement Fund= Rs. 1,500,000
Deduction of 50% of taxable portion or Rs. 500,000 whichever is higher
Gain = Rs. 750,000
Tax @ 5% on Gain = Rs. 37,500
Tax is withheld as per Sec. 88 (1).

b. Amount of Gain from Unapproved Retirement Fund is Rs. 150,000 (interest portion). Tax is
5% of Gain, i.e. Rs. 7,500. Tax is withheld as per Sec. 88 (2).

11) Mr. Ramesh has been retired from Government of Nepal on 15th Jestha, 20X-71. He has
received the following retirement payments in Ashad, 20X-71. He has not any other sources
of income except salary. Remuneration tax already has been deducted and deposited.
Assume no retirement payment was accrued at the commencement of this Act in connection
with this employment. Calculate the amount of tax that must be paid by Mr. Ramesh. (Dec
2014, 5 Marks)
Particulars Amount (Rs.)
Payment from GON against accumulated leave and medical Rs. 6,00,000
allowances
Payment from Employee Provident Fund against contribution Rs. 7,00,000
Payment from Citizen Investment Trust against contribution Rs. 5,00,000

Answer
Total Payment from Approved Retirement Fund- Rs. 1,800,000
Deduction of 50% of payment or Rs. 500,000; whichever is higher
Gain = Rs. 900,000
Tax to be paid Mr. Ramesh = 5% of Gain = Rs. 45,000

12) Explain the following with reasons; on tax/withholding taxes and implication thereon, with
reference to Income Tax Act, 2058. (Dec 2013, 5×2=10)
a. Z & Co. has taken loan from M/s Nabil Bank Ltd. amounting Rs. 10 crores for conducting
its business. The company has paid interest at the rate of 10% amounting to Rs. 1 crore
during income year 20X-69/X-70.

The Institute of Chartered Accountants of Nepal 189


CAP-II Paper 7 - Income Tax and VAT

Answer:
As per Sec. 88 (4), there shall be no withholding of taxes when interest is paid to a resident banks
or financial institutions. Assuming Nabil Bank is resident of Nepal, Z & Co. shall not withhold tax
while making payment of interest.

b. Mr. Gokul has given a contract to Z & engineers, a construction company for
construction of his residential building for Rs. 1 crore. While making payments to Z &
engineers, Mr. Gokul does not deduct any withholding taxes.
Answer
As per Sec. 88 (4), a natural person shall not withhold tax in any payment except when it is related
to operation of business. In the given case, Mr. Gokul is not making payment in connection to
operation of business, as such, there shall be no withholding of taxes.

c. Ms. Rakhi has taken the insurance policy of Rs. 16,00,000 from Rastriya Beema
Sansthan. She pays annual premium of Rs. 80,000. After maturity of the policy, she is
receiving Rs. 24,00,000 from Rastriya Beema Sansthan.
Answer
Assuming that Mr. Rakhi has paid total premium of Rs. 1,600,000 which in turn gives us gain from
investment insurance of Rs. 800,000. Tax withholding @ 5% is applicable on payment of gain by
resident investment insurance company, that means, tax to be withheld by Rastriya Beema
Sansthan is Rs. 40,000 as per Sec. 88 (2).

d. Mr. Smart has won the cash lucky draw of Rs. 1 million from a trading company under
its scheme. He claims that the amount is not subjected to any tax deduction and wants to
receive full payment.
Answer
Winning a lottery is windfall gain, and windfall gain is subject to 25% withholding taxes as per
Sec. 88Ka. As such, trading company shall withhold Rs. 250,000 and pay remaining balance to
Mr. Smart.

e. X & Co. made the following payment to Z & Co., under a contract, for the supply of
materials required for its company. Both the companies are registered under Value
Added Tax (VAT).
Date Amount (Rs.)
20X-69.04.08 25,000
20X-69.04.12 10,000
20X-69.04.13 5,000
20X-69.04.15 10,000
20X-69.04.17 5,000

The Institute of Chartered Accountants of Nepal 190


CAP-II Paper 7 - Income Tax and VAT

Since the payments are less than Rs. 50,000, X & Co. claims no deduction of withholding taxes
on such payments.

Answer
As per Sec. 89, where payment related to a contract exceeds Rs. 50,000 after considering all
payments of moving 11 days, tax shall be withheld @ 1.5% of payment.
In the given case, there is no requirement of withholding taxes on 20X-69 Shrawan 8, 12, 13 and
15 as the payment in moving 11 days is Rs. 25,000, Rs. 35,000, Rs. 40,000 and Rs. 50,000
respectively.

However, payment exceeds Rs. 50,000 on Shrawan 17, total payment in moving 11 days is Rs.
55,000 and provision of Sec. 89 is applicable. Since, no payments before Shrawan 17 is subject to
withholding taxes, tax shall be withheld on Rs. 55,000 @ 1.5% on Shrawan 17.

13) X & Co. has received from Y & Co., a resident company, Rs. 10 lakhs as dividend after
deduction of taxes. The profit of X & Co. before inclusion of such dividend income is Rs. 90
lakhs. X & Co. has decided to declare the dividend distribution to its shareholders during
current fiscal year amounting to Rs. 100 lakhs (including Rs. 10 lakhs of dividend received
from Y & Co.). Mr. Bibhu is one of the shareholders of X & Co. and supposed to receive
from Rs. 5 lakhs as dividend. Mention the relevant provision and calculate the amount of
Tax to be deducted while making payment to Mr. Bibhu. (Dec 2013, 5 Marks)

Answer:
As per Sec. 54 (3), there shall be not be applicability of dividend tax on distribution of such profit,
on which dividend tax as per Sec. 54 is already levied.

In the given case, dividend is distributed out of total profit of Rs. 10 Million, of which Rs. 1 Million
is dividend received from Y & Co., i.e. 10% of total distributable profit is dividend received from
another resident company subject to application of Sec. 54 (3) and hence, tax free.

Out of dividend received by Mr. Bibhu, 10% of dividend is tax free, i.e. Rs. 50,000 is tax free.
Dividend tax is applicable on Rs. 450,000 @ 5%. As such, amount of tax to be withheld u/s 88 (2)
is Rs. 22,500.

14) Explain the followings with reasons; on tax/withholding taxes and implication thereon, with
reference to the Income Tax Act, 2058. (June 2013, 5×2=10)
a. X & Co., Chartered Accountants, has entered into a contract for tax consultancy services
to Z Ltd. for Rs. 200,000 per month. X & Co. raised VAT invoice for the month of
Shrawan, 2068 amounting to Rs. 226,000 (Rs. 200,000+13%VAT = Rs. 226,000).

The Institute of Chartered Accountants of Nepal 191


CAP-II Paper 7 - Income Tax and VAT

Answer
As per Sec. 88 (1), payment of service fee to VAT registered service provider is subject to
withholding tax @ 1.5% of payment. Hence, tax to be withheld in this case is 1.5% of Rs. 200,000
(VAT is payment to Government and not to service provider), i.e., Rs. 3,000.

b. Mr. Hari, an individual person, owned the furnished building situated at Kalanki,
Kathmandu. He has given on lease the furnished building to a development bank for Rs.
40,000 per month from Shrawan 20X-68. Mr. Hari is not conducting any business and
has rent income only.
Answer
As per definition of "Rent", natural person's house rent income that is not generated through a
private firm does not fall under the definition of rent, which means such income is exempt from
income tax. As per Sec. 88 (4), there shall not be tax withholding when an income is exempt from
tax.

c. Mr. Krishna is an economist. He published an article on "Current Status of Economy"


in Kantipur daily and received Rs. 6,000. He also teaches, occasionally, "Economics" in
an educational institution for which he received Rs. 10,000 for a class in Income Year
20X-68/X-69.
Answer
As per Sec. 88 (4), there shall be no withholding of tax in respect of payment by newspaper in
relation to articles published in such newspaper. Therefore, Kantipur daily shall not withhold tax
on honorarium to Mr. Krishna for his article title “Current Status of Economy”. However, Mr.
Krishna’s such income is not exempt from income tax.

Further, occasional lecture fee is service fee is subject to withholding tax @ 15%. Tax to be
withheld by Educational Institution is Rs. 1,500. It is final withholding payment for Mr. Krishna.

d. Mrs. Sila has opened the account in Paschimanchal Rural Development Bank, Parbat
and deposited Rs. 60,000. The interest rate is 8% per annum and Mrs. Sila earned interest
income of Rs. 4,800 during Income Year 20X-68/X-69.
Answer
As per Sec. 11 (2Ka), income tax is exempt on interest income up to Rs. 25,000 earned on deposit
by a natural person during the income year from a rural development bank based on Rural
Municipality. Assuming, the bank in question is located at Rural Municipality, the interest income
of Mrs. Sita is tax exempt, and is not subject to withholding tax as per Sec. 88 (4).

e. A & Company Ltd. decided to declare and distribute the cash dividend at the rate of 10%
amounting to Rs. 50,000 and bonus shares at the rate of 20% amounting to Rs. 100,000

The Institute of Chartered Accountants of Nepal 192


CAP-II Paper 7 - Income Tax and VAT

out of the profit of Income Year 20X-67/X-68, from its annual general meeting dated
Mangsir 28, 20X-68. Determine the amount of tax to be withheld on payment.
Answer
As per Sec. 88 (2), tax shall be withheld @5% on distribution of profit by the company. Total tax
to be withheld is 5% of Rs. 150,000, i.e. Rs. 7,500.

15) Anubhav & Co. has taken on rent the premises from Buddhi Shree & Co for the operation
of its business which pays monthly rent of Rs. 100,000. Anubhav & Co. is deducting
withholding tax at the rate of 10% amounting to Rs. 10,000 per month and making payment
amounting to Rs. 90,000 per month to Buddhi Shree & Co. While submitting Income Tax
Return, tax liability of Buddhi Shree & Co, to be payable during Income Year 20X-68/X-69
came to Rs. 200,000. Answer the following question mentioning the relevant provision of
Income Tax Act, 2058. (Dec 2012, 8 Marks)
a. What is the amount of tax to be deposited?
Answer
As per Sec. 93, M/s Buddhi Shree & Co. can utilize taxes withheld by agent and not in respect of
final withholding payment as tax credit while determining the tax payable for the Income Year.
As such, Rs. 120,000 can be offset against the total tax liability of Rs. 200,000 and M/s Buddhi
Shree & Co. shall pay additional Rs. 80,000 to discharge its tax obligation.

b. Will your answer be different if tax liability of Buddhi Shree & Co comes to Rs. 100,000
instead of Rs. 200,000?
Answer
Yes, in that case M/s Buddhi Shree & Co. has more tax credit than the tax obligation and there
would be no additional outflow of cash to discharge tax obligation. Further, company has unused
advance tax of Rs. 20,000 which can be set-off with tax liability of next income year.

c. What is the implication if Buddhi Shree & Co had not adjusted the amount of
withholding tax paid through Anubhav & Co. while submitting income tax return?
Answer
As per Sec. 93, the amount of withholding taxes shall be adjusted while submitted income return
during the year when the related income is included in the return. If not adjusted, the amount
cannot be adjusted in any other year.
However, the taxpayer has the option to apply for tax refund under the provisions of Sec. 113.

16) Mr. Ram, an employee of a local bank accepted the offer of Voluntary Retirement Scheme
(VRS) as part of the Bank's employee reduction plans. As a VRS package, the bank has
decided to provide 2 months basic salary for every completed year of service. Mr. Ram has
taken retirement as part of VRS scheme. As per the rules of the Bank, employees serving
more than 15 years are entitled for 2 month's basic salary for every completed year of service

The Institute of Chartered Accountants of Nepal 193


CAP-II Paper 7 - Income Tax and VAT

as gratuity and employees serving more than 20 years are entitled for 2.5 month's basic
salary for every completed year of service as gratuity. He has taken retirement in the month
of Ashad 20X-68 and at the time of retirement, his basic salary was Rs. 20,000 per month.
His other monthly allowance is Rs. 10,000 per month. During the year 20X-67/X-68, he also
received one month's basic salary as Dashain Allowance. The bank has also contributed 10%
of his basic salary as Provident Fund contribution. At the time of retirement, his total service
period was 25 years. The gratuity amount accrued and payable to him upto Chaitra 18, 2058
is Rs. 950,000 and the amount of leave payable to him upto Chaitra 18, 2058 is Rs. 50,000.
The number of days leave accrued after Chaitra 18, 2058 and payable on Ashad 20X-68 is
90 days. Compute his Income from Remuneration for the year 20X-67/X-68 and also
calculate the amount of Retirement Payments as per the provision of the Act. What rate of
withholding tax the bank should apply for payments of VRS, Gratuity and Leave at the time
of Retirement and also the amount of Tax to be deducted for payments of VRS, Gratuity and
Leave at the time of Retirement?

On the other hand, Mr. Hari is a low skilled worker and he has been terminated from the
company as part of Compulsory Retirement Scheme (CRS) and for this he got Rs. 400,000
from the Bank and also as part of CRS package, he commuted the amount of Pension and
lump sum pension of Rs. 3,00,000 has been paid to him at the time of Retirement. Compute
the withholding tax to be deducted in this case by also mentioning the withholding rate in
both the cases? (June 2012, 10 Marks, CA Inter)

Answer
Assessable Income from Employment Mr. Ram
Particulars Sec. Ref. Amount Notes
Basic Salary 8 (2) 240,000
Allowances 8 (2) 120,000
Dashain Allowance 8 (2) 20,000
Employers Contribution to RF 8 (2) 24,000 10% of basic salary
Assessable Income from 404,000
Employment

Tax to be deducted at the time of Retirement of Mr. Ram


Particulars Details Total PayTDS Rate TDS
Amount
VRS after 25 2 months basic 50 X 20,000= Rs. 15% Rs.
years of service salary each year for 1,000,000 150,000
25 years

The Institute of Chartered Accountants of Nepal 194


CAP-II Paper 7 - Income Tax and VAT

Gratuity after 2.5% months basic 62.5 X 20,000 - 15% Rs. 45,000
Chaitra 19, 2058 salary each year for 950,000 = Rs.
25 years 300,000
Gratuity after 950,000 Exempt -
Chaitra 19, 2058
Leave until 2058 50,000 Exempt -
Chaitra 18
Leave after 2058 90 days~ 3 months 3 X 20,000= Rs. 15% 9,000
Chaitra 18 60,000

Tax to be deducted at the time of Retirement of Mr. Hari


Particulars Total Pay TDS Rate TDS Amount
CRS payment 400,000 15% Rs. 60,000
Lump sum Pension 300,000 15% Rs. 45,000

17) Mention, with reasons, the amount of tax that need to be deducted at source/accounted for
in the books in following cases: (June 2012, 5×2=10 Marks)
a. M Ltd. has entered into an agreement with PC Pvt. Ltd. to conduct an awareness
program on maternal health in 5 districts. The total amount paid by M Ltd. against such
agreement was Rs. 40,000. In addition to such amount, M Ltd. also reimbursed Rs. 20,000
as expenses on air –ticket, lodging and fooding, daily expenses etc. of staffs of PC Pvt.
Ltd. to PC Pvt. Ltd. as per the terms of the agreement. PC Pvt. Ltd. did not give VAT
invoice.
Answer
Total service fee paid to PC Pvt. Ltd. is Rs. 60,000. As per Sec. 88 (1), service fee is subject to tax
withholding @ 15%, as such, M. Ltd. shall withhold tax amounting to Rs. 9,000.

b. M/S ABC Company Limited has 60 shareholders. The company has distributed iPhone
worth Rs. 38,000 to each shareholder free of cost in FY 20X-68/X-69.
Answer
In case a person makes payment in kind, the market value of such kind is treated as the amount of
payment, as per sec. 27.
Payment in the nature as given in this question, to shareholders is treated as dividend as per Sec.
53, as such, the distribution of iPhone is dividend. There are two possible way to collect
withholding taxes in the given case:
• If the company collects dividend tax from the shareholders, the amount to be collected from
each shareholder is Rs. 1,900 (5% of Rs. 38,000)
• If the company bears the cost of withholding taxes, the shareholders are deemed to receive Rs.
38,000 net of TDS; which means, Rs. 38,000 is 95% of total dividend, and the total dividend
comes to Rs. 40,000. The dividend tax would be Rs. 2,000 per shareholder.

The Institute of Chartered Accountants of Nepal 195


CAP-II Paper 7 - Income Tax and VAT

c. Mahesh is constructing a building for his personal residence. He has appointed Ramesh,
an engineer, to take care of the overall construction activities at monthly remuneration
of Rs. 30,000.
Answer
As per Sec. 88 (4), a natural person shall not withhold tax in any payment except when it is related
to operation of business. In the given case, Mr. Mahesh is not making payment in connection to
operation of business, as such; there shall be no withholding of taxes.

d. MNO Company Pvt. Ltd. has taken loan Rs. 10 lakhs from Hamro Bank Limited @ 12%
Per annum. MNO Company Pvt. Ltd. has paid loan service charge of Rs. 25,000 and
interest Rs. 90,000 during the year to the Hamro Bank Limited.
Answer
As per Sec. 88 (4), there shall be no withholding of taxes when interest is paid to a resident banks
or financial institutions. Assuming Hamro Bank is resident of Nepal, MNO Co. Pvt. Ltd. shall not
withhold tax while making payment of interest.

e. Mr. Ram is invited to deliver lecture on corporate finance by KBC College. KBC College
has paid Rs. 10,000 to Mr. Ram for delivering lectures, Rs. 3,000 for setting up model
questions and Rs. 5,000 for checking the answer-sheets.
Answer
All such activities are subject to withholding tax @ 15% as per Sec. 88 (1). Therefore, the amount
of withholding tax is Rs. 2,700 (15% of Rs.18,000)

18) Adishree & Co. is conducting business in Pokhara and has taken the office premises on rent
from Mr. Garibdash. According to agreement between them, Adishree & Co. is required to
pay the house owner NPR 50,000 per month. Further, Adishree & Co.has taken on rent a
vehicle for NPR 25,000 from Mr. Daman Thapa for its business purpose. Mr. Daman Thapa
has paid the annual tax as per Section 1 (13) of Schedule 1 of Income Tax Act.

State whether the TDS as per Income Tax Act, 2058 is to be deducted in above circumstances.
If yes, what is the amount to be deducted as TDS? (Dec 2012, 3 Marks)

Answer:
As per definition of "Rent", natural person's house rent income that is not generated through a
private firm does not fall under the definition of rent, which means such income is exempt from
income tax. As per Sec. 88 (4), there shall not be tax withholding when an income is exempt from
tax. As such, there shall be no withholding tax while making rental payment to Mr. Garibdash.

The Institute of Chartered Accountants of Nepal 196


CAP-II Paper 7 - Income Tax and VAT

As per Sec. 88, rental payment is subject to withholding tax @ 10%, as such; Adishree & Co. shall
withhold tax while making vehicle rental payment to Mr. Daman Thapa is subject to tax @ 10%.

19) What is the rate of TDS on following payments? Also explain whether such payments are
final withholding or not? (Dec 2011, CA Inter, 3 Marks)
a. Retirement benefit given by Nepal Government or by any approved retirement fund.
Answer: 5% of Gain, Gain is calculated by deducting higher of 50% of payment or Rs. 500,000
from the payment and payment is subject to final withholding u/s 92.

b. Commission payment by resident employment Company to a non-resident person


Answer: Withholding tax @ 5% u/s 88.

c. Rs. 12,000 interest paid by Postal Saving Bank


Answer: No withholding tax, as interest up to Rs. 25,000 is exempt u/s 11 (2Ka) if the interest is
paid to natural person.

d. Disposal of Non-Business Chargeable Assets (land and house owned since last 5 years and
6 months) by a natural person.
Answer: 5% of gain, to be paid as advance tax (not a withholding tax) in Land Revenue Office at
the time of transfer of ownership.

e. Gain from commodities future trading


Answer: 10% of gain, to be collected as advance tax (not as withholding tax) u/s 95Ka.

f. Dividend given by Cooperative Society having registered office in Banepa Municipality?


Answer: 5% of Dividend amount.

20) On the occasion of Deepawali 20X-67, Himal Group organized a nationwide song
competition and Mr. Ghanashyam wins a first prize of Rs. 5,00,000. The organizer of the
program wants to deduct TDS @ 25% on the prize amount and pay the balance to him.
Mr. Ghanashyam was of the view that prize is related to the work of art and TDS is not
applicable on this payment. As a tax consultant, you are required to advice to Mr.
Ghanashyam whether TDS is applicable in this payment or not also mention the relevant
provisions of Income Tax Act 2058. (June 2011, 3 Marks)

Answer:
As per Sec. 88Ka, prizes of national or international level up to Rs. 500,000 is exempt from
windfall gain tax, only if it is received in recognition of somebody’s contribution in the field of
sports (or any other seven fields).

The Institute of Chartered Accountants of Nepal 197


CAP-II Paper 7 - Income Tax and VAT

In the given case, Mr. Ghanshyam has won the prize in competition and it is not awarded to
recognize his contribution, as such; the prize is subject to windfall gain tax @ 25%.

21) What would be the rate of TDS in case of following payments? (June 2011, 0.5×12=6, CA
Inter)
a. Dividend
Answer: 5%

b. Rent
Answer:
• House rent income of an individual that is not generated through operation of private firm-
exempt
• Vehicle rent paid to VAT registered person- 1.5%
• Transport rent to VAT registered person- 1.5%
• Transport rent to person not registered for VAT purpose- 2.5%
• All other cases than above: 10%

c. Commission
Answer:
• By a resident employment agency to non-resident- 5%
• All other cases- 15%

d. Retirement Benefit by approved agency / fund


Answer: 5% of gain

e. Rent paid on Leased Aircrafts


Answer: 10%

f. Interest to Natural Person by Bank & Financial Institutions


Answer: 5% in case it is not related to his/her business and 15% in all other cases

g. Commission to tax payer registered under VAT


Answer: 15%

h. Commission paid by Resident Employer to non-resident Person


Answer: 5%

i. Interest Paid by Person other than Bank & Financial Institution


Answer: 15%

The Institute of Chartered Accountants of Nepal 198


CAP-II Paper 7 - Income Tax and VAT

j. Payment of Benefit on Insurance Investment


Answer: 5%

k. Retirement Benefit by unapproved Fund / Agency


Answer: 5%

l. Windfall gains
Answer: 25%

22) Mr. X held 500 shares of ABC Ltd., a Listed Company, of Rs. 100 each. He acquired those
shares through a stockbroker by paying Rs. 700 each, and incurred Rs. 2,800 towards broker
commission. He sold all those shares through the broker for Rs. 3,500 each with the
brokerage commission of Rs. 8,750. Assuming that Mr. X is a non-resident, compute the gain
amount and withholding tax amount. (June 2010, 5 Marks)

Answer:
Particulars Shares Per share value Total (Rs.)
Incomings 500 3500 1,750,000
Outgoings
Original Cost 500 700 350,000
Brokerage on Purchase 2,800
Brokerage on Sales 8,750
Total Outgoings 361,550
Gain 1,388,450
Advance tax Collection rate 25%
Advance tax to be collected by NEPSE 347,113

23) M/s A. Ltd has entered into a contract with a party for supply of X items. M/s A. Ltd. has
made the payments to the supplier on following dates:
20X-66/4/8 Rs. 25,000
20X-66/4/12 (morning) Rs. 10,000
20X-66/4/12 (at 2 PM) Rs. 5,000
20X-66/4/12 (at 4 PM) Rs. 10,000
20X-66/4/17 Rs. 5,000
M/s A. Ltd contends that it need not deduct TDS on the payment made to the supplier since
each payment does not exceed Rs. 50,000. Give your opinion on this case. (June 2010, 5
Marks, CA Inter)

The Institute of Chartered Accountants of Nepal 199


CAP-II Paper 7 - Income Tax and VAT

Answer
As per Sec. 89, where payment related to a contract exceeds Rs. 50,000 after considering all
payments of moving 11 days, tax shall be withheld @ 1.5% of payment.

In the given case, there is no requirement of withholding taxes on 20X-66 Shrawan 8 and 12 as
the payment in moving 11 days is Rs. 25,000, and Rs. 50,000 respectively.

However, payment exceeds Rs. 50,000 on Shrawan 17, total payment in moving 11 days is Rs.
55,000 and provision of Sec. 89 is applicable. Since, no payments before Shrawan 17 is subject to
withholding taxes, tax shall be withheld on Rs. 55,000 @ 1.5% on Shrawan 17.

24) Calculate the amount of TDS to be deducted by a payer of the following payments as per the
provisions of the Income Tax Act, 2058. Also state whether the payments are final
withholding payments or not? (Dec 2009, 5 Marks)
a. M/S Lama Construction Pvt. Ltd. has received an invoice for Rs. 339,000 including VAT
from M/S Jurist Company Pvt. Ltd. for legal advisory services rendered during fiscal
year 20X-65/X-66.
Answer
The payment of service fee to VAT registered service provider is subject to Withholding tax @
1.5% of payment as per Sec. 88 (1). Therefore, Lama Construction shall withhold tax amounting
Rs. 4,500 (1.5% of Rs. 300,000).

This tax is not final withholding for M/s Jurist Company.

b. Amount paid by an approved retirement fund Rs. 15,00,000. The accumulated principal
and the interest thereon as on Chaitra 19, 2058 for the same amount was Rs. 4,00,000
Answer
The accumulated principal and the interest thereon as on Chaitra 19, 2058 is exempt from tax,
therefore, taxable amount is Rs. 1,100,000 (15,00,000-4,00,000).

Withholding tax rate is 5% of gain, as per Sec. 88 (1). Gain is calculated by deducting higher of
50% of taxable amount or Rs. 500,000 from taxable amount. Therefore, gain is Rs. 550,000, and
the withholding tax is Rs. 27,500.

The amount is final withholding in the hand of recipient.

c. Amount paid by an unapproved retirement fund Rs. 15,00,000. The accumulated


principal and the interest thereon as on Chaitra 19, 2058 for the same amount was Rs.
4,00,000. Total amount deposited by the account holder after Chaitra 19, 2058 to the
retirement fund Rs. 9,00,000.

The Institute of Chartered Accountants of Nepal 200


CAP-II Paper 7 - Income Tax and VAT

Answer
The accumulated principal and the interest thereon as on Chaitra 19, 2058 is exempt from tax,
therefore, the total payment amount subject to taxation is Rs. 1,100,000 (15,00,000-4,00,000).

Withholding tax rate is 5% of gain, as per Sec. 88 (2). Gain is Rs. 200,000 (Rs. 1,100,000 – Rs.
9,00,000). The withholding tax amount is Rs. 10,000.

The amount is final withholding in the hand of recipient.

d. Binod paid Rs. 50,000 as interest to Ram for loan taken from him. Binod has taken the
loan for his personal purpose.
Answer
As per Sec. 88 (4), a natural person shall not withhold tax in any payment except when it is related
to operation of business. In the given case, Mr. Binod is not making payment in connection to
operation of business, as such; there shall be no withholding of taxes.

However, Mr. Ram has to include the amount as part of investment income and pay tax as per
Income Tax Act.

e. A college is paying Rs. 20,000 to Mr. Ramesh for setting various question papers. Mr.
Ramesh is an employee of the college and getting regular remuneration.
Answer
The amount is to be included as part of employment income computation u/s 87 and tax shall be
withheld accordingly. It is not final withholding for Mr. Ramesh.

25) If a resident Airlines company makes payment to a non-resident person for aircraft
maintenance, at which rate TDS shall be deducted? If such payment is made for consultancy
services, will your answer differ? (Dec 2009, 2 Marks, CA Inter)

Answer:
As per Sec. 89 (3), if a resident person makes payment to a non-resident person under any contract
or agreement, tax shall be withheld @ 5%.

Sec. 88 is applicable when a resident person makes payments having source in Nepal. Service fee
is treated as having source in Nepal when the service is rendered to Government of Nepal, or when
the service is rendered in the state of Nepal. If consultancy service is rendered outside the state of
Nepal, there shall be no withholding of tax.

The Institute of Chartered Accountants of Nepal 201


CAP-II Paper 7 - Income Tax and VAT

26) Gain on commodity future market trading is subject to withholding tax or not? (Dec 2009, 2
Marks, CA Inter)

Answer:
No, it is subject to collection of advance tax by agent under Sec. 95Ka.

27) The following issues arise in connection with the deduction of tax at source under Chapter
17 of the Income Tax Act, 2058. Discuss the liability for tax deduction in these cases: (June
2009, 8 Marks, CA Inter)
Note:
Students should note that the specific requirement of this question is not clear. So, the answer may
be multiple, provided logically explained by students.

a. TDS to be deducted in the case of Non-Resident.


Answer
Where a resident person makes payments subject to Sec. 87-89, even when the payment is made
to non-resident, withholding of tax is applicable. These amounts are treated as final withholding
in the hand of non-resident persons.

However, a non-resident person cannot withhold tax u/s 87, 88 and 89. Whether a non-resident
can withhold tax u/s 88Ka is not specifically clear. However, compelling a non-resident to
withhold tax through literal interpretation of tax law is blunder and against the very premise on
which the concept of withholding taxes is introduced in tax law.

b. TDS in the case of co-operative society.


Answer
Where a resident person makes payments subject to Sec. 87-89, even when the payment is made
to non-resident, withholding of tax is applicable. Cooperatives are withholding agent and shall
withhold tax u/s 87-89 of the Act.

c. TDS in case of Contract.


Answer
Withholding of tax is required in case of contract payments within the scope of Sec. 89. Sec. 89
does not define what contract mean; however, it clarifies the types of contract on which
withholding of tax is required.

In case payment related to a contract exceeds Rs. 50,000 when the payment is aggregated over
moving 11 days, tax shall be withheld at the rate 1.5% of payments.

There shall be no double withholding of tax or no non-withholding of tax.

The Institute of Chartered Accountants of Nepal 202


CAP-II Paper 7 - Income Tax and VAT

d. Advance Tax Collection in case of gain on sale of securities of listed companies.


Answer
As per Sec. 95Ka, advance tax shall be collected by Nepal Stock Exchange Limited on gain on
sale of securities of listed companies. The rate of such tax is 5% and 7.5% (based on holding
period) for resident natural person, 10% for resident entity and 25% for non-resident persons.

28) As per the Loan agreement with the UK based company, AA has been assigned to provide
fund management service to Resident company BB. The BB Company has earned one million
rupees from the investment made during the year. Service fees paid to AA company for this
purpose is hundred thousand rupees.

Provide your opinion based on given facts based on the provisions of the Income Tax Act,
2058. The answer must also consist of implication as per the Act and opinion/
recommendation. (June 2007, 1.67 Marks, CA Inter)

Answer:
AA is a resident entity as it provides service to BB throughout the year, which qualifies AA as
Foreign permanent establishment. The income generated by AA is subject to corporate tax under
Income Tax Act. When BB makes payment of service fee to AA, it has to withhold tax @ 15%, if
AA is not registered for VAT and @ 1.5% where it is registered for VAT.

29) Mechi Noodles Pvt. Ltd. has made payments on various dates in the income year 20X-61/X-
62 to Kali Construction Company Pvt. Ltd. towards work done under a contract as follows
:
Date of Payment Amount (Rs.)
Ashad 11, 20X-62 15,000
Ashad 15, 20X-62 17,000
Ashad 17, 20X-62 3,000
Ashad 19, 20X-62 20,000

Mechi Noodles Pvt. Ltd. claims that it is not liable to deduction of withholding tax. Examine
the correctness of the claim made by the Company.

What would be the position if the payment on Ashad 19, 20X-62 is Rs. 5,000 only. (June 2006,
5 Marks, CA Inter)

The Institute of Chartered Accountants of Nepal 203


CAP-II Paper 7 - Income Tax and VAT

Answer
Part (a):
As per Sec. 89, where payment related to a contract exceeds Rs. 50,000 after considering all
payments of moving 11 days, tax shall be withheld @ 1.5% of payment.

In the given case, there is no requirement of withholding taxes on 20X-62 Ashad 11, 15 and 17 as
the payment in moving 11 days is Rs. 15,000, 32,000 and Rs. 35,000 respectively.

However, payment exceeds Rs. 50,000 on Ashad 19, total payment in moving 11 days is Rs. 55,000
and provision of Sec. 89 is applicable. Since, no payments before Ashad 19 since Ashad 9 is subject
to withholding taxes, tax shall be withheld on Rs. 55,000 @ 1.5% on Ashad 19.

Part (b)- what if, question


If the payment on Ashad 19 was Rs. 5,000 only, the total payment over moving 11 days (Ashad 9-
19) would be Rs. 40,000 only that is less than Rs. 50,000 and hence, withholding of tax would not
be applicable.

30) M/S 'X' Co. Ltd. (Hydro Power Generation Electricity Company) whose total turnover in
F.Y. 20X-59/X-60 is Rs. 150 crores and net profit before tax is Rs. 20 crores. The company
has entered into a contract for Rs. 60 crores with 'Y' Ltd. of India (Registered in VAT in
Nepal) to erect and install one Hydropower project in Nepal. 'Y' Co. Ltd. has taken few
equipments on hire from 'X' Co. Ltd. and paid hire charges to 'X' Co. Ltd. amounting to Rs.
18 lacs in the same financial year. What rates of TDS will be applicable on the payment of
this hire charges and also state the implications if any, of value added Tax Act on the said
companies? (Dec 2003, 5 Marks, CA Inter)

Answer
Value Added Tax is applicable on hire charges of equipment. As such, X Co. Ltd. shall raise a
Value Added tax invoice.

Further, while making payment of hire charges , Y. Co. Ltd. shall withhold tax on rental charges
@ 10% as per Sec. 88 (1).

31) State and explain the rates of TDS applicable in the following cases. (Dec 2003, 3 Marks, CA
Inter)
a. Payment to security service Provider Company under their service contract.
Answer: 1.5% under Sec. 89, if payment exceeds Rs. 50,000 over a period of moving 11 days.

b. Labour contract with individual for supply of labour to industry and charging service
fee @5% on total labour payment under contract.

The Institute of Chartered Accountants of Nepal 204


CAP-II Paper 7 - Income Tax and VAT

Answer: 1.5% under Sec. 89, if payment exceeds Rs. 50,000 over a period of moving 11 days.

c. Royalty paid to Indian company.


Answer: 15%

d. Interest received from Bank.


Answer: Natural person not in connection to business: 5% and all other cases: 15%, except the
interest is exempt from income tax.

32) What are the payments for which TDS becomes the final tax? (Dec 2003, 2 Marks, CA Inter)
Answer
The following payments are final withholding as per Sec. 92 (1) of Income Tax Act, 2058:
a. Dividend paid by resident company or partnership firm.
b. Other than in respect of operation of business, rent with source in Nepal paid to a natural person
in consideration of lands or buildings and fittings and equipment connected with them.
c. Payment made by a resident person for gains from investment insurance
d. Gains paid by a resident person in consideration of interest in an unapproved retirement fund
e. The following interest amount paid by bank, financial institution, or any other institution
issuing bonds, or company listed under prevailing laws or cooperatives referred to in
Subsection (3) of Section 88:
i. Payment with source in Nepal made to a natural person and is not in relation of operation
of business,
ii. Payment made to an exempt organization referred to in section 2 (dha)
f. Payments subject to withholding tax under section 87, 88, 88Ka or 89 made to non-resident
persons
g. All types of retirement payments including that made by Government of Nepal or an approved
or unapproved retirement fund (except pension payable on regular basis)
h. Meeting fee up-to Rs. 20,000 per meeting, payment in respect of occasional lecture service,
setting up of question paper or checking answer sheet
i. Payment of Windfall Gain
j. Payment of return made to a natural person by a mutual fund
k. Rental Payment received by a natural person in respect of transport service or vehicle rent other
than that received from operation of private firm

33) Big Traders Pvt. Ltd. dealing with electronic items has annual turnover Rs. 7,890,000 and
taxable income Rs. 1,780,000 for IY 20X-74/X-75. The details of tax paid by the company is
Rs. 130,000 on Poush end 20X-74 and filed the estimated tax return on 28 Poush 20X-74,
paid Rs. 56,000 on Chaitra 27, 20X-74 and a government agency has withheld tax on behalf
of Big Traders during the month of Jestha 20X-75 amounting Rs. 57,500. Final installment

The Institute of Chartered Accountants of Nepal 205


CAP-II Paper 7 - Income Tax and VAT

of tax amounting Rs. 46,500 was deposited during Ashad 20X-75. Examine about the
compliance of Section 94 by the company. (June 2019, 5 Marks)

Answer
Provision of Sec. 94:
A turnover based taxpayer shall be tax on two installments, by the end of Poush and by the end of
Ashad. The amount to be deposited in first installment is applicable tax on turnover until Poush 20
and in second installment is applicable tax on total estimated turnover for the year taking into
account the turnover until Ashad 20 after deducting the amount paid in first installment.

For other taxpayers, who are required to pay tax in installments, tax shall be paid in three
installments as follows:
a. By Poush end- 40% of estimated tax liability
b. By Chaitra end- 70% of estimated tax liability
c. By Ashad end- 100% of estimated tax liability
In the given case, since the estimated liability is not given, therefore, the compliance of Sec. 94
cannot be examined.

In the given case, data for estimated tax liability is not given. So the actual tax liability and
estimated tax liability is considered as same. Estimated Tax = Rs. 1,780,000 x25%= Rs. 445,000
Compliance of section 94 is examined through following:
Annual Estimated Estimated Tax to be Actual Tax Short/(Excess)
Due Date
Tax Paid Paid Tax
Poush 20X-74 445,000 178,000 130,000 48,000
Chaitra 20X-74 445,000 311,500 186,000 125,500
Ashad 20X-75 445,000 445,000 290,000 155,000
Since the company is paying less tax, the company is not complying with section 94 of the Act.

34) Details of annual estimated tax and withholding tax of A & Co. are as follows:
Estimated Tax
Poush end Rs. 10,00,000
Chaitra end Rs. 12,00,000 (Re-estimated)
Ashad end Rs. 12,00,000 (Re-estimated)

Withholding Tax details:


Upto Poush end Rs. 20,000
Magh to Chaitra end Rs. 10,000
Chaitra to Ashad end Rs. 15,000
Compute the total Advance Tax to be paid for each installment. (June 2011, 5 Marks)

The Institute of Chartered Accountants of Nepal 206


CAP-II Paper 7 - Income Tax and VAT

Answer:
With Holding
Already paid
Tax to be Tax (WHT) to Remaining
Due Date Due Amount (Cumulative)
Deposited be considered to Pay
except WHT
(Cumulative)
40% of Estimated Tax
By Poush end 400,000 20,000 - 380,000
Liability
70% of Estimated Tax
By Chaitra end Liability, 840,000 30,000 380,000 430,000
Including re-estimated
100% of Estimated Tax
By Ashad end Liability, 1,200,000 45,000 810,000 345,000
Including re-estimated

35) Discuss the provision for payment of Income Tax under Installments. (Dec 2009, 5 Marks)

Answer
A turnover based taxpayer shall be tax on two installments, by the end of Poush and by the end of
Ashad. The amount to be deposited in first installment is applicable tax on turnover until Poush 20
and in second installment is applicable tax on total estimated turnover for the year taking into
account the turnover until Ashad 20 after deducting the amount paid in first installment.

For other taxpayers, who are required to pay tax in installments, tax shall be paid in three
installments as follows:
a. By Poush end- 40% of estimated tax liability
b. By Chaitra end- 70% of estimated tax liability
c. By Ashad end- 100% of estimated tax liability

The Institute of Chartered Accountants of Nepal 207


CAP-II Paper 7 - Income Tax and VAT

Chapter 15: Income from Business

1) Mohit Shrestha, citizen of Nepal provides rental services/hiring services of Mini Bus, Power
Tiller, Auto Rickshaw and other vehicles. Following is the detail of his business. Vehicles are
owned by him in his individual name for fiscal year 20X-69/X-70. (5 Marks, Dec 2013)
Rates per No. of services Provided Number of vehicles
Services (Rs.) during the year owned
Power Tiller 5,500 220 7
Tractor 5,000 180 11
Auto Rickshaw 5,000 10 1
Micro Bus 8,500 50 4
Car 8,500 310 8
Truck 11,000 200 4
He incurred following expenses during the year.
Particulars Amount (Rs.)
Petrol 1,01,000
Diesel 1,20,000
Oil & Lubricants 21,375
Repair 68,245
Staff's Salary 39,00,000

Required:
a. Calculate the tax to be paid by Mr. Mohit for fiscal year 20X-69/X-70.
b. Mr. Mohit did not submit the Annual Income Tax Return under Section 96. What are its
implications?

Answer:
a. Tax to be paid by Mr. Mohit
Inclusions in income as per Sec. 7 (2) [Note 1 below] 50,000,000
Less: Deductions under Income Tax Act, 2058 (Sec. 13) (4,142,375)
Less: Depreciation u/s 19 -
(Could not be computed in absence of information)
Less: Deduction u/s 16 (Assumed within 7% Limit) (68,245)
Taxable Income 45,857,625

Computation of Tax Liability Assuming Couple


1st Rs. 600,000 0% -
Next Rs. 200,000 10% 20,000

The Institute of Chartered Accountants of Nepal 208


CAP-II Paper 7 - Income Tax and VAT

Next Rs. 300,000 20% 60,000


Next Rs. 900,000 30% 270,000
Balance Rs. 43,857,625 36% 15,788,745
Total 16,138,745

Note 1:
Rates per No. of services
Number of
Services Provided during Total Earned
vehicles owned
(Rs.) the year
Power Tiller 5,500 220 7 8,470,000
Tractor 5,000 180 11 9,900,000
Auto Rickshaw 5,000 10 1 50,000
Micro Bus 8,500 50 4 1,700,000
Car 8,500 310 8 21,080,000
Truck 11,000 200 4 8,800,000
Total 50,000,000

b. Obligation to File Income Return


Since, the total income of Mr. Mohit exceeds Rs. 40 Lakhs during the year; Mr. Mohit must file
income return. The implication of not filing income return is payment of fees u/s 117, which is Rs.
100 per month or 0.1% p.a. of Gross Inclusions (i.e. 0.1% p.a. of Rs. 50,000,000). However, CAP
II syllabus excludes Sec. 117, so the implication is not important for CAP II Students.

2) By virtue of an agreement entered on 20X-61/7/11 between A Ltd. and B Ltd., A Ltd. agree
not to carry on any business relating to washing machine within Kathmandu valley for next
5 years, for which B Ltd. agrees to pay a sum of Rs. 15,00,000 to A Ltd. The said amount was
paid on 20X-62/2/27. Indicate treatment of such receipt in the hands of A Ltd. for the income
year 20X-61/X-62. (June 2006, 5 marks, CA Inter)

Answer:
As per Sec. 7 (2) (Cha), any amount received in connection to acceptance of business restriction
forms part of business income. As such, the amount of Rs. 1,500,000 received by A Ltd. is business
income of A Ltd.
As per Sec. 22, a company must follow accrual basis of accounting for income tax purpose.
Therefore, the amount that is received for five years shall be treated as income of five years.

Conclusion
The amount shall be treated as income for following income years as follows:
20X-61/X-62 [1,500,000/5 (for 8 months 19 days (days can also be taken)] Rs. 215,833
20X-62/X-63 300,000

The Institute of Chartered Accountants of Nepal 209


CAP-II Paper 7 - Income Tax and VAT

20X-63/X-64 300,000
20X-64/X-65 300,000
20X-65/X-66 300,000
20X-66/X-67 84,167

3) Nepal Manufacturing Company a special industry engaged in production of electronic goods.


The income statement of the company for the F.Y. 2077/78 is a follows:

Particulars Amount (in NPR)


Sales of Electronic Goods 3,500,000.00
Other income 1,100,000.00
Bad Debt recovered 900,000.00
Closing stock 500,000.00
Total Income 6,000,000.00
Less: Expenses
Opening Stock 150,000.00
Purchase 1,500,000.00
Water and Electricity 140,000.00
Salary and wages 300,000.00
Provision for Gratuity 120,000.00
Seed capital investment (Grant) 900,000.00
Communication expenses 50,000.00
Duty and Taxes 100,000.00
Bad debt 200,000.00
Miscellaneous expenses 800,000.00
Repair and Maintenance 600,000.00
Depreciation 900,000.00
Total Expenses 5,760,000.00
Net Profit before tax 240,000.00

Additional Information
a) Bad debt recovered was disallowed previously.
b) Other income includes dividend income of Rs. 950,000 after tax received from investment
in share of a listed company. Moreover, miscellaneous expenses include Rs. 600,000
incurred exclusively for earning such dividend.
c) The cost of water and electricity includes payment of Rs. 12,000 for the month of Ashadh
2077.
d) Value of Opening stock was overvalued by Rs. 50,000.

The Institute of Chartered Accountants of Nepal 210


CAP-II Paper 7 - Income Tax and VAT

e) Retirement payment includes Rs. 50,000 which paid to an employee who has retired in
the month of Baisakh, 2077 and 50,000 to an employee who has retired in the month of
Chaitra, 2077.
f) The company has provided Rs. 150,000 each to 6 startup companies as seed capital which
are not associated with Nepal Manufacturing Company.
g) Miscellaneous Expenses of Rs. 5,000 was incurred without PAN bills.
h) Duty and Taxes includes tax paid for the municipality Rs. 10,000 and fine paid for
violation of income tax law Rs. 20,000.
i) Communication expenses include Rs. 40,000 paid for personal internet expenses of
director.
j) Repair and Maintenance includes repair of vehicle amounting to Rs. 31,000 furniture of
Rs. 49,000 and office equipment Rs. 35,000 and remaining repairs is related to factory
building.
Detail of Fixed Assets Opening WDV for the year
Vehicle 1,100,000.00
Furniture 650,000.00
Office Equipment 850,000.00
Factory Building 2,500,000.00
Additional furniture of Rs. 50,000 was purchased on 1st Magh, 2077.

Required: (June 2022, 20 Marks)


a) Calculate allowable repair and maintenance and depreciation expenses for the F.Y.
2077/78.
b) Calculate the Assessable Income from Business and Taxable income from Business.
c) Calculate the Tax liability for the FY 2077/78.

Answer:

a. Calculation of Allowable Repair and Allowable Depreciation:

i. Calculation of depreciation
Particulars Pool A Pool B Pool C Total
Opening WDV 2,500,000.00 1,500,000.00 1,1x00,000.00 5,100,000.00
Addition During the year
Absorbed Addition (2/3rd of 50,000) 33,333.33 33,333.33
Depreciation Base 2,500,000.00 1,533,333.33 1,100,000.00 5,133,333.33
Normal Depreciation Rate 5.00% 25.00% 20.00%
Additional 1/3 (being special industry) 1.67% 8.33% 6.67%
New depreciation rate 6.67% 33.33% 26.67%
Allowable Depreciation 166,666.67 511,111.11 293,333.33 971,111.11

The Institute of Chartered Accountants of Nepal 211


CAP-II Paper 7 - Income Tax and VAT

ii. Calculation of allowable repair and


maintenance
Depreciation base 2,500,000.00 1,533,333.33 1,100,000.00 5,133,333.33
Lower of: 175,000.00 84,000.00 31,000.00 290,000.00
a. 7% of depreciation base, or 175,000.00 107,333.33 77,000.00 359,333.33
b. Actual 485,000.00 84,000.00 31,000.00 600,000.00

b. Calculation of Assessable Income, Taxable income and Tax Liability of Nepal


Manufacturing Company
Particulars Amount Remarks
Sale of Electronic Goods 3,500,000.00
Other Income 150,000.00 Note 1
Bad debt recovered - Note 2
Assessable income from business (A) 3,650,000.00

Cost of Trading Stock 1,100,000.00 Note 3


Water and Electricity 128,000.00 Note 4
Salary and Wage 300,000.00
Gratuity Expense 50,000.00 Note 5
Seed Capital 500,000.00 Note 6
Communication expenses 10,000.00 Note 7
Duty and Taxes 80,000.00 Note 8
Bad debt -
Miscellaneous expenses 195,000.00 Note 9
Repair and maintenance 290,000.00 Note 10
Depreciation 971,111.11 Answer ”a”
Total Deduction (B) 3,624,111.11

Taxable Income (A-B) 25,888.89


Applicable tax rate (25 * (100%-20%) 20%
Tax Liability 5,177.78

Note 1
Other Income
Other Income 1,100,000.00
Less: Dividend Income 950,000.00
Total 150,000.00

The Institute of Chartered Accountants of Nepal 212


CAP-II Paper 7 - Income Tax and VAT

Note 2
Bad Debt Recovered
Since, bad debt was not allowed previously it is not required to be included in calculation of income
from business.

Note 3
Cost of Trading Stock
Opening value of trading stock (1,50,000-50,000) 100,000.00
Add: Purchase 1,500,000.00
Less: Closing Stock 500,000.00
Cost of Trading Stock 1,100,000.00

Note 4
Water and Electricity Expenses
Water and Electricity 140,000.00
Less: Payment of Ashadh 2077 (Prior year expenses) 12,000.00
Total 128,000.00

Note 5
Provision for gratuity
Provision for gratuity cannot be claimed as expenses as per the 50,000.00
Income Tax Act so gratuity payment of Chaitra, 2077 is only allowed
for deduction.

Note 6
Seed Capital
Seed capital provided by any person up toRs. 1 lakh to maximum 5 500,000.00
startup business other than the associated person, is allowed for
deduction at the time of calculation of taxable income.
So in the above question allowable seed capital is Rs. 1,00,000 * 5
[Section 12C]

Note 7
Communication expenses
Communication expenses 50,000.00
Less: Personal internet expenses of director 40,000.00
Total 10,000.00

The Institute of Chartered Accountants of Nepal 213


CAP-II Paper 7 - Income Tax and VAT

Note 8
Duty and Taxes
Duty and taxes 100,000.00
Less: Fine paid as per income tax 20,000.00
Total 80,000.00
(Tax to metropolitan office is allowed as deduction.)

Note 9
Miscellaneous Expenses
Miscellaneous Expenses 800,000.00
Less: Expenses without PAN bill not allowed u/s 21 5,000.00
Less: Expenses incurred for earning dividend (income matching) 600,000.00
Total 195,000.00

4) ABC P. Ltd. is a special industry as per Industrial Enterprise Act 2076. Relevant extracts
related to income and expenditure for income year 2077/78 are as follows:
Amount
Particulars (Rs.) Particulars Amount (Rs.)
Opening Stock 4,500,000.00 Sales 18,000,000.00
Purchase of raw materials 8,500,000.00 Closing stock 4,800,000.00
Gain on sale of business
Freight Inwards 400,000.00 assets 600,000.00
Prize from display
Direct wages 800,000.00 competition 200,000.00
Manufacturing Expenses 1,200,000.00 Miscellaneous Income 750,000.00
Administrative Expenses 400,000.00 Bad debts recovered 300,000.00
Interest Expenses 700,000.00 Dividend received 950,000.00
Repair Expenses 500,000.00
Depreciation 1,200,000.00
Entertainment Expenses 200,000.00
Miscellaneous Expenses 100,000.00
Audit Fee 400,000.00
Pollution control cost 500,000.00
Donation 300,000.00
Research and Development
Cost 200,000.00

Additional Information: (Dec 2021, 20 Marks)


1. Out of bad debts recovered, 20% was not allowed previously.

The Institute of Chartered Accountants of Nepal 214


CAP-II Paper 7 - Income Tax and VAT

2. The opening stock and closing stock has been recorded 45,000 pieces and 50,000 pieces
respectively. The opening stock includes factory fixed overhead Rs. 15 per piece and
repair and maintenance Rs. 10 per piece (related to machinery). During the year, the
company produced 200,000 pieces of garments and the overhead cost includes equal rate
of previous years fixed overhead and repair and maintenance cost.
3. Administration expenses include a cash payment of Rs. 75,000 to a constitutional body.
4. Additionally, administrative expenses includes payment made to wages amounting to Rs.
250,000 The breakup of these payments are as: Rs. 2,500 paid to 60 workers and Rs. 5,000
paid to 20 workers, each.
5. Opening WDV of assets are: Building Rs. 30,000,000, land 5,000,000, Plant & Machinery
Rs. 4,000,000, Vehicle 1,000,000, Office Equipment Rs. 1,000,000.
6. Machinery worth 6,000,000 was purchased on the Month of Jestha, 2078, while vehicle
purchased 3 years ago with a cost of 3,000,000 and WDV of 1,000,0000, as per accounts
was disposed off for value of Rs. 1,500,000 in the month of Baisakh, 2078.
7. Repair and maintenance cost includes Rs. 200,000 for repairs of building and 300,000 for
repairs of machinery.
8. Donation includes Rs. 100,000 donated to Covid relief fund and Rs. 200,000 donated to tax
exempt organization.
9. The Company employees 400 Nepalese citizen throughout the year and was set up in less
developed area five years ago.

Required: Determine tax liability of the Company.

Answer:
Computation of Assessable Income and Tax Liability of the Company
Particulars Rs. Note Section
Sales 18,000,000.00 7
Net gain on sale of business assets 600,000.00 7
Prize from display competition 200,000.00 7
Miscellaneous Income 750,000.00 7
Bad debts recovered (80%) 240,000.00 (W.N.1) 7
Balance Charging on disposal of depreciable assets 500,000.00 (W.N. 4) 7
Dividend Received (Final Withholding) - 92
Less: Allowable Deductions
Cost of trading stock u/s 15 12,225,000.00 (W.N.2) 15
Administration expenses 400,000.00 (W.N.3) 13
Depreciation 3,533,333.00 (W.N.4) 19
Repair & improvement expenses 500,000.00 (W.N.5) 16
Entertainment expenses 200,000.00 13
Miscellaneous expenses 100,000.00 13

The Institute of Chartered Accountants of Nepal 215


CAP-II Paper 7 - Income Tax and VAT

Audit Fee 400,000.00 13


Interest Expenses 700,000.00 13
Assessable Income before PCC, R& D, Donation 2,231,667.00
Less: Pollution control cost 500,000.00 (W.N.6) 17
Less: R & D Cost 200,000.00 (W.N.7) 18
Assessable Income from Business 1,531,667.00
Less: Donation u/s 12 81,583.35 (W.N.8) 12
Taxable Income 1,450,083.65
Effective Tax Rate 6% (W.N.9) 11
Tax Liability 87,005.02

W.N. 1
80 % of Bad Debts was allowed previously, therefore, on recovery 80% of the recovery is only
taxable.
W.N. 2 Calculation of Cost of trading stock
Particulars Amount
Opening Stock 4,500,000.00
Less: Adjustment for Repair & Maintenance Expense (10*45,000) 450,000.00
Value of Adjusted Opening Stock (A) 4,050,000.00
Add: Cost of Production
Purchase of Raw Materials 8,500,000.00
Freight inwards 400,000.00
Direct Wages 800,000.00
Manufacturing Expenses 1,200,000.00
Total Manufacturing Cost (B) 10,900,000.00
Production Unit 200,000.00
Production Cost Per Unit (Using FIFO Method) 54.50
Value of Closing Stock (50,000*54.50) © 2,725,000.00
Cost of Trading Stock (D=A+B-C) 12,225,000.00
(Note: Repair and maintenance of machinery could not be adjusted for during the year’s
transaction as the fixed factory overhead is not given in question. Only value of opening stock is
adjusted.)

Alternatively, students may assume Rs. 3,000,000 (15*200000) as overhead cost of production in
addition to production cost given table. And value closing stock accordingly. Further, repair of
machinery would be Rs. 2,000,000 for the year.

W.N. 3: Administrative Expenses


(Given) 400,000.00

The Institute of Chartered Accountants of Nepal 216


CAP-II Paper 7 - Income Tax and VAT

Cash Paid to Constitutional Body -


Allowed Expenses 400,000.00
As per section 21(2) of the act, if a person, having annual turnover of Rs. 20 lacs, makes an
expenditure by cash for amount more than Rs.50,000, then the expenditure is not deductible. But
the exception is for the constitutional body, hence it is allowable.

Wages paid to workers are allowed as a deduction. The question does not provide the information
about their PAN. It is assumed that all workers have PAN. Moreover, it is assumed that wages are
paid for administrative work.

W.N. 4 Calculation of Depreciation Expense


Particulars Pool A Pool B Pool C Pool D Remarks
I. Depreciation Rate 5.00% 25.00% 20.00% 15%
Additional: (1/3) (Special
Industry) 1.67% 8.33% 6.67% 5%
Applicable Rate of
Depreciation 6.67% 33.33% 26.67% 20%
II. Opening Value 30,000,000.00 1,000,000 1,000,000.00 4,000,000.00
III. Absorbed Additions 2,000,000.00
IV. Disposals 1,500,000.00
V. Depreciation Base 30,000,000.00 1,000,000 - 6,000,000.00
VI. Balance Charging 500,000.00 Inclusion
VI. Depreciation
Expenses 2,000,000.00 3,33,333.33 1,200,000.00

Total Depreciation Expenses: NRs. 35,33,333


Land is a non-depreciable asset
Additional 1/3rd depreciation is provided to special industries.
Rs. 500,000 is included income (sales proceeds less WDV)
W.N. 5: Calculation of Eligible Repair & Improvement Cost
Particulars Pool A Pool B Pool D
I. Depreciation Base 30,000,000.00 1,200,000 6,000,000.00
II. 7% of Depreciation Base 2,100,000.00 84,000.00 420,000.00
III. Actual Repair & Improvement Cost 200,000.00 0 300,000.00
IV. Eligible (Lower of II or III) 200,000.00 0 300,000.00

Total Eligible Repair and Maintenance Expenses: 500,000

The Institute of Chartered Accountants of Nepal 217


CAP-II Paper 7 - Income Tax and VAT

W.N. 6 Pollution Control Expenses


Taxable income without giving effect to sec 12, 14(2), 17, 18 2,331,667.00
Less: Actual R&D 200,000.00
ATI for the purpose of Sec. 17 21,31,667.00
Actual PCC [A] 500,000.00
50% of ATI [B] 10,65,833.50
Allowable PCC 500,000.00

W.N. 7 Research and Development cost


Taxable income without giving effect to sec 12, 14(2), 17, 18 2,331,667.00
Less: Actual PCC 500,000.00
ATI for the purpose of Sec. 18 18,31,667.00
Actual R & D Expenses 200,000.00
50% of ATI 915,833.50
Allowable R & D expenses 200,000.00

W.N. 8 Donation

Taxable income without giving effect to sec 12, 14(2), 17, 18 23,31,667.00
Less: Actual PCC 500,000.00
Less: Actual R&D 200,000.00
ATI for the purpose of Sec. 12 16,31,667.00
Actual Donation (300,000-100,000) 200,000.00
5% of ATI 81,583.35
Maximum 100,000.00
Allowable Donation u/s 12 81,583.35
Donation made to Covid Relief fund is not allowed as deduction. This is allowed as deduction
only for IY 2077/078 through Finance Act.

W.N. 9
Effective Tax Rate
Tax Rate- Special Industry (11(2b)(c)) - 20% exempt 20%
Concession to industry providing employment to more than 300
Nepalese citizen throughout the year(Section 11(3(a))- 80% of
applicable tax 16%

Tax rate of Special industry set up at less developed area, within 10


years of its incorporation 30% of the tax levied (Section 11(3)(b)) 6%

The Institute of Chartered Accountants of Nepal 218


CAP-II Paper 7 - Income Tax and VAT

5) Strong Cement Limited is a company engaged in manufacturing and sale of premium grade
cement. It provides employment opportunity to 400 people. The company is listed in Nepal
Stock Exchange, it has domestic as well as export sales. Following is the provisional Income
Statement of the company for the year ended Ashad 31, 2077.
Particulars Amount in NPR
Income:
Export Sales 9,000,000.00
Domestic Sales 6,000,000.00
Dividend Received (Net of Tax) 150,000.00
Investment Income 150,000.00
Total Income 15,300,000.00
Expenditure:
Cost of Materials Consumed 4,500,000.00
Manufacturing Expenses 900,000.00
Employee Cost 1,300,000.00
Selling and Administrative Expenses 2,000,000.00
Interest and Bank Charges 1,000,000.00
Exchange Loss 250,000.00
Depreciation 450,000.00
Total Expenditure 10,400,000.00
Profit Before Tax 4,900,000.00

Additional information:
i) Cost of material consumed
For export sales: NPR 2,700,000
For local sales: NPR 1,800,000
ii) Selling and Administrative Expenses include NPR 200,000 donation given to Provincial level
Covid Fund and NPR 350,000 given for construction of Private hospital.
iii) You are given following information with regards to some expenses.
• NPR 1,50,000 included in employee cost as staff welfare is personal expenses of directors.
• Manufacturing expenses include NPR 40,000 for electricity bill of previous year.
• Selling and Administrative Expenses include NPR 40,000 for business promotion which is
not related to business.
iv) Exchange loss is fully realized and is related with import of raw materials.
v) From the tax account of the company, details of property, plant and equipment are as
follows:

Opening WDV including Unabsorbed Repair &


Block Maintenance Expenses of previous year (Rs.)
A 500,000

The Institute of Chartered Accountants of Nepal 219


CAP-II Paper 7 - Income Tax and VAT

B 400,000
C 500,000
D 2,000,000

Company has received tax assessment order from IRD for income year 2075/76 and the
company did not file any appeal to concerned authority. As per the assessment order,
following depreciation relating to income year 2075/76 is not allowed as deductible expenses
due to unsupported purchase capitalized on 2075/06/10.
Company has claimed depreciation on such capitalization as per the rates so provided by
Income Tax Act, 2058.

Particulars Disallowed Depreciation (Rs.)


Block A 15,000
Block B 20,000
Block C 30,000
Block D 45,000

No purchase and sales of depreciable assets is done during the year. The company desires to
claim additional depreciation as per Income Tax Act from this year onwards, which is
available to a special industry. This claim is only for tax purpose.
vi) Investment Income includes income from sale of land.

You are required to calculate Assessable income & Income Tax Liability of Strong Cement
Limited for the Income year 2076/77. (Dec 2020 CAP II 20 marks)

Answer:
Computation of Assessable Income, Taxable Income, and Tax Liability:
Particulars Export Sales Local Sales Investment Total Notes
Income
Income 9,000,000.00 6,000,000.00 - 15,000,000.00
Dividend Received - - - - W.N.1
Investment Income - 150,000.00 150,000.00 W.N.2
Total Income 9,000,000.00 6,000,000.00 150,000.00 15,150,000.00
Expenditure:
Cost of Materials Consumed 2,700,000.00 1,800,000.00 - 4,500,000.00 W.N.3
Manufacturing Expenses 516,000.00 344,000.00 - 860,000.00 W.N.4
Employee Cost 690,000.00 460,000.00 - 1,150,000.00 W.N.5
Selling and Administrative Expenses 846,000.00 564,000.00 - 1,610,000.00 W.N.6
Interest and Bank Charges 600,000.00 400,000.00 - 1,000,000.00 W.N.7
Exchange Loss 150,000.00 100,000.00 - 250,000.00 W.N.7
Depreciation 346,800.00 231,200.00 - 578,000.00 W.N.8
Total Expenditure 5,848,800.00 3,899,200.00 - 9,948,000.00

The Institute of Chartered Accountants of Nepal 220


CAP-II Paper 7 - Income Tax and VAT

Assessable income from Business 3,151,200.00 2,100,800.00 150,000.00 5,202,000.00


Donation for construction of Private - - - - W.N. 9
hospital
Contribution to Covid Fund of GON W.N.
10
Taxable Income 3,151,200.00 2,100,800.00 150,000.00 5,202,000.00
Tax Rate 8.00% 16.00% 25.00% W.N.9
Tax Liability 252,096.00 336,128.00 37,500.00 625,724.00

Working Notes:
1. Dividend received is net of tax and is final withholding and hence will not be included in taxable
Income.
2. Investment Income is the income from sale of Land & will be taxed at 25%, hence will be
calculated separately.
3. Cost of Materials Consumed
Particulars Amount (NPR)
Cost of Material for Export Sales (60% of total cost of Material) 2,700,000.00
Cost of Material for Local Sales (40% of total cost of Material) 1,800,000.00
Total 4,500,000.00
4. Manufacturing Expenses
Particulars Amount (NPR)
Total Manufacturing expenses 900,000.00
Less: Disallowed Expenditure
Electricity Expenses (Disallowed due to prior period exp. U/s 13) (40,000.00)
Allowable Expenditure 860,000.00
For Export Sales (60%) 516,000.00
For Domestic Sales (40%) 344,000.00
5. Employee Expenses:
Particulars Amount (NPR)
Total Employee expenses 1,300,000.00
Less: Disallowed Expenditure
Staff welfare (Disallowed personal exp. U/s 13) (150,000.00)
Allowable Expenditure 1,150,000.00
For Export Sales (60%) 690,000.00
For Domestic Sales (40%) 460,000.00
6. Selling & Distribution Expenses
Particulars Amount (NPR)
Selling & Distribution Expenses 2,000,000.00
Less: Disallowed Expenditure
Business Promotion (Disallowed due to Not related to business. (40,000.00)
U/s 13)

The Institute of Chartered Accountants of Nepal 221


CAP-II Paper 7 - Income Tax and VAT

Donation given to construction of hospital and covid fund (550,000.00)


separately treated for tax purpose
Allowable Expenditure 1,410,000.00
For Export Sales (60%) 846,000.00
For Domestic Sales (40%) 564,000.00

7. Interest Expenses & Exchange Loss

Particulars Interest Exp. (NPR) Exchange Loss Exp. (NPR)


Total Interest expenses 1,000,000.00 250,000.00
Allowable Expenditure 1,000,000.00 250,000.00
For Export Sales (60%) 600,000.00 150,000.00
For Domestic Sales (40%) 400,000.00 100,000.00

8. Depreciation Expenses
Particulars Block-A Block-B Block-C Block-D Total
Opening WDV as per Tax 500,000.00 400,000.00 500,000.00 2,000,000.00 3,400,000.00
Disallowed in Opening WDV 285,000.00 60,000.00 120,000.00 255,000.00 720,000.00
(8.1)
Adjusted Opening WDV 215,000.00 340,000.00 380,000.00 1,745,000.00 2,680,000.00
Additions - - - - -
Sales - - - - -
Depreciation Base 215,000.00 340,000.00 380,000.00 1,745,000.00 2,680,000.00
Rate of Depreciation 5% 25% 20% 15%
Depreciation (Normal) 10,750.00 85,000.00 76,000.00 261,750.00 433,500.00
Depreciation (Additional) 3,583.33 28,333.33 25,333.34 87,250.00 144,500.00
Total Depreciation 14,333.33 113,333.33 101,333.34 349,000.00 578,000.00
For Export Business (60%) 346,800.00
For Export Business (40%) 231,200.00

8.1 Calculation of Disallowance in Opening WDV


Particulars Block-A Block-B Block-C Block-D Total
Disallowed Depreciation 15,000.00 20,000.00 30,000.00 45,000.00 110,000.00
(A)
Depreciation Rate (B) 5% 25% 20% 15%
Capitalized Amount 300,000.00 80,000.00 150,000.00 300,000.00 830,000.00
[C=(A/B)]
Disallowed Amount (C-A) 285,000.00 60,000.00 120,000.00 255,000.00 720,000.00

The Institute of Chartered Accountants of Nepal 222


CAP-II Paper 7 - Income Tax and VAT

8.2 Since the Company did not claim additional depreciation in previous year, only depreciation as
per the rates provided by Act is used for backward calculation of capitalized amount.
8.3 As the company did not file any appeal in assessment order of IRD, it is established that the
company accept the disallowed depreciation and capitalization of concerned fixed Assets.
8.4 As per Schedule 2 of Income Tax Act, Special Industry can claim additional 1/3rd depreciation on
depreciable blocks of assets.
9. Donation paid to tax exempt entity can be claimed as deductible expenses within the limit
prescribed under section 12 of Income Tax Act. As the Private Hospital is not a Tax-Exempt
Entity, donation paid Rs. 350,000 cannot be claimed as deductible expenses under Income Tax
Act.
10. Donation to CoVID fund of GON, Province Govt. or Local Govt. is deductible as per Sec. 26 of
Finance Act, 2077. This is not allowed as deduction for IY 2079/080.
11. Tax Rate
Export Domestic Investmen
Particulars Remarks
Income Income t Income
Section 2 of
Normal tax rate 25% 25% 25%
schedule
11(2B) for special
Applicable tax rate 20% 20% 25%
industry
u/s 11 (3) (Ka) for
Rebate-1 for employing
16% 16% - Spl. Industries,
300+ employees (A) 20%
reduced rate
Rebate-2 for Listed W.N. 9.1 reduced
17% 17% -
Company (B) rate
Rebate for Export from W.N. 9.1 Section
8% -
manufacturing industry 11(3E) reduced rate
Selection of Rebate 8% 16% 25%
Net Applicable Tax Rate 8% 16% 25%

9.1 Available Concession/Exemptions:


As per section 11 of Income Tax Act, 2058 below are the exemptions available to Strong Cement
Limited being a Special Industry –
i. 20% rebate for special industry as per section u/s 11 (2kha)(kha)& the industry availing this
rebate will be eligible to enjoy & avail another rebates available in section 11.
Accordingly,
ii. The Company employs more than 400 Nepali employees whole year. So, it is entitled for 20%
tax rebate as per section 11(3)(Ka).
ii. The Company has export sales. So, 20% tax rebate for export Income only as per section 11
(3anga). Additional 50% concession for export of goods produced in Nepal. Applicable Tax
Rate = 20*80%*50%

The Institute of Chartered Accountants of Nepal 223


CAP-II Paper 7 - Income Tax and VAT

iii. The Company is a listed production-based Company. Hence, it is entitled to 15% rebate as per
section 11 (3chha).
Selection of Tax Exemption
As per section 11(5), if more than one tax exemption is available for the same income, then only
one tax exemption should be availed as per choice of the Taxpayer.
Hence, the Company should choose one of the exemptions mentioned above.

6) Kamala Cement Ltd. is a company situated at Biratnagar and listed in Nepal Stock Exchange
and engaged in manufacturing and sale of premium grade cement in Domestic Market as
well as Foreign Market. Following is the provisional Income Statement of Kamala Cement
Ltd., for the year ended Ashadh 31, 2077.
Particulars Amount (Rs.)
Income
Export Sales 6,000,000
Domestic Sales 4,000,000
Dividend Received (Net of Tax) 150,000
Rent Income (Related with Business) 50,000
Total Income 10,200,000

Expenditure
Cost of Materials Consumed 3,000,000
Manufacturing Expenses 500,000
Employee Cost 1,000,000
Selling and Administrative Expenses 1,500,000
Interest and Bank Charges 500,000
Exchange Loss 250,000
Depreciation 500,000
Bad Debt Written Off 70,000
Loss on Sale of Assets 300,000
Total Expenditure 7,620,000

Operating Profit 2,580,000


Add: Provision for Doubtful Receivables Written Back 720,000
Less: Provision for Bonus 300,000
Profit Before Tax 3,000,000
Additional information:
a) Cost of material consumed
For export sales : Rs. 1,800,000
For local sales : Rs. 1,200,000
b) Exchange loss includes Rs. 100,000 against revaluation of creditors at the yearend date.

The Institute of Chartered Accountants of Nepal 224


CAP-II Paper 7 - Income Tax and VAT

c) Asset detail for income tax purpose is as below:


Computer, Office
Plant and
Particulars Building Equipment Vehicles
Machinery
& Furniture
Opening Depreciation 11,00,000 1,00,00,000 15,00,000 29,50,000
Base
Addition up to Poush End - - 2,00,000 -
Addition Magh to Chaitra 6,00,000 - 1,50,000 -
end
Addition Baishakh to - - - 9,00,000
Ashadh End
Sales Proceed - 15,00,000

d) Manufacturing expenses includes repair and maintenance expenses as below:


Building Repair and Maintenance : Rs. 1,50,000
Office Equipment Repair and Maintenance : Rs. 50,000
Vehicle Repair and Maintenance : Rs. 20,000
e) Selling and Distribution Expenses includes Rs. 7,00,000 donations given to Prime
Minister COVID Relief Fund and Rs. 3,00,000 given for construction of private school.
f) Bonus of Rs. 2,00,000 only distributed to the employees till the time of filing of income
tax return. It has been decided by the management not to pay the undistributed portion.
g) Following expenses are of below nature:
i) Rs. 50,000 included in employee co
ii) st for staff welfare is for personal use of directors.
iii) Manufacturing expenses includes Rs. 30,000 for electricity bill of previous years.
iv) Selling and administrative expenses includes Rs. 10,000 without having PAN/VAT
bill.
h) Company employed 1,300 Nepali employees during the whole year out of which 50
employees are foreign employee.

Calculate the income tax liability of Kamala Cement Limited for the income year 2077/78.
(June 2021 20 marks)
Answer
Statement of Assessable income from Business
Sec. Export Other
Particulars Working Notes
Ref Income Income
Inclusions:
Export Sales 7 (2) 6,000,000
Domestic Sales 7 (2) 4,000,000
Rent Income - 50,000 Assumed Rent is gross

The Institute of Chartered Accountants of Nepal 225


CAP-II Paper 7 - Income Tax and VAT

Sec. Export Other


Particulars Working Notes
Ref Income Income
Dividend Income 92 - Assuming dividend received
from resident entity and is
final withholding income.
Total Inclusions 6,000,000 4,050,000
Deductions
Interest Expenses 14 300,000 200,000 In proportion to Cost of
Material Consumed (i.e. 3:2)
Cost of Trading Stock 15 1,950,000 1,300,000 Refer W.N. 1
Depreciation for the Year 19 1,900,000 1,266,667 Refer W.N. 3
Repair and Improvement 16 105,000 70,000 Refer W.N. 3
Cost
Exchange Loss 90,000 60,000 Revaluation loss is not
deductible u/s 28
Employee Cost 13 570,000 380,000 Rs. 50,000 is not deductible
as it is personal expense.
Remaining amount is
distributed in proportion to
Cost of Material Consumed
Selling and Administrative 13 294,000 196,000 Refer W.N. 2
Expenses
Bonus provision for the year
Employee Bonus Provision 180,000 120,000 is deductible as it is fixed
for the Year obligation.
Any amount undistributed
shall be added back to
income in the next income
year.
Bad Debt written off Assumed that conditions of
Sec. 40 (3) (ga) not met.
Loss on Sale of Assets Is considered while
computing depreciation.
Total Deductions 5,389,000 3,592,667
Assessable Income from 611,000 457,333
Business

Statement of Taxable income and tax liability


Total Assessable Income 611,000 457,333
Less:
Reductions u/s 12Kha- 420,000 280,000 In Cost of Material
National Reconstruction Consumed ratio, as the
Fund of GON question suggests

The Institute of Chartered Accountants of Nepal 226


CAP-II Paper 7 - Income Tax and VAT

Taxable Income 191,000 177,333


Tax Rate 8% 14%
Tax Liability 15,280 24,827

Working Notes
1. Cost of Trading Stock for the Year
Cost of Materials consumed 3,000,000
Manufacturing Expenses 500,000
Less: Repair and Maintenance expenses (220,000)
Less: Prior period expenses (30,000)
Total Cost 3,250,000
Towards Export Sales (in Cost of Material Consumed 1,950,000
ratio)
Towards Domestic Sales 1,300,000
2. Selling and Administrative Expense
Given 1,500,000
Less:
Donation to PM Relief Fund (700,000)
Donation for Construction of School (300,000)
Business promotion, without having PAN/VAT Bill (10,000) 490,000
Allocated to Export sales (in Cost of Material Consumed 294,000
ratio)
Allocated to Domestic Sales 196,000
3. Computation of Depreciation Allowances
Calculation of Depreciation and Repair and Improvement Cost
Particulars Block A Block B Block C Block D Total
Opening Depreciation Base 1,100,000 1,500,000 2,950,000 10,000,000
Add: Absorbed additions
Up to Poush (100% of cost) 200,000
From Magh to Chaitra (2/3rd of Cost) 400,000 100,000
From Baishakh to Ashadh (1/3rd of Cost) 300,000
Less: Disposal Proceeds of Assets (Sales of (1,500,000)
Vehicle)
Depreciable Basis 1,500,000 1,800,000 1,750,000 10,000,000
Depreciation Rate 5% 25% 20% 15%
Depreciation Amount (Normal) 75,000 450,000 350,000 1,500,000 2,375,000
One-third additional (as it is special industry) 25,000 150,000 116,667 500,000 791,667
Total Depreciation for the year 100,000 600,000 466,667 2,000,000 3,166,667

7% of Depreciable Basis 105,000 126,000 122,500 700,000


Actual Repair Expense 150,000 50,000 20,000 -
Allowable Repair Expenses (Minimum of 7% of 105,000 50,000 20,000 - 175,000

The Institute of Chartered Accountants of Nepal 227


CAP-II Paper 7 - Income Tax and VAT

Particulars Block A Block B Block C Block D Total


Depreciation Base or Actual)
* Donation for construction of private school is not allowed for deductible expenses u/s 12 of Income
Tax Act 2058. Private school is not a tax exempt entity.
* Provision for doubtful receivable is assumed to be disallowed in the year of provision, hence, it is
not included in taxable income during the year of written back.
4. Identification of Tax Rate
Particulars Details Export Domestic Ref. Sec.
Applicable Tax Rate 25% 25%
Reduction to special industry 20% Sec. 11 (2Kha)
(Kha)
Reduced Tax Rate 20% 20%
Additional Reduction due to 30% Sec. 11 (3) (ka)
employment
Effective Rate due to effect of 14% 14%
employment
Reduction due to export 20% 16% Sec. 11 (3Nga)
(Kha)
Additional Reduction to 25% 8% Sec. 11 (3Nga)
manufacturing exporter (Ga)
Lower Rate 8% 14%
As per Sec. 11 (3) (ka), where a special industry provides employment to 1,000 or more Nepali
nationals, the tax rate is 70% of reduced tax rate, i.e. 14%.
Application of Sec. 11 (3Nga) jointly read with Sec. 11 (3Kha) will give tax rate of 8% in case of
export of goods manufactured in Nepal.
As such, tax rate of 14% is applicable in case of domestic sales income and that of 8% is applicable
in case of export income, as a rational taxpayer opts for beneficial options.

7) Juntara Trades is a partnership firm with 4 partners. It is an authorized distributer of


SUPERB brand of household goods. The firm has drawn income statement relating to
Income Year (IY) 2074/75 as follows:
Particulars Amount (Rs.)
Sales 55,194,100
Less: Cost of Goods Sold 44,155,280
Opening Stock 3,219,660
Materials Purchased 46,455,030
Less: Closing Stock (5,519,410)
Gross Profit 11,038,820
Less: Administrative & Selling Expenses 4,723,196
Salary & Wages 3,311,646

The Institute of Chartered Accountants of Nepal 228


CAP-II Paper 7 - Income Tax and VAT

Business Promotion Expenses 500,000


Repairs & Maintenance 375,000
Office Expenses 50,000
Lawyers Fee 45,000
Marketing Expenses 441,550
Net Profit 6,315,624
The accountant provides the following additional information in relation to the above
transactions:
a. Salary and Wages includes Rs. 50,000 paid for installing plant Rs. 10 lakhs purchased on
Chaitra 2074.
b. The firm didn't deduct tax on salary Rs. 40,000 at the time of payment but it was paid by the
firm during Ashad end. The deposited amount is debited to Salary and Wages.
c. The firm was awarded a motorcycle with market value of Rs. 220,000 as it met Sales target.
The motorcycle was immediately sold to one of the employees at the market value. The
transaction has not been reflected in the income statement above.
d. The details of depreciable assets are as follows:
Block Opening Depreciation Base (Rs.)
A 11,123,750
B 556,190
C 2,500,000
D 4,449,500

e. The sole assets in block C is a delivery van which is 100% financed by AB Bank Ltd. The
delivery van is registered in the name of the bank. The related interest expense of Rs. 250,000
has not been paid by the firm till the end of the year and is not considered as expenses in
preparing the income statement above.
f. 20% of repairs & maintenance is related to Block B and remaining is related to Block D.
g. The firm has a policy to provide commission to persons referring customers. The firm paid
Rs. 40,000 this year as referral commission to persons for referring customers in the previous
year and this has been reduced from Sales above. Similarly, the firm paid Rs. 30,000 after
closure of the financial year as referral commission to persons for referring customers in IY
2074/75 which is not shown as expenses in the income statement above.
h. Lawyer’s fee is paid to a lawyer for defending assessment of the firm made by income tax
officer in relation to IY 2073/74. The tax officer issued an assessment order demanding
additional income tax Rs. 200,000. The related liability is not provided by the firm in the
above income statement.
i. Office expenses include monthly tuition fees of children of marketing director of Rs. 12,000
as per terms of employment.
Required: (June 2019, 20 Marks)
a. Compute tax payable by the firm for IY 20X-74/X-75.

The Institute of Chartered Accountants of Nepal 229


CAP-II Paper 7 - Income Tax and VAT

b. Would the tax liability change if the firm had engaged more than 300 Nepalese employees
throughout the year?

Answer
Statement of Assessable income from Business
Sec.
Particulars Amount Working Notes
Ref
Inclusions:
Sales 7 (2) 55,234,100 Refer W.N. 1
Gift (Motorcycle) (Additional note "c") 7 (2) 220,000
Total Inclusions 55,454,100
Deductions
Interest expense of delivery
Interest Expense 14 250,000
van, additional note "e".
Cost of Trading Stock 15 44,155,280 Refer W.N. 2
Depreciation 19 1,967,660 Refer W.N. 3
Repair and Improvement Cost 16 338,933 Refer W.N. 3
Other Costs deductible u/s 13 13 4,288,196 Refer W.N. 4
Total Deductions 51,000,069
Assessable Income from Business 4,454,031
Less: Reductions u/s 12, 12A, 12B -
Taxable Income 4,454,031
Tax Liability @25% 1,113,508

Answer Part (b)


The concessions in taxation as a result of employing 300 Nepali nationals throughout the Income
Year are entitled to Special Industry. As per Clarification Clause to Sec. 11, Special Industry refers
to Production based industries (except through producing alcohols, beers or products using tobacco
as basic raw materials, agro and forest-based industries and mineral based industries. Since,
Juntara Trades is a trading firm, the tax rate remains 25% irrespective of it being able to employ
300 Nepali nationals.

Working Notes
Note: It is assumed that the firm has followed accrual basis of accounting for tax.
1 Adjusted Value of Sales
Given 55,194,100
Add: Commission of last year reduced from sales 40,000
Sales to be included 55,234,100

2 Cost of Trading Stock

The Institute of Chartered Accountants of Nepal 230


CAP-II Paper 7 - Income Tax and VAT

Particulars Sec. Ref Amount


Opening Stock 3,219,660
Add: Purchases/Cost of Conversion 46,455,030
Less: Closing Stock (5,519,410)
Cost of Trading Stock 44,155,280

3 Calculation of Depreciation and Repair and Improvement Cost


Particulars Block A Block B Block Block D Total
Opening Depreciation Base 11,123,750 556,190 2,500,000 4,449,500
Add: Absorbed additions
Motorcycle obtained as gift
220,000
(assumed until Poush)
Cost of Plant and Plant
Installation Cost (2/3) 700,000
(2/3rd of Rs. 1,050,000)
Less: Disposal Proceeds of Assets
(220,000)
(Sales of Motorcycle to employee)
Depreciable Basis 11,123,750 556,190 2,500,000 5,149,500
Depreciation Rate 5% 25% 20% 15%
Depreciation Amount 556,188 139,048 500,000 772,425 1,967,660

4 Expense u/s 13 Reason


Salary and Wages 3,311,646
Less: Incurred for installation of Plant (50,000) to be capitalized as plant in Block D
to be booked as receivable from
Less: TDS on Salary and Wages (40,000) 3,221,646
employee
Business Promotion Expense 500,000
Commission for 74/75 to be claimed
Add: Referral Commission for the Year 30,000 530,000
during the year
Monthly tuition fee of employees'
children is deductible, as such
Office Expenses 50,000
amount forms part of income of
employee
Expenses related to lawsuit is
Lawyers Fee 45,000
deductible
Marketing Expense 441,550
Total 4,288,196

8) Mega Vision Pvt. Ltd., Pokhara, a manufacturer and exporter of garments; has the following
incomes and expenses for Income Year (IY) 20X-74/X-75: (Dec 2018, 20 Marks)

The Institute of Chartered Accountants of Nepal 231


CAP-II Paper 7 - Income Tax and VAT

Particulars Amount (Rs.)


Export sales 50,000,000
Misc. income 2,850,000
Dividend income 950,000
Opening stock 15,000,000
Raw material import 14,000,000
Freight for raw material 900,000
Custom duty paid for raw material 1,500,000
Custom agent fee 150,000
VAT paid for import 1,500,000
Wages for production 3,000,000
Overhead cost (production) 2,000,000
Administrative expenses 1,500,000
Selling and distribution expenses 1,000,000
Taxes paid to state and local government 1,500,000
Interest paid to financial institution 300,000
Penalties paid to metropolitan office 200,000
Donation 1,100,000
Depreciation 1,200,000

The following information is relevant for income tax calculation:


a. Sales include VAT refund of Rs. 1,500,000 which was paid at the customs point for import
of raw materials.
b. Miscellaneous income Rs. 2,000,000 is the incentive given by the Government of Nepal
(GON) for export of previous year and is received during this year through Nepal Rastra
Bank as the right to such incentive was established during the current income year.
Remaining miscellaneous income Rs. 850,000 is a net of tax amount received from the bank
deposit incidental to the business.
c. Dividend income is also a net of tax amount received from a subsidiary company.
d. The opening stock and closing stock have been recorded 1,000 pieces and 1,200 pieces
respectively. The opening stock includes fixed factory overhead Rs. 10 per piece and
machineries repair and maintenance Rs. 15 per piece. During the year, the Company
produced 2,000 pieces of garments and the overhead cost includes equal rate of previous
years' fixed overhead and repair and maintenance cost.
e. Administrative expenses include employees' travelling expenses Rs. 100,000 during the
import of raw materials and also include entertainment expenses Rs. 50,000 provided to the
Custom's employees.
f. The Company has made employees' bonus provision Rs. 2,200,000 as per the Bonus Act.
During the year, it has paid Rs. 400,000 as full settlement of previous year's bonus. The

The Institute of Chartered Accountants of Nepal 232


CAP-II Paper 7 - Income Tax and VAT

administrative expense presented in the above table includes the previous year's bonus but
excludes the current year's bonus.
g. The Company has made provision for Corporate Social Responsibility (CSR) for Rs. 187,000
as 1 percent of profit after tax as per Section 48 of Industrial Enterprises Act, 2073. The
amount is included in the administrative expenses.
h. Selling and distribution expenses were paid to the cargo agent for export, invoices amounting
Rs. 300,000 from the cargo agent were received, and the remaining amount pertains to goods
transportation from the factory to depot point as per details given by the agent.
i. Interest expense is charged against the loan taken to purchase the factory machinery costing
Rs. 2,000,000. The machinery was purchased on 1st Baishakh, 20X-75 and has been put to
use from 20th Bhadra, 20X-75.
j. Donation includes Rs. 100,000 directly paid to the flood victims; the remaining amount was
deposited into Reconstruction Fund established by GON for earthquake victims.
k. Opening balance of depreciable assets as on 20X-74/4/1 as per tax return records are as
follows:
Building Rs. 6,000,000
Cars Rs. 3,000,000
Plant & Machinery Rs. 8,000,000
l. The company disposed a Santro car for Rs. 800,000 during the year and the written down
value of the car at the disposal time was Rs. 1,000,000.
You are requested to ascertain:
a. Taxable income
b. Tax liability
c. Dividend tax that must be paid by the Company, if the Company has proposed to declare
dividend of Rs. 3,000,000.

Answer:
Statement of Assessable income from Business
Export Other
Particulars Sec. Ref Working Notes
Income Income
Inclusions:
Export Sales 7 (2) 48,500,000 Refer W.N. 1
Miscellaneous Income 7 (2)
As right to receive the payment is
Export Incentive 2,000,000
established during the Income Year
Grossing up net interest applying TDS
Interest Income 1,000,000
rate of 15%
Assuming subsidiary company is
Dividend Income 92 -
resident, the income is final withholding

The Institute of Chartered Accountants of Nepal 233


CAP-II Paper 7 - Income Tax and VAT

Total Inclusions 50,500,000 1,000,000


Deductions
Asset purchased under the loan for which
interest is accrued is put to use only
Interest Expenses 14 -
during next Income year; as such interest
expense is not deductible
Cost of Trading Stock 15 23,593,000 Refer W.N. 2
Depreciation for the Year 19 2,586,667 Refer W.N. 3
Repair and Improvement
Cost 16 30,000 Refer W.N. 3
a. Travel expense of employee is
administrative expense
b. Entertainment expense is excluded
cost, so not deductible u/s 21 (1) (Cha)
Administrative Expenses 13 863,000 c. Previous year bonus does not satisfy
"during the year" criteria for expense
deduction u/s 13
d. Provision for CSR is provision only
and not allowed as deduction u/s/ 13
Selling and Distribution Expense without proper invoice is not
13 300,000
Expenses deductible
Taxes paid to State and
13 1,500,000 Specifically deductible u/s 21
Local Government
Penalties paid to
21 - Penalties against legal non compliances
Metropolitan Office
Corporate Social Expense is not incurred, and this is not a
-
Responsibility fixed obligation
Practice differs in allocating amount,
Employee Bonus
2,100,000 100,000 used 10% of assessable income before
Provision for the Year
bonus expenses.
Total Deductions 30,972,667 100,000
Assessable Income from 19,527,333
900,000
Business

Statement of Taxable income and tax liability


Export Other
Particulars Notes
Income Income
Assessable Income from Business 19,527,333 900,000
Total Assessable Income 19,527,333 900,000
Less:

The Institute of Chartered Accountants of Nepal 234


CAP-II Paper 7 - Income Tax and VAT

Paid directly to flood victim is


Reductions u/s 12 - not a qualified donation for
reduction
Reductions u/s 12Kha- National
955,941.39 44,058.61
Reconstruction Fund of GON
Taxable Income 18,571,392.00 855,941.00
Tax Rate 8% 25%
Tax Liability 1,485,711.00 213,985.00

Working Notes
1 Adjusted Sales
Given 50,000,000
Less: VAT Refund (1,500,000)
Sales to be included 48,500,000

Treatment of VAT Refund


VAT is not the expense of any business person. When a person makes payment of VAT, a VAT
Receivable Account is debited that is an account representing Receivables from Government of
Nepal. When Government refunds VAT amount, the same asset account is credited with
corresponding effect being given to bank or cash account. As such, VAT refund does not have any
impact on profitability of any person.

2 Cost of Trading Stock for the Year


Particulars Details Amount Note
Value of Opening Stock
Given 15,000,000
Repair cost does
Less: Repair of Machineries in the value (15,000) 14,985,000 not form part of
cost of stock
Purchases/Cost of Conversion
Raw Material Import 14,000,000
Freight for Raw Material 900,000
Customs duty paid for Raw Material 1,500,000
Customs Agent Fee 150,000
Receivable
VAT Paid for Import - amount, not an
expense
Wages for Production 3,000,000
Production Overhead 2,000,000
Less: Repair and Maintenance cost on OH (30,000) 21,520,000

The Institute of Chartered Accountants of Nepal 235


CAP-II Paper 7 - Income Tax and VAT

Cost of
Per unit cost of Production 10,760 conversion /
Units produced
Per unit cost of
production * No.
Value of Closing Stock (12,912,000)
of closing stock,
assuming FIFO
Cost of Trading Stock 23,593,000

3 Computation of Depreciation Allowances

Particulars Block A Block B Block C Block D Total


Opening Depreciation Base 6,000,000 - 3,000,000 8,000,000
Add: Absorbed additions
Less: Disposal Proceeds of Assets
(Sales of Santro car) (800,000)
Depreciable Basis 6,000,000 - 2,200,000 8,000,000
Depreciation Rate 5% 25% 20% 15%
Depreciation Amount (Normal) 300,000 - 440,000 1,200,000 1,940,000
One-third additional (as it is special industry) 100,000 - 146,667 400,000 646,667
Total Depreciation for the year 400,000 - 586,667 1,600,000 2,586,667

7% of Depreciable Basis 420,000 - 154,000 560,000


Actual Repair and Improvement Expense - - - 30,000
Allowable Repair and Improvement Cost
(Minimum of 7% of Depreciation Base or - - - 30,000 30,000
Actual)
Comment on Machinery Additions:
There is no addition, as the pooling date is later of use date or purchase date, and the machinery purchased using loan
is used during next I.Y.
4 Determination of Tax Rate
Income from export by manufacturing business is treated as received by separate person, as Sec.
11 is applicable and earning interest income is treated as earned by separate person.
Interest is considered as normal income and shall be taxed at 25%.
Alternatively, interest income may be treated as normal income and taxed at 8% as the
interest is related with normal business of the company.
The tax rate for interest income is 25%.
The tax rate Export by Manufacturing business
Applicable Tax Rate 25% (Schedule 1)
Reduction in Tax rate to Special Industry 20% Sec. 11 (2Kha) (Kha)

The Institute of Chartered Accountants of Nepal 236


CAP-II Paper 7 - Income Tax and VAT

Reduced Tax Rate 20%


Concessions availed to Export business 20% (Sec. 11 (3Nga) (Kha)
Rate after concession 16%
Additional Concessions to Manufacturing industry
50% (Sec. 11 (3Nga) (Ga)
that exports goods
Effective Rate 8%

9) Mustang Cashmere Manufacturing Co. Pvt. Limited is engaged in manufacturing and sale
of high-grade cashmere clothes. It provides employment opportunity to 650 Nepalese people
throughout the year. Following are the extracts of the Income Statement of the company for
the year ended on Ashad 31, 20X-74. (June 2018, 20 Marks)
Particulars Amount Rs.
Income:
Export Sales 6,000,000.00
Domestic Sales 4,000,000.00
Dividend Received (Net of Tax) 150,000.00
Rent Income (Related with Business) 50,000.00
Total Income 10,200,000.00
Expenditure:
Cost of Materials Consumed 3,000,000.00
Manufacturing Expenses 500,000.00
Employee Cost 1,000,000.00
Selling and Administrative Expenses 1,500,000.00
Interest and Bank Charges 500,000.00
Exchange Loss 250,000.00
Loss on Sale of Depreciable Assets 300,000.00
Total Expenditure 7,050,000.00
Operating Profit 3,150,000.00
Less: Provision for Bonus 300,000.00
Profit Before Tax 2,850,000.00

Additional information:
a. Cost of materials is consumed in the ratio of sales.
b. Exchange loss includes Rs 100,000.00 against revaluation of creditors at the year-end.
c. Selling and Administrative Expenses include Rs 70,000.00 donation given to Prime
Minister Disaster Relief Fund and Rs 300,000.00 given for construction of school.
d. Out of total provision for bonus, Rs 200,000.00 was distributed to the employees till the
time of filing income tax return. It has been decided by the management not to pay the
undistributed portion.
e. You are given the following information in regard to some of the expenses:

The Institute of Chartered Accountants of Nepal 237


CAP-II Paper 7 - Income Tax and VAT

i) Rs 50,000.00 included in employee cost as staff welfare is personal expenses of


directors.
ii) Manufacturing expenses include Rs 30,000.00 for electricity bill of previous year.
iii) Selling and Administrative Expenses include Rs 10,000.00 for business promotion
which is not related to business.

Based on the above information, Compute tax liability of the Company

Answer:
Statement of Assessable Income from Business
Sec. Export Other
Particulars Working Notes
Ref Income Income
Inclusions:
Export Sales 7 (2) 6,000,000
Domestic Sales 7 (2) 4,000,000
Rent Income - 50,000
Assuming received
from resident company,
Dividend Income 92 -
the income is final
withholding
Total Inclusions 6,000,000 4,050,000
Deduction of Expenses:
In proportion to Export
Interest Expenses 14 300,000 200,000 Sales and Domestic
Sales (i.e., 3:2)
Cost of Trading Stock 15 2,082,000 1,388,000 Refer W.N. 1
There is loss on sale of
depreciable asset, it is
assumed that the pool
Depreciation for the Year 19 180,000 120,000
was disposed, and
allocated in proportion
of sales.
Revaluation loss is not
deductible u/s 28.
Remaining expenses is
Exchange Loss 90,000 60,000 allocated in proportion
to Export Sales and
Domestic Sales (i.e.,
3:2)
Rs. 50,000 is not
Employee Cost 13 570,000 380,000 deductible as it is
personal expense.

The Institute of Chartered Accountants of Nepal 238


CAP-II Paper 7 - Income Tax and VAT

Remaining amount is
distributed in proportion
to sales
Selling and Administrative Expenses 13 672,000 448,000 Refer W.N. 2
Bonus provision for the
year is deductible as it is
fixed obligation
Employee Bonus Provision for the Year 180,000 120,000 Any amount
undistributed is added
back to income in the
next income year
Total Deductions 4,074,000 2,716,000
Assessable Income from Business 1,926,000 1,334,000
Statement of Taxable income and tax
liability
Assessable Income from Business 1,926,000 1,334,000
Total Assessable Income 1,926,000 1,334,000
Less:
Reductions u/s 12 -
Reductions u/s 12Ka -
Reductions u/s 12Kha- National In sales ratio, as the
42,000 28,000
Reconstruction Fund of GON question suggests
Taxable Income 1,884,000 1,306,000
Tax Rate 8% 15%
Tax Liability 150,720 195,900

Working Notes
1 Cost of Trading Stock for the Year
Cost of Materials consumed 3,000,000
Manufacturing Expenses 500,000
Less: Prior period expenses (30,000)
Total Cost 3,470,000
Towards Export Sales 2,082,000
(in sales ratio)
Towards Domestic Sales 1,388,000

2 Selling and Administrative Expense


Given 1,500,000
Less:
Donation to PM Relief Fund (70,000)
Donation for Construction of School (300,000)
Business promotion, not related to business (10,000) 1,120,000

The Institute of Chartered Accountants of Nepal 239


CAP-II Paper 7 - Income Tax and VAT

Allocated to Export sales 672,000


(in sales ratio)
Allocated to Domestic Sales 448,000

3 Identification of Tax Rate


Export Domestic
Applicable Tax Rate 25% 25%
Reduction to special industry 20% Sec. 11 (2Kha) (Kha)
Reduced Tax Rate 20% 20%
Additional Reduction due to employment 25% Sec. 11 (3) (ka)
Effective Rate due to effect of employment 15% 15%
Reduction due to export 20% 16% Sec. 11 (3Nga) (Kha)
Additional Reduction to manufacturing exporter 50% 8% Sec. 11 (3Nga) (Ga)
Lower Rate 8% 15%

10) I & M Pvt. Ltd. Kathmandu, a manufacturing company, deals in production and sales of
garments. Based upon the following information, you are requested to ascertain taxable
income and tax liability for Income Year 20X-73/X-74. (Dec 2017, 20 Marks)
Particulars Amount (Rs.)
Export Sales 3,00,00,000
Misc. Income 25,00,000
Dividend Income 10,00,000
Opening stock 75,00,000
Raw material import 70,00,000
Freight for raw material 9,00,000
Custom duty paid for raw material 15,00,000
Custom agent fee 1,50,000
VAT paid for import 15,00,000
Wages for production 30,00,000
Overhead cost 20,00,000
Administrative expenses 15,00,000
Selling and distribution expenses 10,00,000
Interest paid to Financial Institution 3,00,000
Penalties paid to Metropolitan Office 2,00,000
Donation 2,00,000
Depreciation 12,00,000
Additional information:
a) Sales include VAT refund of Rs. 10,00,000 paid at custom point for import of raw
material. Miscellaneous income Rs. 20,00,000 is the incentive given by the Nepal

The Institute of Chartered Accountants of Nepal 240


CAP-II Paper 7 - Income Tax and VAT

government for export of previous year and is received during this year through Nepal
Rastra Bank. Remaining Miscellaneous income Rs. 5,00,000 has been generated from the
bank deposit.
b) Dividend income has been received from Joint Venture.
c) The opening stock and closing stock have been recorded 1,000 pieces and 1,200 pieces
respectively. The opening stock includes factory fixed overhead Rs. 10 per piece and
repair and maintenance Rs. 15 per piece. During the year, the company produced 2,000
pieces of garments and the overhead cost includes equal rate of previous years fixed
overhead and repair and maintenance cost.
d) Administrative expenses, includes travelling cost Rs. 1,00,000 of employees during the
import of raw material, entertainment expenses Rs. 50,000 provided to the Custom's
employee and custom agent during the import.
e) The company has the policy for employee bonus; annually it makes 10 % provision from
the profit as per the Bonus Act. During this year, it has provisioned Rs. 200,000 and
booked under the administrative expenses.
f) Selling and distribution expenses were paid to the cargo agent for export, invoices
amounting Rs. 200,000 from the cargo agent were received, and the remaining amount
pertaining to the transportation of the goods from the factory to the depot point as per
details given by the agent.
g) Interest expense has been charged against the loan taken to purchase the factory
machinery costing Rs. 2,000,000. The machinery was purchased on 1st Ashad, 20X-74 and
has been used from 29th Shrawan, 20X-74.
h) The donation includes Rs. 100,000 directly paid to the flood victims; the remaining
amount was deposited into Reconstruction Fund established by Nepal government for
earthquake victims.
i) Opening balance (WDV) of depreciable assets as on 20X-73/4/1 as per books of account
are as follows:
a. Land Rs. 2,000,000
b. Building Rs. 5,000,000
c. Cars Rs. 2,600,000
d. Plant & Machinery Rs. 8,000,000
j) The company disposed a Santro car having written down value of Rs. 10,00,000 for Rs.
8,00,000 during the year.
Answer
Assessable Income from Business
Sec. Export Other
Particulars Working Notes
Ref Income Income
Inclusions:
Export Sales 7 (2) 29,000,000 Refer W.N. 1
Miscellaneous Income 7 (2)

The Institute of Chartered Accountants of Nepal 241


CAP-II Paper 7 - Income Tax and VAT

As right to receive the payment


Export Incentive 2,000,000 is established during the Income
Year
Alternatively, interest income
may be treated as normal income
Interest Income 500,000 and taxed at 8% as the interest is
related with normal business of
the company.
Assuming joint venture is
Dividend Income 92 - resident, the income is final
withholding
Total Inclusions 31,000,000 500,000
Deductions
Asset purchased under the loan
for which interest is accrued is
Interest Expenses 14 - put to use only during next
Income year; as such interest
expense is not deductible
Cost of Trading Stock 15 9,993,000 Refer W.N. 2
Depreciation for the Year 19 2,413,333 Refer W.N. 3
Repair and Improvement Cost 16 30,000 Refer W.N. 3
Net off fixed factory overhead
Overhead Cost 1,980,000
charged to cost of trading stock
a. Travel expense of employee is
administrative expense
b. Entertainment expense is in
Administrative Expenses 13 1,450,000 nature of bribe, so not deductible
u/s 21 (1) (Cha)
c. Current year bonus is eligible
expense
Any expense in excess of Rs.
Selling and Distribution 1,000 require invoice disclosing
13 200,000
Expenses Permanent Account Number of
vendor
Penalties paid to Metropolitan Penalties against legal non
21 -
Office compliances
Total Deductions 16,066,333 -
Assessable Income from
14,933,667 500,000
Business

Statement of Taxable income and tax liability

The Institute of Chartered Accountants of Nepal 242


CAP-II Paper 7 - Income Tax and VAT

Assessable Income from


14,933,667 -
Business
Total Assessable Income 14,933,667 500,000
Less:
Reductions u/s 12 -
Reductions u/s 12Ka -
Paid directly to flood victim is
Reductions u/s 12Kha- not a qualified donation for
National Reconstruction Fund of 96,760 3,240 reduction, remaining amount
GON allocated on the basis of
assessable income
Taxable Income 14,836,907 496,760
Tax Rate 8% 25%
Tax Liability 1,780,429 124,190

Working Notes
1 Adjusted Sales
Given 30,000,000
Less: VAT Refund (1,000,000)
Sales to be included 29,000,000
Treatment of VAT Refund
Creditable VAT is not the expense of any business person registered in VAT. When a person
makes payment of VAT, a VAT Receivable Account is debited that is an account representing
Receivables from Government of Nepal. When Government refunds VAT amount, the same
asset account is credited with corresponding effect being given to bank or cash account. As such,
VAT refund does not have any impact on profitability of any person.

2 Cost of Trading Stock for the Year


Particulars Details Amount Notes
Value of Opening Stock
Given 7,500,000
Repair cost does not form part of cost of
Less: Repair of Machineries in the
(15,000) 7,485,000 stock
value
1000 pieces @ Rs. 15 per piece
Purchases/Cost of Conversion
Raw Material Import 700,000
Freight for Raw Material 900,000
Customs duty paid for Raw Material 1,500,000
Customs Agent Fee 150,000
VAT Paid for Import - Receivable amount, not an expense

The Institute of Chartered Accountants of Nepal 243


CAP-II Paper 7 - Income Tax and VAT

Wages for Production 3,000,000


Fixed factory overhead charged to cost
of conversion
Alternatively, overhead cost can be
Production Overhead 20,000
treated as production overhead with
another effect on deduction of repair
cost of Rs. 30,000
Less: Repair and Maintenance cost on As it has not been allocated to cost of
6,270,000
OH production
Per unit cost of Production 3,135 Cost of conversion / Units produced
Per unit cost of production * No. of
Value of Closing Stock (3,762,000)
closing stock, assuming FIFO
Cost of Trading Stock 9,993,000

3 Computation of Depreciation Allowances

Particulars Block A Block B Block C Block D Total


Opening Depreciation Base 5,000,000 - 2,600,000 8,000,000
Add: Absorbed additions
Less: Disposal Proceeds of Assets
(Sales of Santro car) (800,000)
Depreciable Basis 5,000,000 - 1,800,000 8,000,000
Depreciation Rate 5% 25% 20% 15%
Depreciation Amount (Normal) 250,000 - 360,000 1,200,000 1,810,000
One-third additional (as it is special
industry) 83,333 - 120,000 400,000 603,333
Total Depreciation for the year 333,333 - 480,000 1,600,000 2,413,333

7% of Depreciable Basis 350,000 - 126,000 560,000


Actual Repair and Improvement
Expense
(2000 units @ 15 per unit) - - 30,000
Allowable Repair and
Improvement Cost
- - - 30,000 30,000
(Minimum of 7% of Depreciation
Base or Actual)
Comment on Machinery Additions:
There is no addition, as the pooling date is later of use date or purchase date, and the machinery purchased using
loan is used during next I.Y.
Land is not a depreciable asset

The Institute of Chartered Accountants of Nepal 244


CAP-II Paper 7 - Income Tax and VAT

4 Determination of Tax Rate


Income from export by manufacturing business is treated as received by separate person, as Sec.
11 is applicable and earning interest income is treated as earned by separate person.
The tax rate for interest income is 25%.
The tax rate Export by Manufacturing business
Applicable Tax Rate 25% (Schedule 1)
Reduction in Tax rate to Special Industry 20% Sec. 11 (2Kha) (Kha)
Reduced Tax Rate 20%
Concessions availed to Export business 20% (Sec. 11 (3Nga) (Kha)
Rate after concession 16%
Additional Concessions to Manufacturing industry
50% (Sec. 11 (3Nga) (Ga)
that exports goods
Effective Rate 8%

11) Citizen Cement Ltd. is a company situated in Biratnagar and listed in Nepal Stock Exchange
and engaged in manufacturing and sale of premium grade cement for Nepal and Export.
Following is the provisional Income Statement of Citizen Cement Ltd., for the year ended
Ashad 31, 20X-73.
Particulars Amount (Rs.)
Income
Export Sales 60,00,000
Domestic Sales 40,00,000
Dividend Received (Net of Tax) 1,50,000
Rent Income (Related with Business) 50,000
Total Income 1,02,00,000
Expenditure
Cost of Materials Consumed 30,00,000
Manufacturing Expenses 5,00,000
Employee Cost 10,00,000
Selling and Administrative Expenses 15,00,000
Interest and Bank Charges 5,00,000
Exchange Loss 2,50,000
Depreciation 5,00,000
Bad Debt Written Off 70,000
Loss on Sale of Assets 3,00,000
Total Expenditure 76,20,000
Operating Profit 25,80,000
Add: Provision for Doubtful Receivables Written Back 7,20,000

The Institute of Chartered Accountants of Nepal 245


CAP-II Paper 7 - Income Tax and VAT

Less: Provision for Bonus 3,00,000


Profit Before Tax 30,00,000

Additional information:
a) Cost of material consumed
For export sales : Rs. 18,00,000
For local sales : Rs. 12,00,000
b) Exchange loss includes Rs. 1,00,000 against revaluation of creditors at the year-end date.
c) Asset detail for income tax purpose is as below –
Computer, Office
Plant and
Building Equipment and Vehicles
Machinery
Furniture
Opening Depreciation Base 11,00,000 1,00,00,000 15,00,000 29,50,000
Addition upto Poush End - - 2,00,000 -
Addition Magh to Chaitra end 6,00,000 - 1,50,000 -
Addition Baishak to Ashad - - - 9,00,000
End
Sales Proceed - 15,00,000

d) Manufacturing expenses includes repair and maintenance expenses as below:


Building Repair and Maintenance : Rs. 1,50,000
Office Equipment Repair and Maintenance : Rs. 50,000
Vehicle Repair and Maintenance : Rs. 20,000
e) Selling and Distribution Expenses includes Rs. 7,00,000 donations given to Prime
Minister Disaster Relief Fund and Rs. 3,00,000 given for construction of school.
f) Bonus of Rs. 2,00,000 only distributed to the employees till the time of filing of income
tax return. It has been decided by the management not to pay the undistributed portion.
g) Following expenses are of below nature:
i) Rs. 50,000 included in employee cost for staff welfare is for personal use of directors.
ii) Manufacturing expenses includes Rs. 30,000 for electricity bill of previous years.
iii) Selling and administrative expenses includes Rs. 10,000 for business promotion which
is not related to business.
h) Company employed 1,300 Nepali employees during the whole year out of which 50
employees are foreign employee.
Based on the above information, please advise Citizen Cement Ltd. regarding.
(June 2017, 5+15=20)
a. Various tax exemptions available to them as per Income Tax Act, 2058, and which tax
exemptions Citizen Cement Ltd. should opt for?
b. Compute taxable income and tax liability of the company segregating income on the basis
of cost of material consumed for common cost and income.

The Institute of Chartered Accountants of Nepal 246


CAP-II Paper 7 - Income Tax and VAT

Answer:
Statement of Assessable income from Business
Export Other
Particulars Sec. Ref Working Notes
Income Income
Inclusions:
Export Sales 7 (2) 6,000,000
Domestic Sales 7 (2) 4,000,000
Rent Income - 50,000
Assuming subsidiary
Dividend Income 92 - company is resident, the
income is final withholding
Total Inclusions 6,000,000 4,050,000
Deductions
In proportion to Export Sales
Interest Expenses 14 300,000 200,000
and Domestic Sales (i.e. 3:2)
Cost of Trading Stock 15 1,950,000 1,300,000 Refer W.N. 1
Depreciation for the Year 19 1,900,000 1,266,667 Refer W.N. 3
Repair and Improvement Cost 16 105,000 70,000 Refer W.N. 3
Revaluation loss is not
Exchange Loss 100,000 50,000
deductible u/s 24.
Rs. 50,000 is not deductible
as it is personal expense.
Employee Cost 13 570,000 380,000 Remaining amount is
distributed in proportion to
sales
Selling and Administrative
13 294,000 196,000 Refer W.N. 2
Expenses
Bonus provision for the year
is deductible as it is fixed
Employee Bonus Provision for the obligation.
180,000 120,000
Year Any amount undistributed is
added back to income in the
next income year.
Assumed that conditions of
Bad Debt written off
Sec. 40 (3) (ga) not met
Is considered while
Loss on Sale of Assets computing depreciation
allowances
Total Deductions 5,399,000 3,582,667
Assessable Income from Business 601,000 467,333

The Institute of Chartered Accountants of Nepal 247


CAP-II Paper 7 - Income Tax and VAT

Statement of Taxable income and tax liability


Assessable Income from Business 5,399,000 3,582,667
Total Assessable Income 601,000 467,333
Less:
Reductions u/s 12 -
Reductions u/s 12Ka -
Reductions u/s 12Kha- National In sales ratio, as the question
420,000 280,000
Reconstruction Fund of GON suggests
Taxable Income 181,000 187,333
Tax Rate 8% 14%
Tax Liability 14,480 26,227

Working Notes
1 Cost of Trading Stock for the Year
Cost of Materials consumed 3,000,000
Manufacturing Expenses 500,000
Less: Repair and Maintenance expenses (220,000)
Less: Prior period expenses (30,000)
Total Cost 3,250,000
Towards Export Sales 1,950,000 (in sales ratio)
Towards Domestic Sales 1,300,000

2 Selling and Administrative Expense


Given 1,500,000
Less:
Donation to PM Relief Fund (700,000)
Donation for Construction of School (300,000)
Business promotion, not related to business (10,000) 490,000
Allocated to Export sales 294,000 (in sales ratio)
Allocated to Domestic Sales 196,000

3 Computation of Depreciation Allowances


Calculation of Depreciation and Repair and Improvement Cost
Particulars Block A Block B Block C Block D Total
Opening Depreciation Base 1,100,000 1,500,000 2,950,000 10,000,000
Add: Absorbed additions
Up to Poush (100% of cost) 200,000
From Magh to Chaitra (2/3rd of
Cost) 400,000 100,000

The Institute of Chartered Accountants of Nepal 248


CAP-II Paper 7 - Income Tax and VAT

From Baisakh to Ashad (1/3rd of


Cost) 300,000
Less: Disposal Proceeds of Assets
(Sales of Santro car) (1,500,000)
Depreciable Basis 1,500,000 1,800,000 1,750,000 10,000,000
Depreciation Rate 5% 25% 20% 15%
Depreciation Amount (Normal) 75,000 450,000 350,000 1,500,000 2,375,000
One-third additional (as it is special
industry) 25,000 150,000 116,667 500,000 791,667
Total Depreciation for the year 100,000 600,000 466,667 2,000,000 3,166,667

7% of Depreciable Basis 105,000 126,000 122,500 700,000


Actual Repair and Improvement Expense 150,000 50,000 20,000 -
Allowable Repair and Improvement
Cost
105,000 50,000 20,000 - 175,000
(Minimum of 7% of Depreciation Base
or Actual)

4 Identification of Tax Rate


Particulars Details Export Domestic
Applicable Tax Rate 25% 25%
Sec. 11 (2Kha)
Reduction to special industry 20%
(Kha)
Reduced Tax Rate 20% 20%
Additional Reduction due to employment 30% Sec. 11 (3) (ka)
Effective Rate (Option 1) 14% 14%
Sec. 11 (3Nga)
Reduction due to export 20% 16%
(Kha)
Additional Reduction to manufacturing Sec. 11 (3Nga)
50% 8%
exporter (Ga)
Effective Rate (Option 2) 8%
Applicable Tax Rate 8% 14%

As per Sec. 11 (3) (ka), where a special industry provides employment to 1,000 or more Nepali
nationals, the tax rate is 70% of reduced tax rate, i.e. 14%
Application of Sec. 11 (3Nga) jointly read with Sec. 11 (3Kha) will give tax rate of 50% in case
of export of goods manufactured in Nepal.
As such, tax rate of 14% is applicable in case of domestic sales income and that of 8% is applicable
in case of export income, as a rational taxpayer opts for beneficial options.

The Institute of Chartered Accountants of Nepal 249


CAP-II Paper 7 - Income Tax and VAT

12) Heritage Fashion Pvt. Ltd. is a garment industry; which exports all its garments to USA.
The company maintains its books of account in cash basis. The company records in a loose
sheet of the transactions of sales and purchases before accounting in the books. The receipts
and payments based on the accounting of the company during the Income Year 20X-72/X-
73 are as follows:
Particulars Payments (Rs.) Receipts (Rs.)
Loan 2,500,000 4,200,000
Raw material 6,000,000
Wages for labour 500,000
Variable overhead for production 250,000
Factory rent 1,200,000
Production supervisory salary 250,000
Insurance for factory 117,000
Administrative salary 650,000
Office expenses 400,000
Plant & Machinery 4,200,000
Generator 800,000
Sale of old computer 10,000
Export 12,000,000
Advance Income tax 200,000
17,067,000 16,210,000

You are asked by the Company to assess the tax payable for the Income year based on the
above transactions and following additional information: (December 2016, 20 Marks)
a) Closing balance of Income Year 20X-71/X-72 related to the above transactions were
assessed as follows:
Particulars Amount (Rs.)
Loan 2,500,000
Garment stocks (1000 units) 2,400,000
Plant & Machinery 800,000
Computers 170,000
Cars 1,600,000
b) The company has borrowed the loan for the purchase of plant & Machinery on 1st Magh,
20X-72 from a commercial bank. The opening loan was paid on 30th kartik, 20X-72. The
agreed rate of interest was 12 % for this loan including opening.
c) Normal production capacity of the company is 5000 units per year, but actual production
was 4500 units during the income year. The company has 1,500 units in stocks as per
FIFO method at the end of the year.
d) The company has purchased a generator on 15 Chaitra 20X-72.

The Institute of Chartered Accountants of Nepal 250


CAP-II Paper 7 - Income Tax and VAT

e) Payment of office expense includes advance amounting of Rs. 200,000 to Fulbari printing
Pvt. Ltd.
f) No need to consider the VAT amount on above transactions.

Answer
Statement of Assessable income from Business
Sec. Export
Particulars Working Notes
Ref Income
Inclusions:
Export Sales 7 (2) 12,000,000
Total Inclusions 12,000,000
Deductions
Assuming the asset purchased utilizing loan
has been in use during the year.
Interest Expenses 14 327,000
(2,500,000x12%x3/12+
4,200,000x12%x6/12)
Cost of Trading Stock 15 7,944,667 Refer W.N. 1
Depreciation for the Year 19 1,600,000 Refer W.N. 2
Administrative Salary 13 650,000 Assumed that it is accrued and paid
Office Expenses 13 200,000 Prepaid expense is not deductible
Total Deductions 10,721,667
Assessable Income from Business 1,278,333

Statement of Taxable income and tax liability

Assessable Income from Business 1,278,333


Total Assessable Income 1,278,333
Less:
Reductions u/s 12 -
Reductions u/s 12Ka -
Reductions u/s 12Kha- National
-
Reconstruction Fund of GON
Taxable Income 1,278,333
Tax Rate 8%
Tax Liability 102,267

Working Notes
1 Cost of Trading Stock for the Year
Value of Opening Stock 2,400,000
Cost of Conversion

The Institute of Chartered Accountants of Nepal 251


CAP-II Paper 7 - Income Tax and VAT

Purchase of Raw Material 6,000,000


Wages for Labour 500,000
Variable Factory Overhead 250,000
Factory Rent (Fixed Factory OH) 1,200,000
Production Supervisors Salary 250,000
Insurance for Factory 117,000
Total Cost of Production 8,317,000
Cost of conversion divided
Per unit Cost of Production 1,848.22
by actual production units
Per unit cost of Production
Value of Closing Stock (2,772,333) multiplied by no. of closing
stocks units
Cost of Trading Stock for the Year 7,944,667

Computation of Depreciation
2 Allowances
Calculation of Depreciation and Repair and Improvement Cost
Particulars Block A Block B Block C Block D Total
Opening Depreciation Base - 170,000 1,600,000 800,000
Add: Absorbed additions
From Magh to Chaitra (2/3rd of Cost)
(Purchase of generator and Plant & 3,333,333
machinery- Refer below)
Less: Disposal Proceeds of Assets
(10,000)
(Sales of old Computer)
Depreciable Basis - 160,000 1,600,000 4,133,333
Depreciation Rate 6.67% 33.33% 26.67% 20.00%
Normal Depreciation - 53,333 426,667 826,667 1,306,667
Add: Special depreciation for power
generating asset (800000x50%- - - - 293,333 -
800,000x2/3x20%)
Total Depreciation Expenses - 53,333 426,667 1,120,000 1,600,000
Note: It is assumed that the plant and machinery is used since the date of purchase

3 Identification of Tax Rate


Export
Applicable Tax Rate 25%
Reduction to special industry 20% Sec. 11 (2Kha) (Kha)
Reduced Tax Rate 20%
Reduction due to export 20% 16% Sec. 11 (3Nga) (Kha)

The Institute of Chartered Accountants of Nepal 252


CAP-II Paper 7 - Income Tax and VAT

Additional Reduction to manufacturing exporter 50% 8% Sec. 11 (3Nga) (Ga)


Lower Rate 8%

13) Mr. Hari Shrestha has a business of selling Computers. During the FY 20X-72/X-73, he has
made a profit of Rs. 15,00,000 from sale of Computer business. Further, during the year, he
has sold his entire office equipment and furniture related with computer business for Rs.
10,00,000. He has purchased those office equipment and furniture for Rs. 15,00,000, and
opening WDV as per last year’s tax return of the said office equipment and furniture was
Rs. 7,00,000. He has purchased no any other office equipment and furniture during the year,
and he has no any other office equipment and furniture used in other business as well.
Mr. Hari Shrestha has another business of developing Compute Software. The Computer
software development business was run on rented office space. During the FY 20X-72/X-73,
he has incurred net loss of Rs. 6,00,000 in the said business of developing Compute Software.
His business of developing Compute Software has been in loss for past several years, and has
carried forwarded eligible business loss of Rs 10,00,000 as per his last years tax return.
Mr. Hari Shrestha has incurred loss of Rs 5,00,000 in sale of Shares of a Bank. He has made
profit of Rs. 3,00,000 in sale of shares of an Insurance Company.
Calculate net taxable income of Mr. Hari Shrestha for the FY 20X-72/X-73. (Dec 2016, 5
Marks)

Answer
Assessable Income from Computer Business Rs. 1,800,000
(Rs. 1,500,000+Gain on Disposal of Block B of Depreciable Asset)
Loss from Software Business- Rs. 600,000
Carried forward business loss on Computer software business- Rs. 1,000,000
(assumed to be since last 7 Income Years)
Loss on Sale of Shares of Bank Rs. 500,000
Profit on Sale of shares of insurance companies 300,000

As per 20 (1), business loss of any Income Year can be set off against another business income of
the same person, or investment income of the same person. Here, Computer selling business and
software development business is not treated as conducted by separate person u/s 11 of the Act,
therefore, loss on Computer software business can be set off against computer trading business.
Furthermore, the loss of previous seven income years can be set off against business or investment
income of current Income Year.

As per Sec. 20 (2), investment loss of same year or of previous seven income years can only be set
off against investment gain.

The Institute of Chartered Accountants of Nepal 253


CAP-II Paper 7 - Income Tax and VAT

As per Sec. 20 (6), the taxpayer has the choice to make to set off losses, and a rational taxpayer
always applies the choices such that there is less tax obligation.

Therefore, Mr. Hari Shrestha will set off investment loss of Rs. 5 Lakhs against investment income
of Rs. 300,000 and will carry forward Rs. 2 Lakhs loss for set off in subsequent income year(s).

Mr. Hari Shrestha will apply business income of Rs. 18 Lakhs to set off current year loss of Rs. 6
Lakhs and previous year loss of Rs. 10 Lakhs, and will have assessable income from business of
Rs. 2 Lakhs.

14) Shalimar Trading & Manufacturing Co. Ltd., located at Birjung in which Garima
Charitable Trust an exempt entity holds 35% of share capital. Shalimar Trading &
Manufacturing Co. Ltd. provide you manufacturing and profit and loss account for the
financial year 20X-71/X-72 are as follows:
Particulars Amount Particulars Amount
(Rs.) (Rs.)
To, Opening Stock RM 550,000 By, Sales 3,000,000
To, Purchases 1,150,000
To, Direct mfg. costs 300,000
To, Freight 100,000
To, Gross Profit 1,100,000 By, Closing Stock 200,000
Total 3,200,000 Total 3,200,000
To, Office Expenses 75,000 By, Gross Profit 1,100,000
To, Depreciation calculated 125,000 By, Interest-Business (net) 85,000
as per IT Act
To, Interest expenses 225,000
To, Repair & Maintenance 85,000 By, Bad debts recovered 185,000
To, Salaries 625,000 By, Dividend (net) from 340,000
Resident. Co.
To, Fine & Penalties 5,000
To, Income Tax Provision 18,000
To, Net Profit 552,000
Total 1,710,000 Total 1,710,000

Additional information:
• Interest expenses include Rs. 185,000 paid to Garima Charitable Trust.
• Bad debts recovered during the year were not allowed to deduct during last year.
• Opening stock is undervalued by 20% and closing stock is overvalued by 25%.
• Depreciation is not allowed related to assets under Block-C category by Rs. 25,000, which
is included in calculated figure Rs. 125,000.

The Institute of Chartered Accountants of Nepal 254


CAP-II Paper 7 - Income Tax and VAT

• Repair & maintenance include Rs. 45,500 related to assets under Block-B.
• For the purpose of repair and maintenance, opening depreciation base of Block-B is Rs.
600,000 and addition of Block-B assets as on 25th Jestha 20X-72 amounted to Rs. 150,000.
(Ignore additional depreciation, if applicable, for repair and maintenance limit
calculation).
• Donation provided to tax exempt entity amounted to Rs. 5,500.

Required: (June 2016, 15+5=20)


a. Compute the taxable business income & tax liability of Shalimar trading &
manufacturing Co. Ltd. with working notes for the financial year 20X-71/X-72 as per
Income Tax Act, 2058?
b. Explain “a resident entity controlled by an organization entitled to tax exemption" as per
explanation provided for the purpose of section 14(2) of Income Tax Act, 2058?

Answer
Assessable Income of Shalimar Trading & Manufacturing
Export
Particulars Sec. Ref Working Notes
Income
Inclusions:
Sales 7 (2) 3,000,000
Interest Income 7 (2) 100,000 Gross up 85,000/85%
Not allowed as
Bad debt recovered 25 -
deduction previously
Dividend from resident company 92 - Final withholding

Total Inclusions 3,100,000


Deductions
Cost of Trading Stock 15 1,987,500 Refer W.N. 1
Given less not allowed
Depreciation for the Year 19 100,000
amount of Rs. 25000
Repair and Improvement Cost 16 85,000 Refer W.N. 2
Salaries 13 625,000
Office Expenses 13 75,000
Not deductible, as it is
assumed to be paid for
Fine and Penalties 21 -
non-compliance of
laws and regulations
Tax paid under Income
Income Tax Provision 21 -
Tax Act

The Institute of Chartered Accountants of Nepal 255


CAP-II Paper 7 - Income Tax and VAT

Total deductions before adjustment to determine ATI 2,872,500


Paid to other than
Interest expense where Sec. 14 (2) is not applicable 40,000 entity with controlling
interest
Interest expense where Sec. 14 (2) is applicable 163,750 Refer W.N. 3
Total Deductions 3,076,250
Assessable Income from Business 23,750

Statement of Taxable income and tax liability


Assessable Income from Business 23,750
Total Assessable Income 23,750
Less:
Reductions u/s 12 - Refer W.N. 4
Reductions u/s 12Ka -
Reductions u/s 12Kha -
Taxable Income 23,750
Tax Rate 20%
Tax Liability 4,750
Working Notes
1. Cost of Trading Stock for the Year
Value of Opening Stock
Given 550,000
Add: Undervalued amount 137,500 687,500
Cost of Conversion
Purchase of Raw Material 1,150,000
Direct Manufacturing cost 300,000
Freight 10,000 1,460,000
Value of Closing Stock
Given 200,000
Less: Overvalued amount (40,000) (160,000)

Cost of Trading Stock for the Year 1,987,500

2. Computation of Repair and Improvement Cost


The question specifies that repair cost of Rs. 45,500 is related to Block B, and silent about remaining repair cost.
It is assumed that the remaining repair cost is eligible under Sec. 16
Calculation of Repair and Improvement Cost
Particulars Block B Total
Opening Depreciation Base 600,000
Add: Absorbed additions

The Institute of Chartered Accountants of Nepal 256


CAP-II Paper 7 - Income Tax and VAT

Up to Poush (100% of cost)


From Magh to Chaitra (2/3rd of Cost)
From Baisakh to Ashad (1/3rd of Cost) 50,000
Less: Disposal Proceeds of Assets
Depreciable Basis 650,000

7% of Depreciable Basis 45,500


Actual Repair and Improvement Expense 45,500
Allowable Repair and Improvement Cost
45,500
(Minimum of 7% of Depreciation Base or Actual) 45,500

Add: Remaining Repair and Improvement Cost


39,500

Total Eligible cost


85,000

3. Adjusted Taxable income and the eligible expenses for Sec. 14(2)
Inclusions before Interest Income 3,000,000
Deductions before Interest expenses, PCC, and R&D
(2,872,500)
Cost
Deductions of PCC and R&D Cost (Actual) -
Reductions u/s 12, 12A -
Adjusted Taxable Income 127,500
50% of Adjusted Taxable Income (i) 63,750
Add: Interest Income (ii) 100,000
Maximum Eligible amount (A= i+ii) 163,750
Actual accrued amount (B) 185,000
Eligible amount (Minimum of "A" or "B") 163,750

4. Adjusted Taxable income and the eligible expenses for Sec. 12


Eligible Donation u/s 12
Inclusions before Interest Income 3,000,000
Deductions before Interest expenses, PCC, and R&D
(2,872,500)
Cost
Deductions of PCC and R&D Cost (Actual) -
Deduction of Interest Expense- Sec. 14(2) not applied-
(40,000)
eligible
Deduction of Interest Expense- Sec. 14(2) applied-
(185,000)
actual
Reductions u/s 12, 12A -
Adjusted Taxable Income (97,500)
Since ATI is negative, there is no reduction of donation

The Institute of Chartered Accountants of Nepal 257


CAP-II Paper 7 - Income Tax and VAT

Identification of Tax Rate


Special
Industry
Applicable Tax Rate 25%
Reduction to special industry 20% Sec. 11 (2Kha) (Kha)
Reduced Tax Rate 20%

15) An accounting firm owned by Amber Poudel maintains its books of account on cash basis.
Mr. Poudel submitted his financial information for the financial year 20X-71/X-72 as follows:
Receipts Amount in Payments Amount in
(Rs.) (Rs.)
To, Opening cash/bank balance. 60,000 By, office expenses 75,000
To, Consultancy fees-gross 648,000 By, salary to staff 425,000
To, Book writing fee 1,700,000 By, Life Insurance policy 26,000
To, Bank Loan @ 10 % 600,000 By, Purchase of furniture 90,000
To, Sale of old journals 12,000 By, Office Rent 500,000
To, Lottery prize 150,000 By, Interest Exp. 60,000
To, Paper checking from T.U. 200,000 By, Water & Electricity 125,000
Expenses
To. Part time lecture 135,000 By, Purchase of journals 4,000
By, Transportation Expenses 100,000
By, Closing cash/bank bal. 2100,000
3,505,000 3,505,000

Additional information:
a. Salary expenses include salary paid to a staff for last three years amounted to Rs. 125,000
and advance salary given to another staff amounted to Rs. 25,000.
b. Opening WDV of furniture was Rs. 170,000 and during the year additional furniture was
purchase during the end of Jestha, 20X-72.
c. Consultancy fee received includes advance from Paban Limited Rs. 125,000.
d. Office Rent paid includes payments for financial 207X-2/X-73, 20X-73/X-74 & 20X-74/X-
75
e. Fifty percent of loan used for personal purpose by Mr. Poudel.
Required: (Dec 2015, 5+5=10)
Compute the taxable income and tax liability of accounting firm owned Mr. Poudel for the
financial year 20X-71/X-72.
Answer
Since, a private firm can follow cash basis of accounting, hence, the income is calculated under cash
basis.

The Institute of Chartered Accountants of Nepal 258


CAP-II Paper 7 - Income Tax and VAT

Assessable Income of the


accounting firm
Sec.
Particulars Working Notes
Ref Amounts
Inclusions:
Consultancy Fees 7 (2) 648,000
Book writing Fee 7 (2) 1,700,000
Sale of Old Journals 7 (2) 12,000
Lottery Prize 92 - Final withholding
Paper checking from T.U. 92 - Final withholding
Assumed occasional, and hence, final
Part time lecture 92 -
withholding

Total Inclusions 2,360,000


Deductions
50% loan used for personal purpose,
Interest expense 14 30,000
interest on which is not deductible
Depreciation for the Year 19 50,000 Refer W.N. 1
Salary to Staff 13 425,000 Cash basis, hence all deductible
Office Rent 13 500,000 Cash basis, hence all deductible
Water & Electricity Expense 13 125,000
Purchase of Journals 13 4,000
Transportation Expenses 13 100,000
Total Deductions 1,234,000

Assessable Income from Business 1,126,000

Statement of Taxable Income and Balance Taxable Income


Assessable Income from Business 1,126,000
Assessable Income from Employment -
Assessable Income from Investment -
Total Assessable Income 1,126,000
Less: Reductions u/s 12, 12Kha & 63 -
Taxable Income 1,126,000
Less: Deductions under Sec. 1 of Sch. 1
Life Insurance Premium Paid
(Rs. 40,000 or actual, whichever is lower) 25,000
Balance Taxable Income 1,101,000

The Institute of Chartered Accountants of Nepal 259


CAP-II Paper 7 - Income Tax and VAT

Statement of Tax Liability (Couple)


1st Rs. 600,000 0% -
Next Rs. 200,000 10% 20,000
Next Rs. 300,000 20% 60,000
Balance Rs. 1,000 30% 300
Total Tax Liability 80,300

Working Note
1. Depreciation
Block B
Opening Depreciation Base 170,000
Add: Absorbed Additions 30,000
(1/3rd of 90,000)
Less: Disposal Proceeds -
Depreciable Basis 200,000
Depreciation @ 25% 50,000

16) Gorkhali Cement (P.) Ltd., Rupendehi, has the following transactions during the year 20X-
71/X-72:
Profit & loss Account
Particulars Amount (Rs.)
Sales 31,000,000
Cost of Sales 17,600,000
Gross profit 12,400,000
Other Income 500,000
Administrative expenses 2,000,000
Selling & Distribution Expenses 600,000
Depreciation 6,800,000
Interest expenses 2,000,000
Profit before tax 2,500,000

Additional Information:
a. Other income was the interest from Bank of Kathmandu. It has presented net of Tax.
b. Cost of sales includes the following:
Particulars Amount (Rs.) Amount (Rs.)
Opening stocks 700,000
Clinker and Other raw material purchase 18,000,000
Manufacturing Expenses

The Institute of Chartered Accountants of Nepal 260


CAP-II Paper 7 - Income Tax and VAT

Salary and wages for factory 1,200,000


Repair and maintenance expenses 400,000
Other overhead expenses 300,000
Total manufacturing expenses 1,900,000
Closing stocks 3,000,000
Cost of sales 11,900,000

c. Cost of stocks includes the repair and maintenance expenses, which is approximately 2
% of the valuation.
d. Depreciation includes Rs. 2,000,000 for the construction of Factory which was
completed on 20 Mangsir, 20X-71 with total cost of Rs. 40,000,000 without charging the
interest. Depreciation for others are as follows:
Block Depreciation base (Rs.) Depreciation (Rs.)
B 1,200,000 300,000
D 30,000,000 4,500,000
e. Interest expense was charged for the loan borrowed for factory construction. The loan
Rs. 20,000,000 with 10 percent interest rate was disbursed on 1st Ashad 20X-71 from
concerned bank and remains even during the year.

Based on the above information, you are required to calculate Taxable income and Tax
liability of the Pvt. Ltd. for Income Year 20X-71/X-72. (July 2015, 10 Marks)

Answer
Statement of Assessable income from Business (Gorkhali Cements)
Particulars Sec. Ref Amount Working Notes
Inclusions:
Sales 7 (2) 31,000,000
Separate treatment not done, and
Other Income (500,000 divided by 0.85) 7 (2) 588,235
grossed up to consider TDS
Total Inclusions 31,588,235
Deductions
Factory is used during the Income
Interest Expenses 14 2,000,000
Year, as such, expense is deductible
Cost of Trading Stock 15 17,246,000 Refer W.N. 1
Depreciation for the Year 19 9,079,117 Refer W.N. 2
Refer W.N. 2, assuming the repair
Repair and Improvement Cost 16 240,000
costs belong to Block D
Administrative Expense 13 2,000,000
Selling and Distribution Expense 13 600,000
Total Deductions 31,165,117

The Institute of Chartered Accountants of Nepal 261


CAP-II Paper 7 - Income Tax and VAT

Assessable Income from Business 423,118


Statement of Taxable income and tax
liability
Assessable Income from Business 423,118
Taxable Income 423,118
Tax Rate 20%
Tax Liability 84,624

Working Notes
1. Cost of Trading Stock for the Year
Value of Opening Stock
Given 700,000
Less: Repair and Maintenance of Assumed the repair expense is of
(14,000) 686,000
Depreciable asset depreciable asset

Cost of Conversion
Purchase of Raw Material (Clinker and
Others) 18,000,000
Salary and Wages for Factory 1,200,000
Repair & Maintenance Expenses - Assumed to be of Depreciable asset
Other Overhead Expenses 300,000
19,500,000
2% of value of inventory is repair
Value of Closing Stock (2,940,000)
expenses.
Cost of Trading Stock for the Year 17,246,000

2. Computation of Depreciation Allowances


Calculation of Depreciation and Repair and Improvement Cost
Particulars Block A Block B Block D Total
Opening Depreciation Base 1,200,000 30,000,000
Add: Absorbed additions
Up to Poush (100% of cost)
(Factory construction, interest cost of Ashad 20X-71 40,166,667
capitalized)
From Magh to Chaitra (2/3rd of Cost) -
From Baisakh to Ashad (1/3rd of Cost)
Less: Disposal Proceeds of Assets
Depreciable Basis 40,166,667 1,200,000 30,000,000
Depreciation Rate 6.67% 33.33% 20.00%
Depreciation Amount 2,679,117 400,000 6,000,000 9,079,117

The Institute of Chartered Accountants of Nepal 262


CAP-II Paper 7 - Income Tax and VAT

7% of Depreciable Basis 2,811,667 84,000 2,100,000


Actual Repair and Improvement Expense - - 400,000
Allowable Repair and Improvement Cost
- - 400,000 400,000
(Minimum of 7% of Depreciation Base or Actual)

Identification of Tax Rate Export


Applicable Tax Rate 25%
Reduction to special industry 20% Sec. 11 (2Kha) (Kha)
Reduced Tax Rate 20%

17) Income Statement of a company registered under Company Act, 2063 for the income year
20X-67/X-68 is given below:
Expense Rs. Income Rs.
Cost of Goods sold 7,000,000 Sales 10,000,000
Interest expense 500,000 Interest income 100,000
Personnel expense 1,500,000 Other income 200,000
Administrative expense 900,000
Net profit 400,000
10,300,000 10,300,000
Further information is as follows:
a. Cost of goods sold includes factory depreciation Rs. 300,000, the allowable depreciation
as per Income Tax Act is Rs. 250,000 only. Similarly, administrative expense includes
depreciation of Rs. 100,000 and the allowable depreciation is Rs. 250,000.
b. The shareholders of the company are foreigners and 50% of interest expense is the
amount paid to them, remaining amount is paid for foreign bank loan.
c. Donation of Rs. 20,000 given to an exempt entity is included in administrative expense;
staff themselves donated to the same entity Rs. 70,000.
From the above information, find current tax payable by the Company. (Dec 2011, CA
Inter, 15/ June 2010, 15 Marks, CA Inter))

Answer
Statement of Assessable income from
Business
Particulars Sec. Ref Amount Working Notes
Inclusions:
Sales 7 (2) 10,000,000
Interest Income 7 (2) 100,000
Other Income 7 (2) 200,000
Total Inclusions 10,300,000
Deductions

The Institute of Chartered Accountants of Nepal 263


CAP-II Paper 7 - Income Tax and VAT

Cost of Trading Stock 15 6,700,000 Refer W.N. 1


Refer additional information (a) and
Depreciation for the Year 19 250,000 considering that total allowable
depreciation is Rs. 250,000.s
Deduction of depreciation and
Administrative Expense 13 780,000
donation included in this expense
Personnel Expenses 13 1,500,000
Deductions before considering Interest
9,230,000
Expense, PCC and R&D Cost
Interest expense, u/s 14 (1) 250,000 Paid to foreign bank
Interest expense, where Sec. 14 (2) is Refer W.N. 2, paid to person having
250,000
applicable controlling interest
Total Deductions 9,730,000
Assessable Income from Business 570,000
Statement of Taxable income and tax
liability
Assessable Income from Business 570,000
Total Assessable Income 570,000
Less:
Reductions u/s 12 20,000 Refer W.N. 3
Reductions u/s 12Ka -
Reductions u/s 12Kha- National
-
Reconstruction Fund of GON
Taxable Income 550,000
Tax Rate 25% Since it is a trading company
Tax Liability 137,500

Working Notes
1. Cost of Trading Stock for the Year
Cost of Goods Sold
Given 7,000,000
Less: Depreciation of factory included in COGS (300,000) 6,700,000
Cost of Trading Stock for the Year 6,700,000
2. Calculation of Allowable Interest expense

Inclusions without considering Interest Income 10,200,000


Deductions without considering interest expense, PCC and
9,230,000
R&D Cost
Less:
Actual PCC incurred during the Year -
Actual R&D Cost incurred during the Year -

The Institute of Chartered Accountants of Nepal 264


CAP-II Paper 7 - Income Tax and VAT

Expenditure u/s 12A -


Contribution u/s 12B - -
Adjusted Taxable Income for the purpose of Sec. 14 (2) 970,000

Allowable:
Interest income plus 50% ATI Calculated Above (A) 585,000
Actual interest expenses in respect of controlling person (B) 250,000
Eligible ("A" or "B", whichever is lower) 250,000

3. Calculation of Allowable Donation

Inclusions 10,300,000
Deductions without considering interest expense, PCC and
9,230,000
R&D Cost
Less:
Eligible Interest expense, where Sec. 14 (2) is not applicable 250,000
Actual Interest Expense, where Sec. 14 (2) is applicable 250,000
Actual PCC incurred during the Year -
Actual R&D Cost incurred during the Year -
Expenditure u/s 12A -
Contribution u/s 12B -
Adjusted Taxable Income for the purpose of Sec. 12 570,000

Allowable:
5% of ATI Calculated above (A) 28,500
Maximum (B) 100,000
Actual (C) 20,000 Donated by staff is not the amount paid by company
Eligible (Lower of "A", "B" or "C") 20,000

18) From the following Income Statement of M/S ABC Pvt. Ltd., prepare the taxable income for
the income year 20X-66/X-67. (Dec 2010, 10 Marks)
Rs. In ‘000
Sales 8,000
Cost of sales 5,000
Gross profit 3,000
Administrative expenses 500
Donation 100
Interest expenses 300
Pollution control expenses 1,500
Research & Development expenses 1,000
The Institute of Chartered Accountants of Nepal 265
CAP-II Paper 7 - Income Tax and VAT

Depreciation 50
Balance (450)
Other Income 50
Net Profit/ (loss) (400)

Answer
Statement of Assessable income from Business (ABC Pvt. Ltd.)
Particulars Sec. Ref Amount Working Notes
Inclusions:
Sales 7 (2) 8,000,000
Other Income 7 (2) 50,000

Total Inclusions 8,050,000


Deductions
There is no adjustment, therefore,
Cost of Trading Stock 15 5,000,000
assumed to be as per Tax
There is no adjustment, therefore,
Depreciation for the Year 19 50,000
assumed to be as per Tax
There is no adjustment, therefore,
Administrative Expense 13 500,000
assumed to be as per Tax

Interest expense 300,000

Deductions before considering


Interest Expense, PCC and 5,850,000
R&D Cost
Interest expense, where Sec.
-
14 (2) is applicable
Pollution Control Cost 600,000 Refer W. N. 1
Research and Development
350,000 Refer W. N. 1
Cost

Total Deductions 6,800,000


Assessable Income from
1,250,000
Business
Statement of Taxable income and tax liability
Assessable Income from
1,250,000
Business
Total Assessable Income 1,250,000
Less:
Reductions u/s 12 - Refer W.N. 2
Reductions u/s 12Ka -

The Institute of Chartered Accountants of Nepal 266


CAP-II Paper 7 - Income Tax and VAT

Reductions u/s 12Kha-


National Reconstruction Fund -
of GON
Taxable Income 1,250,000

Working Notes
2. Calculation of Pollution Control cost and
R&D Cost
For PCC For R &D Cost
Inclusions as per Income Tax Act 8,050,000 8,050,000
Deductions without considering interest expense, PCC
5,850,000 5,850,000
and R&D Cost
Less:
Actual interest cost, where Sec. 14 (2) is not
applicable
Actual PCC incurred during the Year 1,500,000
Actual R&D Cost incurred during the Year 1,000,000
Adjusted Taxable Income for the purpose of Sec. 14
1,200,000 700,000
(2)

Allowable:
50% ATI Calculated Above (A) 600,000 350,000
Actual Cost incurred during the year (B) 1,500,000 1,000,000
Eligible ("A" or "B", whichever is lower) 600,000 350,000

3. Calculation of Allowable Donation

Inclusions 8,050,000
Deductions without considering interest expense,
5,550,000
PCC and R&D Cost
Less:
Eligible Interest expense, where Sec. 14 (2) is not
300,000
applicable
Actual Interest Expense, where Sec. 14 (2) is
-
applicable
Actual PCC incurred during the Year 1,500,000
Actual R&D Cost incurred during the Year 1,000,000
Expenditure u/s 12A -
Contribution u/s 12B - 2,800,000
Adjusted Taxable Income for the purpose of Sec.
(300,000)
12

The Institute of Chartered Accountants of Nepal 267


CAP-II Paper 7 - Income Tax and VAT

Allowable:
5% of ATI Calculated above (A) (15,000)
Maximum (B) 100,000
Assumed to be donated to
100,000
Actual (C) exempt entity
Eligible (Lower of "A", "B" or "C") - Since, ATI is negative

19) Kailali (P.) Ltd. engaged in producing edible oils, could not prepare its accounts on accrual
basis of accounting and prepared only receipt and payment accounts. You just joined in the
company as Chief Accountant. With the help of existing assistants, you could gather
following information. Find the tax payable for the income year 20X-66/X-67.
Particulars Rs. Particulars Rs.
Opening Bank 100,000 Loan repaid 300,000
Payment for purchase 7,000,000 Sales realized 10,000,000
Interest paid 500,000 Interest income 100,000
Personnel Administrative Expense paid 1,500,000 Other income 200,000
Wage expense paid 900,000 Bank balance 200,000
Loan received 800,000
10,800,000 10,800,000

Further information gathered by the assistants are as follows:


a)
Particulars Opening Rs. Closing Rs.
Debtors 100,000 1,300,000
Creditors 300,000 100,000
Paid up capital 1,000,000 1,500,000
Machineries 1,500,000 2,100,000
Interest payable 50,000 150,000
Wage payables 50,000 100,000
3,000,000 5,250,000
b) Depreciation has not been charged in the books. Additional investments in machineries
have been done on Ashad 20X-67. Trading stock of Rs. 100,000 has not been shown in the
above information.
c) The shareholders of the company are foreigners and interest are paid to one of the foreign
banks.
d) Donation of Rs. 20,000 has been given to an exempt entity which is included in
administrative expense. Staff also donated Rs. 7,000 to the same entity.
e) Personnel Administrative expense excludes a gratuity provision of Rs. 200,000. During
this year Rs. 30,000 was paid as retirement payment from this provision account.

The Institute of Chartered Accountants of Nepal 268


CAP-II Paper 7 - Income Tax and VAT

From the above information, find current tax expense for income year 20X-66/X-67. (Dec
2010, 20 marks, CA Inter)
Answer
Working Notes
1. Value of Sales
Closing Debtors Values 1,300,000
Add: Sales Realization 10,000,000
Less: Opening Debtors (100,000)
Sales 11,200,000

2. Value of Purchases
Closing Creditors 100,000
Add: Payment for Purchases 7,000,000
Less: Opening Creditor (300,000)
Purchases during the year 6,800,000

3. Interest Expenses
Closing Interest Payable 150,000
Add: Interest paid during the Year 500,000
Less: Opening Interest Payable (50,000)
Interest Expenses 600,000

4. Accrued Wages
Closing Wages Payable 100,000
Add: Wages paid during the Year 900,000
Less: Opening Wages Payable (50,000)
Wage Expenses 950,000

5. It is assumed that:
a. Interest is paid to the extent of accrued.
b. Wages expense is paid to the extent of accrued.

Statement of Assessable Income


from Business
Particulars Sec. Ref Amount Notes
Inclusions
Sales income 7 (2) 11,200,000
Interest Income 7 (2) 100,000

The Institute of Chartered Accountants of Nepal 269


CAP-II Paper 7 - Income Tax and VAT

Other Income 7 (2) 200,000

Total Inclusions 11,500,000


Deductions
Cost of purchases minus value of
Cost of Trading Stock 15 6,700,000 closing stock as in additional
information (a)
Depreciation 19 340,000 W.N. 1 below
Administrative Expense 13 1,310,000 W.N. 2 below
Wage Expense 13 950,000
Interest expense 600,000 Refer W.N. 3 below
Total Deductions 9,900,000
Assessable Income from Business 1,600,000

Statement of Taxable Income


Total Assessable Income 1,600,000
Less:
Reduction u/s 12 20,000 Refer W.N. 4 below
Reduction u/s 12Ka
Reduction u/s 12Kha
Taxable Income 1,580,000

Calculation of Current Tax Expenses


Taxable Income 1,580,000
Corporate Tax @ 20% 316,000 Special Industry
1. Calculation of Depreciation
Block D
Opening Depreciation Base 1,500,000 Assumed this is tax base
Add: Absorbed Additions 200,000
(1/3rd of Rs. 600,000) As machinery is used since Ashad
Less: Disposal Value -
Depreciable Basis 1,700,000
Depreciation @ 20% (Special
340,000
industry)

2. Administrative Expense
Given 1,500,000
Less: Donation included in it (20,000)

The Institute of Chartered Accountants of Nepal 270


CAP-II Paper 7 - Income Tax and VAT

Provision for gratuity is not allowed as


Less: Gratuity provision (200,000)
deduction.
As gratuity is probable obligation, the
expense in such case is deductible
Add: Gratuity actually Paid 30,000
when there is actual outflow of
resource
1,310,000

3. Since the interest is paid to foreign bank not the foreign shareholder (controller). So entire amount of
Rs. 600,000 is allowed as deduction u/s 14(1).
4. Eligible Donation u/s 12
Inclusions as per Income Tax Act 11,500,000
Less: Deductions without Interest
expense u/s 14(2), PCC and R&D 9,900,000
Cost
Less: Actual PCC -
Less: Actual R&D Cost -
Less: Reductions u/s 12Ka -
Less: Reductions u/s 12Kha -
Adjusted Taxable income for the
1,600,000
purpose of Sec. 12
5% of Adjusted Taxable Income (A) 80,000
Actual Donation to Exempt
20,000
Organization (B)
Maximum Limit [C] 100,000
Eligible (Lower of "A", "B" or "C") 20,000

20) Given below is the Trading and Profit and Loss Account of X, a Trader, for the previous
year.
(Dec 2008, 6+4=10, CA Inter)
Particulars Amount (Rs.) Particulars Amount (Rs.)
To Opening Stock 50,000 By Sales 1,400,000
To Purchases 750,000 By Service Charge 48,600
To Salaries & Wages 175,000 By Miscellaneous Income 60,000
To Commission & Discount 30,000 By Agriculture Income 50,000
To Donation to VDC Rural 25,500 By Closing Stock 75,000
Municipality
To Administrative Expenses 32,000
To Insurance Premium 20,500

The Institute of Chartered Accountants of Nepal 271


CAP-II Paper 7 - Income Tax and VAT

To Carriage Outward 18,600


To Interest on Loan 110,000
To Legal Expenses 30,000
To Loss of Stock 25,000
To Miscellaneous Expenses 15,000
To Rent 96,000
To General Expenses 42,000
To Provision for Dividend 20,000
To Compensation to Employee 7,000
To Net Profit 187,000
1,633,600 1,633,600

Additional Information:
a. Salaries & wages include Rs. 20,000 paid to Mr. Bikash for the month of Asadh, manager
without deducting TDS.
b. Compensation of Rs. 20,000 which was received from insurance company due to loss of
stock of Rs. 25,000 (debited in the profit and loss account).
c. Out of the total insurance premium Rs.5,000 is related to the furniture given on hire.
d. Rent Rs. 1,05,000 was paid in a cash payment even though bank facility is available. This
amount has not been included in above Profit and Loss Account.
e. Purchase include Rs. 1,00,000 purchase cost of computer which was purchased on 10th
Magh of the previous income year and Rs. 50,000 being the cost of custom duty and
transport freight in connection with the purchase of computer that was not shown in the
above account. But those computers are booked as assets in company's account.
f. Firm claims to have brought forward previous year cumulative loss Rs. 50,000.
Required:
a. Assessable income from business.
b. Statement of total taxable income.

Answer
Statement of Assessable Income from Business (of Mr. X)
Particulars Sec. Ref. Amount Notes
Inclusions
Sales 7 (2) 1,400,000
Service Charge 7 (2) 48,600
Assumed that compensation under
Miscellaneous Income 7 (2) 60,000 additional information (b), is included in
miscellaneous income
Agriculture income is exempted from tax
Agriculture Income 11 -
even though generated by Company.

The Institute of Chartered Accountants of Nepal 272


CAP-II Paper 7 - Income Tax and VAT

Total Inclusions 1,508,600


Deductions
Interest Expense 14 110,000
Cost of Trading Stock 15 650,000 Refer W. N. 1
Depreciation 19 25,000 Refer W. N. 2
Failure to withhold tax triggers interest and
fees under Sec. 117 and 119 of the Act, and
Salaries & Wages 13 175,000 there is no any provision in Income Tax Act
which makes the expense not deductible
merely due to the failure to withhold tax
Commission and Discount 13 30,000
Administrative Expense 13 32,000
Insurance premium paid for furniture given
on hire is also business expense, as furniture
hire income forms part of business income
Insurance Premium 13 20,500 Alternatively, furniture hire income of a
natural person may be treated as investment
income and accordingly, this expense will
be treated as investment expense
Carriage Outward 13 18,600
Legal Expense 13 30,000
Miscellaneous Expense 13 15,000
96,000+1,05,000=201,000
Though rent is paid in cash, as the turnover
of the person is 1,508,600 that is less than
Rent 13 201,000
Rs. 20 Lakhs, the amount cannot be made
not deductible merely due to the reason of
cash payment
General Expense 13 42,000
Provision for dividend 21 - Not deductible
Compensation to Employee 13 7,000
Total Deductions 1,356,100
Deductible carried forward
20 50,000 Assuming the loss was since 7 years ago
losses
Total deductions including
1,406,100
losses
Assessable Income from Business 102,500

Statement of Taxable Income


Assessable Income from Business 102,500
Assessable Income from
-
Employment

The Institute of Chartered Accountants of Nepal 273


CAP-II Paper 7 - Income Tax and VAT

Assessable Income from


-
Investment
Total Assessable Income 102,500
Less: Reductions
Under Sec. 12 - Rural Municipality is not an exempt entity
Under Sec. 12Kha -
Under Sec. 63 -
Taxable Income 102,500

Working Notes
1. Cost of Trading Stock
Value of Opening Stock 50,000
Add: Purchases 750,000
Purchase of computer included in purchase,
Less: Accounting errors (100,000) 650,000
now rectified
Loss of stock is cost of disposal of stock,
Add: Loss of Stock 25,000
and it is part of cost of stock
Value of Closing Stock (75,000)
Cost of Trading Stock 650,000

2. Depreciation
Block B
Opening Depreciation Base -
Add: Absorbed Additions 100,000
Added in Magh as per additional
(2/3rd of Rs. 150,000)
information point (e)
Less: Disposal Proceeds -
Depreciable Basis 100,000
Depreciation @ 25% 25,000

21) Hard Steel Udyog, proprietorship firm owned by Mr. Jeevan unmarried individual
handicapped has following Trading & Profit and Loss Account for F.Y. 20X-63/X-64 on
accrual basis.
Particulars Rs. Particulars Rs.
To Opening Stock By Sales 1,500,000
Raw Material - By Closing Stock
Finished Goods - - Raw Material -
Raw Material Purchase 500,000 - Finished Goods 560,000
Production Expenses
To Labour Wages 150,000

The Institute of Chartered Accountants of Nepal 274


CAP-II Paper 7 - Income Tax and VAT

To Electricity 200,000
To Production Salary 100,000
To Depreciation (Block D) 200,000
To Plant Repair & Improvement 80,000
To Insurance Premium 10,000
(Machinery)
Gross Profit 820,000
Total 2,060,000 Total 2,060,000

By Gross Profit C/D 820,000


To Selling & Distribution 150,000 By Dividend (Gross) 200,000
To Insurance Premium Expenses 60,000
(Office Equipment)
To Other Expenses 4,000
To Net Profit 806,000
Total 1,020,000 1,020,000

Additional Information
a. It purchases raw material 50,000 kg @ Rs. 10/kg from local market on 20X-63/4/1.
b. Depreciation Base of Block ‘D’ is Rs. 1,000,000.
c. Selling & Distribution Expenses include payment against truck freight Rs. 75,000 in cash
on 20X-63/7/10 to Mr. Rajan, truck driver.
d. Insurance Premium Expenses (Office Equipment) is related to 20X-62/10/01 to 20X-
64/03/32 and Insurance Bill is attached with Voucher.
e. Data for consumption and production were as follows:
Particulars Opening Purchase/ Total Consumption/ Closing
Production Sales
Raw Material - 50,000 50,000 50,000 -
(kg)
Finished Goods - 48,000 48,000 32,000 16,000
(kg)
2,000 kg (50,000 – 48,000) is normal production loss.
f. He has received Salary Rs. 110,000 of F.Y. 20X-62/X-63 and Rs. 50,000 for F.Y. 20X-
63/X-64 in cash from part time employment in other companies.
g. He received dividend from Moon Rise Bank Ltd., registered as per Company Act, 2063.
h. He contributed Rs. 300,000 to Citizen Investment Trust (CIT).
i. Closing Stock of Finished Goods has been valued at Closing Market Rate i.e. Rs. 35/Kg.
Required: (June 2008, 12 Marks, CA Inter)
a. Calculate Taxable Income
b. Calculate Tax Liability

The Institute of Chartered Accountants of Nepal 275


CAP-II Paper 7 - Income Tax and VAT

Answer
Statement of Assessable Income from Business (Mr. Jeevan)
Particulars Sec. Ref. Amount Note
Inclusion
Sales 7 (2) 1,500,000
Final withholding, as it is received
Dividend Income 92 -
from resident company
Total Inclusions 1,500,000
Deductions
Cost of Trading Stock 15 640,000 Refer W.N. 1
Depreciation 19 150,000 Refer W. N. 2
Repair and Improvement Cost 16 70,000 Refer W. N. 2
Cash payment exceeding Rs. 50,000
as truck freight is not not-deductible
Selling and Distribution Expense 13 150,000 merely due to the reason of payment
in cash, as the turnover (15 lakhs plus
1.6 lakhs) is less than Rs. 20 Lakhs
60,000 divided by 18 multiplied by
Insurance Premium 13 40,000
12
Other Expenses 13 40,000
Total Deductions 1,090,000
Assessable Income from Business 410,000

Statement of Taxable Income and Balance Taxable Income


Assessable Income from Business 410,000
Cash basis and assumed that related
Assessable Income from Employment 160,000 to 20X-62/X-63 is received in cash
during 20X-63/X-64
Assessable Income from Investment -
Total Assessable Income 570,000
Less: Reductions-
under Sec. 12
under Sec. 12Kha
Lower of:
a. 1/3rd of Total Assessable Income
(Rs. 190,000)
under Sec. 63 190,000
b. Actual Contribution to ARF (Rs.
300,000)
c. Max. Rs. 300,000
Less:

The Institute of Chartered Accountants of Nepal 276


CAP-II Paper 7 - Income Tax and VAT

Deductions availed under Sch. 1 to handicapped person 200,000 50% of basic exemption limit
Balance Taxable Income 180,000

Statement of Tax Liability


1st Rs. 170,000 1% 1,700 SST on employment Income
Falls within basic exemption
Balance Rs. 10,000 0% -
limit of Rs. 500,000
Total Tax Liability 1,700

Working Note
1. Cost of Trading Stock
Value of Opening Stock -
Cost of Conversion
Raw Material Purchase 500,000
Labour Wages 150,000
Electricity 200,000
Production Salary 100,000
Insurance Premium 10,000
Total Cost of Production 960,000
Cost of conversion divided by
Per unit Cost of Production 20
units produced
Cost or Market Value,
whichever is lower
Value of Closing Stock (320,000) Cost= 16,000*20= 320,000
Market Value= 16,000*35 =
560,000
Cost of Trading Stock 640,000

2. Depreciation and Repair Cost


Block D
In absence of information,
Opening Depreciation Base 1,000,000 depreciation of Block B cannot
be calculated
Add: Absorbed Additions -
Less: Disposal Proceeds -
Depreciable Basis 1,000,000
Depreciation @ 15% 150,000

7% of Depreciable Basis (A) 70,000


Actual Repair Cost (B) 80,000
Eligible Cost ("A" or "B", w/e is lower) 70,000

The Institute of Chartered Accountants of Nepal 277


CAP-II Paper 7 - Income Tax and VAT

22) A proprietorship firm trading on beer earns taxable profit of Rs. 300,000 Calculate Tax
Liability.
(June 2008, 1 Mark, CA Inter)
Answer:
Since the amount falls within basic exemption limit, there is no tax liability of the person.

23) X Ltd. of Delhi has got 70% shares in Yoyo Private Ltd. of Nepal. The Profit and Loss
Account and Balance Sheet of the company for the first year of operation ending on 31.3.2062
was as follows: (Dec 2006, 4 Marks, CA Inter)
Balance Sheet
Rs. in lakhs
Share Capital 270 Plant & Machinery 400
Secured Loan for Building 200 Department Building 350
Unsecured Loan 150 Current Assets 183
Current Liabilities 80 Profit & Loss Account 67
700 700
Profit & Loss Account
Rs. in lakhs
Cost of Sales 450 Sales 500
Administration Expenses 75 Interest Received 2
Interest on Loan 42 Loss for the year 65
567 567

Interest on Bank Loan is 10% and the interest on unsecured loan is15%. Compute the
taxable income.

Answer
Note:
It is assumed that there is no loan from X Ltd. of Delhi
Statement of Assessable income from Business (Yoyo Pvt. Ltd.)
Particulars Sec. Ref. Amount Note
Inclusions
Sales 7 (2) 50,000,000
Interest Received 7 (2) 200,000
Total Inclusions 50,200,000
Deductions
Assuming loan is used evenly
14
Interest Expense 4,250,000 throughout the year
Cost of Sales 15 45,000,000

The Institute of Chartered Accountants of Nepal 278


CAP-II Paper 7 - Income Tax and VAT

Assuming assets are from beginning of


19
Depreciation 7,750,000 year
Administrative Expenses 13 7,500,000
Total Deductions 64,500,000
Assessable Income from Since there is loss, there is no taxable
Business (14,300,000) income

Working Note: Depreciation


Block A Block D
Opening Depreciation Base 35,000,000 40,000,000
Add: Absorbed Additions - -
Less: Disposal Proceeds - -
Depreciable Basis 35,000,000 40,000,000
Depreciation Rate 5% 15%
Depreciation 1,750,000 6,000,000

24) Shri Hari Upadhyaya is a Chartered Accountant practicing at Kathmandu. The following is
the analysis of his receipts and payments account for the year 20X-61/X-62. (Dec 2005, 6
Marks, CA Inter)
Receipts Rs. Payments Rs.
Balance b/d 9,500 Salaries 1,64,000
Professional income 3,90,000 Rent 12,000
House rent for 8 months 40,800 Telephone expenses 5,500
Share of Income from firm 6,250 Professional expenses 3,000
Interest on Bank Deposit 11,280 Motor car expenses 7,500
Insurance policy matured with bonus 77,750 Misc. Office expenses 1,500
Purchase of car 80,000
Advance Income-tax 12,500
Personal expenses 42,400
Entertainment expenses 17,000
House property expenses:
Municipal Taxes 6,000
- Repairs 2,500
- Insurance 2,000
- Collection charges 600
Balance c/d 1,79,080
Total Rs. 5,35,580 Total Rs. 5,35,580

Compute Shri Hari Upadhyaya's total income after taking into account, the following:
a. Value of benefits received from clients during the course of profession is Rs.5,000/-;

The Institute of Chartered Accountants of Nepal 279


CAP-II Paper 7 - Income Tax and VAT

b. Allowable rate of depreciation on motor car is 20%;


c. Municipal value of the house property is Rs.60,000/-. This house was self-occupied for
residence for 4 months during the year.

Answer
Statement of Assessable Income from Business
Sec.
Particulars Notes
Ref. Amount
Inclusions
Professional Income 7 (2) 390,000
House rent income of natural person not related
House Rent Income - with business is not subject to income tax. Refer
Definition of Rent
Share Income from Firm 92 - Assumed to be dividend, which is final withholding
Assumed to be investment nature and bank is
Interest on Bank Deposit 92 -
assumed to be resident bank
Insurance Policy Assumed to be received from resident insurance
92 -
Matured company, final withholding u/s 92.
Benefit received from
7 (2) 50,000
client
Total Inclusions 440,000
Deductions
Salaries 13 164,000
Rent 13 12,000
Telephone expenses 13 5,500
Professional Expenses 13 3,000
Motor Car expenses 13 7,500
Miscellaneous office
13 1,500
expense
Depreciation 19 16,000 Assumed to be put to use until Poush
Advance Income Tax 21 - Not deductible
Personal Expense 21 - Not deductible
House Property Expense 13 - Not deductible, as the income is also not taxable
Total Deductions 209,500
Assessable Income from
230,500
Business

25) M/s Pashupatinath Industries, a partnership firm, submits the following profits and loss
account to you for computation of taxable business income for the income year 20X-60/X-61
: (June 2005, 15 Marks, CA Inter)

The Institute of Chartered Accountants of Nepal 280


CAP-II Paper 7 - Income Tax and VAT

Profit and Loss Account for the year ending 31/3/20X-61


Amount (Rs.) Amount (Rs.)
Cost of Raw materials used 1,500,000 Sales 2,600,000
Production Expenses 500,000 Misc. Income 50,000
Gross Profit 650,000
2,650,000 2,650,000
To Salaries 348,000 By Gross Profit 650,000
To Rent 24,000 By Sundry 7,000
To Printing & Stationery 4,700 creditors w/back 13,500
To Telephone 2,800 By Dividend from
To Conveyance 19,500 Nabil bank
To Traveling 16,000
To Interest 68,000
To Depreciation 20,000
To Legal fees 12,000
To Auditor’s fees 12,000
To Provident Fund 18,000
contribution 125,500
To Net Profit 670,500 670,500
Total Rs.

Additional Information:
a. Salaries include Rs.120, 000 paid to working partner X and Rs.80,000 to working partner
Y.
b. Rent of Rs. 24, 000 is paid to the premises belonging to partner Y who has let it out to the
firm.
c. Interest paid includes Rs.60, 000, being interest paid on loan given by partner Y at the
rate of 15% simple interest.
d. Out of Provident Fund Contribution debited to P & L. Account Rs.7, 000 was outstanding
unpaid under the Income Tax Act.
e. The firm normally purchases goods issuing crossed cheques and Banks drafts only except
in the case of one bill for Rs.75, 000 for which payment has been made by cash.
f. Depreciation debited in the accounts is as per the Income Tax Act.
g. Legal Fees include Rs.10, 000 fees paid in respect of appeal against the income tax
assessment for the earlier year.
Answer
Statement of Assessable Income from Business
Sec.
Particulars Amount Notes
Ref.
Inclusions

The Institute of Chartered Accountants of Nepal 281


CAP-II Paper 7 - Income Tax and VAT

Sales 7 (2) 2,600,000


Miscellaneous Income 7 (2) 50,000
Net Gain on Disposal of
7 (2) 7,000 Sundry creditors written back
Liability
Final withholding assuming Nabil Bank is Resident
Dividend from Nabil Bank 92 -
company

Total Inclusions 2,657,000


Deductions
Interest paid to partner is deductible, but interest
Interest Expenses 14 68,000
shall be charged on arm length price.
Payment of Rs. 75,000 for cash is non deductible as
Cost of Trading Stock 15 1,925,000
the turnover of the firm is more than Rs. 20 lakhs.
Depreciation 19 20,000 Refer additional information (f)
Salaries 13 348,000 Salaries paid to working partners are deductible.
There is no provision in Income Tax Act that makes
Rent 13 24,000
rent paid to owner against not-deductible.
Printing and Stationery 13 4,700
Telephone 13 2,800
Conveyance 13 19,500
Travelling 13 16,000
Appeal against authority's decision is legal right,
Legal Fees 13 12,000
and such expense is allowed as deduction.
Audit Fee 13 12,000
Assumed that the firm has followed accrual basis of
PF Contribution 13 18,000
accounting. Entire amount is allowed as deduction.

Total Deductions 2,470,000


Assessable Income from
187,000
Business

26) Mansubha Ltd is a business house dealing in readymade garments. During the year 20X3/X4,
it had the following transactions. Find its income from exports and the taxable income and
tax liability: (Dec 2004, 10 Marks, CA Inter)
Total Sales 2,000 Lakhs
Export Sales 1,500 Lakhs
Cost of purchase of garments 2,000 Lakhs
Export expenses 100 Lakhs
Administrative expenses 75 Lakhs
Depreciation 9 Lakhs

The Institute of Chartered Accountants of Nepal 282


CAP-II Paper 7 - Income Tax and VAT

Closing stock of garments 200 Lakhs

Answer
Statement of Assessable Income
Sec. Domestic
Particulars Export Sales Notes
Ref Sales
Inclusions
Export Sales 7 (2) 150,000,000
Domestic Sales 7 (2) 50,000,000
Total Inclusions 150,000,000 50,000,000
Deductions
Equals to cost of purchase minus
Cost of Trading Stock 15 135,000,000 45,000,000 closing stock
Apportioned in Sales Ratio
Apportionment not required as per
Export Expenses 13 10,000,000
matching principle
Administrative Expenses 13 5,625,000 1,875,000 Apportioned in Sales ratio
Assuming that the given amount is
Depreciation 19 675,000 225,000 as per Income Tax Act,
Apportioned in Sales ratio
Total Deductions 151,300,000 47,100,000
Assessable Income from
(1,300,000) 2,900,000
Business
Less: Other Business Loss
0 (1,300,000) Horizontal loss set-off u/s 20.
for the Year
Taxable Income 1,600,000
Tax Rate 20% 25% Normal business
Tax Liability - 400,000

Where a person conducts multiple transactions, where exemptions under Sec. 11 are applicable; each
transaction is treated as being conducted by separate person. As such, there cannot be offsetting of loss between
export income and domestic sales income.

27) The Profit and Loss account of PQR & Co., a manufacturing company, shows a Net Profit
of Rs. 50,00,000 (before tax). Calculate the Taxable income of the company for Financial
Year 20X-59/X-60, after considering the following facts.
(June 2004, 8 Marks, CA Inter)
a. Fixed Assets schedule of the Company shows the following balances:
Pool WDV as on 20X-59.3.3
A 20,00,000
B 10,00,000

The Institute of Chartered Accountants of Nepal 283


CAP-II Paper 7 - Income Tax and VAT

C 12,00,000
D 50,00,000
The Company purchased Furniture of Rs. 50,000 on Falgun 15, 20X-59. It also purchased a
vehicle or Rs. 8,00,000 on Baisakh 1, 20X-60.
b. The Company holds 20% share in ABC & Co. Ltd. It received Rs. 1,00,000 (net of tax)
as dividend during the year.
c. The Company has spent Rs. 2,80,000 on the repairs of the Machinery during the year
and charged the whole amount to the Profit and Loss Account.
d. The Company has paid a premium of Rs. 2,50,000 for insurance of its Machinery and
charged the whole amount to the Profit & Loss account. The period covered by the
premium is from 20X-60/1/1 to 20X-60/12/30 (one year).
e. The Company has charged Rs. 15,00,000 as depreciation during the year in its books.

Answer
Assessable Income from Business
Sec.
Particulars Amount Notes
Ref
Net Profit before Tax 5,000,000
Add:
Depreciation Charged in above Profit 1,500,000
Repairs on Machinery charged in above
280,000
profit
Insurance expense charged in above
250,000
profit
Less:
Allowable Depreciation under ITA (1,868,970)
Allowable Repair Cost under ITA (280,000)
250,000 divided by 12
Accrued Insurance Expense 62,500
multiplied by 3
Dividend from Resident company 92 - Final Withholding
There are no reductions u/s
Assessable Income from Business 4,943,530 12, 12Ka and 12Kha. So, this
amount is taxable income

Working Notes
1. Allowable Depreciation
Pool A Pool B Pool C Pool D Notes/Total
Assuming tax
Opening Depreciation Base 2,000,000 1,000,000 1,200,000 5,000,000
bases
Add: Absorbed Additions

The Institute of Chartered Accountants of Nepal 284


CAP-II Paper 7 - Income Tax and VAT

Furniture (two-third of 50,000) 33,333


Vehicle (one-third of Rs.
266,667
8,00,000)
Less: Disposal Proceeds - - - -
Depreciable Basis 2,000,000 1,033,333 1,466,667 5,000,000
Depreciation Rate 6.67% 33.33% 26.67% 20.00%
Depreciation Amount 133,400 344,410 391,160 1,000,000 1,868,970
Manufacturing company is a special industry and hence, depreciation must be computed at accelerated rate.

2. Allowable Repair
Pool A Pool B Pool C Pool D
7% of Depreciable Basis 140,000 72,333 102,667 350,000
Actual Cost - - - 280,000
Eligible (Minimum of above two) - - - 280,000 280,000

28) Mr. Mahesh is operating a business relating to computer parts and the sales during the year
is Rs. 20,00,000. Except the computer sales, other incomes relating to business are:
a. Net profit by disposal of business assets Rs. 25,000.
b. Prizes received from LG Computer Company as best seller Rs. 50,000.
c. Amounts received instead of only trading of LG Computer parts Rs. 40,000.
d. Amounts received by subletting of some space of room hired for business purpose Rs.
25,000.
e. The total expenditure relating to business including cost of business stock is Rs. 18,30,000.

Mrs. Sita, wife of Mahesh, is working in a bank and her taxable employment income is Rs.
1,10,000, before deduction of donation. They have selected the option of dealing a couple as
a single individual. They provided the gift to a tax-exempt organization worth Rs. 15,000.

Compute the taxable income of Mahesh and his wife jointly for the income year 20X-59/X-
60. (June 2003, 6 Marks, CA Inter)

Answer:
Assessable Income from Business (of Mr. Mahesh)
Sec.
Particular Amount Note
Ref.
Inclusions
Sales 7 (2) 2,000,000
Net Gain on Disposal of Business Asset 7 (2) 25,000
Prize received in respect of business 7 (2) 50,000

The Institute of Chartered Accountants of Nepal 285


CAP-II Paper 7 - Income Tax and VAT

Amount received in respect of business


7 (2) 40,000
restrictions
Subletting income 7 (2) 25,000
Total Inclusions 2,140,000
Less: Total Deductions (1,830,000)
Assessable Income from Business 310,000
Add: Employment income of wife 110,000
Total Assessable Income 420,000
As per section 12,
Lower of following
amount is allowed as
deduction in case of
donation given to
exempt organization:
-Rs. 100,000
-5% of Adjusted
Less: Donation to Exempt Organization 15,000 Taxable Income:
420,000x5%= Rs.
21,000
-Actual Donation to
Exempt Organization:
Rs. 15,000

Rs. 15,000 is allowed


as deduction.
Taxable Income 405,000

Slab Taxable Income Rate of Tax Amount of Tax Remarks


1st 110,000 1% 1,100 To the extent of employment income
1st 295,000 0% 0 Business Income
Total Tax 1,100
Liabilities

The Institute of Chartered Accountants of Nepal 286


CAP-II Paper 7 - Income Tax and VAT

Chapter 16: Income From Investment

1) Ms. Anjali Amatya, submitted the following details of income for Income Year 20X-75/X-76.
Particulars Amount (Rs.)
1. House Rental income let out to XYZ Ltd. (Net Rent) 342,000
2. Interest Income from Development Bank in Nepal (Net 85,000
Interest)
3. Interest Income from friends (Gross) 35,000
4. Furniture rental income from Mr. Paras (Gross) 850,000
5. Windfall gain withholding tax not deducted (Gross) 100,000
6. Gift received related to investment (Gross) 225,000
7. Income from natural resources (Net) 8,500
8. Gain recovered from investment insurance (Net) 85,000
9. Recovery of bad debt 200,000
10. Income from sale of listed shares (Net) 277,500

Ms. Amatya submitted the following details of expenses:


Particulars Amount (Rs.)
1. Repairing expenses of house property let out to XYZ Ltd. (Rs.)40,000
(Rs.)
2. Interest expenses paid to borrowed funds to give loan to friends 9,000
3. Allowable depreciation for furniture let out to Mr. Paras 35,000
4. Interest expenses for borrowed to pay insurance premium 6,500
5. Life insurance premium paid 19,000
6. Expenses paid related to natural resources 9,000
7. Bad debts recovered was not allowed for deduction earlier 80,000
8. Contribution to approved retirement funds 90,000

Your assistance is required to compute her taxable income and tax liability for Income Year
20X-75/X-76. (June 2019, 10 Marks)

Answer:
Assessable Income from Investment
Sec.
Particulars Amount Note
Ref.
Inclusions
As per definition of “Rent”, natural person’s
house rent income that is not generated
House Rental Income - through a private firm does not fall under the
definition of rent, which means such income
is exempt from income tax
Final Withholding, assuming not generated
Interest income from bank 92 -
by private firm of Ms. Anjali

The Institute of Chartered Accountants of Nepal 287


CAP-II Paper 7 - Income Tax and VAT
Interest income from Friend 9 (2) 35,000
Furniture rental income 9 (2) 850,000
Windfall Gain 92 - Final Withholding
Gift received 9 (2) 225,000
Grossed up using TDS rate of 15% [8,500
Income from natural resource 9 (2) 10,000
divided by 0.85]
Gain from investment insurance 92 - Final withholding u/s 92.
Included only to such extent that was claimed
Recovery of bad debt 25 120,000 earlier, assumed bad debt was related to
investment
Grossed up using advance tax rate of 5%
Income from sale of listed shares 9 (2) 292,105 [277,500 divided by 0.95]. Alternatively can
be divided by .925.
Total Inclusions 1,532,105
Less: Deductions
As income is exempt, expense is also not
Repair expense of House property 21 -
deductible
Interest Expense 14 9,000
Allowable Depreciation 19 35,000
Assuming borrowed to pay investment
Interest expense borrowed to pay
21 - insurance premium, which is deductible when
insurance premium
calculation gain from investment insurance
Expense related to natural
13 9,000
resources
Bad debt recovered not allowed A mere information to determine, how much
earlier recovery of bad debt is included in income
Total Deductions 53,000
Assessable Income from Investment 1,479,105
Lower of:
a. one-third of total assessable income,
Less:- Contribution to ARF 63 90,000
b. Actual Contribution to ARF- Rs. 90,000
c. Maximum Rs. 300,000
Less: Payment of Life insurance Sch.
19,000 Lower of actual premium of Rs. 40,000
premium 1
Taxable Income 1,370,105
Balance Taxable Income without
including gain on disposal of 1,078,000
NBCA

Computation of Tax Liability


1st Rs. 500,000 0% -
Next Rs. 200,000 10% 20,000
Next Rs. 300,000 20% 60,000

The Institute of Chartered Accountants of Nepal 288


CAP-II Paper 7 - Income Tax and VAT
Next Rs. 78,000 30% 23,400
Balance Rs. 292,105 5% 14,605
Total Tax Liability 118,005

2) Mrs. Kriti is an employee in Nepal Bank Ltd. Besides employment income, she has the
following transactions for Income Year 20X-73/X-74. Sort out the income from investment
and tax liability for the year assuming that 0% and 15 10% tax slabs are already cover by
income from employment. (June 2018, 10 Marks)
S. No. Particulars Amount (Rs)
1 Net gain on disposal of listed shares 100,000.00
2 Purchase of New listed shares 600,000.00
3 Income from Taxi 400,000.00
4 Gain on sale of gold 100,000.00
5 Interest from savings deposit from ‘A’ class bank 4,000.00
6 Interest from fixed deposit from ‘A’ class bank 40,000.00
Net gain from disposal of ancestral house (owned and
7 resided since Bikram Sambat 20X-60) 5,000,000.00
8 Royalty from music video 100,000.00
9 House rent 300,000.00
10 Shares transaction commission expenses 5,000.00
11 Salary to taxi driver and other taxi related expenses 200,000.00
12 Music video expenses 20,000.00
13 House repair and maintenance expenses 20,000.00

Answer
Assessable Income from Investment
Sec.
Particulars Amount Note
Ref.
Inclusions
Net Gain on Disposal of Listed Shares 9 (2) 100,000 Income is booked at gross amount
Outgoings of shares yet to be sold,
Purchase of new listed share - will be considered at the time of
disposal of new shares
It is assumed that the person’s
business is not in gold and gold is
kept for personal purpose, thus the
Gain on Sale of Gold -
gain is not taxable since it is not
business, employment or
investment income.
Interest from Saving Deposit (A Class
92 - Final withholding
Bank)
Interest from Fixed Deposit (A Class
92 - Final withholding
Bank)

The Institute of Chartered Accountants of Nepal 289


CAP-II Paper 7 - Income Tax and VAT
Since the house is owned
continuously for more than 10
years, and assuming that the person
has resided there for 10 years or
more; the asset does not fall under
Net gain on disposal of ancestral house - the category of Non Business
Chargeable asset. Since the asset
does not fall under any category of
asset as defined by Income Tax
Act, the gain is exempt from
income tax.
Royalty from Music Video 9 (2) 100,000
As per definition of “Rent”,
natural person’s house rent
income that is not generated
House Rent - through a private firm does not
fall under the definition of rent,
which means such income is
exempt from income tax
It is assumed that it has already
been considered while determining
Share Transaction Commission Expense - net gain on disposal of listed
shares, or it is the outgoings for
purchase of new listed shares.
Total Inclusions 200,000
Less: Deductions
Music Video related expense 13 20,000
As income is exempt, expense is
House repair and maintenance expense 21 -
also not deductible
Total Deductions 20,000
Assessable Income from Investment 180,000

Assessable Income from Business


Sec.
Particulars Amount Note
Ref.
Income from Taxi is no more
Income from Taxi 7 (2) 400,000 presumptive taxation, and it is a
business income
Less: Salary to Taxi Driver and other taxi
13 (200,000)
related expenses
Assessable Income from Business 200,000

Total Taxable income subject to tax rate above 10%


Assessable Income from Investment 180,000

The Institute of Chartered Accountants of Nepal 290


CAP-II Paper 7 - Income Tax and VAT
Assessable Income from Business 200,000
Less: Income subject to less tax rate (100,000) Gain on disposal of listed shares
Income subject to 20%, 30% or 36% tax
280,000
rate

Computation of Tax Liability


First and second slab is already
Rs. 280,000 20% 56,000
covered by employment income.
Balance Rs. 100,000 5% 5,000
Total remaining Tax Liability 61,000

3) Mr. Dinesh Maharjan, an investor, submitted the following details of income for assessment
of his income tax liability for the assessment year 20X-72/X-73. (Dec 2016, 10 marks)
Particulars Amount in (Rs.)
1. House Rental income let out to ABC Ltd. (Net Rent) 324,000
2. Interest Income from Commercial Bank in Nepal (Net Interest) 95,000
3. Interest Income from his friends (Gross) 25,000
4. Furniture rental income from Mr. Krishna (Gross) 900,000
5. Windfall gain withholding tax not deducted (Gross) 200,000
6. Cash dividend (Gross) from investment in shares of ABC commercial 300,000
Bank, the bank was merged with another bank 18 month ahead with the
approval of NRB and the investment was made by Mr. Maharjan before
merger.
7. Income from sale of bonus shares (Gross) received from investment in 250,000
shares of DEF commercial Bank, the bank was merged with another
bank 23 month ahead with the approval of NRB and the investment was
made by Mr. Maharjan before merger.
8. Gift received related to Investment (Gross) 150,000
9. Income from Natural resources (Net) 8,500
10. Gain recovered from Investment Insurance (Net) 95,000
11. Bad debt recovered 100,000
12. Income from sale of listed shares (Net) 190,000

Mr. Maharjan, who is also a married guy, submitted following details of expenses to you for
assessment of taxable income, tax liability & net tax payable for the financial year 20X-72/X-
73:
Particulars Amount in (Rs.)
1. Repairing expenses of house property let out to ABC Ltd. 35,000
2. Interest exp. Paid to borrowed funds to give loan to friends 10,000
3. Allowable depreciation for furniture’s let out to Mr. Krishna 25,000
4. Interest expenses for borrowed to pay insurance premium 7,500
5. Life Insurance premium paid 18,000

The Institute of Chartered Accountants of Nepal 291


CAP-II Paper 7 - Income Tax and VAT
6. Expenses paid related to natural resources 10,000
7. Bad debts recovered was not allowed for deduction earlier 50,000
8. Contribution to retirement funds 100,000
Your assistance is required to compute his taxable income & tax liability for financial year
20X-72/X-73.

Answer
Assessable Income from Investment
Sec.
Particulars Amount Note
Ref.
Inclusions
As per definition of "Rent", natural person's
house rent income that is not generated
House Rental Income - through a private firm does not fall under the
definition of rent, which means such income
is exempt from income tax
Final Withholding, assuming not generated
Interest income from bank 92 -
by private firm of Ms. Anjali
Interest income from Friend 9 (2) 25,000
Furniture rental income 9 (2) 900,000
Windfall Gain 92 - Final Withholding
Final withholding u/s 92. Merger is subject
Cash dividend from a bank that
47Ka - to Sec 47A which is not applicable to CAP
was merged 18 months before
II.
Gain on disposal of shares from a
bank that was merged 23 months Merger is subject to Sec 47A which is not
47Ka -
before, shares held on the date of applicable to CAP II.
merger
Gift received 9 (2) 150,000
Grossed up using TDS rate of 15% [8,500
Income from natural resource 9 (2) 10,000
divided by 0.85]
Gain from Investment Insurance 92 - Final withholding
Included only to such extent that was
Recovery of bad debt 25 50,000 claimed earlier, assumed bad debt was
related to investment
Grossed up using advance tax rate of 5%
Income from sale of listed shares 9 (2) 200,000 [190,000 divided by 0.95] (Students may
divide by 0.925)
Total Inclusions 1,335,000
Less: Deductions
Repair expense of House As income is exempt, expense is also not
21 -
property deductible
Interest Expense 14 10,000

The Institute of Chartered Accountants of Nepal 292


CAP-II Paper 7 - Income Tax and VAT
Allowable Depreciation 19 25,000
Interest expense borrowed to pay Deductible when calculation gain from
21 -
insurance premium investment insurance.
Expense related to natural
13 10,000
resources
Bad debt recovered not allowed A mere information to determine, how much
earlier recovery of bad debt is included in income
Total Deductions 45,000
Assessable Income from Investment 1,290,000
Less:
Lower of:
a. one-third of total assessable income,
Contribution to ARF 63 100,000
b. Actual Contribution to ARF- Rs. 100,000
c. Maximum Rs. 300,000
Payment of Life insurance Sch.
18,000 Lower of actual premium of Rs. 40,000
premium 1
Balance Taxable Income 1,172,000
Balance Taxable Income without
including gain on disposal of 972,000
NBCA

Computation of Tax Liability


Assuming individual assessment. Married
1st Rs. 500,000 0% - does not mean compulsory couple
assessment
Next Rs. 200,000 10% 20,000
Next Rs. 272,000 20% 54,400
Balance Rs. 200,000 5% 10,000
Total Tax Liability 84,400

4) Mr. Oli, a disable person and is residing in Manang District, disclosed his following income
and expenditure for the income year 20X-71/X-72
Income side:
a. Interest on private money lending activities Rs. 127,500 (after TDS)
b. Royalty from natural resources Rs. 306,000 (after TDS)
c. Interest from bank deposit Rs. 190,000
d. Dividend from domestic companies Rs. 95,000
e. Royalty from writing article Rs. 15,000
f. Royalty from book publication Rs. 170,000 (after TDS)
g. Compensation received Rs. 50,000
h. Money found lying in street Rs. 7,500 (net)
Expense side:
a. Interest on private money lending Rs. 2,000

The Institute of Chartered Accountants of Nepal 293


CAP-II Paper 7 - Income Tax and VAT
b. Natural resources Rs. 1,000
c. Royalty from writing article Rs. 100
d. Dividend from domestic companies Rs. 500
e. Legal expenses (income tax appeal) Rs. 5,000
f. Salary to assistant Rs. 30,000
g. Life insurance premium Rs. 15,000 on his own policy amount Rs. 300,000
h. Contribution to approved retirement fund Rs. 50,000
i. Donation to tax exempt organization Rs. 12,000

Compute the tax liability and net tax payable of Mr. Oli for the income year 20X-71/X-72.
(Assume him as couple for calculation) (June 2016, 10 marks)
Answer
Assessable Income from Investment
Sec.
Particulars Amount Note
Ref.
Inclusions
Interest on Private Money Lending
9 (2) 150,000 Grossed up, 127,500 divided by 0.85
Activity
Royalty from Natural Resources 9 (2) 360,000 Grossed up, 306,000 divided by 0.85
Final withholding, assuming bank is resident
Interest from Bank deposit 92 -
of Nepal
Dividend from resident company 92 - Final withholding
Royalty from book publication 9 (2) 200,000 Grossed up, 170,000 divided by 0.85
Compensation received 31 50,000 Assumed related to investment
As per contract law, it must be produced
before law enforcement authority. If we
Money found lying in street -
assume it as income, it is windfall gain and is
final withholding
Total Inclusions 760,000
Deductions
Interest on Private Money Lending
14 2,000
Activity
Natural resources 13 1,000
Collection fee for final withholding
Dividend from resident company 21 -
payments
Legal expenses 13 5,000 Appeal is legal right
Assuming all incurred for income generating
Salary to Assistant 13 30,000
activities
Total Deductions 38,000
Assessable Income from
722,000
Investment

Assessable Income from Business

The Institute of Chartered Accountants of Nepal 294


CAP-II Paper 7 - Income Tax and VAT
Royalty from writing article 7 (2) 15,000 Service fee
Less: Writing article 13 (100)
Assessable Income from Business 14,900

Total Assessable Income, Taxable Income and Balance Taxable Income


Assessable Income from Investment 9 722,000
Assessable Income from Business 7 14,900
Total Assessable Income 736,900
Less:
Lower of:
a. one-third of total assessable income,
Contribution to ARF 63 50,000
b. Actual Contribution to ARF- Rs. 50,000
c. Maximum Rs. 300,000
Lower of:
a. 5% of ATI, i.e. 5% of Rs. 686,900
Donation 12 12,000 b. Actual Donation to Exempt Entity- Rs.
12,000
c. Maximum Rs. 100,000
Lower of actual premium of Rs. 15,000 or
Payment of Life insurance premium Sch. 1 15,000
Rs. 40,000
Taxable Income 659,900

Calculation of Tax Liability


1st Rs. 500,000 0% -
Balance Rs. 159,900 10% 15,990
Tax Liability 15,990

5) The investment income and expenses related to Mr. Narayan for the financial year 20X-71/X-
72 are as follows:
Particulars: Amount (Rs.)
A. Income:
House rent income net of TDS 180,000
Bank interest income net of TDS- Nepal Bank Ltd. 190,000
Natural resources payments net of TDS 170,000
Interest income net of TDS from ABC Ltd. 2,125,000
Compensation received from loss of last year investment 25,000
Income from investment insurance net of TDS- Rastriya Beema Sanstha 95,000
Gift related to investment income 50,000
Dividend income net of TDS from Sanima Bank Ltd. 47,500
B. Expenses:
Expenses related to collection of house rent 4,000
Expenses related to natural resources 8,500

The Institute of Chartered Accountants of Nepal 295


CAP-II Paper 7 - Income Tax and VAT
Allowable depreciation allowance as per the Act 5,500
Life insurance premium paid to Rastriya Beema Sanstha 22,500
Donation paid to tax exempt organization 40,000

Required: (July 2015, 8+2=10)


a. Compute total Taxable income from investment.
b. Compute net Tax liability for the financial year 20X-71/X-72, assuming Mr. Narayan is
a married and had no other income.
Answer
Assessable Income from Investment
Particulars Sec. Ref. Amount Note
Inclusions
As per definition of "Rent", natural person's
house rent income that is not generated
House Rent - through a private firm does not fall under
the definition of rent, which means such
income is exempt from income tax
Final withholding, assuming not related to
Interest from Banks 92 -
business and the bank is resident of Nepal
Natural Resource Payment 9 (2) 200,000 Grossed up, [=170,000/0.85]
Grossed up, [=2,125,000/0.85] assuming
ABC Ltd. is not a listed company or entity
authorized to issue debenture/bond.
Interest from ABC Ltd. 9 (2) 2,500,000 Alternatively, if we assume that ABC Ltd.
is a listed company or entity authorized to
issue debenture/bond, interest is Final
withholding.
Compensation received 22 and 31 25,000 Cash basis as per Sec. 31
Income from Investment Final withholding, assuming insurance
92 -
Insurance company is resident of Nepal.
Gift related to investment 9 (2) 50,000
Final withholding, assuming Sanima Bank
Dividend 92 -
Ltd. is resident of Nepal.
Total Inclusions 2,775,000
Deductions
As income is exempt, expense is also not
House Rent Collection expense 21
deductible
Natural Resource Expense 13 8,500
Allowable Depreciation 19 5,500
Total Deductions 14,000
Total Assessable Income 2,761,000
Taxable Income, Balance Taxable Income and Tax Liability
Total Assessable Income 2,761,000

The Institute of Chartered Accountants of Nepal 296


CAP-II Paper 7 - Income Tax and VAT
Less:
Lower of:
a. 5% of ATI, i.e. 5% of Rs. 2,761,000
Donation to Exempt Entity 12 40,000 b. Actual Donation to Exempt Entity: Rs.
40,000
c. Maximum Rs. 100,000
Less:
Payment of Life insurance Lower of actual premium of Rs. 22,500 or
Sch. 1 22,500
premium 40,000
Balance Taxable Income 2,698,500
Tax Liability
1st Rs. 600,000 0% - Assuming Couple Assessment
Next Rs. 200,000 10% 20,000
Next Rs. 300,000 20% 60,000
Next Rs. 900,000 30% 270,000
Balance Rs. 698,500 36% 251,460
601,460
Less:
Taxes withheld and paid by
agents
Natural Resource Payment 30,000
Interest from ABC Ltd. 375,000
Net Tax Payable 271,460

6) What is investment income? What sorts of revenue is included in investment income? (Dec
2011, CA Inter, 5 Marks)

Answer:
“Investment” means the holding of one or more properties or the investment in a property except
the following:
a. The use of the property for the private use of the owner, or
b. A business or an employment.
The holding of a non-business chargeable asset is also known as an investment.

Amount to be included in investment income is as follows:


a. Any dividend, interest, natural resource payment, rent, royalty, gain from investment
insurance, gain from unapproved retirement fund pursuant to Sec. 63 (1), or retirement
payment from approved retirement fund,
b. Net gains on disposal of the non-business chargeable assets of the investment of the person
calculated under Chapter 8,
c. Where the incomings exceed the remaining value, including the outgoings in respect of the
pool of depreciable assets under Clause (Ka) of Sub-Section 2 of Section 4 of Schedule 2, at

The Institute of Chartered Accountants of Nepal 297


CAP-II Paper 7 - Income Tax and VAT
the time of disposal of the depreciable assets of the investments made by the person, the excess
amount,
d. Gifts received by the person in respect investment,
e. Retirement payments made in respect of investment, and the retirement contributions along
with amounts deposited in the retirement fund for that person,
f. Amounts derived as consideration for accepting a restriction on the capacity to conduct the
investment
g. Other amounts to be included pursuant to Chapter 6 or 7, or Section 56

7) Mr. Y, a married person has the following incomes/ losses and investments for the year
ending on 31/3/20X5. Calculate his taxable income for the financial year 20X4/X5. (Dec
2006, 8 Marks, CA Inter)
S.N. Particulars Rs.
(a) Interest on debentures 2,000
(b) Interest on bank accounts 10,000
(c) Dividend from companies 20,000
(d) Salary and interest for being a working partner in a firm (share in 64,000
firm's income Rs. 50,000 excluded)
(e) Business loss from proprietary business 81,000
(f) Rental income from house property fully let-out 18,000
(g) House tax paid for let-out property 500
(h) Interest paid on capital borrowed for purchase of self-occupied 18,000
residential flat
(i) Long-term capital gain on sale of plot 1,60,000
(j) Deposit in CIT Account 5,000

Answer
Assessable Income from Investment
Sec.
Particulars Amount Note
Ref.
Final withholding, assuming debenture issuing
Interest on Debenture 92 -
company is resident of Nepal
Final withholding, assuming bank is resident of
Interest on Bank Accounts 92 - Nepal and the interest earned has no connection to
business
Final withholding, assuming the dividend paying
Dividend from Companies 92 -
company is resident of Nepal
Final withholding, assuming the dividend paying
Share in firm's income 92 -
firm is resident of Nepal
As per definition of "Rent", natural person's house
Rental income from house rent income that is not generated through a private
-
property firm does not fall under the definition of rent, which
means such income is exempt from income tax

The Institute of Chartered Accountants of Nepal 298


CAP-II Paper 7 - Income Tax and VAT
Interest on loan for
- It forms part of outgoings of self-occupied house
construction of private house
Assuming the incoming from land exceeds Rs. 10
Gain on Sale of Land 9 (2) 160,000
Lakhs
Total Inclusions 160,000
Less:
Business loss from Private
20 81,000
firm
Assessable Income from
79,000
Investment

Assessable Income from Employment


Salary from Partnership firm 7 (2) 14,000
Assessable Income from
14,000
Employment

Total Assessable Income and Taxable Income


Assessable Income from
79,000
Investment
Assessable Income from
14,000
Employment
Total Assessable Income 93,000
Less:
Lower of:
Contribution to Approved a. one-third of total assessable income,
63 5,000
RF b. Actual Contribution to ARF- Rs. 5,000
c. Maximum Rs. 300,000
Taxable Income 88,000

The Institute of Chartered Accountants of Nepal 299


CAP-II Paper 7 - Income Tax and VAT

Chapter 17: Computation of Tax Liability- Mixed Income

1) Mr. Shrestha, a retired government employee, earns a pension of Rs. 9,500 per month and 1
month Dashain allowance. He also runs a small business with annual income of Rs. 125,000.
Calculate the Taxable Income and the tax liability for income year 20X-66/X-67 assuming
Mr. Shrestha adopts single status as per section 50 of Income Tax Act, 2058. (Dec 2010, 5
Marks, CA Inter)

Answer:
Tax Liability of Mr. Shrestha (assessing Individual)
Assessable Income from Business 125,000
Assessable Income from Employment (9,500 X 13) 123,500
Total Assessable Income 248,500
Less: Deduction for Pension Income (u/s 1(9a) of Sch 1) 125,000
Tax Liability -
(Since the taxable income is within tax-free slab of Rs. 500,000)

Note to students:
Mr. Shrestha cannot pay presumptive tax u/s 4 (4) of the Act, if wishes so, since he has income
also from employment.

2) Super company is a proprietorship industry producing tobaccos. The Industry has total sales
of Rs. 20 million and taxable income of Rs. 3.5 million in Income Year 20X-70/X-71. The
company hasn’t submitted estimated tax return during the year. In addition, it has not paid
any taxes. You are requested to assess the total Tax liability for this industry up to Mangsir
20, 20X-71. (Dec 2014, 5 Marks)

Answer:
Computation of Tax Liability of a private firm (i.e. natural person) assuming single natural person:
Taxable Income Rs. 3,500,000
Slab Taxable Amount Rate of Tax Tax Liability
st
1 Rs. 500,000 0% -
Next Rs. 200,000 10% 20,000
Next Rs. 300,000 20% 60,000
Next Rs. 1,000,000 30% 300,000
Balance Rs. 1,500,000 36% 540,000
Total Tax Liability 920,000

Note:

The Institute of Chartered Accountants of Nepal 300


CAP-II Paper 7 - Income Tax and VAT
Since sec. 114 and above of Income Tax Act, 2058 is out of syllabus for CAP II, fees and interest
u/s 117, 118 and 119 has not been computed.

3) Mr. Nayak Shakya, who is going to retire from 1st Shrawan 20X-67, submits the following
details with respect to his employment for the Income Year 20X-66/X-67.
Net Salary received (after TDS) Rs. 4,57,000
TDS Paid in respect of Salary Rs. 37,000
Contribution by employer to retirement fund (approved) Rs. 1,00,000
His Contribution to approved retirement fund Rs. 1,50,000
Compensation received as per Employee Rule Rs. 70,000
Investment Insurance Premium paid by Employer on his behalf (insured sum Rs. 19,000
of Rs. 2,00,000)
Encashment of un-availed leave upto Chaitra 18, 2058 Rs. 50,000
Encashment of un-availed leave after Chaitra 18, 2058 Rs. 1,70,000
Emergency Medical Treatment paid by Employer Rs. 10,000
(He didn’t claim Medical Tax Credit)
Reimbursement of Tour Expenses during Official visit to Hongkong Rs. 2,40,000
Gratuity Received (lumpsum payment)
Upto 18 Chaitra, 2058 Rs. 4,00,000
After 18 Chaitra, 2058 (approved) Rs. 7,00,000
Provident Fund Received upto Chaitra 18, 2058 (Lumpsum Payment) Rs. 4,00,000
(unapproved)
Provident Fund Received After Chaitra 18, 2058 (approved) Rs. 6,00,000

Mr. Nayak also conducts a small trading business M/S Nayak Concerns of his own.
Information relating to which are as follows:
Particulars Rs. Particulars Rs.
To Opening Stock 80,000 By Sales 5,00,000
To Purchase 3,90,000 By Closing Stock 1,20,000
To Gross Profit 1,50,000
Total 6,20,000 Total 6,20,000
To Personal Drawings 60,000 By Gross Profit 1,50,000
To Office Expenses 18,000 By Office Furniture Sales 35,000
To Printer Purchased 7,000 By Bad Debt Recovered 1,00,000
To Net Profit 2,50,000 By Dividend 50,000

Ms. Nayak works in a private institution & earns a total taxable salary of Rs. 2,00,000. The
couple has opted to be assessed separately for Income Tax Purpose.
Additional Information:
a. M/S Nayak Concern has the following WDV of Fixed Assets as on 01.04.20X-66.
Furniture Rs. 1,00,000

The Institute of Chartered Accountants of Nepal 301


CAP-II Paper 7 - Income Tax and VAT
Computer Rs. 45,000
Motorcycle Rs. 80,000
One Spare Table is sold on Rs. 35,000
b. Above Office Expenses includes Rs. 2,000 for the purpose of rice cooker gifted to a
customer on his wedding party.
c. Printer was purchased on 15th Poush, 20X-66.
d. Bad debt Recovered Rs. 1,00,000 was not allowed as expenses in F.Y 20X-64/X-65.
e. Dividend Rs. 50,000 is from NIC Bank Ltd.
He also provides following further information not related to his employment, and
business.
d. Gain Rs. 1,00,000 on Sale of shares.
e. Donation to a political Party Rs. 50,000.
f. Interest of Rs. 20,000 from personal deposit account of NATURAL BANK LTD.
Required: (Dec 2009, 20 Marks)
a. Assessable Income from Employment
b. Assessable Income from Business
c. Total Assessable Income
d. Tax Liability

Answer:
Computation of Assessable Income from Employment
Particulars Sec. Amount Working Notes
Ref
Net Salary (After TDS) 8 (2) 457,000
TDS paid in respect of salary 8 (2) 37,000 Reimbursement of Personal
Expense
Employers’ Contribution to 8 (2) 100,000
Retirement Fund
Compensation received as per 8 (2) 70,000 Employment related compensation
Employee Rule & 31 – assumed not during employment
Investment insurance premium paid 8 (2) 19,000 Assumed facility from employer as
by employer per Employment Policy
Leave Encashment 8 (3) - Retirement payment, Final
/92 withholding
Emergency medical treatment paid by 8 (2) 10,000 Reimbursement of personal
employer expense and it exceeds Rs. 500
Reimbursement of Tour Expense 8 (3) - Expense serving proper business
during official visit purpose of employer
Gratuity Received 8 (3) - Retirement Payment, Final
Withholding
Provident Fund Received 8 (3) - Retirement Payment, Final
Withholding

The Institute of Chartered Accountants of Nepal 302


CAP-II Paper 7 - Income Tax and VAT
Assessable Income from Employment 693,000

Computation of Assessable Income from Business


Particulars Sec. Amount Working Notes
Ref
Sales 7 (2) 500,000
Office Furniture Sold - Inclusion in Depreciation
calculation
Bad Debt Recovered 7 (2) - Recovery to the extent deducted
/ 25 previously are included, and all bad
debt was not deductible previously
in the given case
Dividend 7 (3) - Final Withholding, assuming NIC
/ 92 Bank Ltd. is resident bank
Total Inclusions (A) 500,000
Less: Deductions
Interest Expense u/s 14 (1) 14 -
Cost of Trading Stock 15 350,000 Opening Stock value + Purchases –
Closing Stock Value
Depreciation 19 45,250 Working Note (1) below
Repair and Improvement cost 16 -
General Deductions 13 16,000 Rice Cooker gifted to a customer in
event of wedding is personal
expense.
Interest Expense u/s 14 (2) 14 -
Pollution Control Cost 17 -
Research and Development Cost 18 -
Total Deductions (B) 411,250
Assessable Income from Business 88,750
(A- B)

Computation of Assessable Income from Investment


Particulars Sec. Amount Working Notes
Ref
Gain on Disposal of Shares 9 (2) 100,000 Assumed from Listed shares
Interest from personal account 9(3) - Final withholding, assuming
& 92 Natural Bank is resident
Assessable Income from Investment 100,000

The Institute of Chartered Accountants of Nepal 303


CAP-II Paper 7 - Income Tax and VAT
Computation of Total Assessable Income, Taxable Income and Balance Taxable Income
Particulars Sec. Amount Working Notes
Ref
Assessable Income from Employment 693,000
Assessable Income from Business 88,750
Assessable Income from Investment 100,000
Total Assessable Income 881,750
Less: Reductions u/s 12, 12Kha & 63,
if any
Reduction u/s 12 - Assuming political party is not
registered with Election
Commission
Reduction u/s 63 250,000 Lower of Rs. 300,000 or 1/3rd of
Assessable income (293,917) or
actual contribution (Rs. 250,000)
Taxable Income 631,750
Less: Investment Insurance Premium 19,000 Lower of Actual Rs. 19,000 or Rs.
paid 40,000
Balance Taxable Income 612,750
Statement of Tax Liability (Assessed as Individual- Working note 2 for the reason)
Balance Taxable Income Less Gain on Disposal of NBCA: 512,750
Apply normal rates to this amount of Rs. 512,750; assuming no contribution to contribution based
SSF; and rates of NBCA on Gain on disposal of NBCA:
1st Rs. 500,000 1% (W.N. 3) 5,000
Next Rs. 12,750 10% 1,275
Gain on Share Rs. 100,000 (NBCA) 5% 5,000
Total tax Liability (apart from final withholding taxes) 11,275

Working Notes:
1. Calculation of Depreciation
Particulars Pool B Pool C
Opening Depreciation Base (I)
Furniture 100,000
Computer 45,000
Motorcycle 80,000
Add: Absorbed Additions (II) 7,000
(100% of printer purchased as added in Poush)
Less: Disposal Value (III) (35,000)
Depreciable basis (or Depreciation Base) 117,000 80,000
(IV= I + II – III)
Rate of Depreciation (V) 25% 20%
Depreciation Amount 29,250 16,000

The Institute of Chartered Accountants of Nepal 304


CAP-II Paper 7 - Income Tax and VAT

2. Since, his wife is also earning income. The spouse will pay tax separately on her income, which
is max. Rs. 2,000.
3. Computation of taxes in respect of final withholding payments [Voluntary (for understanding
purpose of students)]
Income Head Tax Reason
Amount
Leave Encashment (accrued until Not taxable- Rule 20(6), Income
2058/12/18) Tax Regulation
Leave Encashment (accrued after 25,500 15% of Rs. 170,000
2058/12/18)
Gratuity (accrued until 2058/12/18) Not taxable- Rule 20(6), Income
Tax Regulation
Gratuity (accrued after 2058/12/18) 105,000 15% of Rs. 700,000
Provident Fund (accrued until Not taxable- Rule 20(6), Income
2058/12/18) Tax Regulation
Provident Fund (accrued after 5,000 5% of Gain (Rs. 100,000)- Gain is:
2058/12/18) from approved RF Accrued amount less
higher of (Rs. 500,000 or 50% of
accrued amount)]
Total taxes in respect of final 135,500
withholding payments

4) Answer how much income retained after tax for Income Year 20X-66/X-67:
(Dec 2009, 12 Marks, CA Inter)
a. Mrs. B is administrator of incapacitated husband Mr. B. A firm was registered in the
name of Mr. B before he becomes incapacitated and the firm earns taxable income of Rs.
500,000 from export of merchandise. Mrs. B has not any income.
Answer
As per Sec. 2 (5) of Schedule 1 of Income Tax Act, 2058; a trust taking care of property of
incapacitated natural person shall be taxable as resident individual.
Total taxable income 500,000
Less: Deduction availed to disabled person
(Not available, since it’s an entity)
The tax liability is as follows:
1st Rs. 500,000 0% -
Total Tax Liability 0

b. Mrs. C has taxable income from investments of Rs. 161,000.


Answer:
The tax liability is Zero, as it falls within the Basic Exemption Limit.

The Institute of Chartered Accountants of Nepal 305


CAP-II Paper 7 - Income Tax and VAT
c. Mrs. Bantawa have following incomes: Local Himali Herbs Industry Rs. 100,000, trading
of shoes Rs. 100,000, interest from local cooperative Rs. 8,000 and salary from same
cooperative as market executives Rs. 100,000. She opted as couple.
Answer
Interest from local cooperative is final withholding (Sec. 92).

The total Taxable Income is Rs. 300,000; which is within the Basic Exemption Limit. Since, her
employment income is not from pension and assuming she does not contribute to Social Security
Fund, she has to pay 1% tax on Employment Income up to Rs. 600,000.

Therefore, her tax liability:


1st Rs. 100,000 1% 1,000
Remaining Rs. 200,000 0% 0
Total Tax Liabilities 1,000
Female tax credit is not available, since has income from other sources also.

d. Mr. Majgain from Nagarkot has employment income of Rs. 100,000 per month before
deducting the contribution to approved retirement fund 40%. He is widower with a child.
Answer
Total Inclusions in Employment Income 1,200,000
Total Assessable Income 1,200,000
Less:
Contribution to Approved Retirement Fund- lower of: 300,000
i. Actual (40% of salary) 480,000
rd
ii. 1/3 of Assessable Income 400,000
iii. Maximum 300,000
Taxable Income 900,000

Calculation of Tax Liability- Deemed Couple


1st Rs. 600,000 1% 6,000
Next Rs. 200,000 10% 20,000
Next Rs. 100,000 20% 20,000
Total Tax Liability 46,000
Assuming that there is no contribution in Contribution Based Social Security Fund.

5) Mr. A, who is a resident of Nepal and a married person, submits following particulars of his
income for the financial year ending 15.07.20X5. Compute his total income and tax payable.
(Dec 2006, 8 Marks, CA Inter)
Rs.
Salary 1,00,000
Dividends from Indian companies 2,600
Interest on fixed deposit with bank 7,400

The Institute of Chartered Accountants of Nepal 306


CAP-II Paper 7 - Income Tax and VAT
Income from the units of the Unit Trust of India 3,900
Interest accrued on National Savings Certificates VIII Issue 2,000
Honorarium received as examiner not being casual income 3,500
Gift from friend received on 26.1.20X5 30,000
Employer's contribution to Unrecognized Provident Fund and (Interest
interest received thereon 26,000 included is
Rs. 2,000)
Assessee's own contribution to Unrecognized Provident Fund 21,000
Interest on his own contribution to Unrecognized Provident
Fund 5,000
Interest on deposit with private concerns 4,000
Interest on Debentures from listed companies 2,600
Winning from TV Game Show (Gross) 1,50,000
T.D.S. on Winnings from TV Games Show 37,500
Deposit in recognized retirement fund with CIT A/c 58,000
Contribution to Life Insurance Policy (Policy amount Rs.
2,00,000) 10,000
Medi-claim Insurance premium for self and wife 5,000

Answer:
Statement of Assessable Income from Employment
Particulars Sec. Ref Amount Working Notes
Salary 8 (2) 100,000
Assessable Income from
100,000
Employment

Statement of Assessable Income from Investment


Particulars Sec. Ref Amount Working Notes
9 (2) & Assuming Indian Company is not
Dividends from Indian Company 2,600
54 (2) resident of Nepal
Final withholding, assuming not
Interest on Fixed Deposit with Bank 9 (3) -
related to his business
Income from the units of the Unit Trust 9 (2) & Assuming Indian Company is not
3,900
of India 54 (2) resident of Nepal
Not received, accrued, final
Interest accrued on National Savings withholding even obtained.
9 (2) -
Certificates VIII Issue (Investment income accounting on
cash basis).
Honorarium received as examiner not
92 - Final withholding
being casual income

The Institute of Chartered Accountants of Nepal 307


CAP-II Paper 7 - Income Tax and VAT
not taxable, income does not fall
Gift from friend received on 26.1.20X5 5 -
under any income head
It is taxable only when the retirement
Interest on his own contribution to
- fund makes payment at the time of
Unrecognized RF
retirement
Interest on deposit with private concerns 9 (2) 4,000
Interest on Debentures from listed
92 - Final withholding
companies
Winning from TV Game Show (Gross) 92 - Final withholding
Assessable Income from Investment 10,500

Statement of Total Assessable Income, Taxable Income and Balance Taxable Income
Particulars Sec. Ref Amount Working Notes
Assessable Income from Employment 100,000 From above Calculation
Assessable Income from Investment 10,500 From above Calculation
Total Assessable Income 110,500
One-third of Total Assessable
Less: Contribution to Approved
63 36,833 Income, or Actual deposit or Rs.
Retirement Fund
300,000; whichever is lower
Taxable Income 73,667
Less: Deposit of Life Insurance Sch. 1, Actual premium paid or Rs. 40,000;
10,000
Premium 1(12) whichever is lower
Actual premium paid or Rs. 20,000;
Less: Payment of Medical Insurance Sch. 1,
5,000 whichever is lower and assuming
Premium to Resident Insurance Co. 1(17)
Couple Assessment
Balance Taxable Income 58,667

Statement of Tax Liability


Higher taxation in respect of Social
1st Rs. 58,667 1% 587
Security Tax interpreted
Taxes paid in relation to Final Withholding Payments
Interest on Fixed Deposit with bank 5% 370
Honorarium received as Examiner 15% 525
Interest on Debentures from Listed
5% 130
Companies
Winning of TV Game Show 25% 37,500
Total 38,525

The Institute of Chartered Accountants of Nepal 308


CAP-II Paper 7 - Income Tax and VAT

Chapter 18: Tax Administration, Documentation and Payment of Tax

1) What are the rights of a tax payer as per Income Tax Act, 2058? (Dec 2011, 4 marks, CA
Inter)
OR
As Section 74 (2) of Income Tax Act-2058, briefly mention the rights of taxpayers. (June
2003, 5 Marks, CA Inter)
Answer:
The following rights are available to a taxpayer:
a. Right to get respectful behavior;
b. Right to receive any information related to tax as per the prevailing Laws;
c. Right to get an opportunity of submitting a proof in one’s own favor in respect of tax matters;
d. Right to appoint lawyers or auditors for defense; and
e. Right to secrecy in respect of tax matters and to keep it inviolable.

2) Discussing the relevant provisions of Income Tax Act, 2058, and Rules 2059, mention the
Circumstances under which PAN can be suspended. (June 2021, 5 marks)

Answer:
As per section 78(a) of ITA, 2058, the Department can suspend PAN in following cases
i) Stops carrying transaction
ii) In case of entity, closure, sales or transfer of the entity or other similar activities which results
in extinguishment of its existence
iii) In case of personal ownership, death of the person having such ownership
iv) In case the registration is done by mistake

Moreover, Regulation 23a, mentions the process through which PAN is suspended:
1) The person or entity desirous of suspending their PAN as per section 78a of the Act, have to
submit an application to the Department within 30 days from the date in which the cause for
such suspension has arisen.
2) Income tax return should have been filed and tax deposition should have been done at the time
of filing of application under sub-rule (1
3) After inquiry, on the application filed under sub-rule (1), the Department should inform about
the cancellation of the PAN or the fact that the PAN cannot be cancelled.

3) What is Permanent Account Number? Explain the provisions relating to Advance Rulings.
(Dec 2003, 1+3, CA Inter)
Answer:
Permanent Account Number:
IRD can issue a unique identification number to identify a taxpayer. The unique identification
number is Permanent Account number (interchangeably called Taxpayer’s Identification number-

The Institute of Chartered Accountants of Nepal 309


CAP-II Paper 7 - Income Tax and VAT
TPIN), which is issued by IRD to identify a taxpayer under the power conferred by Sec. 78 (1) of
the Act.

Besides IRD, the PAN can also be issued by entity authorized by Department to issue such number.
The list of such authorized department is produced in the website of IRD, and extracted (as on the
date of writing this book) as part of End Note of this book. The authorized entity shall issue
permanent account number by abiding the provisions of Income Tax Act. But there is restriction
on such taxpayer obtaining PAN number from other authorized entity than IRD to conduct foreign
trade, i.e. the taxpayer cannot conduct import or export transaction for particular period after
obtaining PAN.

Provisions related to Advance Rulings


In case a person makes a written application to IRD seeking IRD’s position regarding the
application of this Act with respect to an arrangement proposed or entered into by the person, IRD
under signature of DG may issue, in writing, an advance ruling in this regard.
IRD shall not issue such an advance ruling on the matters under consideration of any court or
decided by a court.

Under the following circumstances the ruling shall be binding on the every officer of IRD, on IROs
and a civil servant who is authorized to work as tax officer:

The applicant has made a full and true disclosure of all aspects of the arrangement relevant to the
ruling; and
a. The arrangement proceeds in all material respects as described in the application for the ruling.
b. Where the advance ruling issued by IRD contradicts with a public circular, for the person who
applied for the advance ruling, the advance ruling shall be applicable.

4) What is the provision under sec 12Ka of Income Tax Act 2058 as regards expenses incurred
towards ancient, religious and cultural heritage conservation and sports development
activities? (Dec 2010, 5 Marks)

Answer:
Conditions to claim Expenditure u/s 12Ka
All the following conditions shall be satisfied for any expenditure to be eligible to be claimed
under this Section:
a. The person shall be a Company,
b. The person shall obtain prior approval of IRD to incur such expenditure, and
c. The expenditure shall be incurred for conservation and promotion of ancient, religious or
cultural heritages established in Nepal; and/or construction of public infrastructure related to
the Sports

The Institute of Chartered Accountants of Nepal 310


CAP-II Paper 7 - Income Tax and VAT
Eligible Amount for deduction to arrive at Taxable Income
The lower of the following amount is eligible for deduction:
a. Actual expenditure incurred
b. 10% of the Assessable income during the year
c. Rs. 10 Lakhs

5) Mention the timings of payment of taxes under the Income Tax Act, 2058 prescribed in the
following cases: (July 2015, 5×1=5)
a. Deposit of amount of withholding taxes by the person who is required to withhold Taxes.
Answer:
Within 25 days of end of month where TDS is deducted (deemed or actual), month implies the
calendar month as per Nepali Calendar.

b. Amount of Tax payable by a person who has submitted the Income Tax Return as per
section 96 and who has due to be paid as per assessment made under section 99
Answer
Within due date of filing income return under Sec. 96, which is usually three months of the end of
Income Year

c. Amount of Tax payable under jeopardy assessment under section 100 (2)
Answer
Within such due date that is specified in the notice issued by IRD u/s 102

d. Amount of Tax payable under amended assessment under section 101


Answer
Within such due date that is specified in the notice issued by IRD u/s 102

e. Installments amount of Tax payable based on estimated Tax.


Answer
Within Poush, Chaitra and Ashad, an amount not less than 40%, 70% and 100% of estimated tax
liability respectively on cumulative basis in case of taxpayers who has to pay tax on installments
other than a turnover based tax payer
For turnover based taxpayer, due date is Poush and Ashad.

The Institute of Chartered Accountants of Nepal 311


CAP-II Paper 7 - Income Tax and VAT

Chapter 19: Tax Return, Assessment and Collection of Tax

1) Mention whether the return filing is required or not on the following circumstances: (Dec
2017, 5 Marks)
Answer
In any of the following circumstances, a person is relieved from submitting income returns unless
otherwise required by IRD in writing or through public circular or if the person is a natural person,
his/her income for any income year exceeds Rs. 4 Million:
1. Where a person has no income tax payable under Sec. 3 (Ka) during the Income Year,
2. Where a person’s income consists exclusively of Final Withholding Payments during the
Income Year,
3. In case of a resident natural person, who satisfies all the following conditions:
➢ The person’s income for the year consists exclusively of income from any employment
having a source in Nepal,
➢ The person has only one employment at a time during the year, even if the employment
changes during the year, and each employment is by a resident employer, and
➢ The person does not claim the following for the Income Year:
✓ a medical tax credit under section 51, other than with respect to medical tax credit paid
through the employer,
✓ a reduction in taxable income under section 63 (Contribution to Approved Retirement
Fund), other than with respect to retirement contributions paid through the employer, and
✓ a reduction in taxable income under Sec. 12 (Donation)
4. Where a vehicle owner paying tax pursuant to Sec. 1 (13) of Schedule 1 of the Act is a natural
person not operating such vehicle through a private firm,
5. Where a person generating income only from disposal of non-business chargeable asset desires
not to file return

a. Mr. X, a natural person having employment income of Rs. 4,00,000 in 20X-73/X-74, has
worked under a foreign consultancy firm, i.e. non-resident for Nepalese tax purpose.
Assume no deduction of retirement fund, insurance, remote area allowances etc.
Answer
Since any of the conditions above is not satisfied, Mr. X has to file income return.

b. Mr. X with Mrs. X, a resident natural person having employment income of Rs. 2.1
million (Rs. 1 million of Mr. X and Rs. 1.1 million of Mrs. X) in the year 20X-73/X-74.
Assume no deduction of retirement fund, insurance, remote area allowances etc.
Answer
If the employer is resident, these persons fulfill condition (3) specified above, and hence, are
relived from filing income return.
If the employer is non-resident, they have to file income return.

The Institute of Chartered Accountants of Nepal 312


CAP-II Paper 7 - Income Tax and VAT
c. S Enterprises, Kathmandu registered in Small and Cottage Industries Office is dealing
with VAT exempt goods. It has annual sales Rs. 15,00,000 in 20X-72/X-73 and Rs.
25,00,000 in 20X-73/X-74. It has assessable incomes Rs. 1,50,000 and Rs. 1,00,000 in 20X-
72/X-73 and 20X-73/X-74 respectively.
Answer
Since any of the conditions above is not satisfied, the owner of S Enterprises has to file income
return.

d. V Enterprises Kathmandu registered in Small and Cottage Industries Office is dealing


with VAT exempted and taxable goods. It has annual sales Rs. 5,00,000 in 20X-72/X-73
and Rs. 30,00,000 in 20X-73/X-74. It has assessable income Rs. 1,00,000 and Rs. 5,00,000
in 20X-72/X-73 and 20X-73/X-74 respectively.
Answer
Since any of the conditions above is not satisfied, the owner of V Enterprises has to file income
return.

e. Mrs. Chaulagai, a single natural person has income from various sources like dividend
income Rs. 2,00,000, interest income Rs. 3,00,000 from bank, meeting allowances Rs.
20,000 and capital gain from sale of shares Rs. 3,00,000.
Answer
Since any of the conditions above is not satisfied, Mr. Chaulagain has to file income return

2) Mr. Naresh Yadav has been working as a Chief Legal Advisor of a bank. During the income
year 20X-72/X-73, he has received remuneration income of Rs. 50,00,000 from the bank after
deducting TDS of Rs. 22,00,000. During the year he has won a lottery of Rs. 10,00,000.
Company deducted TDS of Rs. 2,50,000 on the said lottery and paid him Rs. 7,50,000. He has
income of Rs. 5,00,000 from his agricultural farm. He has also received dividend of Rs.
5,00,000 on the shares invested by him in an insurance company. Insurance Company has
deducted TDS of Rs. 25,000 on the said amount, and paid balance Rs. 4,75,000. He has also
received bonus share of Rs. 2,00,000 on the shares invested by him in a Hydro Power
Company. Hydro Power Company has deposited TDS of Rs. 10,000 on the said bonus share.
Discussing the relevant provisions of the Income Tax Act, 2058 determine whether he is
required to file Income Tax Return or not, if yes, prepare his Income Tax Return. (June
2017, 5 Marks)

Answer:
In circumstances as per Sec. 97 (1), a person is relieved from submitting income returns unless
otherwise required by IRD in writing or through public circular or if the person is a natural person,
his/her income for any income year exceeds Rs. 4 Million.
In the given case, the income of Mr. Naresh Yadav has income exceeding Rs. 4 Million during the
Income Year; therefore, he needs to file income return as per Sec. 97 (2).

The Institute of Chartered Accountants of Nepal 313


CAP-II Paper 7 - Income Tax and VAT
3) Mention the cases when the income tax returns are not required to be filed as per the
provisions of the Act. Are there any conditions/circumstances in which the department can
order a person to submit a tax return before the time given for filing income tax return? (Dec
2012, 5 Marks, CA Inter)
Or
When Return of Income is not required to be filed? (June 2006, 2 Marks, CA Inter)

Answer:
In any of the following circumstances, a person is relieved from submitting income returns unless
otherwise required by IRD in writing or through public circular or if the person is a natural person,
his/her income for any income year exceeds Rs. 4 Million:
1. Where a person has no income tax payable under Sec. 3 (Ka) during the Income Year,
2. Where a person’s income consists exclusively of Final Withholding Payments during the Income
Year,
3. In case of a resident natural person, who satisfies all the following conditions:
a) The person’s income for the year consists exclusively of income from any employment
having a source in Nepal,
b) The person has only one employment at a time during the year, even if the employment
changes during the year, and each employment is by a resident employer, and
c) The person does not claim the following for the Income Year:
✓ a medical tax credit under section 51, other than with respect to medical tax credit paid
through the employer,
✓ a reduction in taxable income under section 63 (Contribution to Approved Retirement
Fund), other than with respect to retirement contributions paid through the employer,
and
✓ a reduction in taxable income under Sec. 12 (Donation)
4. Where a vehicle owner paying tax pursuant to Sec. 1 (13) of Schedule 1 of the Act is a natural
person not operating such vehicle through a private firm,
5. Where a person generating income only from disposal of non business chargeable asset desires
not to file return

4) Mrs. Sarita is working as a lecturer in Bidhata Nursing Campus. She is the Director of
National Maternity Hospital in Lamadanda. After deduction of tax in source, during the
income Year 20X2/X3, she has got Rs. 17,000 as meeting allowance. She has one house that
has been provided in rent to Health Clinic and the Health Clinic pays the rent amount after
deduction of tax on source. She has deposited Rs. 5,00,000 in fix account in Agriculture
Development Bank and the gross interest receipt as per bank statement is Rs. 35,000. Mrs.
Sarita is asking regading the filing of return. State your view regarding the submission of
file by Sarita. (June 2003, 3 Marks, CA Inter)

The Institute of Chartered Accountants of Nepal 314


CAP-II Paper 7 - Income Tax and VAT
Answer:
In any of the following circumstances, a person is relieved from submitting income returns unless
otherwise required by IRD in writing or through public circular or if the person is a natural person,
his/her income for any income year exceeds Rs. 4 Million:
1. Where a person has no income tax payable under Sec. 3 (Ka) during the Income Year,
2. Where a person’s income consists exclusively of Final Withholding Payments during the
Income Year,
3. In case of a resident natural person, who satisfies all the following conditions:
a) The person’s income for the year consists exclusively of income from any employment
having a source in Nepal,
b) The person has only one employment at a time during the year, even if the employment
changes during the year, and each employment is by a resident employer, and
c) The person does not claim the following for the Income Year:
✓ a medical tax credit under section 51, other than with respect to medical tax credit paid
through the employer,
✓ a reduction in taxable income under section 63 (Contribution to Approved Retirement
Fund), other than with respect to retirement contributions paid through the employer,
and
✓ a reduction in taxable income under Sec. 12 (Donation)
4. Where a vehicle owner paying tax pursuant to Sec. 1 (13) of Schedule 1 of the Act is a natural
person not operating such vehicle through a private firm,
5. Where a person generating income only from disposal of non business chargeable asset desires
not to file return

In the given case, Mr. Sarita does not fulfil the exclusivity criteria as per condition (b) and (c)
above, as she has multiple incomes. Therefore, principally, she needs to file income return.
However, practically IRD has not designed or circulated such forms to require a natural person to
file such return.

5) During the income year 20X-74/X-75, Mr. Anand won DV lottery of America and he left
permanently with his family members on 25th Poush 20X-75. Mr. Anand has already filed
Income Return for income year 20X-74/X-75, however he has not filed Income return for
income year 20X-75/X-76. Mr. Anand has earned Rs. 1,500,000 taxable income up to 25th
Poush 20X-75.
Can income tax authority assess the income tax liability of Mr. Anand immediately? (June
2019, 5 Marks)

Answer
As per Sec. 96 (5), Inland Revenue Department may demand income return for an Income year or
part of income year before the due date of filing return, in any of the following conditions:
a. Where a person is bankrupt, goes into liquidation or wound up,

The Institute of Chartered Accountants of Nepal 315


CAP-II Paper 7 - Income Tax and VAT
b. Where a person leaves Nepal for indefinite period of time (Note: don’t mix up with Value
Added tax, the word there is “leaving Nepal permanently”. There is difference between two
terms)
c. Where is person is ceasing its activity in Nepal, or
d. Where IRD otherwise thinks it appropriate

When such condition arises, IRD has the right to demand income return for part of Income Year
as well. Further, IRD may also conduct jeopardy assessment of tax under Sec. 100.
In the given case, Mr. Anand is leaving Nepal for indefinite period of time. Therefore, IRD can
assess the income tax liability of Mr. Anand immediately as per the power given to it u/s 96 (5)
and 100 of the Act.

6) Sitaram Poudel has received a notice from Tax officer on 1st Chaitra 20X-72 for submission
of Income Tax Return for income year 20X-72/X-73 by 1st Baishakh 20X-73. He has been
planning to migrate to Australia. Sitaram Poudel’s contention is he need not have to submit
tax return since he has income from employment only, and due date of filing income tax
return is also end of Asoj.
Stating the relevant provisions of Income Tax Act, 2058, please advise Sitaram Poudel on the
above matter. (June 2017, 5 Marks)

Answer
As per Sec. 96 (5), Inland Revenue Department may demand income return for an Income year or
part of income year before the due date of filing return, in any of the following conditions:
a. Where a person is bankrupt, goes into liquidation or wound up,
b. Where a person leaves Nepal for indefinite period of time (Note: don’t mix up with Value
Added tax, the word there is “leaving Nepal permanently”. There is difference between two
terms)
c. Where is person is ceasing its activity in Nepal, or
d. Where IRD otherwise thinks it appropriate

When such condition arises, IRD has the right to demand income return for part of Income Year
as well. Further, IRD may also conduct jeopardy assessment of tax under Sec. 100.
In the given case, Mr. Sitaram is leaving Nepal for indefinite period of time. Therefore, IRD can
assess the income tax liability of Mr. Anand immediately as per the power given to it u/s 96 (5)
and 100 of the Act.
Further, provisions of Sec. 97 of Income Tax Act, 2058 are not blanket exemption for relief in
filing income return, but are conditional. If IRD summons a notice in writing requiring a person to
file return, every person is required to file income return regardless of whether conditions of Sec.
97 (1) are satisfied.
In view of above, Mr. Sitaram has to submit return demanded by IRD.

The Institute of Chartered Accountants of Nepal 316


CAP-II Paper 7 - Income Tax and VAT
7) What is the information to be included while issuing a reassessment notice under section 102
of the Income Tax Act? (Dec 2012, 3 Marks, CA Inter)
Or
What are the matters that should be included in the assessment notice issued by Inland
Revenue Department? (June 2010, 5 Marks)

Answer:
The jeopardy assessment order shall include all the following matters:
a) The total tax payable by the taxpayer for the period of assessment and the tax due to him;
b) The method of calculation of the tax liability;
c) The reason of the amended/jeopardy assessment by the IRO;
d) The period within which the tax due is payable; and
e) Where, when and how to appeal against the order if the taxpayer is not satisfied with the
amended assessment.

8) What are the methods of tax assessment as per the Income Tax Act, 2058? (Dec 2008, 10
Marks, CA Inter)

Answer
a. Self -assessment
Actual/Self-Assessment [Sec. 99 (1)]
Where a person files a return of income for an income year disclosing the following matters, self-
assessment is treated as made on the due date for filing the return:
• the income tax payable by the person for the year under section 3 (Ka) and (Kha), and
• the amount of that tax still to be paid for the year in the amount shown in the return

Deemed Self-Assessment [Sec. 99 (2)]


Where a person fails to file a return of income for a tax year then, until such time as a return is
filed, an assessment is treated as made on the due date for filing the return as follows:
• the income tax payable by the person for the year is equal to the sum of any income tax withheld
from payments derived by the person during the year under Chapter 17 of the Act and any
income tax paid by the person by installment for the year under Chapter 18 of the Act, and
• There is no tax payable on the assessment.

b. Jeopardy Assessment
Conditions for Jeopardy Assessment [Sec. 96 (5)]
A written notice may be served upon a person by IRD requiring submission of income return
before the due date of filing return for a full Income Year or part of Income year by the date
specified in the notice, in any of the following four conditions:
• Where the person becomes bankrupt, is wound-up, or goes into liquidation,
• Where the person is about to leave Nepal for indefinite period of time,
• Where the person is otherwise about to cease the activity in Nepal, or

The Institute of Chartered Accountants of Nepal 317


CAP-II Paper 7 - Income Tax and VAT
• Where IRD otherwise considers it appropriate.

Jeopardy Self-Assessment [Sec. 100 (1)]


Where a written notice is served upon a person to submit income return for an Income Year in the
conditions as above, the person shall make self-assessment of tax pursuant to Sec. 99 and submit
income return by the date specified in the notice.

Jeopardy Assessment by Tax officer [Sec. 100 (2)]


In the circumstances giving rise to jeopardy assessment, instead of requiring a person to file a
return of income, the IRD may, according to the IRD's best judgment, make an assessment of
income for a full Income year or part of income year.

Information to be considered for Jeopardy Assessment [Sec. 100 (2)]


The following information is considered for the jeopardy assessment:
• Assessable income of the taxpayer from business, employment or investment, and the source
of such income,
• Taxable income of the taxpayer during the year and the total amount of tax due to the taxpayer,
and
• In the case of a taxpayer, which is a foreign permanent establishment, the income repatriated
to a foreign country during the period and tax payable on such repatriation.

Show-cause Notice [Sec. 100 (5)]


A show-cause notice shall be served upon the person with time limit of 7 days to provide the proof
in own defense before the issuance of jeopardy assessment order.

Effect of Jeopardy Assessment [Sec. 100 (3)]


The effect of jeopardy assessment shall be as follows:
• Where the jeopardy assessment is conducted for a full income year, the taxpayer is not required
to submit a self-assessed income return, or
• Where the jeopardy assessment is conducted for a part of income year, the person shall file an
income return for the full income year

Treatment of Tax paid under Jeopardy Assessment [Sec. 100 (4)]


The tax paid by the taxpayer under jeopardy tax assessment is treated as advance tax paid for the
Income Year and the person can claim credit of such tax while filing the income return of full
income year.

c. Amended Assessment
IRD may amend an assessment made under self and jeopardy assessment provisions. The
amendment in assessment is to adjust the assessed person's liability to tax, including any tax

The Institute of Chartered Accountants of Nepal 318


CAP-II Paper 7 - Income Tax and VAT
payable on the assessment. The adjustment shall be in a manner consistent with the intention of
the Act and according to IRD’s best judgment.

Number of Amended Assessment [Sec. 100 (2)]


In case IRD thinks it proper to do so, the assessments can be amended again and according to the
IRO’s best judgment for as many times as it thinks appropriate.

Time Limit to make amendment in Assessment [Sec. 100 (3)]


The amended assessment shall be completed within four years of the date mentioned below, unless
there is evidence that assessment is inaccurate by reason of fraud:
• In the case of self-assessment: within four years from the due date for filing the return; or
• In the case of jeopardy assessment: within four years from the date on which the notice of
assessment is served to the taxpayer under Sec. 102.
• Reassessment of amended assessment: within four years from the date mentioned above, as
the case may be.

Amended Assessment of inaccurate assessment by reason of Fraud [Sec. 101 (4)]


In case a person’s assessment is inaccurate by reason of fraud, the limitation of four years as above
is not applied. The due date of completing amended assessment of inaccurate assessment by reason
of fraud is one year from the date of receipt of information of fraud.

Limitation of IRD if the competent court has settled the assessment [Sec. 101 (5)]
IRD cannot amend an assessment to the extent that the assessment has been amended or reduced
pursuant to an order of Revenue Tribunal or any other authorized court unless the order is vacated.

Show-cause Notice [Sec. 100 (6)]


A show-cause notice shall be served upon a taxpayer providing 15 days’ time limit to submit
clarification and proof in one’s own favor. The show-cause notice shall include the clear basis for
the amendment.

9) The Inland Revenue Department has issued notice to Mr. Rohit Shrestha, a renowned
business tycoon of Nepal; to pay the due tax liability amounting to Rs. 25,566,725 within
Baisakh 25, 20X-71 and the same notice was also published in the National Level Newspaper.
The notice also contains that if Mr. Shrestha does not pay the tax dues on or before the
specified time limit then appropriate action shall be taken against him. Mr. Shrestha
aggressively trying to arrange the money but he could not be able to pay the due tax on time.
In the meantime, Mr. Shrestha had a prescheduled visit to New Delhi on Baisakh 26, 20X-71
to attend a marriage ceremony of one of his friend's daughter and accordingly he went to
Tribhuvan International Airport (TIA) on the schedule time to fly to New Delhi. At the TIA,
the immigration staff stopped him to enter to the aircraft stating that the Inland Revenue
Department has given notice to stop Mr. Rohit Shrestha to going abroad. In the light of the

The Institute of Chartered Accountants of Nepal 319


CAP-II Paper 7 - Income Tax and VAT
above fact, please state whether the immigration staff at TIA has got the right to do so? If
not, what remedy is available to Mr. Shrestha for the unauthorized act of the immigration
staff as per the provisions of Income Tax Act, 2058. (June 2016, 5 Marks)

Answer
As per Sec. 106, when a person becomes a tax defaulter, the related tax office may serve a notice
in writing to the concerned Department of GON to prevent the tax defaulter from leaving Nepal
for a period of not more than 72 hours from the time of the expiry of deadline mentioned in the
notice served to the defaulter to pay due tax amount.

In case the time limit is to be extended, the tax office has to take a prior approval of the concerned
appellate court.

Once the tax defaulter pays the tax or makes an arrangement for payment in satisfaction to the tax
office, the office may withdraw the order.

In the given case, the deadline to pay tax by tax defaulter Mr. Rohit Shrestha is 25th Baisakh 20X-
71. As Mr. Shrestha did not make payment of tax dues within the deadline mentioned by Inland
Revenue Department, IRD has the right under Sec. 106 to serve a notice in writing to the concerned
Department of GON to prevent the tax defaulter from leaving Nepal for a period of not more than
72 hours from the time of the expiry of deadline mentioned; and the immigration department may
bar him from travelling outside Nepal.

10) What are the various actions that can be resorted by the tax office, if a taxpayer does not pay
the tax in time as prescribed in the Income Tax Act, 2058? (June 2009 CAP II/CA Inter June
2009/ CA Inter, June 2004)
Answer
The various actions are as follows:
a. Creating charge over asset and auctioning such asset to recover dues
b. Collection of tax from officer of the entity
c. Issue an order to prohibit a person from travelling abroad
d. Collection of tax from receiver of any person
e. Collection of tax from the person who owes money to tax defaulter
f. Collection of tax from the agent of Non-resident person
Note: Students are required to elaborate each topic in short based on the marks allocated.

The Institute of Chartered Accountants of Nepal 320


CAP-II Paper 7 - Income Tax and VAT

Chapter 20: Miscellaneous

1) Write in brief the provisions of the Income Tax Act in case of a married resident couple elect
to be treated as a couple. (Dec 2010, 5 Marks)
Answer
A resident natural person and his/her resident spouse (husband or wife) may elect (select) to
be treated as Couple by giving written notification to IRD for any Income year for Tax purpose.

In case a resident natural person & his/her resident spouse elect to be assessed as Couple for the
income pertaining to any Income Year, the natural person & his/her spouse is jointly & severally
(separately) responsible for payment of tax pertaining to that Income Year.

A Widower or widow is treated as Couple for tax purpose, if s/he has to look after the dependent.

2) The paid-up share capital of ABC Ltd. consists of 150,000 shares of Rs. 100 each. As of Ashad
end 20X-68, the Profit and Loss Appropriation Accounts of the company is abstracted as
below:
Profit and Loss Appropriation Account:
Profit for the year Rs. 1.000,000
Accumulated profit b/d Rs. 1,500,000
Total distributable profit Rs. 2,500,000

The Board of Directors (BOD) of the company proposed to issue bonus share at 10%. Also,
it is proposed that dividend tax on the capitalized profit (sec 88(1)) is to be borne by company
by distributing cash dividend so that shareholders do not have to pay any tax at source as
required under sec. 88(1).

You are required to compute the amount of such cash dividend. (Dec 2011, 5 Marks)
Answer
The dividend tax @ 5% is to be borne by the shareholders. If shareholders receive any amount, the
amount is always 95% of total dividend declared by the company.

In the given case, amount of bonus share is 10% of paid up capital of the company, i.e. 10% of Rs.
15,000,000 which comes to Rs. 1,500,000.

As per the information in the question, the company does not collect cash from shareholders in
order to pay dividend tax, but is proposing cash dividend in order to fulfill that obligation, which
means, the amount of bonus share is 95% of total dividend (total dividend being sum of bonus
share and cash dividend). When we develop a mathematical formulation:
Total dividend = Bonus share + Cash dividend-------- (i)
Bonus share = 95% of Total Dividend------- (ii)

The Institute of Chartered Accountants of Nepal 321


CAP-II Paper 7 - Income Tax and VAT
Equating (ii)-
Rs. 1,500,000 = 95% of Total Dividend
Or, Total Dividend = Rs. 1,500,000/0.95 = Rs. 1,578,947.37

From equation (i), pasting value of total dividend and bonus share gives us cash dividend to be
proposed of Rs. 78,947.37.

3) Discuss the taxability of the gain from the amount received from Un-approved Retirement
Fund. (June 2007, 5 Marks, CA Inter)
Answer
The gain amount from unapproved retirement fund is taxed at the rate 5% as per Sec. 88 (2) and it
is final withholding as per Sec. 92, when it is paid by a resident unapproved retirement fund.

The gain amount is included in calculating investment income of a resident natural person, when
it is paid by a non-resident unapproved retirement fund, and the taxation on such income is subject
to Sec. 1 of Schedule 1 of the Act.

For a non-resident person, gain from non-resident unapproved retirement fund is not taxable in
Nepal as per Sec. 6.

The Institute of Chartered Accountants of Nepal 322


CAP-II Paper 7 - Income Tax and VAT

Chapter 21: True False Question

1) State whether following statements are true or false with appropriate provision of Income
Tax Act, 2058: (Dec 2015, 5×2=10)
a. Progressive Income Tax rate is only applicable to entire income of resident natural
person.
Answer
True, progressive tax rate is applicable to entire income of resident natural person.

b. If tax-exempt entities are unable to submit their financial statement within the stipulated
time limit, fee of 0.1% per annum on income shall be levied.
Answer
False, there is no such provision specifically to exempt organization.
(Not applicable to CAP-II)

c. TDS applicable on payment of vehicle hired by resident person to vehicle provider


registered in VAT is 1.5%.
Answer
True

d. Tax exemption has been increased for annual interest income up to of Rs. 25,000 from
Rs. 10,000 in the amount deposited to the microcredit institutions, rural development
banks, postal saving bank, and cooperatives in the Rural Municipality.
Answer
True

e. Finance Act, 2072 has introduced mandatory requirement to submit income returns for
the natural person with yearly income exceeding Rs. 4 million.
Answer
True

2) State whether following statements are true or false with appropriate provision of Income
Tax Act, 2058: (July 2015, 5×2=10)
a. Any person who fails to pay Tax within prescribed time without reasonable excuse shall
be liable on contravention for a fine of an amount ranging from Rs. 50,000 to Rs. 80,000
or an imprisonment for a term of not less than one month and not more than three
months or both.
Answer
False, the penalty ranges from Rs. 5,000 to 30,000 or imprisonment from one to six months or
both.
(Not applicable to CAP II)

The Institute of Chartered Accountants of Nepal 323


CAP-II Paper 7 - Income Tax and VAT

b. In case of person fails to maintain books of account and records as per section 81, the
person is responsible to pay fee higher of following amounts:
i. 0.1 % of turnover or gross receipt during the period for which the person fails to
maintain the accounts and records or
ii. Rs. 1000 per annum.
Answer
True
(Not applicable to CAP II)

c. The listed resident company deducted 15% withholding tax while paying interest having
source in Nepal to Tax exempt organization. The Tax exempt organization claims interest
earned by it from listed resident company as final withholding income.
Answer
False, as per Sec. 88 (1), interest is subject to 15% withholding tax and as per Sec. 92, interest paid
to an exempt entity by a resident bank and financial institution is final withholding.

d. In case of gain on the disposal of land & buildings owned by M/S ABC limited which has
been owned for less than 5 years, applicable Tax rate will be 5% and if the disposed land
& buildings has been owned for more than 5 years, applicable tax rate will be 2.5%.
Answer
False, the gain on disposal of business asset or depreciable assets of an entity is subject to tax at
corporate tax rate applicable to the entity.

e. Cooperatives societies are allowed to claim as expenses an amount equivalent to the


amount set aside in risk bearing fund to the extent of 15% of outstanding loan.
Answer
False, as per Sec. 59 (1Kha), it is to the extent 5% of loan outstanding.
(Not applicable to CAP II)

3) State with reason whether the following statements are True or False. (Dec 2009, 5 Marks)
a. As per the definition of Income Tax Act, 2058 “every entity is a person but every person
is not an entity”.
Answer
True, the definition of person includes both natural person and entity, however, entity cannot cover
natural person, which means all persons are not entity.

b. A non-resident has to pay income tax on income accrued in Nepal only.


Answer
False, there is no concept of accrual of Income in Nepal. Nepal has embraced source concept and
a non-resident has to pay tax in Nepal sourced income only in Nepal.

The Institute of Chartered Accountants of Nepal 324


CAP-II Paper 7 - Income Tax and VAT
c. Dividend of a co-operative society is exempt from tax.
Answer
Partially true, since the dividend by cooperatives specified in Sec. 11(2) is exempt and that by
other cooperatives is subject to dividend tax.

d. An individual has to follow accrual basis of accounting while calculating income under
the head investment.
Answer
False, as per Sec. 22, it shall be cash basis.

e. Meeting allowance shall not be included in the taxable income of an individual.


Answer
True, since it is final withholding payment if payment is up-to Rs. 20,000 per meeting.

4) Fill in the blanks. (June 2009, 10 Marks)


a. TDS return should be submitted to the department under Sec. 90 (1) of the Act from the end
of every month within …… days.
Answer: 25

b. Tax deducted should be deposited to the account of the government from the end of every
month within …… days.
Answer: 25

c. Interest on deficiency of installments of advance tax payable against installments paid will be
calculated on (daily/month-to-month/per annum) basis.
Answer: month-to-month basis
(Not applicable to CAP II)

d. Interest chargeable on deficiency of installments of advance tax payable against installments


paid will be at the rate of …….
Answer: 15%
(Not applicable to CAP II)

e. Income tax return under Sec. 99 is to be filed for the year 20X-65/X-66 before …………
Answer: Ashoj end 20X-66 unless otherwise required by Sec. 97, 98 and 100.

f. Estimate of tax to be submitted by a person liable to pay tax in installment is to be submitted


every year up to …… end.
Answer: Poush

The Institute of Chartered Accountants of Nepal 325


CAP-II Paper 7 - Income Tax and VAT
g. The certificate of tax deducted at source should be provided by the withholding agent to the
withholdee within 55 days from (Date of deduction / The end of the month in which deducted/
the end of the financial year).
Answer: the end of the month in which tax is withheld

h. Tax to be deducted from payment to a person carrying on business of letting out buildings
on rent is …………….%
Answer: not required if owned by a natural person

i. Tax to be deducted from the bill of service provider registered under VAT against VAT bill is
….%
Answer: 1.5%

j. If the total tax to be paid for the year is less than that, no installment tax need be paid is …….
Answer: Rs. 7,500

5) State with reason whether the following statements are True or False (Dec 2007, 10×1=10,
CA Inter)
a. For the purpose of calculation of the income for an income year from business, any
unrelieved loss of the previous eight years from any business of the same person is allowed
for deduction.
False, the unrelieved loss of last seven years in all cases and twelve years in case of projects
involving building power houses, generation and transmission of electricity and those doing
transactions under Petroleum Act, 2040 is allowed for deduction.

b. Standard interest rate means 15 percent per annum.


True

c. Any income of Government of Nepal is exempt from tax.


True

d. Dividend distributed by the industry established at special economic zone is fully


exempted for seven years from the commencement of its business.
False, the exemption is for first five years.

e. Withholding tax at the rate of 10 percent is applicable on the payment of dividend by a


resident person to non-resident.
False, it is 5%.

f. Income of the King of Nepal is not taxable.


Irrelevant in current scenario

The Institute of Chartered Accountants of Nepal 326


CAP-II Paper 7 - Income Tax and VAT
g. Natural person is required to follow cash basis of accounting for the computation of
income from investment.
True

h. Income of approved retirement fund is taxable.


False

i. Senior citizen allowance paid by the Government of Nepal is taxable.


False, it is exempt under Sec. 10

j. Department is required to furnish decision within 60 days on the application for advance
ruling.
False, it shall be within 45 days.

The Institute of Chartered Accountants of Nepal 327


CAP-II Paper 7 - Income Tax and VAT

Chapter 22: Short Notes

1) Write Short Notes


1. Public Circular (June 2021, 2 Marks)
Answer:
In order to bring about uniformity in the implementation of Income Tax Act & VAT Act and
simplify tax administration and give guidelines to the officers of the Inland Revenue Department
as well as the persons affected by the Acts, the Department may issue written public circulars,
accompanied by explanations, on the provisions made in this Act.
The Department shall make available the circular issued in the Department or in any other places
as per necessity or through any other means. The Department shall be compelled to take action
according to the circular issued unless and until such circular is revoked."

2. Assessable income (June 2021, 2 Marks)


Answer:
As per Section 6 of Income Tax Act 2058, subject to the Act, the following incomes earned by any
person for any employment or business or investment or windfall gains in any income year shall
be considered assessable income:
(a) Income earned by any resident person from his employment or business or investment or
windfall gains in that income year irrespective of the place of his source of income, and
(b) Income earned in that income year by any non-resident person from employment or business
or investment or windfall gains having income source in Nepal.
Provided that, the assessable income shall not include any income exempted from tax pursuant to
section 11 or 64 or both.

3. Controlled Foreign Entity (Dec 2020, 2.5 Marks)


Answer:
As per section 69 Clarification (b) of Income Tax Act, 2058, controlled foreign entity for an
income-year means a non-resident entity in which a resident person holds an interest, directly or
indirectly through one or more interposed non-resident entities, and the person is associated with
the entity or would be if the person and not more than four other resident persons were associated.
Similarly, section 69 clarifications (a) defines, Attributable income of a controlled foreign entity
for an income year is its taxable income for the year calculated as if the entity were a resident
entity.

4. Advance ruling (June 2019, 2.5 Marks)


Answer:
In case a person makes a written application to IRD seeking IRD’s position regarding the
application of this Act with respect to an arrangement proposed or entered into by the person, IRD
under signature of DG may issue, in writing, an advance ruling in this regard.

The Institute of Chartered Accountants of Nepal 328


CAP-II Paper 7 - Income Tax and VAT
IRD shall not issue such an advance ruling on the matters under consideration of any court or
decided by a court.

Under the following circumstances the ruling shall be binding on the every officer of IRD, on IROs
and a civil servant who is authorized to work as tax officer:

a. The applicant has made a full and true disclosure of all aspects of the arrangement relevant to
the ruling; and
b. The arrangement proceeds in all material respects as described in the application for the ruling.
Where the advance ruling issued by IRD contradicts with a public circular, for the person who
applied for the advance ruling, the advance ruling shall be applicable

5. Deductible expenses from cash payment


(June 2019, June 2018 and June 2016, 2.5 Marks)
Answer:
The cash payments as follows are always deductible, subject to other provisions of the Act:
a. If the turnover of the person is not more than Rs. 20 lakhs during the Income Year
b. Payment made to Government of Nepal, Constitutional bodies, corporate having ownership of
GON, Banks, and Financial institutions.
c. Payment to a farmer or a producer of primary agro products even in the case where the farmer
himself primarily processes the product.
d. Payment of a retirement contribution or a retirement payment.
e. Payment made in such areas where banking services are not available.
An area not having banking services means the area where there are no banking facilities within
a periphery of 10 kilometers.
f. Payment made on the day when banking services are closed or there is unavoidable compulsion
that the payment shall be made in cash.
g. Payment is made through the bank account of the payee.

6. Tax on agricultural income (Dec 2019 & June 2019, 2.5 Marks)
Answer:
Agriculture income of a natural person, except derived as follows, is exempt from tax:
a. In case the person holds land for industrial purpose as an industrial unit as specified by a
notification from GON and as per the terms of the notification under Section 12 (d) of Land
Act, 2021 generates agriculture income from such land; and
b. An agro-industrial unit holding land for agro- based industries as specified by a notification
from GON and as per the terms of the notification under Section 12 (e) of Land Act, 2021,
generates agriculture income from such land.

The Institute of Chartered Accountants of Nepal 329


CAP-II Paper 7 - Income Tax and VAT
7. Approved medical expenses (Dec 2018, 2.5 Marks)
Answer:
Medical expenses incurred for the treatment of a natural person in an approved hospital, nursing
home, health center, or by a medical practitioner on the basis of actual invoice produced, except
the following, is approved medical expenditure:
a. Expenses incurred on cosmetic surgery,
b. To the extent the expenses as above is reimbursed form the insurance company against any
medical insurance, or
c. To the extent the medical cost is incurred to treat the injured bodily part of an individual where
the compensation is exempted from tax

8. Deductible research and development expenses (Dec 2018, 2.5 Marks)


Answer:
‘Research and development’ cost is the cost incurred by a person for the purpose of further
development of an established business and for improvement of the production process and the
products.

Research and development cost is eligible for deduction, if it satisfies both the following
conditions:
a. The Cost shall be incurred for the operation of business, and
b. The cost shall be incurred during the Income Year.

Research & Development Cost is deductible for Taxation purpose subject to the minimum of
following TWO:
i. Actual Cost incurred during the Income Year [Sec 18 (1)], or
ii. 50% of Adjusted Taxable Income from all businesses [Sec 18 (2)]

In case incurred Research and Development Cost cannot be claimed due to the maximum limit of
50% of Adjusted Taxable Income from all Businesses during the Income Year, such excess of
incurred R & D Cost over 50% of Adjusted Taxable Income from all Businesses shall be
capitalized in Pool D to arrive at the Opening Depreciation Base for following Income Year.

9. Withholding agent (Dec 2018, 2.5 Marks)


Answer:
Withholding Agent means the person who has the obligation to withhold tax pursuant to Sec. 87,
88, 88Ka and 89 of Income Tax Act, 2058.

10. Interest (June 2018 & June 2014, 2.5 Marks)


Answer:
Interest means the following payment or gain:
a. A payment made or accrued under a debt obligation that is not a repayment of capital

The Institute of Chartered Accountants of Nepal 330


CAP-II Paper 7 - Income Tax and VAT
b. Any gain realized by way of a discount, premium, swap payment, or similar payment under
debt obligation; and
c. The amount treated as Interest under Sec 32 of the Act while receiving/making payments for
using any assets under finance lease or under annuity or installment sales

11. Amended assessment (June 2018, 2.5 Marks)


Answer:
IRD may amend an assessment made under self and jeopardy assessment provisions. The
amendment in assessment is to adjust the assessed person's liability to tax, including any tax
payable on the assessment. The adjustment shall be in a manner consistent with the intention of
the Act and according to IRD’s best judgment.

Number of Amended Assessment [Sec. 100 (2)]


In case IRD thinks it proper to do so, the assessments can be amended again and according to the
IRO’s best judgment for as many times as it thinks appropriate.

Time Limit to make amendment in Assessment [Sec. 100 (3)]


The amended assessment shall be completed within four years of the date mentioned below, unless
there is evidence that assessment is inaccurate by reason of fraud:
• In the case of self assessment: within four years from the due date for filing the return; or
• In the case of jeopardy assessment: within four years from the date on which the notice of
assessment is served to the taxpayer under Sec. 102.
• Reassessment of amended assessment: within four years from the date mentioned above, as
the case may be.
Amended Assessment of inaccurate assessment by reason of Fraud [Sec. 101 (4)]
In case a person’s assessment is inaccurate by reason of fraud, the limitation of four years as above
is not applied. The due date of completing amended assessment of inaccurate assessment by reason
of fraud is one year from the date of receipt of information of fraud.
Limitation of IRD if the competent court has settled the assessment [Sec. 101 (5)]
IRD cannot amend an assessment to the extent that the assessment has been amended or reduced
pursuant to an order of Revenue Tribunal or any other authorized court unless the order is vacated.
Show-cause Notice [Sec. 100 (6)]
A show-cause notice shall be served upon a taxpayer providing 15 days’ time limit to submit
clarification and proof in one’s own favor. The show cause notice shall include the clear basis for
the amendment.

12. Permanent Account Number (June 2018, 2.5 Marks)


Answer:
IRD can issue a unique identification number to identify a taxpayer. The unique identification
number is Permanent Account number (interchangeably called Taxpayer’s Identification number-
TPIN), which is issued by IRD to identify a taxpayer under the power conferred by Sec. 78 (1) of
the Act.

The Institute of Chartered Accountants of Nepal 331


CAP-II Paper 7 - Income Tax and VAT

Besides IRD, the PAN can also be issued by entity authorized by Department to issue such number.
The list of such authorized department is produced in the website of IRD, and extracted (as on the
date of writing this book) as part of End Note of this book. The authorized entity shall issue
permanent account number by abiding the provisions of Income Tax Act. But there is restriction
on such taxpayer obtaining PAN number from other authorized entity than IRD to conduct foreign
trade, i.e. the taxpayer cannot conduct import or export transaction for particular period after
obtaining PAN

13. Valuation of Closing Stock (Dec 2017, 2.5 Marks)


Answer:
Closing stock is valued at lower of cost or Market value.

The cost of closing stock is determined using Specific Identification Method, however, when this
method cannot be used, it is determined using First-In-First-Out Cost Formula or Weighted
Average Cost formula.

With regards to calculation of cost of inventory in production process, a person following cash
basis may use prime costing or absorption costing; however, the person following accrual basis
must use absorption costing.

In absorption and prime costing, both, the depreciation and repair and improvement cost of
depreciable asset cannot be allocated as cost of inventory.

14. Associated Person (Dec 2017, 2.5 Marks)


Answer:
“Associated Person” means one or more persons or group of such persons where the relationship
between the two is such that one may reasonably be expected to act in accordance with the
intentions of the other, and the term includes:
1. an individual and a relative of the individual or
2. any person or his/her partner
3. Foreign Permanent Establishment and the person having ownership over such foreign
permanent establishment, and
4. an entity and a person who
• either alone or together with an associate or associates, and
• whether directly or through one or more interposed entities controls or may benefit from
50 percent or more of the rights to income or contributed capital or voting power of the
entity
Provided that the following persons are not associated persons:
5. Employee,
6. Persons prescribed by Department as “not an associated person”

The Institute of Chartered Accountants of Nepal 332


CAP-II Paper 7 - Income Tax and VAT
15. Taxpayer’s Rights (Dec 2014/2017/2021)
Answer:
The following rights are available to a taxpayer:
1. Right to get respectful behavior;
2. Right to receive any information related to tax as per the prevailing Laws;
3. Right to get an opportunity of submitting a proof in one’s own favor in respect of tax matters;
4. Right to appoint lawyers or auditors for defense; and
5. Right to secrecy in respect of tax matters and to keep it inviolable.

16. Self-assessment (Dec 2017, 2.5 Marks)


Answer:
Actual Self Assessment [Sec. 99 (1)]
Where a person files a return of income for an income year disclosing the following matters, self
assessment is treated as made on the due date for filing the return:
a. the income tax payable by the person for the year under section 3 (Ka) and (Kha), and
b. the amount of that tax still to be paid for the year in the amount shown in the return

Deemed Self Assessment [Sec. 99 (2)]


Where a person fails to file a return of income for a tax year then, until such time as a return is
filed, an assessment is treated as made on the due date for filing the return as follows:
a. the income tax payable by the person for the year is equal to the sum of any income tax withheld
from payments derived by the person during the year under Chapter 17 of the Act and any
income tax paid by the person by installment for the year under Chapter 18 of the Act, and
b. There is no tax payable on the assessment.

17. Presumptive Tax for small resident Vendors (Dec 2016, 2.5 Marks)
Answer:
If all the following conditions are satisfied, the tax paid as explained below shall be the tax liability
of such person, and the person is relieved from detailed calculation of tax liability:
a. The person shall be a Resident Natural Person,
b. The person shall derive Income only from Business having source in Nepal during the Income
Year,
c. The person shall not claim Medical Tax Credit (as per Sec 51) and Advance tax arising out of
Withholding Taxes (as per Sec 93),
d. The Income from the Business (Sales or Turnover) shall not be more than Rs. 20 Lakhs and
the profit from such business shall not exceed Rs. 2 Lakhs.
e. The person shall elect this Section to be applied in writing, i.e. the person shall make a written
application (a specific form is developed by IRD on which the person shall make a tick to opt
this facility) to IRD to pay presumptive tax.

The Institute of Chartered Accountants of Nepal 333


CAP-II Paper 7 - Income Tax and VAT
Amount of Tax to be paid [Schedule 1- Section 1 (7)]
Area of conducting Business Amount of Tax (Rs.)
Metropolitan City and Sub Metropolitan City 7,500
Municipality 4,000
Area other than above (Rural Municipality) 2,500

18. Presumptive Tax for Public Transport Carrier Business run by Natural Person
(Dec 2016, 2.5 Marks)
Answer
With amendment in Sec. 1 (13) of Schedule 1 of Income Tax Act, 2058, there shall not be
presumptive tax for public transport carrier business run by natural person.

19. Assessable Income (Dec 2016, 2.5 Marks)


Answer
The following income of a person during any Income Year under head business, employment,
investment and windfall gain shall be the Assessable Income of such person subject to Income Tax
Act, 2058:
i. Income of a resident person from business, employment, investment & windfall gain regardless
of the Source of such payments, and
ii. Income of a nonresident person from business, employment, investment & windfall gain
having source in Nepal
The Assessable Income shall not include any amounts exempted u/s 11 or u/s 64 or under both
Sections of the Act.

20. Bad debts recovered (June 2016, 2.5 Marks)


Answer
When there is writing off of debts for accounting purpose and if the write off is not deductible for
tax purpose, the accounting base of debtor decreases with no change in the tax base of debtor.
When such written off debts are recovered, the accounting impact is charging such recovery as
income, however, since there is tax base of such debtor, there would be a credit to debtor account
for tax purpose, which means the recovery of such bad debts does not have any tax impact.

If the write offs are deductible for tax purpose, thereby decreasing the tax base of debtor; the
recovery is treated as income during the year of recovery.

Simple words:
If the bad debt was allowed for tax purpose at the time the related debt was written off, recovery
is treated as income for tax purpose and vice-versa.

The Institute of Chartered Accountants of Nepal 334


CAP-II Paper 7 - Income Tax and VAT
21. Entity (Dec 2014, 2.5 Marks)
Answer
Entity means the following organizations:
1. Partnership, Trust or Company
2. Rural Municipality, Urban Municipality or District Coordination Committee
3. Government of Nepal or Province Government or Local Government
4. Public International Organization established under any foreign government or state
government of such foreign government or local bodies of such foreign government or under
any treaty
5. Foreign Permanent Establishment

22. Company (June 2014)


Answer
Company means any company incorporated under prevailing company law and the following
organizations are also treated as company for tax purpose:
1. Any organized institutions established under prevailing law (e.g. ICAN, Tribhuwan
University, Kathmandu University, etc.)
2. Any unorganized association, committee, organization, or society, or group of persons except
private firm and partnership, or trust
3. Partnership firm having 20 or more partners whether registered or not under prevailing law,
Retirement Fund, Cooperatives, Unit Trust, Joint Venture
4. Foreign Company
5. Any foreign organization as prescribed by Director General of IRD

23. Lease (June 2014)


Answer
Lease means temporary right of a person to use any asset other than cash or cash equivalents of
another person and the term includes license, rent agreement, option, royalty agreement or right of
lessee.

24. Investment (June 2014)


Answer
“Investment” means the holding of one or more properties or the investment in a property except
the following:
1. The use of the property for the private use of the owner, or
2. A business or an employment.
However, the holding of a non-business chargeable asset is also known as an investment.

25. Receiver (June 2014)


Answer
As per Clarification Clause to Sec. 108, the Receiver means the following person:
1. A liquidator;

The Institute of Chartered Accountants of Nepal 335


CAP-II Paper 7 - Income Tax and VAT
2. A receiver appointed by a court or out of a court in respect of an asset or an entity;
3. A person who has taken the assets in possession in case the asset is mortgaged to him;
4. An executor, administrator, or direct heir of a deceased individual’s estate; or
5. Any person conducting the affairs of an incapacitated individual.

26. Trustee (Dec 2013)


Answer
Trustee means a natural person, guthi or any organized institutions that holds assets in fiduciary
capacity either singly or jointly in association with another natural person, guthi or other organized
institutions and the term includes:
1. Operator or Administrator of property of a deceased natural person
2. Liquidator, Receiver or Trustee
3. Person controlling, directing, managing or preserving asset of an incapacitated person either
in private or authorized capacity
4. Private Enterprises or any person managing assets under private enterprises
5. Any other person acting in the capacity as described in four points above

27. Royalty (Dec 2013)


Answer
‘Royalty’ means any payments for the lease of intangible asset and the term also includes any
payment made for the purpose of following:
1. Use of or right to use copyright, patent, design, model, plan, confidential formula, or process
or trademark;
2. Rendering of technological knowledge;
3. Use of or right to use cinematographic movie, video tape, sound recording, or similar other
mediums, or use of or rendering of industrial, trading or scientific experience;
4. Any type of assistance in a manner to be ancillary to the matters referred to in clauses (a), (b),
or (c) above, or
5. Complete or partial restriction for the matters referred in (a), (b), or (c) above
But the term ‘royalty’ does not include the payment for natural resources.

28. Permanent Establishment (Dec 2013)


Answer
“Permanent establishment" means a place where a person carries on business whether fully or
partially, and includes-
1. a place where a person is carrying on business partially or fully through an agent, other than a
general agent of independent status acting in the ordinary course of business as such;
2. a place where a person has, is using, or is installing substantial equipment or substantial
machinery;
3. one or more places of a country where a person provides technical, business or consultancy
services through employee or otherwise for more than 90 days in any 12 months period

The Institute of Chartered Accountants of Nepal 336


CAP-II Paper 7 - Income Tax and VAT
4. a place where a person is engaged in a construction, assembly, or installation project for 90
days or more, including a place where a person is conducting supervisory activities in relation
to such a project

29. Investment Insurance (Dec 2013)


Answer
“Investment Insurance” means insurance of any of the following class:
a. insurance where the specified event is the death of an individual who is the insured or an
associated person of the insured;
b. insurance where the specified event is an individual who is the insured or an associated
person of the insured sustaining personal injury or becoming incapacitated in a particular
manner
Provided that: the insurance agreement is expressed to be in effect for at least five years or
without limit of time and is not terminable by the insurer before the expiry of five years
except in special circumstances prescribed in the insurance contract;
c. insurance under which an amount or series of amounts is to become payable to the insured in
the future;
d. Reinsurance of insurance as specified in (a), (b) or (c) above, and
e. Reinsurance of reinsurance as specified in (d) above

30. Tax (June 2013)


Answer
“Tax” means the tax imposed under this Act; the term also includes the following payments:
1. Expenses incurred and claimed by the Department in respect of claim over asset and auction
sales of a tax debtor pursuant to Sec. 104 (8) (Ka),
2. Amount payable withholding agent or withholdee pursuant to under Sec. 90 or the amount
payable under Sec. 94 in installment or the amount payable by an advance tax collection agent
pursuant to Sec. 95Ka or amount payable after tax assessment pursuant to Sec. 99, 100 and
101,
3. Amounts payable to the Department in respect of tax liability of a third party under Section
107 (2), Section 108 (3) or (4), Section 109 (1), Section 110 (1),
4. Amounts as referred to in Chapter 22 which are to be paid in respect of fees and interest, and
5. Fines as referred to in Section 129 which are to be paid on the order of the Department.

31. Debt Claim (June 2013)


Answer
‘Debt Claim’ means the right to receive payment by any person from another person and the term
also includes right to refund of amount advanced to another person, deposit kept in Banks and
Financial Institutions, receivable, debenture, negotiable instruments, bond, rights under annuity,
or right to receive payments under installment sale or finance lease.

The Institute of Chartered Accountants of Nepal 337


CAP-II Paper 7 - Income Tax and VAT
32. Underlying Ownership (June 2013)
Answer
It means the ownership as follows:
a. in relation to an entity, means interests owned in the entity, directly or indirectly through one
or more interposed entities, by individuals or by entities in which no person has an interest; or
b. in relation to an asset owned by an entity, is determined as though the asset is owned by the
persons having underlying ownership of the entity in proportion to that ownership of the entity;

33. Payment (June 2013)


Answer
“Payment” means the following:
a. the transfer by one person of an asset or money to another person or the transfer by another
person of a liability to the one person;
b. the creation by one person of an asset that on creation is owned by another person or the
decrease by one person of a liability owed by another person;
c. the provision by one person of services to another person; and
d. the making available of an asset or money owned by one person for use by another person or
the granting of use of such an asset or money to another person

34. Long Term Contract (June 2021, 2 Marks)


Answer:
"Long-Term Contract” means a contract as referred to in Section 26 with term more than 12
months. For the purpose of Section 26, “long-term contract” means a contract of following
conditions:
a. the term of which exceeds 12 months, and
b. that is either a contract for manufacture, installation, or construction, or, in relation to each,
the performance of related services; or a contract with a deferred return that is not an
excluded contract.

35. General rule against tax avoidance (Dec 2021, 2.5 Marks)
Answer:
As per Section 35 of Income Tax Act 2058, for purposes of ascertaining the tax liability
pursuant to this Act, the Department may carry out the followings:
(a) To re-characterize any arrangement or any part of such arrangement made or attempted to
be made as a part of a tax avoidance scheme,
(b) To disregard any arrangement or any part of such arrangement that does not show any
substantial effect, or
(c) To re-characterize any arrangement or any part of such arrangement that does not show
any substantial element.
Explanation: For purposes of this Section, "tax avoidance scheme" means any arrangement
with a main objective to have avoidance of tax liability or to lessen the tax liability.

The Institute of Chartered Accountants of Nepal 338


CAP-II Paper 7 - Income Tax and VAT
36. Tax assessment notice (Dec 2021, 2.5 Marks)
Answer:
The Department shall give the person, whose tax is assessed, a written notice of tax assessment
made pursuant to sub-section (2) of Section 100 or Section 101 setting out the following matters:
a. The assessed tax to be paid and due and payable by the person mentioned in clauses (a) and
(b) of Section 3 for the income year or period related with assessment of tax,
b. The method of computation of tax in the tax assessment as mentioned in clause (a),
c. The reason why the Department has to assess the tax,
d. The time for payment of the assessed tax due and payable, and
e. The time, place and mode for making a petition if one is not satisfied with the assessment of
tax.
37. Departmental action against tax officer (Dec 2021, 2.5 Marks)
Answer:
As per section 133 of Income Tax Act, 2058 Director General of IRD can take a departmental
action against such Tax Officer who has, negligently, assessed any tax liability and there-by, the
tax liability of an assessee either increased or decreased or the tax officer has not completed the
amended assessment within the time limit prescribed by section 101(3).
However, section 136 of Income Tax Act provides relaxation to the tax officer by saying that an
officer, who takes any action with good intention in course of performing duties, shall not be
personally liable to the action.
(Not applicable to CAP II).
38. Income from Windfall gains (June 2022, 2.5 Marks)
Answer:
As per sec 2(h1), windfall gains means lottery, gift, prize, service tips, winning tips and any other
gift received in the form of windfall gain.
As per section 88A of the Income Tax Act, in a payment of windfall gain , income tax shall be
levied at the rate of 25%.
Provided that GoN may provide exemptions in the windfall gain by publishing a notification in
official gazette on national , international awards received for contribution in the sector of
literature, art, culture, sports, journalism, science, technology and public administration.
Notwithstanding anything contained above, no windfall gain tax shall be levied on the national or
international awards received for up to Rs 5 lacs for contributing in the sector of literature, art,
culture, sports, journalism, science, technology and public administration.
39. Imposition of Tax (June 2022, 2.5 Marks)
As per section 3 of Income Tax Act, 2058
Tax shall be imposed and collected according to the Income Tax Act from each of the following
persons on every income year:
a. A person having taxable income in an income year
b. A foreign permanent establishment of a Non resident person repatriating income
c. A person receiving final withholding income

The Institute of Chartered Accountants of Nepal 339


CAP-II Paper 7 - Income Tax and VAT

Part 2: VAT Act

The Institute of Chartered Accountants of Nepal 340


CAP-II Paper 7 - Income Tax and VAT

Chapter 1: Basic Concept of VAT


1. Hub Trade Pvt. Ltd. is engaged in import and sales of various types of goods from India, China,
Hongkong and America. During Income Year 2075/76, it imported following goods from various
countries:

India INR 100,000 (1 INR = 1.6 NPR)


China RMB 15,000 (1 RMB = 15 NPR)
Hongkong HKD 15,000 (1 HKD = 15 NPR)
America USD 25,000 (1 USD = 111.06 NPR)
• All goods are imported through Calcutta Port through Karkarvitta Customs.
• Custom charged on goods by Kakarvitta Custom is 15%, and VAT is imposed at normal
rate except for some goods imported from China which are 45% VAT exempted goods.
• Hub Trade Pvt. Ltd. sells the goods to a wholesaler, who in turn sales it to a distributor
and who inturn sales it to the final consumer.
There is following value additions at each level:
• Hub Trade Pvt. Ltd.: Rs. 350,000 administrative expenses and 10% mark up on sales
• Wholesaler: Rs. 200,000 administrative expense and 10% profit on cost
• Distributor: Rs. 220,000 administrative expenses and 20% mark up on sales

Required:
(Dec 2019, 10 Marks)
a. VAT paid by each business at each stage of sales channel.
b. Final Cost to consumer for the goods of VAT attractive goods and VAT exempted goods.

Answer:

Calculation for First Chain of Business (Hub Trade Pvt. Ltd.)


Amount (NRs.)
Foreign Rate of
Imported From Tax
Currency exchange Total Taxable
Exempt
India 100,000 1.6 160,000.00 160,000.00
Hong Kong 15,000 15 225,000.00 225,000.00
America 25,000 111.06 2,776,500.00 2,776,500.00
China 15,000 15 225,000.00 123,750.00 101,250.00
Total 3,386,500.00 3,285,250.00 101,250.00
Add: 15% Custom 507,975.00 492,787.50 15,187.50
Taxable Amount 3,894,475.00 3,778,037.50 116,437.50
VAT 13 % paid on import 491,144.88 491,144.88 Exempt

The Institute of Chartered Accountants of Nepal 341


CAP-II Paper 7 - Income Tax and VAT
Add : 350,000
administrative expenses
(Note 1) 350,000.00 339,535.66 10,464.34
Cost of sales 4,244,475.00 4,117,573.16 126,901.84
Value Addition (10% margin on sales) (The margin is
10% on sales price, the cost is 90% of sales) 471,608.33 457,508.13 14,100.20
Sales Price 4,716,083.33 4,575,081.28 141,002.05
VAT on sales 594,760.57 Exempt
Less: VAT paid on
purchase/import 491,144.88
Net VAT Payable 103,615.69

Note 1: Administrative expenses distributed in the ratio of total cost for taxable and tax-exempt
goods, as per question the total administrative expense is for sales of total of goods. Normally
administrative expenses do not form a part of value addition. This shall be recovered by the profit
margin. In the given case we consider it as per the requirement of question.
Calculation for Second Chain of Business Tax
Total Taxable
(Wholesaler) Exempt
Purchase Cost 4,716,083.33 4,575,081.28 141,002.05
VAT on purchase 0.00 594,760.57 Exempt
Add : administrative expenses (200,000 distributed in
the ratio of purchase cost) 200,000.00 194,020.38 5,979.62
Total Cost 4,916,083.33 4,769,101.66 146,981.67
Add: Value Addition (10% margin on cost) 491,608.33 476,910.17 14,698.17
Sales Price 5,407,691.67 5,246,011.83 161,679.84
Add: 13% VAT on sales 681,981.54 681,981.54 0.00
Less: VAT paid on purchase 0.00 594,760.57 0
Net VAT Payable 681,981.54 87,220.97 0.00

Calculation for Third Chain of Business Tax


Total Taxable
(Distributor) Exempt
Purchase Cost 5,407,691.67 5,246,011.83 161,679.84
VAT on purchase 681,981.54 681,981.54 0.00
Add: administrative expenses (220,000 distributed in
the ratio of purchase cost) 220,000.00 213,422.41 6,577.59
Total Cost 5,627,691.67 5,459,434.24 168,257.43
Add: Value Addition (20% margin on sales) 1,406,922.92 1,364,858.56 42,064.36
Sales Price 7,034,614.58 6,824,292.80 210,321.79
Add: 13% VAT on sales 887,158.06 887,158.06 Exempt
Less: VAT paid on purchase 681,981.54 681,981.54
Net VAT Payable 205,176.53 205,176.53

The Institute of Chartered Accountants of Nepal 342


CAP-II Paper 7 - Income Tax and VAT

Calculation for Final Consumer (Final Chain on Tax


Total Taxable
business) Exempt
Purchase Cost 7,034,614.58 6,824,292.80 210,321.79
VAT on purchase 887,158.06 887,158.06 Exempt
Total Cost 7,921,772.65 7,711,450.86 210,321.79

2. Morang Distributers (Pvt.) Ltd is a sole purchaser of VAT attractive goods from the manufacturer. Per
unit cost price of the goods is Rs. 40,000.00. The company distributes the goods to the wholesaler,
wholesaler sells the goods to the retailer. The company and the wholesaler, each incurs transportation
expenses Rs. 500.00 for the goods and they charge 10 % profit margin to determine the selling price.
The wholesaler provides 5% discount on selling price to the retailer. The retailer sells the goods on the
same selling price fixed by the wholesaler. The goods are taxable as per VAT Act, 2052.

Required: (June 2018, 5 Marks)


a. Actual cost with VAT to the final consumer.
b. Total VAT payable to the government.
Ignore the VAT payable by the manufacturer

Answer
Profit margins are expressed in terms of percentage of sales unless otherwise specified in question.
Assumption: The retailer works on a commission basis. 5% discount on selling price is commission to
retailer by Wholesaler.
Level of Cost Other Profit Taxable Value VAT Input VAT VAT
Business Price costs Margin Amount credit payable
Manufacturer - - - 40,000 5,200 Not known Not
known
Morang 40,000 500 45,000 * 40,500/ 0.9 = 45,000 5,850 5,200 650
Distributors 0.1=
4,500
Wholesaler 45,000 500 2,527.25 45,500/ 0.9 *.95= 6,243.54 5,850 393.54
50,555*95%=48,027.25
Retailer 48,027.25 - 2,527.75 50,555 6,572.15 6,243.54 328.61
Final Cost to customer= 50,555*1.13= Rs. 57,127.15

3. Nepal Surgical House (Pvt.) Limited is a principal stockiest for Nepal of Medtronic Inc, Hong Kong (MIH).
The company has policy to sell the items to other distributors with 15% markup at landed cost. Medical
and surgical equipment manufactured by MIH are both VAT attractive and VAT exempt. MIH makes
the shipment by billing on FOB CIF Price and import is done through irrevocable letter of credit. The
Company provides following information for the month of Chaitra 20X-74:

a) All items are subject to 5% custom duty under HS code 84.

The Institute of Chartered Accountants of Nepal 343


CAP-II Paper 7 - Income Tax and VAT
b) Shipment against the LC established in Falgun 20X-74 having FOB value USD
126,300.00 (CIF Birgunj) of which USD 43,500.00 is subject to VAT was released during
the month and entire stock was sold on the same month.
c) Of the total import of taxable goods during the month, items having FOB Price USD
26,400.00 was revalued to USD 33,100.00 by the custom authorities.
d) Terms of Payment 20% during the establishment of LC and 80% on release of
documents.
e) Effective exchange rate per USD was 102.6, 103.8 & 103.45 during the date of LC
establishment, document release and custom clearance respectively.
f) Previous non-VAT stock having landed cost of Rs. 2,526,200.00 was also sold during the
month.
g) The company paid Rs. 146,800.00 inclusive of tax for Birgunj Kathmandu
transportation and insurance of the consignment.

On the basis of above information, prepare VAT statement of Nepal Surgical House (Pvt.) Limited for
the month of Chaitra 20X-74 considering opening VAT credit of Rs. 73,612. (June 2018, 10 Marks)

Answer
Taxable Value as per Sec. 12 in case of Import and determination of VAT paid on Import:
Taxable value shall be landed cost up to border, plus any other indirect taxes levied at the Customs
Frontier. When Customs officer revises the valuation of goods, VAT shall be computed based on revised
cost. Customs valuation is based on foreign currency and the conversion rate shall be the rate prevailing
on the date of customs clearance.

Particulars CU Total Taxable VAT exempt Notes


CIF value of imported goods USD 126,300 43,500 82,800 Given
Add: Revaluation made by
USD 6,700 -
Customs
CIF value of
Revised Price of the Goods USD 50,200 82,800
goods for tax
Rate of customs
Exchange Rate 103.45 103.45
clearance date.
Transaction value for
NPR 5,193,190 8,565,660
Customs purpose
Customs duty @ 5% 259,659.50 428,283.00
Taxable Value 5,452,849.50 8,993,943.00
VAT Paid on Import @13% 708,870.44 Exempt

Determination of Selling Price of Goods & Output VAT

Particulars CU Total Taxable VAT exempt Notes


CIF value of goods imported
USD 126,300 43,500 82,800 Given
in Chaitra
Payment for goods

The Institute of Chartered Accountants of Nepal 344


CAP-II Paper 7 - Income Tax and VAT
During LC Establishment
USD 8,700 16,560
(20%)
At the time of release of
USD 34,800 66,240
documents (80%)
Conversion into NPR
For payments during LC
NPR 892,620 1,699,056
Establishment
For payments at the time of
NPR 3,612,240 6,875,712
release of documents
Total NPR Cost 13,079,628 4,504,860 8,574,768
Calculated as
Add: Customs duty 259,659.50 428,283.00
above
Apportioned on
the basis of total
Add: Transportation and NPR Cost. Portion
NPR 129,911.50 *44,743.87 **96,239.43
insurance in Nepal of VAT is added
at cost for exempt
goods
Total Landed Cost 4,809,263.37 9,099,290.43
Markup @ 15% on Landed
721,389.50 1,364,893.56
Cost
Selling Price 5,530,652.87 10,464,183.99
Output VAT on Goods
718,984.87 Exempt
imported during Chaitra

*((146800/1.13)/13079628*8574768)
** (146800/13079628*8574768)

VAT Statement for the month of Chaitra

Output VAT 718,985


Less: Input VAT Credit
VAT Paid on Import -708,870 On Import of VAT attractive items
VAT Paid on insurance and transportation -5,817 Proportionate credit
Opening VAT Credit -73,612
VAT Payable/(Receivable) -69,314

4. A lawyer practicing in Nepal has provided consultancy services to the Legal Firm situated in United
Kingdom. The legal firm situated in United Kingdom has used such services for the release of persons
in USA. Where is place of supply of services as per Value Added Tax Act/Rules? (Dec 2012, 5 Marks)

Answer
As per Rule 16 of Value Added Tax Regulations, 2058; the place of supply of service is the place
where the benefit of service is received.

The Institute of Chartered Accountants of Nepal 345


CAP-II Paper 7 - Income Tax and VAT
In the given circumstances, the benefit of service rendered by a lawyer in Nepal is received in
United Kingdom, no matter where the service is used by the legal firm in United Kingdom.
Hence, the place of service is United Kingdom.

5. Moonlight Traders is engaged in the business of selling Television sets on installment basis. As per the
policy and contract agreement, forty percent of the Price of the Television Rs. 20,000, i.e. Rs. 8,000 has
to be paid as upfront payment in cash and thereafter monthly installment of Rs.1,000 along with
interest 1% per month has to be paid at the end of the month. Mr. Ram Binod has purchased a
television set by paying Rs. 8000 in cash on 20X-66.09.07. Installment of Magh and Falgun has been
paid on Falgun 05, 20X-66. By mentioning the relevant provision, determine the time of supply for
Magh and Falgun Month installments. (Dec 2012, 4 Marks)

Answer
As per Sec. 6 (3) of Value Added Tax Act, 2052; the time of supply, in case where the goods or
services are supplied so as to obtain the consideration in tranches under an installment agreement,
is earlier of payment of installment or due date for payment of installment.
The time of supply for the given case is as follows:
Particulars Due date Payment date Time of Supply
Installment of Magh Magh end Falgun 5, 20X-66 Magh end
Installment of Falgun Falgun end Falgun 5, 20X-66 Falgun 5, 20X-66

6. Mr. Deshmukh, an importer imported certain goods at NRs. 50,000. No VAT was paid on its import.
The goods passed to the final consumer through a retailer. Both middlemen incurred NRs. 1,000 each
for administration expenses. Both middlemen charged 15% profit margin on selling price.
Required: (June 2009, 5+3)
a) Cost price to the final consumer
b) VAT payable to the government at each stage
Answer
Level of Cost Other Profit Taxable VAT Input VAT
Business Price costs Margin Value Amount VAT payable
credit
Importer 50,000 1,000 9,000 (60,000 51,000/ 7,800 0 7,800
* 15%) 0.85 =
60,000
Retailer 60,000 1,000 71,764.71 * 61,000/ 9,329.41 7,800 1,529.41
0.15= 0.85 =
10,764.71 71,764.71
Cost to the retailer is Rs. 81,094.12 (71,764.71+9,329.41)

7. Tax Accountant of Subhashree & Co. contends that the terms “Zero VAT” and “No VAT” has the same
meaning. As a Tax Expert, express your opinion within the framework of VAT Act, 2052. (Dec 2011, 5
Marks)

The Institute of Chartered Accountants of Nepal 346


CAP-II Paper 7 - Income Tax and VAT
Answer
The terms “Zero VAT” and “No VAT” does not have the same meaning. It can be distinguished
as follows:
Basis Zero Rate No VAT
Rate of VAT is levied at Zero Percent. VAT is not levied at all.
VAT
VAT A VAT Invoice shall be issued as per VAT Invoice is not required to
Invoice the format prescribed by VAT be issued for transaction of such
regulation. items.
Registration The person dealing in Zero rated items The person dealing exclusively
Requirement shall be registered for VAT purpose. in No VAT items is relieved
from registration formality.
Other The person dealing in Zero Rated Items The person dealing exclusively
formalities shall observe other formality as per in No VAT items is relieved
as per the VAT Act, such as- maintenance of sales from all formalities to be
Act and purchase register duly certified observed as per the Act.
from Tax Officer, submission of VAT
return, etc.
Input Tax For goods on which VAT is levied on For goods on which VAT is not
Credit Zero Percent, input tax credit can be levied at all, input tax credit can
claimed. not be claimed.

8. What is the value-add taxable transactions? At what point of time and place of the transaction, the
value added tax is liable to be determined and collected and paid to the Government. (June 2009&
2004, 2+4+4=10, CA Inter)

Answer:
Value Added Tax is applicable on following transactions:
a. Supply of goods or services within the state of Nepal,
b. Import of goods or services into the state of Nepal,
c. Export of goods or services from the state of Nepal.

However, where the goods or services are listed in Schedule 1 of the Value Added Tax Act, there shall
not be applicability of Value Added Tax. Further, value added tax is exempt on transfer of ownership of
business to a registered person or a legal heir in the event of death of owner.

The Institute of Chartered Accountants of Nepal 347


CAP-II Paper 7 - Income Tax and VAT
Point of time when VAT is levied:
Supply of goods or services shall be considered to have taken place at the earliest time of the following
times:
a. When an invoice is issued by a supplier;
b. In the case of supply of goods, when the acquirer removes or takes possession of the goods from the
supplier's transaction place;
c. In the case of supply of services, when the service is rendered;
d. When supplier receives the consideration for goods and services

However, in the following cases, the time of supply shall be as follows:


a. In the case of services which are continuously provided, namely, telecommunication services or
similar other public services, when the invoice is issued;
b. Where there is a contractual provision for paying partially the value of goods or services in more than
one day on contractual basis, the supply time shall be the earliest day on which the payment is made
or the day on which the payment is to be made according to the contract;
c. In the case of goods or service which are so used as not to be allowed as offset under this Act, the
time when such goods or services are used

Place of Supply
Place of supply of Goods:
The following places shall be deemed to be the place of supply of goods:
a) In the case of movable goods transferred by sale, the place where such goods were sold or
transferred,
b) In the case of any immovable goods where location can’t be transferred even if change in ownership,
the place where such goods are located,
c) In the case of imported goods, the Customs point in Nepal through which goods are imported into
Nepal,
d) In case any producer or vendor supplies the goods to the self, the place where the producer or the
vendor of such goods resides.

Place of Supply of Services


The place of supply of a service shall be the place where the benefit of that service is received.

9. State whether the following items are liable to VAT or not: (Dec 2008, 5 Marks, CA Inter)
a) Drinking water supplied by the Nepal Drinking Water Company
Exempt

b) Green Tea leaves sold by the farmer


Exempt

The Institute of Chartered Accountants of Nepal 348


CAP-II Paper 7 - Income Tax and VAT
c) Unprocessed tobacco sold by the farmer
Exempt

d) Tea chests imported for packing by a trader for resale


Exempt

e) Medical services provided by private hospitals and clinics


Exempt

f) Carpets made from raw wool imported from New Zealand & Tibet
VAT attractive

g) Cinema Hall tickets


VAT attractive

h) Elevator /Lift imported by an individual from India for putting in High rise building of residential
flats
Exempt

i) 11 KV Electric Power supplied by Nepal Electricity Authority to an industry


Exempt

j) 50 Kg Jute bags for packing sugar by a sugar mill


VAT attractive

10. State whether VAT is applicable to the following cases: (June 2008, 10 Marks, CA Inter)
a) Import of water pumps by a dealer in water pumps.
Exempt

b) Bulldozer imported by a construction company


Exempt

c) Mineral water
VAT attractive

d) Cut fruits supplied to customer at a restaurant and registered under VAT


VAT Attractive

e) Tractor imported by a trader in Tractors


VAT Attractive

The Institute of Chartered Accountants of Nepal 349


CAP-II Paper 7 - Income Tax and VAT
f) Telephone services charged by the telecom company
VAT attractive

g) Electricity charges billed by NEA


Exempt

h) 2 Kg gold brought as personal baggage while coming from a foreign country after staying there for
more than 1 year
Exempt

i) Rent received on letting out residential quarter by an individual


Exempt

j) Rent charged by the hotel for a room occupied by a company for providing accommodation to the
manager of a trading company
VAT attractive

11. State whether the following are liable to Value Added Tax or not and the rate if any: (June
2007, 3 Marks, CA Inter)
a) A local dairy sells Curd worth Rs. 4,00,000 during the month
Answer
Exempt

b) Global Enterprises Organizes a Cultural Program at the National Academy Hall and collects Rs. 2
Lacs as ticket charges.
Answer
The ticket fee is VAT attractive and the rate is 13%.

c) LBL, a Legal Consultancy Firm, registered to VAT provides legal services to Mr. X residing outside
the Country worth Rs. 50,000. Mr. X does not have any business, business agent or legal
representative in Nepal.
Answer:
It is the case of export of service; therefore, the VAT is applicable @ 0%.

The Institute of Chartered Accountants of Nepal 350


CAP-II Paper 7 - Income Tax and VAT

Chapter 2: Registration and De-registration


1) Tasty Food Pvt. Ltd., a company registered in VAT decided to close down its business on Magh, 2076
due to slowdown in sales. It provides the following details:
a. VAT Credit in tax return of Poush is Rs. 5 lacs.
b. Value of asset and liability as on Magh, 2076 is:

Book Value Market Value


i. Land and building 100 lakhs 150 lakhs
ii. Plant and Machinery 40 lakhs 50 lakhs
iii. Vehicle 1 60 lakhs 30 lakhs
iv. Vehicle 2 50 lakhs 55 lakhs
v. Sundry Debtors 60 lakhs only 35 lakhs is recoverable
vi. Finished goods 40 lakhs 35 lakhs
vii. Damaged goods 30 lakhs 10 lakhs
viii. Raw Materials 10 lakhs 8 lakhs
ix. Sundry Creditors 40 lakhs all payable
Suggest:
i) Provisions for cancellation of VAT registration of registered taxpayer.
ii) VAT payable or refundable by Tasty Foods. (Dec 2020, 5 Marks)

Answer:
Conditions for Cancellation of Registration:
As per Sec. 11 (1) of VAT Act, 2052, in the event of occurrence of any of the following
conditions, VAT registration may be cancelled:
Tax officer shall cancel registration of a registered person in the following cases:
a. In the case of an incorporated body, if the incorporated body is closed down, sold or
transferred or if the incorporated body otherwise ceases to exist"
b. In case of ownership of Individual: In the event of death of such individual
c. In case of a partnership firm: Dissolution of partnership or the death of a partner
d. In case a registered person ceases to be engaged in taxable transaction
e. In case of a taxpayer who submits Zero Return continuously for a period of one year or does
not submit VAT Return at all
f. In case where the total taxable turnover of a person does not exceed Rs. 5 Million (2 million
in case of service business or mixed business of goods and services) in preceding consecutive
12 months period
g. In case of error in registration

Procedure for Cancellation of Registration


Application to be filed within 30 days of occurrence of events mentioned in conditions above.
VAT registered entity should file application in a prescribed form (Schedule 11) stating reason
for cancellation of VAT registration within 30 days from the date of occurrence of events
The Institute of Chartered Accountants of Nepal 351
CAP-II Paper 7 - Income Tax and VAT
mentioned in section 11(1) of Value Added Tax Act, 2058. If the tax officer is satisfied that events
stated in section 11(1) has occurred and the registration of the taxpayer should be cancelled, then
the tax officer shall demand deposition of additional tax, if any, and cancel the registration of the
tax payer and inform the tax payer regarding such cancellation.

Submission of documents for VAT audit by tax officer


VAT registered taxpayer should, within 15 days of submission of return, submit documents for
examination and the tax officer shall conduct tax examination of such taxpayer and shall inform,
within three months, whether the registration shall be cancelled or not. If the tax officer does not
cancel the registration or does not decide to cancel the registration, then the tax payer shall not be
required to submit tax returns after such period
Relief on submitting VAT return if the tax officer is not decisive within 90 days of application by
taxpayer

However, such person cannot be relieved of any liability which may be incurred due to something
which has been done which should not have been done or something which has not been done
which should have been done during the period when such person was registered under VAT
simply due to the reason that VAT registration of such person has been cancelled.

In case a person files an application for cancellation of registration, the person shall deem that
closing stock of goods (including capital goods) in which VAT credit had been taken has been
disposed off at the market value. In line with this provision, VAT to be collected and paid shall
be as follows, after considering other transactions of the person:
Particulars Market value (Lakhs) 13% VAT (Lakhs)
Land and building 150 Exempted
Plant and Machinery 50 6.5
Vehicle 1 30 3.9
Vehicle 2 55 7.15
VAT is not applicable on Sundry
Sundry Debtors
- Debtors
Finished goods 35 4.55
Damaged goods 10 1.3
Raw Materials 8 1.04
VAT is not applicable on Sundry
Sundry Creditors
- Creditors
Total VAT Payable 24.44
Less: opening VAT credit -5
Net VAT Payable 19.44

2) Manaslu Enterprises, Kathmandu is trading in clothes, and has the following transactions in the last
twelve months from the end of Bhadra, 20X-75. The Enterprises is not registered in VAT.
a. Export sales Rs. 20 lakh (Kartik 20X-74 to Magh 20X-74)

The Institute of Chartered Accountants of Nepal 352


CAP-II Paper 7 - Income Tax and VAT
b. Sales in foreign currency from the Shop: equivalent Rs. 10 lakh (Bhadra 20X-75)
c. Sales in local currency Rs. 25 lakh (Ashwin 20X-74 to Bhadra 20X-75)
d. Purchase Rs. 40 lakh
e. Loan from bank Rs. 15 lakh
f. Loan payment Rs. 15 lakh
g. Salary and other expenses Rs. 10 lakh
h. Rent expenses Rs. 2 lakh
i. Telephone expenses Rs. 1 lakh

Mention the VAT implication and registration in the above transaction. (Dec 2018, 5 Marks)

Answer:
Conditions for Registration of VAT
As per Rule. 6 and 7 of VAT Rule, 2053 (as amended until the date), the conditions for registration
for VAT are as follows:
(a) In case the taxable turnover of a person exceeds as follows during last 12 months period:
i. Person dealing in goods- Rs. 50 Lakhs
ii. Person dealing in services- Rs. 20 Lakhs
iii. Person dealing mixed transactions of goods and services- Rs. 20 Lakhs
(b) Import of goods having value more than Rs 10,000 for self-consumption except by the person dealing
in exempted items Shall be registered in VAT.

Turnover means the higher of sales or purchase.

Analysis of Question
The person is dealing in clothes, therefore the enterprises’ total taxable turnover for the period of
12 months from Ashoj 20X-74 until Bhadra end 20X-75 is:
Particulars Amount (In Rs.)
Export sales 2,000,000
Sales in Foreign Currency 1,000,000
Sales in local currency 2,500,000
Total sales in last 12 months [A] 5,500,000
Purchase in last twelve months [B] 4,000,000
Turnover [A or B, whichever is higher] 5,500,000
The question does not specify when the threshold of Rs. 50 lakhs exceeded. Therefore, the exact
due date for registration cannot be mentioned. Registration shall be within 30 days from the date
when the threshold of Rs. 50 Lakhs is exceeded.

Hence, if we assume, the threshold exceeded on Bhadra end, 20X-75; an application for VAT
registration shall be filed within 30 days of Bhadra end, 20X-75.

VAT shall be collected by the Enterprises once it is registered for VAT purpose.

The Institute of Chartered Accountants of Nepal 353


CAP-II Paper 7 - Income Tax and VAT

In case, the turnover threshold of Rs 50 Lakhs exceeded before Bhadra 20X-75, the tax officer
deems that any sales by person after 30 days when the turnover threshold exceeded is inclusive
of VAT.

3) Horizon Enterprise, Kathmandu has wholesale transactions of clothes and rice from two business
houses with same PAN. It has the following transactions for 20X-73 Bhadra to Kartik:

Months Clothes Sales Rs. Rice Sales Rs.


Bhadra 20X-73 1,00,000 3,00,000
Aswin 20X-73 5,00,000 5,00,000
Kartik 20X-73 5,00,000 6,00,000

The enterprise recorded its clothes sales Rs. 31 lakh and rice sales Rs. 20 lakh from Bhadra
20X-73 to Shrawan 20X-74. Then, it has following sales in 20X-74 Bhadra to Kartik:
Months Clothes Sales Rs. Rice Sales Rs.
Bhadra 20X-74 10,00,000 3,00,000
Aswin 20X-74 16,00,000 5,00,000
Kartik 20X-74 10,00,000 6,00,000
Tax officer has issued a notice to register the business in VAT on 20X-74/05/10 to the
enterprise. The tax officer has calculated interest, additional fees, and penalties as per VAT
Act. Justify whether the enterprise is required to register in VAT or not. If it is required to
register, then when it has to do so and from when it has to collect tax? (Dec 2017, 5 Marks)

Answer:
Conditions for VAT Registration and whether the conditions are fulfilled as per given question:
Condition
Condition Type of Business satisfied or
not
Compulsory registration where tax officer, during the time of time of N/A, as it is
due to holding of stock inspection, finds stock more than the limit decided by
prescribed by Inland Revenue Department Tax officer
Compulsory Registration For any other the registration is required when To be tested
due to turnover threshold taxable turnover of a person exceeds as follows below
during last 12 months period:
“Turnover” means Person dealing in goods- Rs. 50 Lakhs
a.
higher of Sales orb. Person dealing in services- Rs. 20 Lakhs
c. Person dealing mixed transactions of goods and services-
purchase amount
Rs. 20 Lakhs

The Institute of Chartered Accountants of Nepal 354


CAP-II Paper 7 - Income Tax and VAT
Testing of whether turnover exceeded threshold in any 12 months period between 20X-73 Bhadra
to 20X-74 Kartik:
Rice is VAT exempt
Twelve months to Total Total taxable Turnover
taxable Purchase
Sales
Shrawan 20X-74 (Clothes) 3,100,000 3,100,000
Bhadra 20X-74 3,100,000+ 3,900,000
{Sales from Bhadra 20X-73 to Shrawan 1,000,000 -
20X-74 minus Sales of Bhadra 20X-73 plus 100,000 =
Not given,
Sales of Bhadra 20X-74) 3,900,000
assumed to be
Ashoj 20X-74 3,100,000+ 5,100,000
less than Rs.
{Sales from Bhadra 20X-73 to Shrawan 1,000,000 -
5,000,000
20X-74 minus Sales of Bhadra 20X-73 100,000 +
minus sales of Ashoj 20X-73 plus sales of 1,600,000-
Bhadra 20X-74 plus Sales of Ashoj 20X-74) 500,000 =
5,100,000
Since, taxable turnover for 12 months ending Shrawan 20X-74 is less than Rs. 50 lakhs, the tax
officer’s order for registration on 20X-74 Bhadra 10 is not lawful.

The taxable turnover for the 12-month period from Kartik 20X-73 until Ashoj 20X-74 exceeded
Rs. 50 Lakhs. As such, the person has to register for VAT within 30 days of the end of Ashoj
20X-74.

The person can collect VAT on its sales only after its registration for VAT [as per Sec. 15 (1), an
unregistered person cannot collect VAT unless it falls in the category of Sec. 15 (3)and it does
not fall in category of Sec. 15 (3)].

4) Namaste Steel Private Limited, a VAT registered Company, has witnessed stiff downfall in demand of
its products, and hence its operation has been shut down since the past 3 months. The company is
considering for closure of the business.

Below is the detail of its assets and liabilities, and VAT receivable payable:
a. As per VAT return filed for the previous VAT period, it has VAT credit of Rs. 1 million.
b. Below is the detail of its stock and capital goods and other assets and liabilities as on the
immediately previous VAT period:
i. Finished Goods Inventory (VAT applicable) – Rs. 5 million (Market Value Rs. 4
million)
ii. Raw Material (VAT applicable) – Rs. 1 million (Market Value Rs. 1.5 million)
iii. Damaged Goods (VAT applicable) – Rs. 5,00,000 (Market Value Rs. 1,00,000)
iv. Plant and Machinery (VAT applicable) – Rs. 9 million (Market Value Rs. 4 million)
v. Vehicle – 4 Wheeler (VAT applicable) – Rs. 3 million (Market Value Rs. 4 million)

The Institute of Chartered Accountants of Nepal 355


CAP-II Paper 7 - Income Tax and VAT
vi. Vehicle – 2 Wheeler (VAT applicable) – Rs. 1,00,000 (Market Value Rs. 1,00,000)
vii. Land – Rs. 20 million (Market Value Rs. 50 million)
viii. Sundry Debtors – Rs. 6 million (Rs. 3 million is bad debts)
ix. Sundry Creditors – Rs. 4 million (all amount payable)

Suggest the management of the company regarding: (Dec 2017, 10 Marks)


a. The provisions in the Value Added Tax Act for cancellation of registration of VAT, and
b. Amount of VAT payable or refundable to the company.

Answer
Part (a):
Conditions for Cancellation of Registration u/s 11:
a) In the case of a body corporate: where such body corporate is closed, sold or transferred or ceases
to exist due to any reason,
b) In the case of individual ownership, if the owner dies,
c) In the case of a partnership firm, if it is dissolved or where a partner die,
d) In case the registered person ceases to be engaged in taxable transaction,
e) In case a registered person submits Zero Return continuously for twelve months or is a taxpayer
that does not submit Tax Return,
f) Where the taxpayer’s taxable transaction does not exceed Rs. 50 Lakhs in case of transaction of
goods and Rs. 20 lakhs in case of transaction of service or mixed transaction of goods or services in
last twelve months period,
g) Where there is registration in error.

Procedure for Cancellation of Registration


In case any of the conditions as discussed above arise and a person is required to cancel VAT registration,
the registered person or his/her legal representative/heir in his/her absence shall submit an application
in the format as prescribed by VAT Regulation within 30 days of happening of such conditions along with
the VAT return and the amount of VAT payable in front of tax officer.
The registered person shall submit all the relevant documents for tax assessment within 15 days of
submission of VAT return.

The tax officer shall assess the of such taxpayer and take decision on the matter of cancellation within 3
months of the submission of application. The matters of decision whether to cancel or not shall also be
notified to registered person within 3 months of the application of cancellation.
The registered person is required to submit tax return for another three months until the decision of tax
officer regarding cancellation is served to him.

In case the tax officer fails to decide on matter of cancellation or does not serve notification with regard
to the cancellation of registration within 3 months of application, the registered person is not required
submit any tax return pertaining to periods after the elapse of such three months.

The Institute of Chartered Accountants of Nepal 356


CAP-II Paper 7 - Income Tax and VAT
Effect of Cancellation of Registration
The taxpayer shall assume the following items to be supplied at Market Value at the time of
cancellation of registration and shall assess and collect/levy tax:
a. Stock lying at the time of cancellation on which Input Tax Credit is already enjoyed; and
b. Capital Items still in use, the VAT paid on purchase of which has already been claimed as Input Tax
Credit

Part (b)- Answer


When there is cancellation of registration, the capital items and stocks on which VAT credit was
already claimed are treated to be supplied at market value. The person shall collect VAT on such
market value and pay such collected VAT.
Whether
VAT
Capital VAT
credit Market
Particulars goods or payable @ Remarks
claimed at Value
inventory- 13%
the time of
purchase
Finished Goods Inventory Yes 4,000,000 520,000
inventory
Raw material Inventory Yes 1,500,000 195,000
Damaged Inventory Yes 100,000 13,000
Goods
Plant and Capital Yes 4,000,000 520,000
Machinery Goods
Vehicle- 4 Capital goods Yes 4,000,000 520,000
wheeler
Vehicle- 2 Capital goods Yes 100,000 13,000
wheeler
Land Capital No - - Land is VAT
Goods exempt
Sundry Debtor Not a capital No - - There is no
Sundry Creditor goods, nor an - - applicability of
inventory VAT on debtors and
creditors, since
these are neither
service, nor goods
Total VAT to be collected on cancellation of Registration 1,781,000
Less: VAT Credit (from previous Tax period) (1,000,000)
VAT payable 781,000

5) ABC Trading Company, which is dealing in VAT-attractive goods, has been submitting return of income
mentioning its transactions by understating annual turnover of less than Rs. 50 lakh to avoid from

The Institute of Chartered Accountants of Nepal 357


CAP-II Paper 7 - Income Tax and VAT
registering with VAT and file return on monthly basis. During the course of investigation in the month
of Kartik 20X-71, the tax officer found that the above entity’s transactions stood at more than Rs.50
lakh during the 6 month for the period from Bhadra 20X-70 to Magh 20X-70. The threshold of Rs. 50
lakh has already crossed and its actual turnover has been Rs. 150 lakh during such 6 months.
By stating the relevant provision of VAT Act, give your opinion on registration in this case
and compute only VAT payable amount ignoring the amount of interest, additional fees and
penalty. (June 2016, 5 Marks)

Answer
As per Section 5B of Value Added Tax Act, 2052, in case tax officer identifies that a person
requiring registration as per the Act is conducting business without registration and reasonably
believes so, s/he may demand VAT registration of such person.

In case the person defends the exemption of registration requirement as a result of turnover of the
person for any period of consecutive 12 months not exceeding the threshold (i.e. Rs. 50 lakhs),
the obligation to prove the same is upon the person on whom the registration is demanded.

In the given circumstances, the tax officer has the authority to demand VAT registration. The
obligation to prove that the turnover is within threshold limit is upon the taxpayer.

As the assessee as aforesaid had already exceeded its threshold of Rs.50 lakh in the month of
Magh 20X-70, the assessee has the obligation to get registered with VAT office within the
following 30 days i.e. Falgun 20X-70 itself, and thereafter to file VAT return on monthly basis.

In the case of above problem, despite crossing threshold of Rs.50 lakh, the assesse firm has
ignored to get registered, the firm is liable to pay VAT on Rs.150 lakh considering this figure as
VAT inclusive for the period from Falgun 20X-70 to Kartik 20X-71. Also, such person is liable
extra fees, interest and penalty as per sections 19, 26 and 29 respectively.

Computation of VAT payable=Rs.150, 00,000 X 13/ 113=Rs. 17, 25,664

6) Nepal Philanthropic Society is a “Company not Distributing Profit” registered under the Company Act,
2063. The source of fund of this company comes from membership fees and donation from the
member groups and other outsiders. The company deals with various goods other than those
mentioned in schedule 1 to Value Added Tax Act, 2052. Annual turnover of this company ranges from
Rs. 50 lakh to 75 lakh every year.

Though profits earned by the company are not for the distribution amongst its members,
more than 50% of its profits are utilized towards activities related to the Corporate Social
Responsibilities (CSR) every year.

The members of the company when approached by IRD staffs for VAT registration
contended that the company is not required to be registered in VAT. As you are the tax

The Institute of Chartered Accountants of Nepal 358


CAP-II Paper 7 - Income Tax and VAT
consultant of the entity you are required to comment this statement by the company. (July
2015, 5 Marks)

Answer
Registration for Value Added Tax is compulsory when a person conducting taxable transaction
satisfies any of the following conditions, regardless of whether the person involves in charitable
work or profit-making work:
Condition Type of Business
Compulsory registration where tax officer, during the time of time of inspection, finds stock
due to holding of stock more than the limit prescribed by Inland Revenue Department
Compulsory Registration For any other person than described above, the registration is
due to turnover threshold required when taxable turnover of a person exceeds as follows
during last 12 months period:
“Turnover” means higher Person dealing in goods- Rs. 50 Lakhs
a.
of Sales or purchaseb. Person dealing in services- Rs. 20 Lakhs
c. Person dealing mixed transactions of goods and services- Rs. 20 Lakhs
amount
Since, as given, the taxable turnover of goods of the company exceeds Rs. 50 lakhs in a period of
consecutive 12 months, the company is required to register itself for VAT purposes.

7) Nepali Rice Mill Industries purchases the rice in the husk (Dhan) to produce the rice. The Chitwan based
mill sales its packed rice to the local markets. The company has the following transactions from Bhadra
20X-71 to Chaitra 20X-71.
Sales Rs. 3,000,000
Dhan purchase Rs. 2,000,000
Machinery purchase Rs. 2,500,000 (Net of VAT)
Factory construction Rs. 5,500,000 (Payment to the Contractor)
State the relevant provisions for VAT implications on this case with reference to the VAT
Act, 2052 and VAT Rules, 2053. (July 2015, 5 Marks)

Answer
As per Sec. 10 (3), a person dealing only in such transactions that are not subject to VAT
(i.e.,transactions of items listed in Schedule 1 of the Act, is not required to be registered for VAT.
Therefore, M/s Nepali Rice Mill Industries is not required to be registered for VAT.

However, there are two VAT obligations in case of an unregistered person, unless Sec. 12Ka and
Sec. 15 (3) is applicable (In this case, Sec. 12K and Sec. 15 (3) is not applicable):
a. Collection of VAT from itself when it purchases service from a foreign person that is not registered
for VAT purpose in Nepal. The collected VAT shall be deposited within 25 days of the month when
the amount has to be collected.

b. Collection of VAT from itself, if it constructs building, apartment, shopping complex or similar other
structures for commercial purpose costing more than 50 Lakhs, and the contractor who constructs

The Institute of Chartered Accountants of Nepal 359


CAP-II Paper 7 - Income Tax and VAT
the commercial structure is not registered for VAT purpose. This provision is applicable for factory
building construction as well.

The provision is applicable only if the contractor is not registered in VAT.


Through the analysis of question, condition (a) does not prevail.

As to the prevalence of condition (b), there is construction of factory (for Commercial purpose- building
to be used as fixed asset) that costs Rs. 5,500,000; which is more than 50 Lakhs. The question is not clear
regarding the registration status of the contractor. If we assume that the contractor is a VAT registered
person who shall issue VAT invoice, M/s Nepali Rice Mill must not collect VAT from itself.

However, if we assume that the contractor is not registered for VAT, M/s Nepali Rice Mill Industries must
collect VAT from itself on such costs where VAT is usually applicable and not paid by M/s Nepali Rice Mill
Industries.

8) What are conditions for compulsory registration under the Value Added Tax Act, 2052 and Rules 2053?
(June 2014, 10 Marks& June 2010, 5 Marks)/(Dec 2007, 2 Marks, CA Inter)
Answer
Conditions for Compulsory Registration:

As per Rule. 6 and 7 of VAT Rule, 2053 (as amended until the date), the conditions for registration
for VAT are as follows:
a. In case the taxable turnover of a person exceeds as follows during last 12 months period:
i. Person dealing in goods- Rs. 50 Lakhs
ii. Person dealing in services- Rs. 20 Lakhs
iii. Person dealing mixed transactions of goods and services- Rs. 20 Lakhs
b. Import of goods having value more than Rs 10,000 for self-consumption except by the person dealing
in exempted items Shall be registered in VAT.

Turnover means higher of sales or purchase.

9) National Mobile Hub without being registered under VAT Act is dealing in retail of various branded
mobile sets at NLIC, Kamalpokhari. During the F.Y. 20X-69/X-70, the firm returned its file to the IRO on
20X-70.06.25 with the following Income Statement:
Income Statement for F.Y. 20X-69/X-70
Expenses Amount (Rs.) Income Amount (Rs.)
Opening Stock 175,870 Sales 1,787,654
Purchases 1,576,543 Closing Stock 545,387
Gross Profit C/D 580,628
2,333,041 2,333,041

Salary 91,000 Gross Profit B/D 580,628


Showroom Rent 108,000

The Institute of Chartered Accountants of Nepal 360


CAP-II Paper 7 - Income Tax and VAT
Telephone Charges 18,000
Printing & Stationery 5,075
Local Conveyance 2,400
Miscellaneous Expenses 1,556
Net Profit Before Tax 354,597
580,628 580,628

The Tax Officer made an assessment on 20X-70.08.15 demanding a sum of Rs. 445,778 as VAT along
with interest and penalty and also ordered to register the firm under VAT. The proprietor of National
Mobile Hub was surprised by the assessment order and rushed to you. He explained you that though
his firm not able to sale the goods above the threshold limit fixed under VAT Act, 2052, the IRD
assessed the VAT. He feels that the VAT should not be applicable on him. Is the contention of the
proprietor of the firm is correct? (June 2014, 5 Marks)

Answer:
Conditions for VAT registration, if any of the following conditions are satisfied:

Condition
Condition Particulars satisfied or
not
Compulsory registration where tax officer, during the time of time of N/A, as it is
due to holding of stock inspection, finds stock more than the limit decided by
prescribed by Inland Revenue Department Tax officer
Compulsory Registration For any other person than described above, the Not satisfied,
due to turnover threshold registration is required when taxable turnover of a as total sales
person exceeds as follows during last 12 months or purchase is
“Turnover” means higher period: less than Rs.
of Sales or purchasea. Person dealing in goods- Rs. 50 Lakhs 20 Lakhs
amount b. Person dealing in services- Rs. 20 Lakhs
c. Person dealing mixed transactions of goods and services-
Rs. 20 Lakhs

Import of goods having Import of goods having value more than Rs 10,000 Not Given
value more than Rs. for self-consumption except by the person dealing
10,000 for self- in exempted items Shall be registered in VAT.
consumption.
Since, none of the above conditions are satisfied, M/s National Mobile Hub is not required to get
registered for VAT purpose, unless it desires so voluntarily.
The contention of the proprietor is correct.

10) State the conditions, procedures for and effect of cancellation of registration under Value Added Tax
Act/Rules. (Dec 2013, 10 Marks)

The Institute of Chartered Accountants of Nepal 361


CAP-II Paper 7 - Income Tax and VAT
Answer
Conditions for Cancellation of Registration
a) In the case of a body corporate: where such body corporate is closed, sold or transferred or ceases
to exist due to any reason,
b) In the case of individual ownership, if the owner dies,
c) In the case of a partnership firm; if it is dissolved or where a partner dies,
d) In case the registered person ceases to be engaged in taxable transaction,
e) In case a registered person submits Zero Return continuously for twelve months or is a taxpayer
that does not submit Tax Return,
f) Where the taxpayer’s taxable transaction does not exceed Rs. 50 Lakhs in case of transaction of
goods and Rs. 20 lakhs in case of transaction of service or mixed transaction of goods or services in
last twelve months period,
g) Where there is registration in error.

Procedure for Cancellation of Registration


In case any of the conditions as discussed above arise and a person is required to cancel VAT registration,
the registered person or his/her legal representative/heir in his/her absence shall submit an application
in the format as prescribed by VAT Regulation within 30 days of happening of such conditions along with
the VAT return and the amount of VAT payable in front of tax officer.

The registered person shall submit all the relevant documents for tax assessment within 15 days of
submission of VAT return.

The tax officer shall assess the of such taxpayer and take decision on the matter of cancellation within 3
months of the submission of application. The matters of decision whether to cancel or not shall also be
notified to registered person within 3 months of the application of cancellation.
The registered person is required to submit tax returns for another three months until the decision of
the tax officer regarding cancellation is served to him.

In case the tax officer fails to decide on matter of cancellation or does not serve notification with regard
to the cancellation of registration within 3 months of application, the registered person is not required
submit any tax return pertaining to periods after the elapse of such three months.

Effect of Cancellation of Registration


The taxpayer shall assume the following items to be supplied at Market Value at the time of
cancellation of registration and shall assess and collect/levy tax:
a. Stock lying at the time of cancellation on which Input Tax Credit is already enjoyed; and
b. Capital Items still in use, the VAT paid on purchase of which has already been claimed as Input Tax
Credit

11) What are the special provisions relating to Tax Payer Registration under VAT Act, 2052? (Dec 2010, 5
Marks, CA Inter)

The Institute of Chartered Accountants of Nepal 362


CAP-II Paper 7 - Income Tax and VAT
Answer
Temporary Registration as organizer or Participant as exhibitor of Fair, Exhibition, etc. [sec. 10A]
Conditions for Temporary Registration of Unregistered Person
The following persons shall get temporary registration:
a. Any unregistered person desiring to engage in any short-term taxable transactions of goods or
services at fair, show, demonstration, display, exhibition etc., or
b. The unregistered organizer of such event

Stock Transfer Facility for Existing Registered Person


The person who is already registered and is participating the fair, exhibition, etc. as exhibitor can transfer
goods for transaction to the place of exhibition or fair.

Procedures for Temporary Registration (Rule 7Ka)


Application to be filed along with recommendation from Organizer [Rule 7Ka (1) & Rule 7Ka (2)]
Any unregistered person desiring to engage in any short-term taxable transactions of goods or services
at fair, show, demonstration, display, exhibition etc., shall make an application along with the following:
a. the recommendation from the organizer in the concerned Tax Office or Taxpayer’s Service Office,
and
b. Proof of deposit of amount equal to 2% of estimated revenue from the event

Issuance of Temporary Registration Certificate [Rule 7Ka (3)]


The tax officer at concerned Inland Revenue Office or Taxpayers Service Office shall issue a temporary
registration certificate once the application for registration along with deposit is received.

Special Provision for Registration of Joint Venture


Where two or more persons form a Joint Venture dealing in taxable transaction for a specific period of
time, such co-venturers shall file an application for the purpose of registration of Joint Venture at such
Tax Office where one of the co-venturer is registered.
The Tax Officer shall provide a Registration Certificate in the format as prescribed where an application
is filed for registration.
Joint Venture registered as above shall cancel its registration upon the completion of period specified in
the agreement forming such Joint Venture.
The co-venturers are jointly and severally responsible to discharge tax obligation of Joint ventures in
which they are involved.

12) Binod & Company, a proprietor engaged in the business of selling battery operated tempo, its chassis
and battery. Its annual turnover of the business was estimated of Rs. 10 million. Binod & Company not
registered with VAT. The advisor of the company suggested to get registered with VAT or ready to pay
penalty as per VAT Act. Assuming you are the advisor of Binod & Company, what would be your advice
to the company?
Would your answer be different if Binod & Company is engaged in the business of solar energy
equipments and operates under an umbrella of Alternative Resources Promotion Centre? (Dec 2009,
5 Marks)

The Institute of Chartered Accountants of Nepal 363


CAP-II Paper 7 - Income Tax and VAT
Answer
Conditions for VAT registration:
A person must register for VAT, if any of the following conditions is satisfied:

Condition Particulars
Compulsory registration where tax officer, during the time of time of inspection, finds
due to holding of stock stock more than the limit prescribed by Inland Revenue
Department.
Compulsory Registration For any other person than described above, the registration is
due to turnover threshold required when taxable turnover of a person exceeds as follows
during last 12 months period:
“Turnover” means higher ofa. Person dealing in goods- Rs. 50 Lakhs
Sales or purchase amount b. Person dealing in services- Rs. 20 Lakhs
c. Person dealing mixed transactions of goods and services- Rs. 20 Lakhs
Import of goods having Import of goods having value more than Rs 10,000 for self-
value more than Rs. 10,000 consumption except by the person dealing in exempted items
for self-consumption. Shall be registered in VAT.

A person can register himself voluntarily with VAT even though his VAT applicable annual turnover is less
than Rs. 5 million.
A person dealing with the goods and services exclusively covered in Schedule 1 cannot register for VAT
[Section 10(3)].
In the given case, Binod & Co. is engaged in the business of selling battery operated tempo, its chassis
and battery and its estimated annual turnover is Rs. 10 million. In battery operated tempo VAT is
applicable. But chassis of battery operated tempo is exempt from VAT. Also the battery is the VAT
applicable item. If the turnover of battery and tempo is Rs. 5 million or more, then only Binod & Company
is required to get registered with VAT. The situation of compulsory registration as suggested by the
advisor will arise only when the turnover of battery will exceed Rs. 5 million.

If Binod & Company was engaged in the following business of solar energy equipment and operates
under an umbrella of Alternative Resources Promotion Centre, it is not required to get registered with
VAT as this item is covered by schedule I of VAT Act:
a. Equipment falling under the subheads 7410.21.00, 7607.19.00, 8403.10.00, 8403.90.00, 8412.80.00,
8419.19.00, 8419.31.00 and 8516.60.00 that can only be operated from solar heat energy
b. Lithium Ion Battery under subhead 8507.60.00 with capacity less than 12 volt and ampere that are
used in solar tuki.

13) Mr. Shrestha, a small business proprietor not required to be registered to VAT, got voluntarily
registered on 1.4.20X-63. After two years, he now feels that he should not have gone for registration
as it is creating unnecessary burden on his part.
He doesn't know whether he can go for cancellation or not and the procedure he is required
to follow. He approaches you for the advice. Suggest him. (June 2009, 7 Marks)

The Institute of Chartered Accountants of Nepal 364


CAP-II Paper 7 - Income Tax and VAT
Answer:
The person can apply for cancellation of registration any time after the registration. The procedure
for cancellation of registration is as follows:
a) In the case of a body corporate: where such body corporate is closed, sold or transferred or ceases
to exist due to any reason,
b) In the case of individual ownership, if the owner dies,
c) In the case of a partnership firm, if it is dissolved or where a partner die,
d) In case the registered person ceases to be engaged in taxable transaction,
e) In case a registered person submits Zero Return continuously for twelve months or is a taxpayer that
does not submit Tax Return,
f) Where the taxpayer’s taxable transaction does not exceed Rs. 50 Lakhs in case of transaction of
goods and Rs. 20 lakhs in case of transaction of service or mixed transaction of goods or services in
last twelve months period,
g) Where there is registration in error.

In case any of the conditions as discussed above arise and a person is required to cancel VAT registration,
the registered person or his/her legal representative/heir in his/her absence shall submit an application
in the format as prescribed by VAT Regulation within 30 days of happening of such conditions along with
the VAT return and the amount of VAT payable in front of tax officer.

The registered person shall submit all the relevant documents for tax assessment within 15 days of
submission of VAT return.

The tax officer shall assess the of such taxpayer and take decision on the matter of cancellation within 3
months of the submission of application. The matters of decision whether to cancel or not shall also be
notified to registered person within 3 months of the application of cancellation.

The registered person is required to submit tax return for another three months until the decision of tax
officer regarding cancellation is served to him.

In case the tax officer fails to decide on matter of cancellation or does not serve notification with
regard to the cancellation of registration within 3 months of application, the registered person is
not required submit any tax return pertaining to periods after the elapse of such three months.

The Institute of Chartered Accountants of Nepal 365


CAP-II Paper 7 - Income Tax and VAT

Chapter 2: Time and Place of Supply


1) State the place of supply as per Value Added Tax Act, 2052/Value Added Rules, 2053, in case of: (June
2012/2022, 5 Marks)
a) Sale of movable goods
Answer: the place from where the buyer takes possession of the goods.

b) Immovable goods
Answer: the place where the goods are situated

c) Imported goods
Answer: the customs frontier from where the goods first enter into Nepal

d) Self-consumption
Answer: the place where vendor resides

e) Service
Answer: the place where the benefit of service is received.

2) You are a tax expert. Answer stating the provisions relating to “Time of Supply” as per Value Added
Tax (VAT) Act, 2052, what happens when: (Dec 2011, 10 Marks)
a) A customer takes delivery of the goods from the business place before the supplier issues invoice.
Answer:
The time of supply of goods is earliest of “issuance of invoice”, or “at the time of taking possession of the
goods by buyer”, or “at the time of receipt of consideration”.

In this case, the time of supply is when the buyer takes possession of the goods.

b) A customer paid NPR 10,000 to a supplier along with a list of certain goods to be supplied to the
customer. As per the list the total goods are worth NPR 12,000
Answer:
The time of supply of goods is earliest of “issuance of invoice”, or “at the time of taking possession of the
goods by buyer”, or “at the time of receipt of consideration”.

Since the payment is advance is not treated as consideration, the time of supply would be when any of
the above event occur.

c) A supplier at Kathmandu receives order for supply goods from a customer from Pokhara. The
supplier packs the goods as per the order, asks a labor to deliver the goods to the transporter and
the transporter gives delivery of the goods to the party of Pokhara at Pokhara. When the buyer
receives the goods, the supplier issues the invoices.

The Institute of Chartered Accountants of Nepal 366


CAP-II Paper 7 - Income Tax and VAT
Answer
The time of supply of goods is earliest of “issuance of invoice”, or “at the time of taking possession of the
goods by buyer”, or “at the time of receipt of consideration”.
In the given case, the buyer is deemed to take possession the goods when the supplier delivers the goods
to the transporter and it is the earliest event, as such, it is the time of supply.

3) How the place of supply of goods and services is determined under the VAT laws?
(June 2009, 7 Marks)
Answer
Place of supply of Goods:
The following places shall be deemed to be the place of supply of goods:
a) In the case of movable goods transferred by sale, the place where such goods were sold or
transferred,
b) In the case of any immovable goods where location can’t be transferred even if change in ownership
, the place where such goods are located,
c) In the case of imported goods, the Customs point in Nepal through which goods are imported into
Nepal,
d) In case any producer or vendor supplies the goods to the self, the place where the producer or the
vendor of such goods resides.

Place of Supply of Services


The place of supply of a service shall be the place where the benefit of that service is received.

The Institute of Chartered Accountants of Nepal 367


CAP-II Paper 7 - Income Tax and VAT

Chapter 3: Taxable Value


1) Global Visa Support Service Pvt. Ltd. is operating as approved education consultancy at
Dillibazar. The Occupation English Test (OET) facilitation is done by the company to the
Nepal service seekers. The cost of OET is 250 USD and need to be paid to US based Global
Services Centre. Each Nepali who wish to appear on the test should go through the Nepali
company for the exam appearance. The company further collects Rs. 15,000 as test
preparation and facilitation services. Discuss the impact of VAT on above transaction. (Dec
2021, 5 Marks)

Answer:
As per the Schedule 1 of VAT Act, 2052, The educational services by the Universities and
Schools are exempted from VAT, but the case above mentioned for OET test is not exempted.

Thus, the Nepali company Global Visa Support Service Pvt. Ltd. need to collect VAT on exam
fee 250 USD (equivalent NRs) from each Nepali service seeker, and on the Rs. 15,000 test
preparation and facilitation fee is also VAT attractive.
On payment to US based company by Nepali Company, reverse VAT is applicable.

2) Mr. Ram Prasai a VAT registered person purchased wood/timber from Gairigaun
Community Forest of Jhapa District for business purposes through auction. The auction
price of wood at the time of auction, release order and (cutting)/saw order was Rs. 2,100,000,
Rs. 2,200,000 and Rs. 2,300,000 respectively. The release order was issued earlier than
auction time and cutting order issued by Gairigaun Community Forest.
Required: (Dec 2019, 5 Marks)
a. Explain the applicability of VAT on sale of wood/timber as per the provision of VAT Act, 2052?

Answer
As per section 12A of VAT Act, 2052 VAT is applicable on sales of wood by community, private cultivated
or private forest when the wood buyer buys it for commercial purpose. Where there is applicability of
Value added tax, the time of supply is earliest of issuance of cutting order, issuance of release order or
auction (sales) of the wood.

The taxable value of the wood shall be deemed royalty (royalty applicable for the wood if the wood were
sold by national forest) or auction value (sales value is treated as auction value), whichever is higher.

In case of a national forest, VAT is applicable on sales of wood regardless of purpose of purchase by
buyer. Taxable value and time of supply is as explained above.

The Institute of Chartered Accountants of Nepal 368


CAP-II Paper 7 - Income Tax and VAT

b. Compute the liability of VAT amount if any under the provision of VAT Act, 2052?
Answer
To determine the amount of VAT, the taxable value shall be determined. Since taxable value is higher of
auction price (sales value) or royalty.
In the given case auction price of the wood is Rs. 2,100,000 and amount of royalty is not given. We
assume that the auction price and royalty amount is equal. So, taxable value would be Rs. 2,100,000. VAT
shall be applicable on Rs. 2,100,000.

3) Sussex (Pvt) Ltd, Kathmandu imports goods and directly supplies to the retailers. From the
following information, compute the amount of VAT on sales. (June 2018, 5 Marks)

Total cost of goods declared by importer Rs. 350,000.00


Valuation for cost of goods by Custom Officer Rs. 400,000.00
Freight & Insurance up to custom point Rs. 17,000.00
Freight from custom point to warehouse Rs. 20,000.00
Freight from warehouse to the retailers Rs. 8,000.00

Import Duty @ 10 %, excise duty @ 5% applicable only on import. Gross profit 10 %

Answer
Calculation of Customs Duty and Excise duty to be paid at the time of import

Cost Component Notes


Amount
When value is revised by Customs
Valuation for Cost of Goods by Customs officer, the revised value is
400,000
Officer considered for determination of
taxable amount.
Add: Freight and insurance up to Customs 17,000
Transaction Value for Customs purpose 417,000
Customs duty @10% of valuation 41,700
Value for Excise Duty 458,700
Excise duty @5% 22,935

Note:
Gross profit is usually expressed as a percentage of sales, so, 10% gross profit is deemed to be as
percentage of sales
Determination of Selling price and VAT on Sales

Particulars Notes
Amount
Total Cost of Goods declared by Importer 350,000

The Institute of Chartered Accountants of Nepal 369


CAP-II Paper 7 - Income Tax and VAT
Add: Freight and insurance up to Customs 17,000
Add: Freight from Customs point to
20,000
warehouse
This cost is not direct cost, therefore,
Freight from warehouse to retailers - is not considered in determining the
profit margin
Customs duty 41,700
Excise Duty 22,935
Total Cost of Purchase of goods 451,635
Cost is 90% of Sales, i.e. Profit= Cost
Add: Profit Margin (10% of Sales) 50,182
divided by 0.9 multiplied by 0.1
Sales 501,817
VAT @13% on sales 65,236

4) Recondition House Baneshwor dealing in used goods made following Purchase & sales
during Kartik 20X-73:
Item Purchase with VAT (Rs.) Sales without VAT (Rs.)
Sofa 14000 21000
Freeze 32000 43000
Wooden Furniture 45000 38000

Calculate the amount of VAT payable by recondition house. (June 2017, 5 Marks)

Answer:
As per Rule 33, the taxable value in case of dealer of secondhand goods is the difference
between selling price (excluding VAT) and cost of goods (including VAT). If purchase price
is higher than sales price, there will not be any taxable value resulting into zero VAT.
Wooden
Particulars Sofa Freeze Furniture
Sales without VAT 21,000 43,000 38,000
Purchase with VAT 14,000 32,000 45,000
Difference between Sales without VAT and Purchase
with VAT 7,000 11,000 (7,000)
Taxable Value (=Profit) 7,000 11,000 -
VAT @13% 910 1,430 -

5) Few years ago, due to the blockade in Indo-Nepal border that lasted for quite a few months,
almost all imported goods including cooking fuels were obstructed. To avoid the shortage
of cooking fuel, the User Forest Group (UFG) decided to cut down community forest trees
for timbers to be used as cooking fuel. The UFG accordingly sold out such timber in the
market to be distributed amongst the households. The sold out timbers fetched Rs. 300,000
meant ultimately to be distributed to the user community group. Had those timbers been
The Institute of Chartered Accountants of Nepal 370
CAP-II Paper 7 - Income Tax and VAT
sold out to the state owned forest, the forest offices would have paid Rs. 400,000 as royalty
charges.

Compute the amount of VAT in case: (June 2016, 5 Marks)


a) Timbers are sold through auctions and sale proceeds are distributed amongst the UFG.
b) Timbers are distributed amongst the UFG for their own domestic usages.

Answer:
As per Sec. 12Ka, where a national forest sales wood, VAT shall be levied at such time, that is
earlier of issuance of cutting order, release order or auction. The taxable value shall be higher of
auction value or royalty fixed for such wood.
Where a community forest, private cultivated forest or private forest sells wood for commercial
use by buyer, taxable value shall be higher of sales value (deemed auction value) or royalty fixed
for such wood. Tax shall be collected at the time that is earlier of issuance of cutting order, release
order or sales (deemed auction)
(a) In the given case, timbers are sold for the domestic purpose of buyer. For sales of wood from
community forest, private cultivated forest or private forest to become taxable, the buyer must
purchase it for commercial use. Therefore, VAT is not applicable on sales wood in the given
circumstance.
(b) Where wood are distributed for free, there is commercial transaction and buyer uses it for domestic
purpose, therefore, VAT is not applicable.

6) Discuss the amount to be included in Taxable Value as per section 12 of Value Added Tax Act, 2052.
(Dec 2014, 5 Marks)
Or
How taxable value is determined to levy value Added Tax under Section 12 of VAT Act? (Dec 2007, 5
Marks, CA Inter)

Answer
General Provision of Taxable Value
The taxable value of goods or services shall be the cash consideration received or receivable by the
supplier (i.e. value of goods or services so received) for the goods or services where the consideration is
payable in cash, after deducting trade discount and trade rebate.
The following amounts are also included while computing taxable value:
• Transportation cost borne by the supplier at the time of transaction and distribution expenses and
the amount of profit,
• Excise duty, ownership tax and other tax amount except the VAT to be levied under VAT Act.
For this purpose, “Other Tax Amount” means duty, fees and taxes levied prescribed by annual
Finance Bill.

Specific Provision:
In the following cases, the taxable value shall be as follows:
a. Goods exchanged under barter system- market value of goods exchanged

The Institute of Chartered Accountants of Nepal 371


CAP-II Paper 7 - Income Tax and VAT
b. In case of import of goods- landed cost up-to border (cost paid to vendor, transit cost or other cost)
plus duty charged by the Customs Officer except VAT
c. In case of significant under-invoicing- Market value
d. In case of sales through obtaining part consideration- Market value
e. Taxable value of wood of national forest- higher of auction price or royalty amount
f. Taxable value of wood of community forest, private cultivated forest or private forest- higher of
auction price (sales value) or royalty amount (deemed) where the buyer buys wood for commercial
purpose
g. In case of dealer of used goods- profit, i.e. difference between selling price of goods and cost of such
goods
h. In case of notified goods sold to unregistered person- trade discount cannot be deducted while
determining taxable value
i. At the event of cancellation of registration- the stock and capital items on which input tax credit was
already claimed and remaining at the time of cancellation of registration- deemed to be supplied at
market value
j. Discontinuance of use of such capital goods, on which input tax credit was claimed, for taxable
transaction- market value
k. In case of shortage of stock at the time of inspection by tax officer- shortage is deemed to be supplied
at market value

7) Apar International Pvt. Ltd. is a Recondition House involved in Purchase/Sale of Used Vehicle,
Motorcycle, TV and Refrigerator. The company purchases the used/old Vehicle, Motorcycle, TV and
Refrigerator and Sale those goods to the consumer by repairing and improving the same. Company
purchased from Mr. Ram residing in Satungal at Rs. 20 lakhs and after repairing the same sold to Ms.
Sunita at Rs. 25 lakhs Apar Ltd. seeks your advise as how much VAT shall he collect on such sales
amount. (Dec 2014, 5 Marks)

Answer
Apar International Pvt. Ltd. is dealer of used goods, as such, the taxable value is difference between sales
value and cost of such goods.

In the given case, Apar International shall collect VAT on Rs. 5 Lakhs @ 13%, i.e. Rs. 65,000; and shall be
advised accordingly.

8) The Z Pvt. Ltd. has booked following expenses of imported electronic goods in the ledger. (June 2013,
8 Marks)
Cost of material as per invoice Rs. 200,000
Bank charges of L/C (Letter of Credit) Rs. 15,000
Insurance Rs. 5,000
Calcutta port clearing expense Rs. 20,000
Freight up to custom point Rs. 14,000
Fright from custom point to Kathmandu Rs. 7,000
Other tax paid at custom including local development tax Rs. 2,500
Nepal custom expenses including 1 % custom duty Rs. 30,000

The Institute of Chartered Accountants of Nepal 372


CAP-II Paper 7 - Income Tax and VAT
Discount receivable Rs. 15,000

Further information:
Discount amount has not been deducted in invoices. The above cost is the gross purchase.
a) Explain the taxable value in case of imported goods with reference to the provisions of the Value
Added Tax Act, 2052.
b) Compute the Value Added Tax payable at custom point as per above information.
c) Assume that the company has 20 % gross profit. What would be its sales price and VAT amount?

Answer
a. Taxable Value in case of Import
Taxable value for the purpose of import shall be landed cost up-to border (cost paid to vendor, transit
cost or other cost) plus duty charged by the Customs Officer except VAT.

b. Value Added Tax Payable at Customs Frontier

Particulars Amount Notes


Cost of material as per invoice 200,000
Bank charges of L/C (Letter of Credit) - It is cost incurred inside Nepal
Assumed to be incurred
Insurance 5,000 outside Nepal
Calcutta port clearing expense 20,000
Freight up to custom point 14,000
Fright from custom point to Kathmandu - It is cost incurred inside Nepal
Transaction Value for Customs Purpose 239,000
Customs duty @ 1% 2,390 Given in question
Other tax paid at customs including local
development tax 2,500 Part of taxable value for VAT
Taxable Value for VAT at Customs
Frontier 243,890
VAT recoverable by Customs Officer
@13% 31,706

c. Value Added Tax on Sales


Note: Gross profit is usually expressed in terms of percentage of sales, unless otherwise stated
Gross profit is computed by deducting direct costs from sales. Therefore, only the direct costs
are considered in determining selling price
Particulars Amount Notes
Cost of material as per invoice 200,000 Part of direct cost
Bank charges of L/C (Letter of Credit) 15,000 Part of direct cost
Insurance 5,000 Part of direct cost
Calcutta port clearing expense 20,000 Part of direct cost

The Institute of Chartered Accountants of Nepal 373


CAP-II Paper 7 - Income Tax and VAT
Freight up to custom point 14,000 Part of direct cost
Fright from custom point to Kathmandu 7,000 Part of direct cost
Other tax paid at custom including local
development tax 2,500 Part of direct cost
Customs duty 2,390 Part of direct cost
Part of direct cost, difference
Nepal custom expenses excluding 1 % custom between 30,000 and customs
duty 27,610 duty
It is cash discount and is not
considered in determining gross
Discount Receivable - profit
Cost of Materials including all direct costs 293,500
cost divided by 0.8 multiplied
Gross profit @20% of Sales 73,375 by 0.2
Selling price (Taxable Value) 366,875
VAT @13% 47,694

9) ABC Ltd., a VAT registered person, has exported Noodles to a party based in China. In exchange, ABC
Ltd. has received Garments from the Chinese Party. How Taxable Value of the Noodles is determined
in this case as per the provision of the VAT Act, 2052. (Dec 2012, CA Inter, 2 Marks)

Answer:
Taxable Value in case of goods supplied under barter system is market value of goods so exchanged. As
such, the market value of noodles or garments shall be the taxable value of noodles.

10) Royalty for the national wood has been fixed at Rs. 600 per cu. feet. Wood from the private forest is
sold for Rs. 500 per cu. feet. How the taxable value is determined in this case for the wood of private
forest as per the provision of VAT Act? How the taxable value is determined in case of Wood of national
forest? (Dec 2012, CA Inter, 3 Marks)

Answer
Taxable value of wood bought by buyer for commercial purpose from community forest, private
cultivated forest or private forest shall be higher of follows:
a. Royalty, if the wood were sold from national forest (i.e. Rs. 600 per cu. Ft.), or
b. Auction value (i.e. selling price) – i.e. Rs. 500 per cu. Ft.

In the given case, assuming the buyer buys the wood from private forest for commercial purpose; taxable
value is higher of above, i.e. Rs. 600 per cu. Ft.

In case of wood of national forest, taxable value shall be higher of auction price or royalty value of wood.

The Institute of Chartered Accountants of Nepal 374


CAP-II Paper 7 - Income Tax and VAT
11) Sharma & co, a registered person dealing in second hand vehicles has purchased a second-hand
motorcycle for NPR 10,000 plus chargeable VAT on it. The following expenses are incurred for repair
of the motorcycle to make it saleable:
Particulars Amount (NPR) VAT (NPR)
Spare Parts 4,000 520
Labor Charges 2,000 0

The person sells the motorcycle for NPR 20,000. Calculate the taxable value of the motorcycle as per
VAT Act, 2052. (Dec 2011, 5 Marks)

Answer
As per Rule 33, the taxable value in case of dealer of second hand goods are the profit on sales of such
goods. Since, input credit is not available on purchase of goods, VAT paid on purchase is treated as cost
while determining profit. If there is loss, there will not be any taxable value resulting into zero VAT.

Particulars Details Amount


Sales without VAT 20,000
Purchase with VAT 17,820
Purchase cost with VAT 11,300
Spare parts with VAT 4,520
Labor Charges 2,000
Difference between Sales without VAT and Purchase with VAT 2,180
Taxable Value (=Profit) 2,180
VAT @13% 283

12) M/s XYZ Drinkers Ltd. has sold a kind of drink to non VAT registered party for Rs.10,000. As per the
retail price published by the same company under the direction of IRD, the retail price is Rs. 12,000.
The company has collected VAT from the party on the amount of Rs.10,000 as it says it has given trade
discount to the party and the trade discount can be deducted to arrive at the transaction value. But
the assessing officer insists to collect VAT on the published price. As an expert, give your opinion on
this. (June 2011, 5 Marks)

Answer:
As per Sec. 14 (6) of Value Added Tax Act, 2052; where Inland Revenue Department requires to publish
retail price for the purpose of sales of any goods, trade discount shall not be deductible while determining
taxable value in case when such goods are sold to unregistered person.

In the given case, there is IRD’s notification to publish retail price to sell the kind of drinks, as such, the
taxable value shall be the retail price published by manufacturer/importer while selling such goods to
unregistered person.

The Institute of Chartered Accountants of Nepal 375


CAP-II Paper 7 - Income Tax and VAT
Therefore, the contention of assessing officer is correct, and the taxable value is Rs. 12,000.

13) How is the taxable value for second hand or used goods computed? Similarly how the taxable value
for wood of national forest, private and community forest is computed?
(June 2011, 5 Marks)
Answer
For the dealer of used goods, the taxable value is difference between sales value and cost of such goods,
i.e. Gross profit amount.

In case of wood of national forest, taxable value shall be higher of auction price or royalty value of wood.

Taxable value of wood bought by buyer for commercial purpose from community forest, private
cultivated forest or private forest shall be higher of follows:
a. Royalty, if the wood were sold from national forest, or
b. Auction value (i.e. selling price)

14) Mention the provisions regarding collection of tax by custom authorities. (June 2010, 5 Marks)

Answer:
The Taxable Value of imported goods shall be determined by adding Customs Duty, countervailing duties,
and any other taxes to levied at the time of import otherwise than VAT on the Importation Value
determined by including transportation costs, insurance, freight, commissions of agents and other
persons.

The Customs Officer is responsible to collect VAT except as otherwise specified by Ministry of Finance in
case of import of goods.

The Customs officer may exercise right pursuant to VAT Act and Customs Act for the collection of VAT on
import of goods.

The applicable VAT rate on import of goods shall be equal to the rate applicable for goods or services
supplied within the state of Nepal, which is 13%.

15) Elaborate the provisions relating the market value and its determination. (June 2010, 5 Marks)
Answer:
As per Sec. 13 of Value Added Tax Act, 2052; Market value of supplied goods or services shall be
determined on the basis of the fair value of consideration received or receivable in the transaction
between unrelated independent parties at arm’s length under similar circumstances as to characteristics,
quality, quantity of materials, and any other relevant factors.

As per Rule 22 of Value Added Tax Regulations, 2053; Tax Officer has right to fix the market price based
on the transaction of similar goods. In case a Tax Officer cannot ascertain market price, the Director

The Institute of Chartered Accountants of Nepal 376


CAP-II Paper 7 - Income Tax and VAT
General has the right to prescribe the method on the basis of price of similar goods transacted by
different suppliers.

As per Value Added Tax Directive issued by IRD, tax officer may take the following bases in determination
of market value:
a. Tax officer shall collect taxable value determined by other taxpayers dealing in similar goods
b. The officer shall consider the similarity in time, place, date and area while collecting data.
c. Comparison shall be made between identical and similar goods. Where there is policy of discount by
the taxpayer, the price before such discount shall be compared.
d. The seller and buyer shall be independent of each other.
e. Taxable value shall be determined independently.
f. The consideration against the supply of compared goods shall be identical or similar.
g. The below market price sales shall be focused on sales promotion.

16) A company dealing with computer parts sold the goods worth Rs. 25 lacs in the month of Baisakh 20X-
67. It has imported the computer parts on which input VAT has been paid as per the cost
fixed/determined by Custom Offices on CIF basis with details below:
Cost declared by importer Rs. 200,000
Revised cost fixed by Custom Officer Rs. 250,000
(based on prevailing international price)
Transportation Rs. 25,000
Insurance Rs. 5,000
Import Duty @ 6%
Besides, the credit balances up to Chaitra 20X-66 is Rs. 20,500. In addition, it has omitted to adjust
input tax credit on the purchase of Rs. 30,000 pertaining to the month of Chaitra 20X-66.

Compute the Input and Output VAT. (June 2010, 10 Marks)

Answer:

VAT paid at the time of import

Taxable value in case of import:


All costs incurred on importing goods up to border plus customs duty plus excise duty levied
at customs frontier plus any other indirect taxes levied at Customs Frontier, except VAT. If
Customs officer revises costs up to border, the revised cost shall be considered for customs
valuation and determination of taxable value at the time of import.

Particulars Amount Note


Revised cost fixed by Custom Officer 250,000
Assumed to be incurred
Transportation 25,000
outside Nepal
Assumed to be incurred
Insurance 5,000
outside Nepal

The Institute of Chartered Accountants of Nepal 377


CAP-II Paper 7 - Income Tax and VAT
Transaction Value for Customs purpose 280,000
Import Duty @ 6% 16,800
Taxable Value 296,800
VAT @13% Paid on Import 38,584

Calculation of Output VAT


Particulars Amount
Taxable Sales 2,500,000
VAT @13% 325,000

Total eligible VAT credit for the month of Baisakh


Particulars Amount Note
VAT paid on Import 38,584
Carried forward excess VAT of Chaitra 20,500
VAT credit on purchase can
Purchase of Chaitra on which VAT credit was
3,900 be claimed within 12 months
erroneously not claimed
of purchase
Eligible VAT Credit 62,984

VAT Payable/(Receivable) for the month of Baisakh


Particulars Amount
Output VAT 325,000
Eligible Input VAT credit 62,984
VAT payable/(Receivable) 262,016

17) How much value added tax is to be collected in the following cases? (June 2010, 10 Marks, CA Inter)
a) Sale of wood from community forest at Rs. 90,000, governmental royalty if would be sold the same
from national forest would be Rs. 100,000.
Answer
If the wood is bought by buyer for commercial use, taxable value shall be higher of sales price (deemed
auction value) or royalty applicable when the wood was sold by national forest, therefore, taxable value
would be Rs. 100,000 (higher of Rs. 90 thousand and Rs. 1 Lakhs). VAT to be collected by Community
Forest would be Rs. 13,000 (13% of taxable value)

If the wood is not bought by buyer for commercial use, there would not be any applicability of VAT.

b) Export of rice costing Rs. 80,000 at Rs. 100,000.

Answer
Rice is listed in Schedule 1 of Value Added Tax Act, 2052. Therefore, transaction of rice is exempt from
VAT. There will not be applicability of VAT.

The Institute of Chartered Accountants of Nepal 378


CAP-II Paper 7 - Income Tax and VAT
c) Telephone bill was Rs. 10,000 and delay penalty is Rs. 2,500 levied on settlement.
Answer
Fees, Penalty, etc. of Telecommunication companies does not form part of taxable value (M/s Saroj
Shrestha Vs. Nepal Telecommunication Corporation, Writ Number 3719 of 2056 B.S.)
Therefore, taxable value is Rs. 10,000 and VAT is Rs. 1,300.

d) Second hand goods seller sales Rs. 100,000 goods, cost in total was Rs. 80,000.
Answer
As per Rule 33, the taxable value in case of dealer of second hand goods is the profit on sales of such
goods. Since, input credit is not available on purchase of goods, VAT paid on purchase is treated as cost
while determining profit. If there is loss, there will not be any taxable value resulting into zero VAT.

Therefore, taxable value is Rs. 20,000 and VAT is Rs. 2,600.

e) Sale of furniture by a trader at Rs 50,000, dealer is not registered in VAT.


Answer
VAT is collected by registered person, except in the circumstances mentioned in Sec. 8 (2), 8 (3), 12Ka
and 15 (3). In the given scenario, such circumstances do not prevail. As such, the furniture trader cannot
collect VAT.

18) From the following information, compute the amount of VAT. (Dec 2009, 5 Marks)
Total cost of goods declared by importer Rs. 250,000
Revised cost fixed by Custom Officers (based on prevailing International Price) Rs. 300,000
Freight Rs. 10,000
Insurance Rs. 2,000
Import Duty @ 10 %

Answer:

Taxable value in case of import:


All costs incurred on importing goods up to border plus customs duty plus excise duty
levied at customs frontier plus any other indirect taxes levied at Customs Frontier, except
VAT. If Customs officer revises costs up to border, the revised cost shall be considered
for customs valuation and determination of taxable value at the time of import.

Particulars Amount Note


Revised cost fixed by Custom Officer 300,000
Assumed to be incurred
Transportation 10,000
outside Nepal
Assumed to be incurred
Insurance 2,000
outside Nepal
Transaction Value for Customs purpose 312,000

The Institute of Chartered Accountants of Nepal 379


CAP-II Paper 7 - Income Tax and VAT
Import Duty @ 10% 31,200
Taxable Value 343,200
VAT @13% 44,616

19) Mr. Ram, a furniture supplier produced some woods from a community forest through auction and has
not paid any Royalty or VAT on the deal. IRD Officer assessed the transaction and ordered to Mr. Ram
to pay VAT on the auctioned price. Mr. Ram disputed the order and argued that since there was no
Royalty amount it cannot be required to pay VAT. Mr. Ram appoints you as a professional advisor.
Please advice Mr. Ram on the applicability of VAT and whether he is required to pay the amount on
the agreed price. (Dec 2009, 5 Marks)

Answer:

As per Sec. 12Ka, where a national forest sales wood, VAT shall be levied at such time, that is
earlier of issuance of cutting order, release order or auction. The taxable value shall be higher of
auction value or royalty fixed for such wood.
Where a community forest, private cultivated forest or private forest sells wood for commercial use by
buyer, taxable value shall be higher of sales value (deemed auction value) or royalty fixed for such wood.
Tax shall be collected at the time that is earlier of issuance of cutting order, release order or sales
(deemed auction).
Since, Mr. Ram has purchased the wood for commercial use (he is furniture supplier); VAT is applicable
on the auction price or royalty applicable on such wood for national forest, whichever is higher.

The provision of Sec. 12Ka requires the seller (i.e. community forest) to collect such VAT, however, IRD
has applied the provision in reverse charging mechanism. That means, Tax officer will assess and collect
VAT from the buyer of wood.

As such, the assessment of tax officer (as per IRD’s circular) is correct. However, Mr. Ram can still
challenge the decision of tax officer, since tax has to be assessed and collected from Community Forest,
not from Ram.

20) Determine Taxable Value in the following cases: (June 2005, 10 Marks, CA Inter)
a) Ex-factory price Rs.78, 000. Freight and insurance up to the buyer’s place, 2000. Trade Discount
7%, special Discount 10%.
Answer
Ex-Factory Price 78,000
Freight & Insurance 2,000
Total 80,000
Less: Trade Discount @7% 5,600
Less: Special discount @10% (before issuance of invoice) 8,000
Taxable value 66,400

b) Ex-factory price Rs.50 000 Quantity Rebate 5%, Trade Discount 10%
Answer

The Institute of Chartered Accountants of Nepal 380


CAP-II Paper 7 - Income Tax and VAT
Ex-factory Price 50,000
Less: Trade Discount @10% 5,000
Less: Quantity Rebate at 5% 2,500
Taxable Value 42,500

c) Ex-factory price Rs.18,000 Quantity Rebate on a previous consignment billed and delivered in
Kartik, 20X-61 Rs. 2,000
Answer
Ex-Factory Price 18,000

d) Goods imported for Rs.100, 000 by paying VAT at 13% at custom point. The quality deteriorated
and could not be used in production. Hence it is to be sold for Rs. 10,000 as scrap
Answer
The taxable value can be taken as Rs. 10,000; if adequate documentation is maintained.

e) Ex-factory price Rs.25, 000, Trade Discount 10%, Cash Discount 2% commission 5%
Answer
Ex- Factory Price 25, 000
Less: Trade Discount @ 10% 2,500
Taxable value 22, 500

Cash discount provided before the issuance of invoice is deductible while calculating taxable value.

Commission payable as per contractual obligation will not be reduced from the invoice and same is
available deduction separately.

21) Saurav Publication prints the various books and sells to the wholesaler. The Company itself has
constructed a building for its Publication House. The cost of construction was as follows:
Labour charges Rs. 30,00,000
All Material (Cement, rod etc) purchase cost Rs. 550,000 inclusive VAT.
State the VAT Implications on this case, if applicable with references to VAT Act, 2052 and rules, 2053.
(Dec 2014, 5 Marks)

Answer:
As per Sec. 8 (3), Saurav Publication shall collection of VAT from itself, if it constructs building, apartment,
shopping complex or similar other structures for commercial purpose costing more than 50 Lakhs, and
the contractor who constructs the commercial structure is not registered for VAT purpose. This provision
is applicable for factory building construction as well.

Since, the total construction cost does not exceed Rs. 5,000,000; therefore, this provision is not
applicable.

The Institute of Chartered Accountants of Nepal 381


CAP-II Paper 7 - Income Tax and VAT
22) "Supertech Infosys Pvt. Ltd." deals in computer & related IT technology business. In the course of its
business, it received service from "Key Technology Corp., Japan”, against which, Technology Corp.
raised bill of Rs. 21.60 lakhs. Management of Supertech is confused as to whether VAT shall be charged
on same.

Advise him as regards to VAT applicable on same and net payment to be send. Specify the provision of
VAT Act, 2052 in this regard. (Ignore other taxes, if applicable). (Dec 2013, 5 Marks)

Answer:
As per Sec. 8 (2) of Value Added Tax Act, 2052 (as amended); a person (whether registered or not)
obtaining service from party outside Nepal that is not registered for VAT purpose in Nepal, shall collect
VAT from itself on taxable value determined applying rules u/s 12 of the Act at the time, which is earlier
of payment for service or receipt of service.

In the given case, Supertech Infosys Pvt. Ltd. obtains service from Japanese company. Japanese company
is not registered for VAT purpose in Nepal. Cash consideration payable by Supertech Infosys is Rs. 21.60
Lakhs, therefore, it shall collect VAT @13% on Rs. 21.60 lakhs and pay the VAT amount in Revenue
account of Nepal Government of Nepal.

It shall pay Rs. 21.60 Lakhs to Japanese company. The VAT shall be collected at such time, which is earlier
of payment for service or obtaining service.

23) Mercantile Pvt. Ltd. is a company providing telecommunication services in Nepal. It has purchased a
consultancy service from a company in US for setting up and expanding its operation in Karnali zone
of Nepal. The company has paid USD 500,000 against such services in the month of Jestha 20X-69.
Mercantile Pvt. Ltd. is of the view that since payment is made to foreign company there is no
implication of Value Added Tax. State your view referring relevant provision of the Value Added Tax
Act, 2052. (June 2013, 4 Marks)

Answer:
As per Sec. 8 (2) of Value Added Tax Act, 2052 (as amended); a person (whether registered or not)
obtaining service from party outside Nepal that is not registered for VAT purpose in Nepal, shall collect
VAT from itself on taxable value determined applying rules u/s 12 of the Act at the time, which is earlier
of payment for service or receipt of service.

In the given case, Mercantile Pvt. Ltd has obtained service from a company in US. The company in US is
not registered for VAT purpose in Nepal. Therefore, Mercantile Pvt. Ltd. has the obligation to collect VAT
from itself on such service. The VAT shall be collected at the time, which is earlier of payment date (Jestha
20X-69) or time of receipt of service (question does not specify the time of receipt of service).

The VAT so collected, shall be paid in revenue account of Government of Nepal within 25 days of the end
of month when the VAT was collected (as per Sec. 19).

The Institute of Chartered Accountants of Nepal 382


CAP-II Paper 7 - Income Tax and VAT
Based on the above explanation, Mercantile Pvt. Ltd. shall collect VAT @13% on Nepalese rupee
equivalent of USD 500,000 (Conversion rate is buying rate of NRB on the date when VAT is to be
collected).

24) EFG Ltd. has given a contract to Garibdas construction, a party not registered in VAT, for construction
of a Shopping Complex built exclusively for commercial purposes. The cost of the shopping complex is
Rs.55 lacs. By mentioning the relevant provision of the act and rules, comment on the taxability of the
transaction. (Dec 2012, 4 Marks)

Answer:
As per Sec. 8 (3), any person (registered or not) in Nepal engaged in construction of buildings,
apartments, shopping malls or similar constructions for commercial purpose, the cost of which is more
than Rs. 5 millions shall collect VAT on the construction cost from themselves and deposit it to Revenue
Authority in case the construction work is not carried out through a registered person.
In case of failure to levy tax as such, the tax shall be assessed and recovered from the owner of such
building or structure.

Definition of Commercial purpose


For the purpose of Sec. 8 (3), Commercial Purpose means construction of building, apartment, shopping
complex or similar other structures prescribed by IRD for the purpose of sales or using such asset by
accounting the asset as current or fixed asset.

Conclusion
In the given case, since, M/s EFG Ltd. has given a contract to unregistered contractor and the value of
work of construction of shopping complex for commercial purpose exceeds Rs. 50 Lakhs, M/s EFG Ltd.
shall collect VAT on such costs of shopping complex on which VAT is applicable and VAT is not paid
already by the company.

The Institute of Chartered Accountants of Nepal 383


CAP-II Paper 7 - Income Tax and VAT

Chapter 3: Computation of Tax Liability


1) Cosmos Pvt. Ltd. having registered office in Jawalakhel, Lalitpur transferred some goods costing Rs.
1,000,000 to its branch in Bangladesh. In the same month, branch in Bangladesh transferred some
goods costing Rs. 500,000 to its head office in Nepal. Gross Profit margin of head office is 20% and
branch is 25%. The profit of branch is consolidated at the end of each financial year. Find the VAT
impact on the above transactions. (June 2012, 5 Marks)

Answer:
In the given case, there are two transactions:
a. Import from branch in Bangladesh (transfer of goods costing Rs. 500,000 to Head office from Branch
in Bangladesh), and
b. Export to the branch in Bangladesh (registered office in Jawalakhel, Lalitpur transferred some goods
costing Rs. 1,000,000 to its branch in Bangladesh)

Therefore, assuming the goods to be exported and imported is VAT-attractive:


a. Cosmos Pvt. Ltd. has to pay VAT @ 13% at Customs Frontier on the value of goods determined as per
Sec. 12 of Value Added Tax Act, 2052 for the goods to be imported from Bangladesh. The taxable
value is landed cost plus customs duty plus excise duty plus any other indirect taxes applicable at
Customs frontier.
b. Cosmos Pvt. Ltd. has to pay VAT @ 0% on goods to be exported.

2) Pokhara Automobiles Pvt. Ltd., a registered entity dealing in second hand vehicles has purchased a
Hyundai i10 vehicle for NPR 1,500,000 registered in the name of footballer Paras Gauchan. The following
expenses are incurred for repair of the vehicle:
Spare parts without VAT: NPR 250,000
Labour charges: NPR 120,000
The company sells the vehicle for NPR 2,500,000. Calculate the taxable value of the vehicle
as per VAT Act, 2052. (June 2022, 5 Marks)

Answer:
As per Rule 33, the taxable value in case of dealer of second-hand goods is the profit on sales
of such goods. Since, input credit is not available on purchase of goods, as per rule 44 VAT
paid on purchase is treated as cost while determining profit. If there is loss, there will not be
any taxable value resulting into zero VAT.

Particulars Details (NPR) Amount (NPR)


Sales without VAT 2,500,000
Purchase with VAT 1,902,500
Purchase cost 1,500,000
Spare parts with VAT 282,500
Labour charges 120,000

The Institute of Chartered Accountants of Nepal 384


CAP-II Paper 7 - Income Tax and VAT
Difference between sales without VAT and 597,500
purchases with VAT
Taxable value 597,500
VAT @ 13% 77,675
Note: Such second hand goods dealer should maintain separate records for deal of every item.

3) MM Pvt. Ltd. had the following transactions in Baishakh 2078. Calculate the VAT payable/receivable
from the information below: (Dec 2021 , Marks 10)
Particulars Amount (Rs.)
Sales:
Local 5,000,000.00
Export 15,000,000.00
Purchases:
Raw materials 7,200,000.00
Special packing for export 500,000.00
Consultancy charges purchased from abroad 800,000.00
Bus for staff transportation 200,000.00
Motorcycle hire purchase 500,000.00
Telephone expenses 750,000.00
Diesel for generator 100,000.00
Diesel for bus 250,000.00
Petrol for motorcycle 300,000.00
Computers 150,000.00
Soft drinks 25,000.00

Additional information:
Opening VAT receivable for the month was Rs. 100,000. Diesel for bus for Rs. 20,000 and
soft drinks for Rs. 10,000 was purchased through abbreviated tax invoice. Items above are
exclusive of VAT.
Staff quarter of Rs. 6,000,000/- was constructed by the company in Month of Baisakh from builders
not registered in VAT and without payment of VAT.

Answer:
Particulars Amount (Rs.) VAT (Rs.) Remarks
Output VAT
Local Sales 5,000,000.00 650,000.00 Full
Export Sales 15,000,000.00 - 0% for Export
Total Output VAT (A) 650,000.00
Less: Input VAT
Raw materials 7,200,000.00 936,000.00 Full
Special packing for export 500,000.00 65,000.00 Full

The Institute of Chartered Accountants of Nepal 385


CAP-II Paper 7 - Income Tax and VAT
Reverse Charging,
Payment of consultancy assuming VAT
charges abroad 800,000.00 104,000.00 already paid
Purchase of bus for staff
transportation 200,000.00 10,400.00 Only 40% Allowed
Purchase of motorcycle hire
purchase 500,000.00 65,000.00 Full
Telephone expenses 750,000.00 97,500.00 Full
Purchase of diesel for
generator 100,000.00 13,000.00 Full
Abbreviated Tax not
Purchase of diesel for bus 230,000.00 29,900.00 allowed
Purchase of petrol for No credit as per Rule
motorcycle 300,000.00 - 41(1) (Ga)
Purchase of computers 150,000.00 19,500.00 Full
VAT on Beverages
not allowed as per
Purchase of soft drinks 25,000.00 - Rule 41(1) (Ka)
Total Input tax credit (B) 1,340,300.00
Opening VAT Receivable (C) 100,000.00
Excess Input VAT (A-B-C)
to be carried forward -790,300.00

As per section 8(3) of Value Added Tax Act, 2052, if a construction of commercial purpose
building or apartment or shopping complex or similar other structure as specified by the
Department, of which value is more than Five Million Rupees, has been made from a person who
is not registered, it shall be deemed as it has been constructed from registered person and shall
deposit the tax. In case tax is not deposited, it shall be assessed and collected from the owner of
such structure.
So, VAT needs to be paid in Staff quarter construction, as it is not done through VAT registered
person.
VAT payable =Rs. 6000,000*13%= Rs.7,80,000
Further, input tax credit is not allowed on this VAT.

4) ABC Ltd. provides you the following transaction during Ashwin to Mangsir, 2078:
Amount (Rs.)
Particulars Ashwin Kartik Mangsir

Sales:

Taxable Sales 100,000 200,000 300,000

The Institute of Chartered Accountants of Nepal 386


CAP-II Paper 7 - Income Tax and VAT
Zero Rated Sales 300,000 100,000 200,000

Exempted Sales 200,000 300,000 100,000

Total Sales (A) 600,000 600,000 600,000

VAT paid/payable on purchases related to:

Taxable Sales 13,000 26,000 39,000

Zero Rated Sales 12,000 3,000 6,000

Exempted Sales 12,000 24000 6,000

Total VAT paid (B) 37,000 53,000 51,000

VAT paid/payable on expenses:

Overhead expenses 12,000 12,000 12,000

Motor Car 26,000 0 0

No credit (goods/services) 8,000 8,000 8,000

Total VAT paid (C) 46,000 20,000 20,000

Required: (Dec 2021, 5 Marks)


Compute the amount of eligible VAT credit available to the company for each month?

Assume there was opening VAT credit of Rs. 1,500 at the beginning of Ashwin, 2078.

Answer:
Computation of eligible VAT credit available to ABC Ltd for each month:

Particulars Ashwin Kartik Mangsir

Percentage of Taxable Sales 66.67 50.00 83.33


Input tax credit (VAT receivables):
Opening VAT credit 1,500 28,433 37,433
Input tax credit related to taxable sales 13,000 26,000 39,000
Input tax credit related to Zero Rated Sales 12,000 3000 6,000
Input tax credit related to Exempted Sales 0 0 0
Input tax credit on Overhead Exp. (Note -2) 8,000 6,000 10,000
Input tax credit on Motor Vehicle (W.N.-3) 6,933 0 0
Input tax credit on No credit (goods/Services) 0 0 0
Total eligible VAT credit available (A) 41,433 63,433 92,433

The Institute of Chartered Accountants of Nepal 387


CAP-II Paper 7 - Income Tax and VAT
Output Tax (VAT Payable):
Total Taxable Sales 13,000 26,000 39,000
Zero Rate Sales 0 0 0
Exempted Sales 0 0 0
Total output VAT payable (B) 13,000 26,000 39,000
Net VAT credit Receivable/ (Payable) to be
28,433 37,433 53,433
carried forward to next month A-B

Note 1:
Particulars Ashwin
Taxable Sales (Taxable + Zero Rated Sales) (a) 400,000
Exempted Sales 200,000
Total Sales (b) 600,000
% of Taxable Sales (a/b*100) 66.67

Note 2:
Computation of VAT credit allowed on overhead expenses
Particulars Ashwin
Overhead Expenses 12,000
% of Taxable Sales as per Note 1 66.67
Credit Allowed 8,000
Note 3:
Computation of VAT credit allowed on Motor Car
Particulars Ashwin
VAT paid on purchase of Motor Car 26,000
Credit Allowed @ 40% 10,400
% of Taxable Sales as per Note 1 66.67
Credit Allowed 6,933

5) Manjushree Pvt. Ltd. deals with cosmetic products. In previous month the export sales was 65% of
total sales. The company anticipates increase in export sales demand from following month quantified
as 20% of previous month's total sales. To increase the capacity to meet anticipated increase in
demand, the company imports a machinery from Singapore. Other details of transaction of the
company related to Baishakh 2078 are as follows:
a. While importing a machinery from Singapore, Manjushree Pvt. Ltd. paid Rs. 848,000 as custom
duty and Rs. 245,000 value added tax (VAT) at custom point of Tribhuvan International Airport.
b. Raw materials purchased during current month from local farmers not registered with VAT Rs.
5,300,000.
c. Consumables purchased from local vendors registered with VAT amounted to Rs. 1,000,000
without VAT.
d. Purchased a car for office use amounted to Rs. 4,000,000 without VAT.

The Institute of Chartered Accountants of Nepal 388


CAP-II Paper 7 - Income Tax and VAT
e. The company uses labors supplied by labor contractor and the amount of labor cost payable to
labor contractor is Rs. 1,356,000 including VAT.
f. The company incurred overseas marketing expenses to boost up export sales through advertising
agency based on Hong Kong and paid Rs. 900,000 for the service and applicable reverse VAT was
assessed and paid while remitting the amount to the agency.
g. As at the end of previous month, the company had opening balance of Rs.180,000 VAT credit and
out of which company had claimed vat refund of Rs. 120,000.
h. During the month of Baishakh, 2078 there as Domestic sales of Rs. 4,100,000 and Export sales of
Rs. 4,100,000 without VAT on both sales.

Required: (June 2021, 10 marks)

a. Compute net VAT receivable/payable of Manjushree Pvt. Ltd. for the month of Baishakh, 2078.
b. Is Manjushree Pvt. Ltd. eligible to claim refund immediately after filling vat return of Baishakh,
2078?

Answer

a)

Statement showing computation of Net VAT receivables/(payable)


of Manjushree Pvt. Ltd. for the month of Baishakh, 2078
Particulars Taxable VAT Receivables Remarks
A. VAT Receivable:
Rs. 180,000-Rs.
Opening VAT credit 60,000 120,000
Vat paid on Import of
Machinery 245,000
Domestic-Purchase Raw
materials from farmers 5,300,000 - No vat credit
Domestic-Purchase
consumables 1,000,000 130,000 Full Credit available
Labour 1,200,000 156,000 Full Credit available
(Rs. 40 Lakh *0.13) x
Vat paid on car 4,000,000 208,000 40% partial cr.
Vat paid on advertisement 900,000 117,000 Reverse VAT sec 8(2)
Total VAT Receivables (A) 916,000
B. VAT Payable:
Particulars Taxable VAT Payable
Sales- Domestic 4,100,000 533,000 @ Normal rate
Sales – Export 4,100,000 - @ Zero rate
Total VAT Payable on Sales
(B) - 533,000

The Institute of Chartered Accountants of Nepal 389


CAP-II Paper 7 - Income Tax and VAT
Net Vat receivables/(Payable)
(A-B) 383,000
b) The export sales is 50% of total sales of Baishakh, 2078. Yes, Manjushree Pvt. Ltd. is eligible to claim
refund immediately after filling vat return of Baishakh, 2078 as its export turnover was more than 40%
of total sales of Baishakh, 2078 as per the provision of section 24(4) of Vat Act, 2052.

6) X Ltd. was incorporated and registered with VAT. During the month of Kartik 20X-76 the company had
following transaction:
NPR.
Purchases/Expenses:
Purchase of raw materials- 175,000 (for taxable transaction)
Electricity expenses 25,000.00
Cold drinks 12,000.00
Diesel 20,000.00
LP Gas 7,000.00
Printing and stationery 35,000.00
Services availed from UK 100,000.00
Sales
Taxable sales 500,000.00
Exempted sales 25,000.00
Export sales 10,000.00
All purchases, expenses and sales are exclusive of VAT.
The company had opening balance of VAT credit of NPR. 1,150,000

You are required to calculate VAT payable or VAT credit available for the month of Kartik
20X-76. (Dec 2019, 5 Marks)
Answer
A. Output VAT (or VAT Collected on Sales)

Particulars VAT Note


Amount
Taxable Domestic Sales 500,000 65,000 Tax rate of 13%
Exempted Sales 10,000 - VAT is not applicable
Assumed to be that of
Taxable Export Sales 25,000 -
taxable items, taxed @0%
Total Output VAT for the
65,000
month

B. Eligible Input tax credit on purchases during the month

The Institute of Chartered Accountants of Nepal 390


CAP-II Paper 7 - Income Tax and VAT
Eligible
Particulars VAT Amount Notes
Amount
for Credit
Full credit, directly
Purchase of Raw Materials 175,000 22,750 22,750
related to taxable sales
Electricity Expenses 25,000 - - Electricity is exempt item
No VAT credit, beverage
Cold drinks 12,000 1,560 -
item (rule 41)

Diesel 20,000 2,600 2,548


Proportionate VAT
Credit, as the use is for
LP Gas 7,000 910 892 both taxable and non
taxable output.
Printing and Stationery 35,000 4,550 4,459

Total Eligible VAT Credit 30,649


C. Input VAT Credit on VAT paid as per Sec. 8 (2), 12Ka and 15 (3)
Eligible
Particulars VAT Amount Notes
Amount
for Credit
Proportionate VAT
Credit, as the use is for
Services availed from UK 100,000 13,000 12,740
both taxable and non
taxable output
Total 12,740

Ratio of Taxable Sales: 525,000/535,000*100= 98% (Rounded off)


D. Opening VAT credit (excess of Previous month,
eligible for set off) 1,150,000

E. Payable/ (Receivable) VAT [A-B-C-D]


(1,128,389)

7) Not Only For Profit Pvt. Ltd., Kathmandu imports luxury cars and track-laying tractors and sells in
Nepal. Following details pertain to Ashwin, 20X-75.
a) The showroom price of luxury car is Rs. 5 million. Two percent Dashain discount was offered on
sale of the cars. 10 cars were sold to VAT registered customers and 6 cars were sold to individual
customers not registered in VAT.
b) The showroom price of Track-laying tractors is Rs. 2.5 million. Two percent Dashain discount was
offered on the tractors. 10 tractors were sold to individual customers who were not registered for
VAT.

The Institute of Chartered Accountants of Nepal 391


CAP-II Paper 7 - Income Tax and VAT
c) The company offers additional 5% discount for prompt payment. 5 customers purchasing luxury
car and 6 customers purchasing tractor made prompt payment and benefitted from the discount
offer.
d) Import price of luxury cars was Indian Rupees (INR) 2 million per car. Total transportation expenses
upto custom point was INR 200,000 (paid to Indian transporter) and total insurance premium Rs.
200,000 was paid to an insurance company in Nepal. 16 cars were imported in the same month
and sold as above.
e) Import price of per unit Track-laying tractors was INR 1.5 million. Total transportation expenses up
to custom point were INR 100,000 (paid to Indian transporter) and total insurance premium Rs.
50,000 was paid to an insurance company in Nepal. 10 tractors were imported in the same month
and sold as above.
f) During the month, Rs. 90,000 was spent on mobile telephone calls, of which 40% relates to private
calls.
g) On 30 Ashwin 20X-75, office equipment was purchased for Rs. 1,000,000.
h) Custom duty was Rs. 41,376,000 for the luxury cars in total.
i) The company also constructed a showroom in the month of Ashwin, 20X-75. The total cost of
construction was Rs. 6 million, which was constructed by a VAT unregistered builder.
j) All of the figures are exclusive of VAT and use exchange rate @ Rs. 1.6015 per INR.

Calculate the amount of VAT payable/receivable by the company for Ashwin 20X-75. (Dec 2018, 10
Marks)

Answer:

Calculation of VAT payable/receivable for the month ended Bhadra 20X-75


Particulars Amount Rs.
Output tax (WN 1) 10,192,000
Less: Input tax for imported cars (WN 2) (12,108,759)
Input tax credit on expenses (WN 3) (130,396)
Net VAT receivable (2,047,155)
Reverse charge (VAT) on construction (WN 4- to be paid) 780,000

WN 1: Calculation of output tax


Particulars Car sale Rs. Tractor sale Rs.
Showroom price 5,000,000 2,500,000
Dashain Discount (trade discount) 100,000 50,000
Sales price 4,900,000 2,450,000
Sales to VAT registered Customers (10 cars) 49,000,000 0.00
Sales to Unregistered Customers (6 cars&10 tractors) 29,400,000 24,500,000
Total 78,400,000 24,500,000
Proportionate sales ratio 76.19% 23.81%

Track-laying Tractor is VAT exempt as per schedule 1 of VAT Act, 2052.

The Institute of Chartered Accountants of Nepal 392


CAP-II Paper 7 - Income Tax and VAT
WN 2: Calculation of VAT on Import at custom point for Luxury cars:
Particulars Amount
Qty
Import price (16*2,000,000*1.6015) 51,248,000
Transportation expenses IRs. 200,000 * 1.6015 320,300
Insurance premium 200,000
Customs duty 41,376,000
Total taxable value for VAT 93,144,300
VAT paid at custom point 12,108,759
Track-laying tractors are VAT exempt items so no need to calculate input VAT on import

WN 3: Calculation of input tax credit on expenses:


Particulars Amount Rs.
Full tax credit:
Insurance for Car Rs. 200,000 * 13% 26,000
Proportionate tax
credit:
Mobile expenses Rs. 90,000*13%* 60%* 76.19% 5,349
Office Equipment Rs. 1,000,000 * 13%*76.19% 99,047
Total 130,396

WN 4: Reverse VAT on construction of structures for commercial use through VAT


Unregistered builders.
As per section 8(3), if a building, apartment or similar structures are constructed for
commercial purposes from VAT-unregistered builders, then VAT should be deposited as if
it was constructed from VAT-registered person.
Total cost of construction 6,000,000
VAT 780,000

8) LUC Group Nepal is an emerging business house established by group of clever business minds. It
carries diversified business activities under same permanent account number and group motto is halo
to hydro. Business activities of the group for the month of Bhadra 20X-73 is as follows:
a. Account receivable from a client for contract completed, approved and invoiced during
the month Rs. 3,655,434.
b. Sales of cellular mobile phone during the month Rs. 1,255,660 of which amount Rs.
456,360 was sold to non registered customers.
c. Fruits and vegetable sales during the month Rs. 5,34,600 of which Rs. 1,53,600 was sales
of packed garlic in stock since last 3 months.
d. Deep cycled lid acid battery sales Rs. 1,256,400 out of total Rs. 3,25,600 was sold to Safa
Tempo workshop with recommendation from Ministry of Population & Environments.
e. Industrial machineries for bakery, confectionary & distillery falling under Harmonized
code 84.39.10.00, 8439.20.00 and 8439.30.00 sold during the month Rs. 55,24,300

The Institute of Chartered Accountants of Nepal 393


CAP-II Paper 7 - Income Tax and VAT
f. The group has following purchase/Import during the month exclusive of VAT, where
applicable
• Cement, Iron Nails, Bricks Rs. 21,22,456
• Cellular Mobile & its parts Rs. 16,56,400
• Battery Rs. 7,85,900
• Tower cranes for construction use Rs. 55,60,300
• Fruits and Vegetable Rs. 4,26,300
• Bakery, confectionary & distillery machineries (falling under Harmonized code
8439.10.00, 8439.20.00 and 8439.30.00) for trading purpose Rs. 60,65,480
• Statutory & internal audit fees Rs. 4,75,000
• Computer for official use Rs. 3,25,400
• Stationeries Rs. 5,25,600
• Purchase of car for group president Rs. 40,00,000
g. During the same month internal auditor of the group submitted their report to the
management that reveals:
• Purchase book of VAT for the month of Shrawan 20X-72 was over casted by Rs.
87,500.
• No credit claim for the VAT amount Rs. 56,980 paid to department of roads for
auction purchase of road roller during the month of Kartik 20X-72.
• No adjustment for batteries used for office purpose amounting costing Rs. 45,600
during the month of Jestha 20X-73.

Required: (June 2017, 8+2=10)


i) VAT statement for the month of Bhadra 20X-73 and VAT payable by the group assuming there was
no opening VAT credit.

Answer:
I. Calculation of Output VAT
Taxable Taxable
Particulars Exempt Total
@13% @0%
Construction Revenue billed
3,234,897 3,234,897
(3655434/1.13)
Sales of Cellular mobile phone 1,255,600 1,255,600
Fruits & Vegetable 153,600 381,000 534,600
Battery Sales 930,800 325,600 1,256,400
Industrial Machineries 5,524,300 5,524,300
Deemed sale of battery self use
during Jestha 20X-73 reported by 45,600 45,600
internal auditor
Total 5,620,497 325,600 5,905,300 11,851,397
Tax Rate 13% -

The Institute of Chartered Accountants of Nepal 394


CAP-II Paper 7 - Income Tax and VAT
Tax collection 730,665 - 730,665

Ratio of Taxable Sales and Non-taxable Sales

Total Transaction 11,851,397


Taxable Transaction 5,946,097 50.17%
Exempt Transaction 5,905,300 49.83%

II. Calculation of Eligible Input Tax Credit

Partial/
Particulars Full No Total
Proportionate

Cement, iron, nail and bricks 2,122,456


Cellular mobile 1,656,400
Battery 785,900
Statutory & internal Audit
475,000
fees
Computer for office 325,400
Stationeries 525,600
Car Purchase (4000000*0.4) 1,600,000
Total 4,564,756 2,926,000 -
VAT paid on Purchase 593,418 380,380 -
Credit eligible VAT Ratio Full 50.17% -
Credit eligible VAT
593,418 190,845 - 784,263
amount
Note:
Fruits and vegetables, machineries and tower crane are non VAT items listed in Schedule 1. Therefore,
VAT is not paid at the time of purchase

III. Calculation of VAT Payable/(Receivable)


VAT Collected on sales ("I" above) 730,665
VAT Paid on Purchase ("II" above) 784,263
VAT payable/(credit forward) (53,598)
Opening VAT Credit -
Adjusted VAT credit (Road Roller Purchased from DOR during Kartik 20X-
(56,980)
72)
Total VAT credit forwarded for Aswin 20X-73 (110,578)

ii) Discuss about VAT refundable, if any.


Answer
As per Sec. 24 of Value Added Tax Act, 2052, unless there is export sales exceeding 40% of total sales
during any month, any excess receivable VAT (i.e. when input VAT exceeds Output VAT) of a month shall
be utilized to set off any payable VAT of next four months. Any remaining excess receivable VAT after

The Institute of Chartered Accountants of Nepal 395


CAP-II Paper 7 - Income Tax and VAT
such action for consecutive four months can be refunded by making an application by the concerned
taxpayer along with the VAT return of fourth month or any months thereafter.

In case of export sales exceeding 40 percent of total sales during any month, the taxpayer may make an
application to refund any excess receivable VAT of such month along with the VAT return of the same
month.

Conclusion
In the given case, there are no export sales. Therefore, VAT can be refunded only after applying the
excess to payable VAT of next four months and if there still remain excess VAT.

9) ABC Ltd. was incorporated and registered with VAT on 2065. During the month of Jestha 20X-73 the
company had following transaction:
Purchases/Expenses:
Purchase of raw materials Rs. 150,000 (Exclusively used for taxable transaction)
Electricity Expenses Rs. 125,000
Cold Drinks Rs. 25,000
Motorbike Rs. 250,000
Printing & Stationeries Rs. 50,000
Services availed from America Rs. 150,000
Sales:
Taxable sales -Rs. 500,000
Exempted Sales -Rs. 10,000
Export Sales -Rs. 100,000

All purchases, expenses and sales are exclusive of VAT.

The company had opening balance of VAT credit of Rs. 2,750,000 out of which 2,000,000 is credit prior
to financial year 20X-68/X-69.
Required:
(Dec 2016, 5+2=7)
a. Vat payable or vat credit available for the month of Jestha 20X-73 without setting off opening vat
credit. Write the due date of filling vat return?

Answer:
VAT payable or VAT credit available for the month of Jestha 20X-73 without setting off opening vat credit
Particulars Amount
A. VAT Collected on Sales (Refer "I" below) 65,000
B. Eligible VAT Credit (Refer "II" below) 57,861
C. VAT Credit on VAT paid under Reverse Charge Principle (Refer "III" Below) 19,180
D. VAT Payable/(Receivable) [A-B-C] (12,041)

The Institute of Chartered Accountants of Nepal 396


CAP-II Paper 7 - Income Tax and VAT
Working Notes
I. VAT Collected on Sales

Particulars VAT Note


Amount
Taxable Domestic
500,000 65,000 Tax rate of 13%
Sales
Exempted Sales 25,000 - VAT is not applicable
Assumed to be that of taxable
Taxable Export Sales 100,000 -
items, taxed @0%
Total Output VAT for the
65,000
month

Ratio of Taxable Sales to Total Sales

Taxable Sales 600,000


Non Taxable Sales 10,000
Total Sales 610,000
Percentage of Taxable Sales 98.36%

II. Eligible VAT Credit on Purchases during the month

Eligible
Particulars VAT Amount for Notes
Amount
Credit
Full credit, directly
Purchase of Raw Materials 150,000 19,500 19,500
related to taxable sales
Electricity is exempt
Electricity Expenses 125,000 - -
item
No VAT credit,
Cold drinks 25,000 3,250 -
beverage item (rule 41)
Proportionate VAT
31,967 Credit, as the use is for
Motorbike 250,000 32,500
(32,500*98.36%) both taxable and non
taxable output
Proportionate VAT
6,393 Credit, as the use is for
Printing and Stationery 50,000 6,500
(6,500*98.36%) both taxable and non
taxable output
Total Eligible VAT Credit 57,861

The Institute of Chartered Accountants of Nepal 397


CAP-II Paper 7 - Income Tax and VAT

III. VAT Credit on VAT paid under Reverse Charge Principle as per Sec . 8 (2)

Eligible Amount
Particulars VAT Notes
Amount for Credit
Proportionate VAT Credit as the input's
Services direct relation with any output cannot be
19,180
availed from 150,000 19,500 established
(19,500*98.36%)
America It is assumed that VAT is already paid
in respect of receipt of this service
Total 19,180
Note: X Ltd must have deposited value added tax of Rs. 19,500 in respect of services availed from UK, in
order for it to be able to claim credit of such VAT

Due date of filing VAT Return


VAT return must be filed within 25 days of end of tax period.

b. Is it wise to shut down the business of company based on the information provided by the company?
Answer:
It is not wise to shut down the business of company because the company has huge balance of VAT credit
which is not eligible to get refund. VAT refund is permissible under section 24 of Value Added Tax Act,
2052 to registered person. The refund application shall be filed within three years of the end of tax
period. The company had huge balance of VAT credit generated before three years; hence, advised to
continue their business and claim vat set off from future transaction.

10) Sujha Pvt. Ltd. imports the Honda cars and track-laying tractors and sells in Nepal. It has the following
transactions and information relevant to the VAT calculation in the month of Bhadra, 20X-73.
a. Honda cars sales invoices of Rs. 30 million were issued to VAT registered customers and invoices
of Rs. l0 million were issued to individual customers (Not registered in VAT).
b. Track-laying tractors sales invoices totaling Rs. 10 million were issued to individual customers that
were not registered for VAT.
c. The company offers a 5% discount for prompt payment.
d. Import price of Honda cars was IRs. 15 million, transportation expense upto custom point was IRs
100,000 (Paid to Indian transporter) and insurance premium Rs. 100,000 paid to Nepal insurance
company. One of the imported cars valued costing Rs. 50 lakhs were used itself for the pickup and
drop of its executive director.
e. Import price of Track-laying tractors was IRs. 5 million, transportation expense up to custom point
was IRs 50,000 (Paid to Indian transporter) and insurance premium Rs. 50,000 paid to Nepal
insurance company.
f. During the month, Rs. 30,000 was spent on mobile telephone calls, of which 40% relates to private
calls.
g. On 30 Bhadra, an office equipment was purchased for Rs. 7,00,000.
h. Custom duty was 80% in the case of Honda cars and 1 % in case of tractors.
i. All of the figures except IRs are exclusive of VAT.

The Institute of Chartered Accountants of Nepal 398


CAP-II Paper 7 - Income Tax and VAT

Calculate the amount of VAT payable/receivable by the company for the month ended Bhadra 20X-73.
(Dec 2016, 10 Marks)

Answer
Calculation of VAT payable/receivable for the month ended Bhadra 20X-73.

Particulars Amount Rs.


Output tax (WN 1) 5,850,000
Less: Input tax for imported cars (WN 2) (5,676,840)
Input tax credit on expenses (WN 3) (89,371)
Input tax on deemed sales (WN 4) (212,732)
Net VAT receivable 128,943

WN 1: Calculation of output tax

Particulars Cars’ sale Rs. Tractors sale Rs. VAT amount Rs.
Sales to VAT registered Customers 30 million --- 3,900,000
Sales to Unregistered Customers 10 million 10 million 1,300,000
Deemed Sales- Owned Consumption 5 million --- 650,000
Total 45 million 10 million 5,850,000
Proportionate sales ratio 81.82 % 18.18 %
Note 1: Track-laying Tractor is VAT exempt as per schedule 1 of VAT Act, 2052.
Note 2: Prompt Payment discount is deemed to be given after issuance of invoice.
Note 3: As per Value Added Tax Directive, 2069 (Updated 2076), VAT shall be collected at cost at the time
of self-use of goods. Self-use includes using the asset of one form into another form.

WN 2: Calculation of VAT on Import at custom point for Honda cars:


Particulars Amount Rs.

Import price IRs. 15 million * 1.6015 (Marks shall be allowed even if 24,000,000
1.6 rate is used taking the level of students)
Transportation expenses IRs. 100,000 * 1.6015 160,000

Insurance premium 100,000

Total value for custom 24,260,000

80 % custom duty 19,408,000

Total taxable value for VAT 43,668,000

VAT paid at custom point 5,676,840

Track-laying tractors are vat exempt items. Therefore, there is no need to calculate input vat on import.

The Institute of Chartered Accountants of Nepal 399


CAP-II Paper 7 - Income Tax and VAT
WN 3: Calculation of input tax credit on expenses:
Particulars Amount Rs.

Full tax credit:

Insurance for Car Rs. 100,000 * 13% 13,000

Proportionate tax credit:

Mobile expenses Rs. 30,000*13%* 60%* 81.82% 1,915

Office Equipment Rs. 7,00,000 * 13%*81.82% 74,456

Total 89,371

WN 4: VAT credit on deemed sale (for own use in business)


VAT Sale invoice is issued whether it is sold to others or use for business, so, input tax for the car is
included in import and collection is included in output (Sales to VAT registers). Input can be claimed for
the car as follows:

Debit Adjustment for own used car


Cost price Rs. 5,000,000
VAT Rs. 650,000
Eligible for input Rs. 212,732 (650,000*40%*81.82%)

11) Ustab publication house is VAT registered firm, it prints the books and copy. It has the following
transactions excluding VAT for the month of Falgun, 2072.
Particulars Cost/ Sales Price Rs.
Purchase
Printing machine 30,000,000
Car for official purpose 3,000,000
Car for directors household purpose 2,600,000
Petrol to run generator set 500,000
Petrol to run car 200,000
Diesel for transport on truck 400,000
Papers Purchased 5,000,000
Motorcycle for official purpose 280,000
Advertisement 200,000
Royalty paid to the writers 2,500,000
Sales
Copy 6,000,000
Stationary 4,000,000

The Institute of Chartered Accountants of Nepal 400


CAP-II Paper 7 - Income Tax and VAT
Books 5,000,000
Patro (Calendar) 1,000,000
You are required to calculate the amount of input tax credit and VAT to be collected on sales. If Input
tax credit is not set off against the collection, when it can claim for refund? (June 2016, 10 Marks)

Answer:
A. Output VAT (or VAT Collected on Sales)

Particulars Amount VAT Note


Copy 6,000,000 780,000 Tax rate of 13%
Stationery 4,000,000 520,000 Tax rate of 13%
Books 5,000,000 - VAT exempt
Patro 1,000,000 - VAT exempt
Total Output VAT for the month 1,300,000

B. Eligible VAT Credit on Purchases during the month

Eligible
VAT Amount
Particulars Amount Notes
paid for
Credit
Proportionate VAT Credit, as
Printing Machine 30,000,000 3,900,000 2,437,500 the use is for both taxable and
non taxable output
Partial VAT credit @ 40% of
VAT paid, and since the
Car for official person deals both in taxable
3,000,000 390,000 97,500
purpose and non taxable items,
proportionate credit shall also
be claimed
Car for directors No VAT credit, not related to
2,600,000 338,000 -
household purpose taxable transaction
Petrol to run
500,000 65,000 40,625 Proportionate VAT Credit
generator
No VAT Credit, consumption
Petrol to run car 200,000 26,000 -
of petroleum (Rule 41(1)
Diesel to Transport
400,000 52,000 32,500
on truck
Papers 5,000,000 650,000 406,250 Proportionate VAT Credit, as
Motorcycle for the use is for both taxable and
280,000 36,400 22,750
official purpose non taxable output
Advertisement 200,000 26,000 16,250

The Institute of Chartered Accountants of Nepal 401


CAP-II Paper 7 - Income Tax and VAT
Royalty paid to
2,500,000 - - VAT exempt
writers
Total Eligible VAT Credit 3,053,375

C. Payable/ (Receivable VAT)

Payable/ (Receivable VAT) [A-B] (1,753,375)

Working Note

1. Ratio of Taxable Sales (For Proportionate VAT Credit)


Taxable Sales 10,000,000
Non Taxable Sales 6,000,000
Total Sales 16,000,000
Percentage of Taxable Sales 62.50%

12) Liza enterprise, a dealer of Toyota cars in Nepal, imported 5 cars for the Value @ US $ 50,000 on 12
Jestha, 20X-72. The payment was made through the Maya Bank Ltd. on exchange rate of 1 US$ = Rs.
103.60. Others information are as follows:
a. Transportation cost upto Calcutta, India : @ US $ 2,000 (Payment made to transporter @ Rs. 102.40
per US $)
b. Marine insurance paid to Nepal Insurance Company Ltd. Rs. 33,900
c. Transportation cost from Calcutta to Birgunj Customs Office IRs. 24,000
d. Driver's salary for the transportation from Birgunj to Kathmandu @ Rs. 6,000
e. Petrol cost from Birgunj to Kathmandu Rs. 9,040 with tax invoice
f. The custom duty is 80 %, ignore the excise duty and other taxes, duties
g. The company sells the cars adding 12 % on above cost.
h. Opening VAT account: Rs. 34,000 payable and it has no opening stocks.
i. The exchange rate published by Nepal Rastra bank on the day of clearing the cars at custom: Buying
rate is Rs. 102.40 per US $ and selling rate is Rs. 103.00 per US$

You are required to calculate the VAT payable/receivable on the following situations:
(Ignore Import Service Fee): (Dec 2015, 6+4=10)
a. The Enterprise has used a car for own business and sold the remaining cars during the month.
Answer

Output VAT for 4 cars (Sale) 5,650,472 (13% of 10,866,290)*4


Output VAT for 1 car (Self Consumption) 1,261,266 (13% of 9,702,045)*1
Input VAT for 5 cars (WN 1 & 2) 6,286,435 (1,256,507+780)*5
Input VAT for a car (Self Consumption) 504,506 (1,261,266)*40%
VAT Payable (Output-input) 120,797

The Institute of Chartered Accountants of Nepal 402


CAP-II Paper 7 - Income Tax and VAT
Note:
VAT paid on petrol cannot be claimed as credit.

b. The Enterprise sold 4 cars and one remains in stock. Round up the fraction.
Answer

Output VAT for 4 cars (WN 2) 5,650,472 (13% of 10,866,290)*4


Input VAT for 5 cars (WN 1 & 2) 6,286,435 (1,256,507+780)*5
VAT receivable (Input-Output) 635,963

Note:
VAT paid on petrol cannot be claimed as credit.

Working Notes
WN 1
Calculation of per unit VAT paid at Custom
Particulars Amount (Rs.)
Imported cost 5,150,000 (50,000*103)
Insurance 6,000 (33,900/5=6,780/1.13)
Transportation up to Calcutta 206,000 (2000*103)
(24,000/5=
Transportation Calcutta to custom 7,687
4,800*1.6015)
Transaction value 5,369,687
Custom duty @ 80 % 4,295,750
Taxable value 9,665,437
VAT @ 13 % 1,256,507
WN 2: Calculation of per unit cost, sales price and VAT
Particulars Amount (Rs). VAT (Rs.)
Imported cost (50,000*103.60) 5,180,000
Insurance (33,900/5=6,780/1.13) 6,000 780
Transportation up to Calcutta
204,800
(2000*102.4)
Transportation Calcutta to custom
7,687
(24,000/5= 4,800*1.6015)
Custom Duty and VAT (WN 1) 4,295,750 1,256,507
Driver's salary 6,000 Per Car
Petrol cost (9,040/5) 1,808 Input tax not allowed
Total Cost 9,702,045 1,257,495
Profit addition (12 % on 97,03,037) 1,164,245
Selling price per unit 10,866,290
Output VAT per Car 1,412,618

The Institute of Chartered Accountants of Nepal 403


CAP-II Paper 7 - Income Tax and VAT
13) M/S Prakarti Limited is a VAT registered company and registered with company Registrar's Office in
Nepal. The company is producing equal number of both vat exempted & vat attractive goods on each
day of production. The company has following transaction during the month of Baishakh, 20X-72:
Opening VAT credit Rs. 125,000
Sales:
Vat exempted sales Rs. 4,000,000
Vat attractive sales Rs. 6,000,000
The company exports 59% of VAT attractive and VAT exempted sales
Purchases of raw materials Rs. 6,000,000
Other office expenses incurred Rs. 450,000
Cost of soft drinks for office party Rs. 200,000
Purchase of car Rs. 2,500,000
Purchase of two-wheeler Rs. 225,000
Purchase of petrol for car & two-wheeler Rs. 185,000
The given purchases & expenses are excluding applicable VAT. Other office expenses include one
invoice of Rs. 100,000 pertaining to the period of Chaitra 20X-70.
Required: (Dec 2015, 2+3+4+1=10)
a. Compute the VAT payable on sales, VAT paid on purchase & expenses incurred by the company
and VAT credit available to the company.
Answer:
VAT Payable (i.e. collected) on Sales

Particulars Total Value VAT Collected Note


VAT Attractive Sales:
Domestic 2,460,000 319,800 13 Percent
Export 3,540,000 - Zero percent
VAT Exempt Sales:
Domestic 1,640,000 - Exempt
Export 2,360,000 - Exempt
Total Output VAT 319,800

VAT paid on Purchases and Expenses

Particulars Total Value VAT Paid Note


Purchases of raw materials 6,000,000 780,000
Other office expenses incurred during the month Assumed
350,000
(Excludes purchase of Rs. 100,000 made for Chaitra) 45,500 that
Other office expenses incurred during Chaitra 20X-70 100,000 13,000 purchase is
Cost of soft drinks for office party 200,000 26,000 made from
Purchase of car 2,500,000 325,000 registered
Purchase of two wheeler 225,000 29,250 person
Purchase of petrol for car & two wheeler 185,000 24,050
VAT Paid on Purchases 1,242,800

The Institute of Chartered Accountants of Nepal 404


CAP-II Paper 7 - Income Tax and VAT

VAT Credit available to the Company


Ratio of Taxable Sales to Total Sales is 60%

VAT Eligible
Particulars Note
Paid Credit
Purchases of raw materials 780,000 468,000 Proportionate Credit
Other office expenses incurred
45,500 27,300 Proportionate Credit
during the month
The date from purchase
Other office expenses incurred
13,000 - exceeded one year, and hence,
during Chaitra 20X-70
no credit
Cost of soft drinks for office party 26,000 - No VAT Credit
40% of VAT paid assuming
used for administrative
Purchase of car 325,000 78,000
purpose, and to the extent of
taxable sales to total sales
Purchase of two wheeler 29,250 17,550 Proportionate Credit
Purchase of petrol for car & two
24,050 - No VAT Credit
wheeler
Eligible VAT Credit 590,850

b. Compute VAT refundable if any under the provision of VAT Act, 2052.
Answer

Particulars Amount
Total Output VAT 319,800
Eligible VAT Credit 590,850
Opening VAT Credit 125,000
VAT Payable/ (Receivable) (396,050)

14) OLIZ Trading Pvt. Ltd. trades in various vatable goods. The company sold goods on credit for 60 days
whereas creditors provide credit facility for 50 days. The company’s receivables on account of taxable
goods sold is Rs. 3,390,000 for the month of Kartik 20X-71. Input Tax credit of previous month is Rs.
21,000.
Details of taxable purchase of the company in the month of Kartik are as follows:
a. Total payable to Supplier for the Taxable purchase of the trading goods- Rs. 2,260,000
b. Total payable for computer and printer purchased Rs. 67, 800
c. Payable made for motorcycle purchased Rs. 226,000
d. Payable made to Hotel Rs. 24, 860 for dinner party organized for party meeting.
Find out the VAT amount paid on purchases and collected on sales. Also, workout amount of allowable
input Tax credit and VAT payable. (July 2015, 10 Marks)

The Institute of Chartered Accountants of Nepal 405


CAP-II Paper 7 - Income Tax and VAT
Answer:
Calculation of Output VAT
Particulars Total Value VAT Note
including VAT Collected
Receivable on account of Taxable Goods 3,390,000 390,000 3,390,000/1.13 *0.13
Total Output VAT 390,000
Calculation of Eligible Input Tax Credit
Particulars Total Value VAT Paid Eligible Note
including VAT VAT
Credit
Payable to Supplier for the Taxable purchase of the 2,260,000 260,000 260,000
trading goods
Payable for computer and printer purchased 67,800 7,800 7,800
Payable made for motorcycle purchased 226,000 26,000 26,000
Payable on account of dinner party 24,860 2,860 - No Credit
Total Eligible Input Tax Credit 293,800

Calculation of VAT Payable/(Receivable)


Particulars Amount
Total Output VAT 390,000
Total Eligible Input Tax Credit 293,800
Opening VAT Credit 21,000
VAT Payable/(Receivable) 75,200

15) Cycle Tyre Co. Ltd. of Biratnagar imports raw materials from India for manufacturing of BLAST brand
of bicycle wheels. It sells its product both in Nepal and India. During Magh, finance controller of the
company resigned and the company has not paid any amount of VAT nor has filed monthly VAT return
for Magh and Falgun and you are appointed as finance officer of the company during Chaitra.

The details of VAT related transaction of company are:

Month Magh Falgun


Particulars Taxable VAT Taxable VAT
Opening VAT credit 20,000
Domestic-purchase raw materials 21,000 2,730 30,000 3,900
Import of raw materials 280,000 36,400 400,000 52,000
Raw materials consumed out of domestic
purchased 18,900 2,457 27,000 3,510
Raw materials consumed out of imports 252,000 32,760 360,000 46,800
Diesel consumed 22,400 2,912 32,000 4,160
Diesel purchased 63,000 8,190 90,000 11,700
Payment to labour contractor for supply of
workers 36,400 4,732 52,000 6,760
Consumables used from stock 14,000 1,820 20,000 2,600

The Institute of Chartered Accountants of Nepal 406


CAP-II Paper 7 - Income Tax and VAT
R&D service procured from Germany
(Unregistered) 300,000
Value of finished goods produced 900,000 1,200,000
Sales domestic 810,000 345,600
Sales export - 734,400
Required: (July 2015/June 2022, 10 Marks)
Prepare a memo to be forwarded to the Board of Directors of the company referring the relevant
provisions of VAT Act, amount indicating VAT payable, refund receivable and amount of VAT credit
available for each month.

Answer
Office Memo

Subject: Internal Memorandum

The Board of Directors,


Cycle Tyre Co. Ltd.

Based on the attached VAT computation sheet, the company owes Rs. 41,438 as VAT obligation payable
to Government of Nepal during the month of Magh. In addition to Rs. 41,438; the company also owes
Rs. 39,000 as VAT payable in respect of purchase of service from outside Nepal under reverse charging
principle as per Sec. 8 (2) of Value Added Tax Act, 2052 as soon as possible.

Further, the company has receivable VAT of Rs. 56,732 during the month of Chaitra.

The detailed computation of VAT payable, refund receivable and amount of VAT credit available for each
month is presented in Annex 1, Annex 2 and Annex 3 below.

Annex 1: Calculation of Payable/(Receivable) VAT

Particulars Magh Falgun


Total Output VAT for the month 105,300 44,928
Total Eligible Input Tax Credit for the purchase during month 52,052 74,360
Opening VAT credit 20,000 -
Input Tax credit on R&D service procured from Germany 39,000
VAT Payable/(Receivable) 33,248 (68,432)

Annex 2: Calculation of Output VAT

Magh Falgun
Particulars Note
Value VAT Value VAT
Sales domestic 810,000 105,300 345,600 44,928 @13%
Sales export - - 734,400 - @0%

The Institute of Chartered Accountants of Nepal 407


CAP-II Paper 7 - Income Tax and VAT
Output VAT is
collected on
Value of finished goods produced 900,000 - 1,200,000 -
Sales and not
on Production
Total Output VAT for the
105,300 44,928
month

W.N.3: Calculation of Eligible Input Tax Credit on Purchases made during the month, ignoring any VAT
paid u/s 8 (2), 12Ka or 15 (3)

Magh Falgun
Particulars Note
Value VAT Value VAT
Domestic- purchase raw
21,000 2,730 30,000 3,900 Full VAT credit
materials
Import of raw materials 280,000 36,400 400,000 52,000 Full VAT credit
Raw materials consumed out Input Credit is
18,900 - 27,000 -
of domestic purchased claimed on purchase
Raw materials consumed out and not on
252,000 - 360,000 -
of imports consumption.

Diesel consumed 22,400 - 32,000 -

Diesel purchased 63,000 8,190 90,000 11,700 Full VAT credit


Payment to labour contractor
36,400 4,732 52,000 6,760 Full VAT credit
for supply of workers
Input Credit is
Consumables used from claimed on purchase
14,000 - 20,000 -
stock and not on
consumption
Total Eligible Input Tax Credit for
52,052 74,360
the purchase during month

16) M/s RK Trading Private Limited is a Value Added Tax Registered company. It is engaged in the business
of importing wines & liquor and selling in local market. The company is in confusion to take tax credit
on procurement of wines and liquor as per Value Added Tax Act/Rules, Advise. What will be your
answer if the company does not have such primary business? (Dec 2014, 5 Marks)

Answer:
As per Rule 41, there shall be no VAT credit on consumption of liquor, even if the person deals in taxable
transaction otherwise than in such circumstances when the main business of the person is to deal in
liquor.

The Institute of Chartered Accountants of Nepal 408


CAP-II Paper 7 - Income Tax and VAT
Conclusion
Since the main business of M/s RK Trading Private Limited is to deal in liquor items, the person is eligible
to claim full VAT credit on the VAT paid on import of wines and liquor.

If M/s RK Trading Private Limited does not sell wines and liquor, VAT paid on such wines and liquor cannot
be claimed as input tax credit.

17) Ilam Tea Pvt. Ltd. is a tea processing and producing industry, located in Ilam. It sells the tea within
Nepal. It has the following transaction in the month of Bhadra, 20X-71.
Particulars Transactions Amount (Rs.)
Sales:
VAT registered party 20,00,000
VAT Non-registered party 5,00,000
Purchase:
Wages and others that are VAT exempt 12,00,000
Compute the VAT amount payable/receivable by the Tea Industry with reference to the VAT Act, 2052
for the Month. Assume no VAT on opening. (Dec 2014, 5 Marks)

Answer:
Output VAT 325,000
(13% of 25 Lakhs)
Eligible Input VAT credit -
(As all purchases are VAT exempt)
Payable VAT for the month 325,000

18) JivanJyoti Gases Industries Pvt. Ltd. is engaged in manufacturing of Oxygen & Nitrogen Gases. The
transactions of the industry for the month of Kartik, 20X-70 is as follows:
Sales Details (Excluding
VAT)
Oxygen Gases Nitrogen Gases
Sold to Date
(in Rs.) (In Rs.)
Teaching Hospital 20X-70.07.02 50,000 -
Ram Metal Works 20X-70.07.05 40,000 25,000
B & B Hospital 20X-70.07.10 55,000 -
Ghorahi Cement Industries 20X-70.07.14 125,000 250,000
Sine Hydro Corporation 20X-70.07.25 150,000 30,000
Total Sales for the month 420,000 305,000

Purchase Details (Excluding VAT)


Particulars Amount ( in Rs.)
Diesel purchase for Generator Running 90,000

The Institute of Chartered Accountants of Nepal 409


CAP-II Paper 7 - Income Tax and VAT
Telephone Chagres 18,000
Purchase of Vehicle for General Manager 2,250,000

The Industries also paid the consultancy fee amounting to IRS 50,000 to Sigma Consultancy, New Delhi,
India.
VAT receivable as per Ashwin, 20X-70 VAT Return is NRS 28,000.
You are requested to compute the VAT collected on Sales, VAT Paid on Purchase and Net VAT
payable/receivable for the Kartik end 20X-70. (June 2014, 10 Marks)

Answer:
Calculation of Output VAT
Particulars Sales Value Output VAT Reason
Sales of Oxygen 420,000 - Exempt
Sales of Nitrogen 305,000 39,650 @13%
Total Output VAT 39,650
Taxable Sales 42.07%
Non-taxable sales 57.93%

Calculation of Eligible Input Tax Credit

Amount VAT Eligible VAT


Particulars Notes
(in Rs.) paid Credit
Diesel purchase for Generator
Running 90,000 11,700 4,922 Proportionate
Telephone Chagres 18,000 2,340 984 credit
40% of VAT
paid in
Purchase of Vehicle for General proportion of
Manager 2,250,000 292,500 49,222 taxable sales
Total Eligible Input VAT 55,128

Calculation of Net VAT Payable or Receivable


Total Output VAT 39,650
Total Eligible Input VAT 55,128
VAT Payable/(Receivable) (15,478)

19) AMP Wire Industries Pvt. Ltd. purchased the following in the month of Kartik 20X-70.
Particulars VAT Paid (Rs.)
Used photocopy machine 1,30,000
Brand new Fax machine 2,600
Used generator set 3,90,000
Import of Brand new generator set 5,20,000
Required:
Calculate the VAT amount to be claimed for offset. (Dec 2013, 5 Marks)

The Institute of Chartered Accountants of Nepal 410


CAP-II Paper 7 - Income Tax and VAT

Answer
AMP Wire Industries Pvt. Ltd. deals in taxable transaction, therefore, there shall be full VAT credit on VAT
paid on purchases that are intended to be used for taxable output. As such, the amount to be claimed
for set off is as follows:
Particulars VAT paid
Used photocopy machine 130,000
Brand new Fax machine 2,600
Used generator set 390,000
Import of Brand new generator set 520,000
Total 1,042,600

20) EFG Garment Pvt. Ltd. had the following transaction in the month of Shrawan 20X-70. Calculate the
VAT payable/receivable from the information below. (Dec 2013, 10 Marks)
Particulars Amount (Rs.)
Local sale 50,00,000
Export sale 1,00,00,000
Purchase of clothes, Stitching, Packing Materials, loose tools for 81,00,000
machineries
Special packing for export 4,00,000
Payment of consultancy charges abroad 5,00,000
Purchase of bus for staff transportation 20,00,000
Purchase of motorcycle of hire purchase 4,00,000
Telephone expenses 76,000
Purchase of diesel for generator 80,000
Purchase of diesel for bus 24,000
Purchase of petrol for motorcycle 30,000
Purchase of computers 90,000
Purchase of soft drinks 12,000

Additional information:
Opening VAT receivable for the month was Rs. 91,560. Diesel for bus for Rs. 16,000 and soft drinks for
Rs. 6,000 was purchased through abbreviated tax invoice. Items above are exclusive of VAT.
Answer

A. Output VAT (or VAT Collected on Sales)


Particulars Amount VAT Note
Domestic Sales 5,000,000 650,000 @ 13%
Export Sales 10,000,000 - @ 0%
Total Output VAT for the month 650,000

B. Eligible Input tax credit on purchases during the month

The Institute of Chartered Accountants of Nepal 411


CAP-II Paper 7 - Income Tax and VAT
Eligible
VAT
Particulars Amount Amount Notes
paid
for Credit
Purchase of clothes, Stitching, 8,100,000
Packing Materials, loose tools 1,053,000 1,053,000 Full VAT credit
for machineries
Special packing for export 400,000 52,000 52,000 Full VAT credit
40% of VAT paid
can be claimed as
Purchase of bus for staff credit, as the bus is
2,000,000 260,000 104,000
transportation used for
administrative
purpose
Purchase of motorcycle of hire 400,000
52,000 52,000 Full VAT credit
purchase
Telephone expenses 76,000 9,880 9,880 Full VAT credit
Purchase of diesel for generator 80,000 10,400 10,400 Full VAT credit
Purchase of diesel for bus 24,000 3,120 3,120 Full VAT credit
Purchase of petrol for 30,000
3,900 - No VAT Credit
motorcycle
Purchase of computers 90,000 11,700 11,700 Full VAT credit
Purchase of soft drinks 12,000 1,560 - No VAT Credit
Total Eligible VAT Credit 1,296,100

C. Input VAT Credit on VAT paid as per Sec. 8 (2), 12Ka and 15 (3)
Eligible Amount
Particulars VAT Notes
Amount for Credit
Payment of Assuming that the VAT in
Consultancy 500,000 65,000 65,000 respect of this service is
Charges abroad already paid
Total 65,000

D. Opening VAT credit (excess of


Previous month, eligible for set off) -

E. Payable/ (Receivable VAT)


[A-B-C-D] (711,100)

21) Gramin Krishi Sewa Pvt. Ltd. sales vegetables and fruits. During the month of Baishakh 20X-70
following sales and purchase made. Compute the net Value Added Tax payable under the Value Added
Tax Act, 2052. (June 2013, 8 Marks)
Items Purchase (Rs.) Sales (Rs.)
The Institute of Chartered Accountants of Nepal 412
CAP-II Paper 7 - Income Tax and VAT
Green Tea 50,000 57,500
Barley 40,000 48,000
Garlic 30,000 37,500
Black Tea 80,000 100,000
Bhakti Fish 150,000 175,000
Millet 45,000 49,500
Paneer 70,000 78,400
Curd 60,000 64,375
Telephone Bill 5,000
Stationery Purchase 20,000

Answer:
Calculation of Output VAT
Sales VAT
Particulars Reason
Value amount
Green Tea 57,500 7,475 Taxable, & @13%
Barley 48,000 - VAT exempt
Garlic 37,500 4,875 Taxable, & @13%
Black Tea 100,000 13,000 Taxable, & @13%
Bhakti Fish 175,000 22,750 Taxable, & @13%
Millet 49,500 - VAT exempt
Paneer 78,400 10,192 Taxable, & @13%
Curd 64,375 8,369 Taxable, & @13%
Total Output VAT 66,661

Taxable Sales 512,775 84.02%


Non-taxable Sales 97,500 15.98%
610,275

The Institute of Chartered Accountants of Nepal 413


CAP-II Paper 7 - Income Tax and VAT
Calculation of Eligible Input Tax Credit
Particulars Purchase Value VAT paid Eligible Credit Reason
Green Tea 50,000 6,500 6,500 Full VAT Credit
Barley 40,000 - - VAT Exempt
Garlic 30,000 3,900 3,900 Full VAT Credit
Black Tea 80,000 10,400 10,400 Full VAT Credit
Bhakti Fish 150,000 19,500 19,500 Full VAT Credit
Millet 45,000 - - VAT Exempt
Paneer 70,000 9,100 9,100 Full VAT Credit
Curd 60,000 7,800 7,800 Full VAT Credit
Telephone Bill 5,000 650 546 Proportionate Credit
Stationery Purchase 20,000 2,600 2,185 Proportionate Credit
Total Eligible VAT Credit 59,931

Calculation of VAT payable or Receivable


Particulars Amount
Total Output VAT 66,661
Total Eligible VAT Credit 59,931
VAT Payable/(Receivable) 6,730

22) There are some cases where Value Added Tax (VAT) paid on purchase is not allowed- no credit, even
if output of registered person is VAT attractive. Mention those cases as prescribed under Value Added
Tax/Rules. (Dec 2012, 5 Marks)

Answer
The circumstances when Value Added Tax paid on purchase is not allowed, even if output of registered
person is VAT attractive are as follows:
a. In case there is no proper Value Added Tax invoice (other than abbreviated tax invoice) to
substantiate VAT paid except when VAT is paid under import (where import documents substitute
VAT invoice), u/s 8 (2), 12Ka and 15 (3).
b. In case such purchase is listed as below and the main business of the person is not dealing in the
items listed below:
(i) Entertainment expense
(ii) Beverages
(iii) Liquor
(iv) Petrol used for vehicles

23) M/s Kiran Distributors is engaged in the wholesale business of various types of noodles. The firm is
registered in VAT. The firm has taken a loan of Rs. 10 lacs from M/s Saraswati Bank for purchasing a
delivery van. Bank has paid the total amount for the purchase of Van but has kept the ownership of
van in the name of the bank till the loan is fully repaid by Kiran Distributors. Kiran has to pay Rs. 25,000
per month towards the principal and interest to the Bank as part of their loan agreement. Rs. 130,000

The Institute of Chartered Accountants of Nepal 414


CAP-II Paper 7 - Income Tax and VAT
was paid as VAT during the purchase of van. By mentioning the relevant provision of the Act, can Kiran
Distributors claim the input tax credit? (Dec 2012, 4 Marks)

Answer:
As per section 17(5ka) of Value Added Tax Act, 2052, if a tax payer purchases or imports capital goods
under a loan agreement under financial lease, then input tax paid can be claimed by the concerned tax
payer.

In this case, although the legal title still vests with the bank, the firm can take the input tax credit as per
section 17(5ka).

24) PQR Ltd's Stock was damaged by fire on 20X-68.05.20. PQR Ltd. enquires on taking the input tax credit
paid on the lost stock. By citing the relevant provisions of the act and rules, advise PQR Ltd. (Dec 2012,
3 Marks, CA Inter)

Answer:
Nepal adopts invoice-credit method. That means, a taxpayer claims input tax credit on every purchase
that are intended to be used for taxable output. When the related stock is lost due to any reason, the
taxpayer shall comply with provisions of VAT law to get the requisite support from tax officer to
legitimatize the credit claimed at the time of purchase of such stocks.

The related provision for the same is mentioned in Rule 39Ka of Value Added Tax Regulation, 2053, which
is extracted as under:

Application for Input Tax Credit & Time Limit for such application
Where there is loss of goods by fire, theft, accident, accidental damages, or terror and it compels a person
to write off the goods (assets) or sale such goods at lower selling rate, the person shall make an
application in writing to respective Inland Revenue Office along with evidence within 30 days of
happening of such events.

Investigation and permission to claim as input tax credit


The tax office shall investigate the matter and finalize the quantum of tax credits to be allowed. On the
basis of such investigation, the tax office may allow the taxpayer to claim Input Tax Credit of VAT paid on
such assets (goods).

Right to claim Input tax Credit without confirmation from Tax Officer
In case the goods are insured, the tax office may allow the taxpayer to claim Input Tax Credit on such
goods to the extent of compensation paid by the Insurance Company, without any investigation from tax
officer.

The Institute of Chartered Accountants of Nepal 415


CAP-II Paper 7 - Income Tax and VAT
Conclusion
In the given case, since the stock is lost by fire, procedure of Rule 39Ka shall be followed to obtain support
from tax officer to legitimatize the credit claim made by PQR Ltd. at the time of purchase of such stock.
For this, PQR Ltd. shall file an application within 30 days of 20X-68.05.20, along with the substantiating
documents, after which the tax officer will investigate the matter and legitimize the credit claimed by
PQR Ltd. as per Rule 39Ka.

25) Mercantile Limited is providing service in the field of information technology and its sales in the month
of Chaitra 20X-67 was Rs. 5 million. Out of total sales Rs. 3 million was export. The company has got
some service from Star Inc. registered and working in USA. Cost paid for the services was Rs. 2 million
and the company paid Rs. 260,000 in IRO as reverse charging payment. Total purchases for the month
of Chaitra were Rs. 1 million and it has also incurred the following expenditures:
a. Telephone charges Rs. 10,000
b. Petrol for vehicle Rs. 5,000
c. Printing and stationery Rs. 5,000
The company has taken VAT bill for all the above purchases. You are required to calculate the actual
amount of VAT payable by Mercantile Limited for the month of Chaitra 20X-67. (June 2012, 5 Marks)

Answer:
A. Output VAT (or VAT Collected on Sales)
Particulars Amount VAT Note
Domestic Sales 2,000,000 260,000 Tax rate of 13%
Export Sales 3,000,000 - Tax rate of 0%
Total Output VAT for the
260,000
month (A)

B. Eligible Input tax credit on purchases during the month


Eligible
Amount
Particulars Amount VAT Notes
for
Credit
Purchase 1,000,000 130,000 130,000 Full VAT credit
Telephone Charges 10,000 1,300 1,300 Full VAT credit
No VAT Credit, consumption of
Petrol for vehicle 5,000 650 -
petroleum
Printing and
5,000 650 650 Full VAT credit
stationery
Total Eligible VAT
131,950
Credit (B)

C.
D.
E.

The Institute of Chartered Accountants of Nepal 416


CAP-II Paper 7 - Income Tax and VAT
F. Input VAT Credit on VAT paid as per Sec. 8 (2), 12Ka and 15 (3)

Eligible
Particulars Amount VAT Notes
Credit

The VAT in respect of this service


Service from Star
2,000,000 260,000 260,000 is already paid and therefore, the
Inc., USA
amount can be claimed as credit
Total ( C) 260,000
G. Opening VAT credit (excess of Previous month,
-
eligible for set off) (D)

H. Payable/ (Receivable VAT)


(131,950)
[A-B-C-D-E]

26) Satellite Telecom Pvt. Ltd., a telecom operator company in Nepal, seeks your advice whether it can
take credit of the VAT paid in the following transactions. (June 2012, 5 Marks)
a) VAT paid Rs. 18,000 for the purchase of petrol to be used for vehicle operation.
Answer
No, since the main business of Satellite Telecom Pvt. Ltd. is not dealing in petrol. There shall be no VAT
credit in respect of purchase of petrol for vehicle.

b) VAT paid at Birgunj Customs Office Rs. 1,000,000 while importing the telecom equipments from
China.
Answer
Full VAT credit, as the intended use of input is to generate taxable output, which shall be demonstrated
through use later.

c) The company deposited VAT amount Rs. 175,000 in Inland Revenue Office on the service charges
payable to satellite bandwidth provider and want to take VAT credit of this whole amount.
Answer
Assuming the amount is paid under reverse charging principle as per Sec. 8 (2) of the Act, the amount is
eligible for VAT credit.

d) The company purchases beverage in the occasion of annual day and paid VAT Rs. 10,000 on this
bill.
Answer
No, since the main business of Satellite Telecom Pvt. Ltd. is not dealing in beverage and there shall be no
VAT credit in respect of purchase of beverage by any person whose main business is not dealing in
beverage.

e) VAT paid Rs. 13,000 for the purchase of petrol to be used for the operation of generators as BTS
sites.
Answer

The Institute of Chartered Accountants of Nepal 417


CAP-II Paper 7 - Income Tax and VAT
Yes, it is allowed as input tax credit. If VAT is paid for petrol for vehicle, input tax credit is not allowed.

27) XYZ Pvt. Ltd. imports/purchases the taxable raw materials such as Iron Ingots and Steel Plates to
manufacture them into different furniture items as well as agriculture tools.
Details of total imports excluding Input VAT are as follows:
Cost of Iron Ingots declared by XYZ Pvt. Ltd. Rs. 3,00,000
Revised cost fixed by Customs Officer Rs. 3,50,000
Freight Rs. 50,000
Insurance Rs. 5,000
Import Duty @ 6%
In addition, it has also purchased following raw materials:
Steel plates (exclusive of VAT) Rs. 1,00,000
Woods (exclusive of VAT) Rs. 35,000

The products manufactured out of the above raw materials are sold out with the details as given
below:
Agriculture tools (VAT exempt items) Rs. 2,50,000
Furniture Rs. 3,00,000
Find the ratio between taxable and non taxable sales and calculate the amount of Input VAT that XYZ
Pvt. Ltd. is entitled to claim under sec 17 (3) of VAT Act. (June 2011, 10 Marks)

Answer:
Ratio of Taxable and Non-taxable Sales
Particulars Amount Ratio Calculation
Taxable Sales 300,000 54.55% 300,000 divided by 550,000 multiplied by 100
Non-taxable sales 250,000 45.45% 250,000 divided by 550,000 multiplied by 100
Total Sales 550,000 100%

Calculation of Output VAT


Particulars Sales Amount VAT Amount Reason
Taxable Sales 300,000 39,000 @13% on domestic sales
Non-taxable sales 250,000 - Exempt
Total Output VAT 39,000

Calculation of Eligible Input VAT credit

Eligible
VAT
Particulars VAT Reason
Amount paid
amount
Import (Refer W.N. 1 below) 429,300 55,809 30,441 Proportionate credit in
Steel Plates 100,000 13,000 7,091 proportion of taxable sales
Woods 35,000 4,550 2,482 to total sales
Total Eligible Input Tax Credit 40,014

The Institute of Chartered Accountants of Nepal 418


CAP-II Paper 7 - Income Tax and VAT
Calculation of Payable or Receivable VAT for the month
Particulars Amount
Total Output VAT 39,000
Total Eligible Input Tax Credit 40,014
Payable /(Receivable) VAT for the month (1,014)

Working Note 1: VAT paid on Import

Particulars Amount
Revised cost fixed by Customs Officer 350,000
Freight 50,000
Insurance 5,000
Transaction Value for Import duty 405,000
Import Duty @ 6% 24,300
Taxable Value for VAT 429,300
VAT @13% 55,809

28) Sharma & Sharma Co. has been a self employed VAT registered trader since 2064 B.S, and is in the
process of completing the VAT return for the month ended 31 Baisakh 20X-68. The following
information is relevant to the completion of the VAT return:
a. Sales invoices totaling Rs. 44,00,000 (excluding VAT) were issued to VAT registered customers in
respect of VAT sales. The firm offers a 5% discount for prompt payment.
b. Sales invoices totaling Rs. 16,92,000 were issued to customers that were not registered for VAT.
Of this figure, Rs. 51,700 was in respect of zero-rated VAT sales.
c. During the month of Baisakh 20X-68, goods amounted to Rs. 11,20,000 were purchased. Of this
figure, Rs. 80,000 was used for Mr. Sharma’s private purpose.
d. On 15 Baisakh 20X-68, a Toyota Hilux vehicle was purchased for Rs. 24,00,000. The cost is inclusive
of VAT.
e. During the month ended 31 Baisakh 20X-68, Rs. 40,000 was spent on mobile telephone calls, of
which 30% relates to private calls.
f. On 17 Baisakh 20X-68, an office equipment was purchased for Rs. 6,00,000. The purchase was
partly financed by a bank loan of Rs. 5,00,000.

Unless stated otherwise, all of the figures are exclusive of VAT.


Calculate the amount of VAT payable by Sharma & Sharma Co. for the month ended 31 Baisakh 20X-
68. (June 2011, 10 Marks)

Answer:
Calculation of Output VAT

Particulars Amount
Sales VAT Registered Customers 4,400,000
Less: Discount at the time of sales 220,000

The Institute of Chartered Accountants of Nepal 419


CAP-II Paper 7 - Income Tax and VAT
Taxable Value 4,180,000
VAT @13% 543,400
(Alternatively, students may assume discount as settlement discount. In this case, value of
sales would be Rs. 4,400,000.)
Particulars Amount Reason
Sales to Customers not registered for VAT 1,692,000 Assuming excluding VAT
Less: Zero rated sales 51,700
Taxable Value for non-zero rated sales 1,640,300
VAT @13% 213,239
Total Output VAT
Particulars Amount Reason
VAT collected from Registered Customers 543,400
VAT collected from Unregistered
213,239
Customers
VAT collected on Zero rated sales - 51,700 multiply by Zero
Total Output VAT 756,639
Calculation of Eligible Input VAT Credit on Purchases during the month

VAT Eligible
Particulars Amount Reason
paid Credit
There shall not be input tax credit on
purchases consumed for personal
Purchase
1,120,000 145,600 135,200 purpose.
(excluding VAT)
Amount of credit = Rs. 10,40,000 *
13%
VAT paid is 13 % of (2400000
Purchase of divided by 1.13)
Toyota Hilux 2,400,000 276,106 110,442 40% of VAT paid is eligible to claim
(Including VAT) credit, assuming the vehicle is used for
personal (administrative) purpose
No VAT credit with regards to
Telephone call
40,000 5,200 3,640 personal call, VAT credit is equal to
(excluding VAT)
70% of VAT paid
VAT credit is available to the lessee in
Office Equipment 600,000 78,000 78,000
respect of goods under finance lease
Eligible VAT Credit 327,282
Calculation of VAT Payable/(Receivable)

Particulars Amount
Output VAT 756,639
Less: Eligible Input VAT credit 327,282
VAT Payable/(Receivable) 429,357

The Institute of Chartered Accountants of Nepal 420


CAP-II Paper 7 - Income Tax and VAT

29) Samal Ltd. has the following transactions during the month. Find the amount of VAT
payable/receivable for the coming month: (June 2011, 5 Marks, CA Inter)
Purchase (excluding VAT) 10,00,000
Salary for the month 1,00,000
Purchase of a car with VAT 11,30,000
Purchase of Office Supplies (excluding VAT) 1,10,000
Sales 20,00,000
Office supplies include Rs. 10,000 purchased under abbreviated tax invoice.
50% of the sales represent export sales.

Answer:
Calculation of Output VAT

Particulars Amount VAT collected Reason


Domestic Sales 1,000,000 130,000 VAT @13% on Domestic Taxable Sales
Export Sales 1,000,000 - VAT @ 0% on Export Taxable Sales
Total Output VAT 130,000
Calculation of Eligible Input VAT credit

Amount Eligible
Amount VAT
Particulars excluding VAT Reason
(Rs.) paid
VAT credit
Purchase (excluding VAT) 1,000,000 1,000,000 130,000 130,000 Full VAT credit
Salary for the month 100,000 100,000 Exempt - Exempt
40% of VAT
paid, assuming
car is used for
Purchase of car with VAT 1,130,000 1,000,000 130,000 52,000
personal
(administrative)
purpose
Purchase of office supplies
(excluding VAT)
There cannot be
tax credit using
In abbreviated tax invoice 10,000 10,000 1,300 -
abbreviated tax
invoice

With Proper VAT invoice 100,000 100,000 13,000 13,000


Full VAT credit
Total Eligible Tax Credit 195,000
Calculation of VAT payable or Receivable
VAT payable/(Receivable)
Output VAT 130,000

The Institute of Chartered Accountants of Nepal 421


CAP-II Paper 7 - Income Tax and VAT
Less: Input VAT (195.000)
VAT Payable/(Receivable) (65,000)

30) Fat Limited has following transactions during the month of Baisakh, 20X-67, find the amount of VAT
credit available for the month:
Amount (Rs.)
Opening Credit available 12,000
Purchases net of VAT 10,00,000
Salary for the month 1,50,000
Electricity expenses 10,000
Telephone expenses with VAT 13,560
Fuel expenses with VAT 28,250
Purchase of car with VAT 11,30,000
Purchase of office supplies net of VAT 1,50,000
Total sales for the month 15,00,000
Additional information:
All the sales of the company were VAT applicable. Out of office supplies, Rs. 20,000 was purchased by
obtaining abbreviated tax invoice. In purchases, Rs. 375,000 is import of raw material and customs
office has valued these goods Rs. 450,000. Fuel expenses consists expenses for petrol used for office
vehicle. (Dec 2010, 6 Marks)

Answer:
VAT credit available for the month

Amount Amount
VAT Eligible
Particulars (Rs.)- excluding Reason
paid VAT
Given VAT
credit
Purchases net of VAT
Full VAT
Import where Customs Officer
375,000 375,000 58,500 58,500 credit, Note (1)
revised Value
below
Full VAT
Other purchases 625,000 625,000 81,250 81,250
credit
Salary for the month 150,000 150,000 Exempt - Exempt
Electricity expenses 10,000 10,000 Exempt - Exempt
Full VAT
Telephone expenses with VAT 13,560 12,000 1,560 1,560
credit
No VAT credit,
as no input is
Fuel expenses with VAT 28,250 25,000 3,250 -
allowed for
petrol used for

The Institute of Chartered Accountants of Nepal 422


CAP-II Paper 7 - Income Tax and VAT
vehicle. (Rule
41(1)).

40% of VAT
paid, assuming
Purchase of car with VAT 1,130,000 1,000,000 130,000 52,000
car is used for
business
Purchase of office supplies net
of VAT
No tax credit
In abbreviated tax invoice 20,000 20,000 2,600 - for abbreviated
tax invoice
Full VAT
With Proper VAT invoice 130,000 130,000 16,900 16,900
credit
Total Eligible Tax Credit 210,210
Add: Opening VAT Credit 12,000
Total Credit Available for the month 222,210

Note 1:
VAT paid on import of Rs. 375,000 is 13% of Rs. 450,000 as VAT is collected by Customs officer on revised
cost, assuming the taxable value is Rs. 450,000.

31) Goodwill Ltd. has purchased a motorcycle from a VAT registered dealer and paid Rs. 10,000 as VAT.
Goodwill Ltd. wants to take input credit of Rs. 10,000 as per section 17 of the Act. But the tax officer
contends that as per rule 41 of VAT Rules, only 40% of the VAT paid i.e. Rs. 4,000 can be claimed. Give
your opinion with regards to this transaction by citing the provision of Section 17 and Rule 41. (Dec
2010, 5 Marks, CA Inter)

Answer:
Partial VAT Credit:
As per Rule 41 of Value Added Tax Regulation, 2053, input Tax credit on purchase of automobile by a
registered person dealing wholly in taxable items can be claimed to the extent of 40% of Value Added
Tax paid on such purchase, if the automobile is used for personal (administrative use) purpose.
The term "automobiles" means automobiles having three or more wheels and that runs in road with
passenger

Analysis of Fact in the Question


The item that has been purchased is motorcycle. A motorcycle has two wheels; therefore, it does not fall
in the category of "automobiles" within the meaning of Rule 41 of Value Added Tax Regulation, 2053.

The Institute of Chartered Accountants of Nepal 423


CAP-II Paper 7 - Income Tax and VAT
Conclusion
The contention of tax officer is incorrect. Full VAT credit is allowed in respect of purchase of motorcycle
by a registered person dealing only in taxable items, since motorcycle does not fall in the category of
"automobiles" within the meaning of Rule 41.

32) Explain the following with reference to VAT Act (Dec 2009, 10 Marks)
a) VAT credit on the damaged goods.
Answer
Application for Input Tax Credit & Time Limit for such application
Where there is loss of goods by fire, theft, accident, accidental damages, or terror and it compels a person
to write off the goods (assets) or sale such goods at lower selling rate, the person shall make an
application in writing to respective Inland Revenue Office along with evidence within 30 days of
happening of such events.

Investigation and permission to claim as input tax credit


The tax office shall investigate the matter and finalize the quantum of tax credits to be allowed. On the
basis of such investigation, the tax office may allow the taxpayer to claim Input Tax Credit of VAT paid on
such assets (goods).

Right to claim Input tax Credit without confirmation from Tax Officer
In case the goods is insured, the tax office may allow the taxpayer to claim Input Tax Credit on such goods
to the extent of compensation paid by the Insurance Company, without any investigation from tax
officer.

b) VAT credit facility in case of used goods


Answer
The VAT paid on purchase of used goods (second hand goods) as follows shall not be eligible for VAT
credit:
(i) Purchased from unregistered persons
(ii) Purchased from registered person and where such goods are used for personal purpose

33) A manufacturer purchases the taxable raw materials such as iron and steel goods. He manufactures
iron and steel goods including metal office furniture and metal fencing. He also makes agriculture
implements such as hoes and rakes. These supplies are exempt from VAT. Details of his purchases and
sales are given below:
Purchase and Overheads Rs.
Iron and Steel goods 100,000
Woods 40,000
Overheads (telephone and electricity) 10,000
Overheads (stationery and furniture) 25,000
Machine Parts 35,000
Total purchase excluding VAT 210,000

The Institute of Chartered Accountants of Nepal 424


CAP-II Paper 7 - Income Tax and VAT
Sales
Agriculture Implements 100,000
Steel Appliances 100,000
Office Furniture and Fencing 300,000
Total sales excluding VAT 500,000

You are required to find the ratio between taxable and non-taxable sales and calculate the value of
Input Tax Credit. (Dec 2009, 5 Marks)

Answer:

Ratio of Taxable and Non-taxable Sales


Non-Taxable Sales
Agriculture Implements 100,000
Taxable Sales
Steel Appliances 100,000
Office Furniture and Fencing 300,000
Total Taxable Sales 400,000

Percentage of Non-taxable sales 20%


(Total Non-taxable sales divided by total sales)

Percentage of Taxable Sales 80%


(Total Taxable sales divided by total sales)

Value of Input Tax credit


All input items are used to produce taxable and non-taxable output, which means, there shall be
proportionate VAT credit on all input items, if VAT has been paid.

Purchase Eligible
cost VAT Input
Particulars Note
excluding paid Tax
VAT credit
Iron and Steel goods 100,000 13,000 10,400 Proportionate VAT
Woods 40,000 5,200 4,160 credit, in proportion
Overheads (stationery and furniture) 25,000 3,250 2,600 of taxable sales to
Machine Parts 35,000 4,550 3,640 total sales
Check note 1 below,
and proportionate
Telephone 5,000 650 520 credit on sales ratio
VAT exempt, check
Electricity 5,000 - - note 1 below
Eligible VAT credit 21,320

The Institute of Chartered Accountants of Nepal 425


CAP-II Paper 7 - Income Tax and VAT

Note 1:
There is combined total of telephone and electricity expense. Telephone expense is VAT attractive and
electricity expense is exempt from VAT. Therefore, it is assumed that the composition of telephone
expense and electricity expense is fifty-fifty.

34) Write short note on Partial Credit on VAT (Dec 2009, 2 Marks, CA Inter)
Or
Explain 'no-tax credit' and 'partial tax credit' under the VAT Act. (June 2009, 5 Marks)
Or
What are the items in which no input tax credit is allowed under Rule 41 of the VAT Regulations. (Dec
2006, 4 Marks, CA Inter)

Answer:
Partial VAT Credit:
Input Tax credit on purchase of automobile by a registered person dealing wholly in taxable items can be
claimed to the extent of 40% of Value Added Tax paid on such purchase, if the automobile is used for
personal (administrative use) purpose.
The term "automobiles" means automobiles having three or more wheels and that runs in road.

No VAT credit:
In any of the following circumstances, VAT paid on purchases cannot be claimed as input tax credit:
a. If the intended use of purchase is to generate non-taxable output,
b. In case the purchase is made by non-registered person,
c. In case there is no proper Value Added Tax invoice (other than abbreviated tax invoice) to
substantiate VAT paid except when VAT is paid under import (where import documents substitute
VAT invoice), u/s 8 (2), 12Ka and 15 (3).
d. In case such purchase is listed as below and the main business of the registered person is not dealing
in the items listed below:
i. Entertainment expense
ii. Beverages
iii. Liquor
iv. Petrol Used for Vehicle

35) Write about the treatment of VAT paid on purchases made by the following: (Dec 2007, 3 Marks, CA
Inter)
a) Registered Person
Answer
In case a registered person pays VAT on purchases, the VAT may be claimed as follows:
i. Full VAT credit, if the intended sales of products from purchase is taxable.
ii. Proportionate Tax credit, if the intended sales of products from purchase is both taxable and
non-taxable.

The Institute of Chartered Accountants of Nepal 426


CAP-II Paper 7 - Income Tax and VAT
iii. Partial VAT credit, if the purchase is automobiles and is intended to be used for administrative
purpose, when the person deals wholly in taxable items. If he deals both in taxable and non-
taxable items, the applicability is partial credit cum proportionate credit.
iv. No VAT Credit, if the purchase is entertainment, beverage, liquor, diesel, petrol or liquefied
petroleum gases and the person's main business does not include transaction of those items.

b) Unregistered Person
Answer
The VAT paid cannot be treated as credit and forms part of cost of purchase.

c) Person producing VAT Exempt goods


Answer
VAT paid is treated as cost of purchase.

36) From the following figures of M/s Light Life Pvt. Ltd., calculate the VAT payable or receivable for Magh
20X-63. (June 2007, 7 Marks, CA Inter)
The Sales during the month:

Oil Rs. 5,00,000


Rice Rs. 2,00,000
Scraps Rs. 1,00,000

Total import during the month is as follows:

Particulars (Amount in Rs.)

Oil Rice
Invoice Value 4,00,000 2,50,000
Transportation etc. up to boarder 25,000 7,000
Custom Duty and Other Duties 40,000 13,000
Transportation etc. expenses from boarder to godown 20,000 10,000
Local Purchase from a VAT Registered Firm 2,50,000 80,000

Other administrative expenses for which VAT was charged, as follows:

Stationery Items 3,000


Telephone Bills 8,000
Hotel Bills 6,000

During the month, the Company had purchased a Van for transportation of goods for Rs. 10,00,000.
The Van has been financed by a Finance Company and hence the invoice is in the name of the Finance
Company.

The Institute of Chartered Accountants of Nepal 427


CAP-II Paper 7 - Income Tax and VAT
Answer:
Calculation of Output VAT
Particulars Amount Output VAT Reason
Oil 5,00,000 65,000 Taxable @13%
Rice 2,00,000 - VAT exempt
Scraps 1,00,000 13,000 taxable @13%
Total 800,000 78,000

Ratio of Taxable and Non-Taxable Sales


Taxable Sales 600,000 75%
Non-Taxable Sales 200,000 25%

Calculation of eligible Input Tax Credit


Particulars Amount VAT Eligible Reason
Paid Input VAT
Import of Oil (Note 1 60,450 60,450 Full VAT credit
below)
Local purchase of oil 250,000 32,500 32,500 Full VAT Credit
Stationery Items 3,000 390 292.50 Proportionate credit, to the extent of
75% of VAT paid
Telephone bill 8,000 1,040 780 Proportionate credit, to the extent of
75% of VAT paid
Hotel bill 6,000 780 - No VAT Credit, entertainment
expense
Purchase of Van 1,000,000 130,000 97,500 Proportionate VAT credit to the
extent of 75% of paid VAT.
VAT credit is claimed by Lessee
when an asset is purchased under
finance lease arrangement.
Total Eligible VAT Credit 191,522.50

Calculation of VAT payable or Receivable


Total Output VAT 78,000
Eligible Input VAT credit 191,522.50
VAT Payable/(Receivable) (113,522.50)

Note:
(1) VAT paid on import of oil at Customs Frontier
Particulars Amount
Invoice Value 400,000
Transportation etc. up to boarder 25,000
Custom Duty and Other Duties 40,000
Taxable Value 465,000
VAT @13% 60,450

The Institute of Chartered Accountants of Nepal 428


CAP-II Paper 7 - Income Tax and VAT
(2) There is no applicability of VAT on import and domestic purchase of rice.

37) A dealer manufactured goods under exempt under schedule I worth Rs. 5 Crores out of goods
purchased partly VAT exempt and partly goods liable to VAT. VAT exempt goods used were Rs. 3 Crore
were exempted. He claims are zero and he should get full credit for VAT on exported goods. Consider
his claim to set off the VAT paid by him on the inputs. (Dec 2004, CA Inter)

Answer:
As per Sec. 5 (3) of Value Added Tax Act, 2053, no VAT shall be levied on goods and services listed in
Schedule 1 of the Act. The VAT paid on purchase of goods utilized to produce VAT exempt output cannot
be claimed as credit and such VAT cannot be refunded as well. Though the VAT exempt goods or services
are exported, there shall not be applicability of VAT.

As such, there cannot be refund of VAT when the exempt goods are exported.

38) What are the provisions relating to the deduction and adjustment of Value Added Tax? (Dec 2004, CA
Inter)
Answer:
In case a registered person pays VAT on purchases, the VAT may be claimed as follows:
a. Full VAT credit, if the intended sales of products from purchase is taxable.
b. Proportionate Tax credit, if the intended sales of products from purchase is both taxable and non-
taxable.
c. Partial VAT credit, if the purchase is automobiles and is intended to be used for administrative
purpose, when the person deals wholly in taxable items. If he deals both in taxable and non-taxable
items, the applicability is partial credit cum proportionate credit.
d. No VAT Credit, if the purchase is entertainment, beverage, liquor, diesel, petrol or liquefied
petroleum gases and the person's main business does not include transaction of those items.

VAT credit can be claimed even without VAT invoice in case of import of goods (based on import
documents) and in case of VAT paid under Sec. 8 (2), 12ka and 15 (3) [with documents proving VAT has
been paid). In case of assets purchased under finance lease, lessee can claim input VAT credit even
though the legal title is vested on lessor.

The Institute of Chartered Accountants of Nepal 429


CAP-II Paper 7 - Income Tax and VAT

Chapter 4: Return, Tax Payment, Fine and Assessment


1) PQR Pvt. Ltd., a company registered under VAT and submitting the return on monthly basis. It has
submitted the tax return on Jestha 25, 20X-72 for the month of Baisakh 20X-72. Also, the amount of
tax was Rs. 100,000 and it has paid via 'Account Payee' Cheque dated Jestha 25, 20X-72. The Cheque
got cleared on Jestha 26, 20X-72. Is it considered as payment of tax within due date as pet Act/Rules?

Will your answer be the same if the company paid via 'Account Payee' Cheque guaranteed by bank
(good for payment Cheque) and the Cheque got cleared on Jestha 27, 20X-72? (Dec 2015, 5 Marks)
Answer
As per Sec. 19 (7) of Value Added Tax Act, 2052, in case a person makes payment of VAT through
electronic means or account payee cheque, the date when the pay order was issues in respect of use of
electronic means and the date when cheque was submitted to revenue authority in respect of good for
payment cheque shall be considered as the date of payment.

In the given case, since the payment was not made through electronic means or good for payment
cheque, the date of payment of VAT is the date when the amount is cleared by the respective bank, i.e.
Jestha 26, 20X-72.

In the contrary, the cheque was marked "Good for payment", i.e. guaranteed by bank, the date of
payment would be the date when the cheque was deposited with revenue authority. That means,
although the cheque was cleared on 27th Jestha 20X-72, the date of payment by PQR Pvt. Ltd. is 25th
Jestha 20X-72.

2) MNP Pvt. Ltd. is a company voluntarily registered to VAT on 1st Shrawan, 2067. Details of his
transactions are as follows:

Fiscal Year (FY) 20X-68/X-69 Amount (Rs.)


Shrawan- Kartik 550,000
Mangsir- Falgun 500,000
Chaitra- Ashadh 930,000
1,980,000
Fiscal Year 20X-69/X-70
Shrawan- Kartik 400,000
Mangsir- Falgun 680,000
Chaitra- Ashadh 590,000
1,670,000
The company is submitting VAT return on four monthly bases, since he it is voluntarily registered, and
the turnover of any F/Y doesn't exceed Rs. 20 lakhs, it's claiming that it need not have to submit its
VAT return on monthly basis. Comment on the company's contention with reference to Value Added
Tax Act/Rules. (June 2014, 5 Marks)

The Institute of Chartered Accountants of Nepal 430


CAP-II Paper 7 - Income Tax and VAT
Answer
As per Sec. 11 (1) (Cha), the VAT registration of a person who is registered voluntarily is cancelled
automatically by revenue authority if the turnover of such person in last twelve months does not exceed
the threshold of Rs. 50 lakhs in case he is dealing only in goods and Rs. 20 lakhs in all other cases. If the
person wishes to retain the registration status, an application along with the proof of submission of VAT
returns in twelfth month out of last twelve months shall be filed.

There is no provision in VAT law barring a person to apply for cancellation of VAT registration.
Therefore, if M/s MNP Pvt. Ltd. wishes to cancel registration; it can do so by filing an application in this
regard.

3) Explain the provisions of Tax Assessment by Tax Officers (Dec 2009, 5 Marks)

Answer
Conditions for VAT assessment by Tax Officer:
Tax Officer can make tax assessment under any of the following grounds and conditions:
a) In case VAT return is not submitted within the time limit;
b) In case of filing of an incomplete or erroneous return;
c) In case of filing of a fraudulent return;
d) In case the Tax Officer has a reason to believe that the amount of tax is understated or otherwise
incorrect;
e) In case the Tax Officer has a reason to believe that the taxpayer has caused under-invoicing;
f) In case of supply to group companies through under-invoicing;
g) In case person requiring registration operates business without registration;
h) In case of sales without issuing invoices;
i) In case an unregistered person collects the tax;
j) In case a person does not deposit VAT to be levied u/s 8 (2) or 8 (3) under the principle of reverse
charging; or
k) In case a person does not levy VAT on goods that are left to be used for taxable transaction for which
input tax credit was enjoyed as per Sec. 17 (4)
Assessment by tax officer is risk-based sampling based on above conditions

Bases of the assessment


The Tax Officer may make such tax assessment on one or more of the following bases:
a) Proof of transactions;
b) A tax audit report on transactions submitted by the concerned Tax Officer; and
c) Tax paid on a similar transaction by another person.

Burden of proof
The burden of proof lies on Tax Officer for reasons of assessment.

The Institute of Chartered Accountants of Nepal 431


CAP-II Paper 7 - Income Tax and VAT
Time limit for assessment
Assessment by Tax Officer needs to be finalized within four years from the date of filing of tax return. If
the stipulated time expires and the tax officer fails to assess tax, the return so filed by taxpayer shall be
considered to be true and valid.
But, in case of fraudulent return, reassessment tenure is unlimited.

Show cause Notice


The tax officer shall issue a Preliminary Assessment Order (PAO) after evaluating the basis as described
above and determining additional price, if any and send the notice to the taxpayer for it to be able to
defend the allegations made by tax officer providing a time limit of 15 days from the receipt of such
notice.
In case the Tax Officer is satisfied with the reply on PAO, the points raised by the tax officer shall be
omitted and the tax officer shall issue a Final Assessment order considering the replies of taxpayer on
PAO considered unsatisfactory.

Power of Inland Revenue Department in relation to Assessment pending or completed by Tax Officer
In case where, before the assessment of tax is complete, it appears from the information received by
Director General any action relating to the assessment of tax is about to be taken in irregular way or has
been irregular, the Director General may by executing the memorandum (Parcha Khada gari) clearly
setting out the reasons, give direction to the concerned tax officer to make reassessment of tax or order
any other tax officer to do that act.

In case where it appears that the tax liability has decreased because of the tax assessment made by the
tax officer recklessly or with the malafide intention, the Director General may order amendment in such
tax assessment order within 4 years after the date of initial tax assessment.

4) Healthy Bottlers Pvt. Ltd. is a manufacturer of glass bottle packaged beverages. The Company supplies
packaged bottles of beverage, billing the selling price of the beverage. In addition to that, it gets certain
amount from the dealers as deposit for the glass bottles supplied to them. When the dealer returns
the empty bottles, the amount of deposit is refunded. A tax officer, during his inspection of the
company, found that bottles worth NRs. 200,000 were not in physical stock. He made tax assessment
under section 20 of the VAT Act and treated the shortage of bottles as sold by the company. The
company had proved that the bottles were in stock with various dealers in due course of return.
However, the tax officer did not accept the contention and charged tax and penalty on the deemed
sales of the bottles. Critically examine the contention of the tax officer. (June 2009, 5 Marks)

Answer
Healthy Bottlers Pvt. Ltd. is selling beverages only. When beverage is consumed by consumer, the empty
bottle is returned to the producer. Thus, the supply of empty bottles on deposit does not constitute
transaction as per VAT Act, 2052.

Section 2 (g) of the Act, defined the term supply which means act of selling, exchanging and delivering
any goods or services, or the act of granting permission thereto or of contract thereof for a consideration.

The Institute of Chartered Accountants of Nepal 432


CAP-II Paper 7 - Income Tax and VAT
In this case the supply of bottle is not for consideration and so it is not treated as supply for transaction
purpose.

It was held in Pepsi Cola Nepal Pvt. Ltd. vs. Inland Revenue Department; the Revenue Tribunal has
interpreted that the bottles would be treated as sold in case of breakage or lost by the dealer and when
the producer charging for such breakage or shortage to the dealer. The Tribunal has pointed out Section
12 (8) of the Act, according to which deposit amount could be treated as sales when it is adjusted against
consideration receivable for supply of goods or services.

That is why; the shortage of bottles at factory premises cannot be treated as sold in case it is proved that
the bottles ate lying with the dealers.

5) As per the provisions of Value Added Tax Act, the tax officer has to complete the assessment within 4
years from date of submission of VAT return. A VAT Registrant destroyed the VAT records on expiry of
above-mentioned period. On inspection, the tax officer imposed a fine under section 29 stating the
reason that the company did not produced the record for his inspection. The company wants to apply
for administrative review and seeks your advice in this regard. Please give your advice.
(Dec 2015, 5 Marks)
Answer
Section 20(4) of Value Added Tax Act, 2052 provides for the assessment by tax officer within the
prescribed time limit of four years from the date of filing of the tax return. However, the taxpayer shall
maintain VAT records for six years. This is the responsibility of the taxpayer to maintain the records for
the period mentioned in the Act.

The taxpayer in the question destroyed the records before the time limit the tax officer has rightly
imposed penalty under the Act. The Tax officer is correct in his action and there is no meaning in filing
for administrative review.

6) Mr. Krishna K.C is a proprietor of KKC & Co. which is a VAT registered firm. Due to the occurrence of
destructive Earthquake on Baisakh 12, 2072, Mr. K.C could not deposit VAT amount of Rs. 10 lakh
collected by the goods sold by his firm for the month of Chaitra 2071.
IRD vide public notice has extended the VAT submission date from 2072/01/25 to 2072/02/07. Finally,
Mr. K.C has been successful to deposit the said VAT amount on Jestha 08, 2072.
The Tax officer intends to levy additional fee of 10% p.a. due to failure to pay VAT even within the
extended time of 2072/02/07. Since the failure to pay VAT was due to the circumstances beyond his
control, suggest Mr. K.C to obtain waiver from paying such extra charges on total VAT amount
due. (July 2015, 5 Marks)

Answer
Additional fees @ 10% p.a. on payable VAT obligation, as per Sec. 19 (2), can be waived by Director
General in the event of circumstances beyond control.

The Institute of Chartered Accountants of Nepal 433


CAP-II Paper 7 - Income Tax and VAT
In case there is circumstances beyond control as a result of fire, earthquake or similar other natural
disaster, a period of 30 days from the date of happening of such event is treated as circumstances beyond
control.

In order to obtain waiver, the concerned taxpayer shall file an application for waiver within 30 days of
the due date of payment of VAT in front of Director General.

Since, Mr. Krishna has filed an application for waiver within 30 days of due date of payment of VAT of
Chaitra 2071, Director General may waive additional duty. For this, tax shall be paid after 30th day of
12th Baisakh 2072.

7) What shall be the fine and penalty chargeable under the following situations as per VAT Act,
2052? (Dec 2013, 2×2=4)
a) Late payment of VAT amount
Answer
Fees @ 10% per annum on payable VAT and interest @ 15% per annum on payable VAT.

b) Tax plate not kept/misplaced


Answer
Rs. 1,000 every time

8) Describe the circumstances beyond the control under the Value Added Tax Act, 2052.
(June 2013, 10 Marks& Dec 2011, 5 Marks)
Answer
Director General may waive additional fees @10% per annum applicable on payable VAT, in case when
a person fails to make payment of VAT by due date.

In case of any of the following circumstances, the circumstances are treated as circumstances beyond
control:

Due date
Period to which the due Other
for filing
Conditions date is extended (i.e. tax formalities, if
Waiver
payment due date) any
Application
In case of incapacity of a Within 7 days after the
30 days of
person as a result of illness person recovers from
expiry of
illness
due date for
In case the person responsible Within 7 days of the A
payment of
to pay tax has to perform completion of such death recommendation
tax
death ritual ritual letter of

The Institute of Chartered Accountants of Nepal 434


CAP-II Paper 7 - Income Tax and VAT
In case the woman concerned local
Within 35 days of giving
responsible to pay tax gives level must be
birth to a child
birth to a child submitted
In case of death, insanity or
35 days of
absconding of person Within 7 days of
happening
responsible to pay tax and the receiving such
of such
rightful beneficiary or legal application
event
representative
In case of failure to reach Tax
Office as a result of blockage
Within 7 days after the
of road due to flood,
road is resumed
landslide, snowfall or similar
other reasons
In case of failure to reach tax A
office due to complete recommendation
blockage of means of letter of
transportation 30 days of concerned local
expiry of The next day when the level where the
due date for transportation is resumed complete
payment of blockage of road
tax has happened
must be
submitted
In case of creation of A
circumstances beyond control recommendation
as a result of fire, earthquake Within 30 days of the letter of
or similar other natural happening of such event concerned local
disaster level must be
submitted

Due Date for application for Waiver of Fees (Rule 36)


The concerned taxpayer shall file an application for waiver of fees within 30 days of the expiry
of due date of payment of tax in relation to the period to which the Circumstances beyond
control is related.
Where it is related to absconding or death of a person, the application must be filed within 35 days of
happening of such event.

9) Mention the penalties prescribed under Value Added Tax Act, 2052 on the following
infringements: (June 2012, 5 Marks)
a) Failure to issue a tax invoice and in case of transportation of goods out of the area specified by
Inland Revenue Department, worth more than Rs. 10,000 without accompanied with a tax
invoice.

The Institute of Chartered Accountants of Nepal 435


CAP-II Paper 7 - Income Tax and VAT
Answer
Rs. 10,000 every time

b) Registration as mentioned in section 10(1) and (2) related infringement.


Answer
Rs. 20,000 every time

c) Failure to display the tax board as per VAT Act/Rules.


Answer
Rs. 1,000 every time

d) An unregistered person issuing an invoice or documents showing collection of tax.


Answer
100% of tax so collected or imprisonment up to Six months or both.

e) Tax officer finding that a taxpayer has reduced tax liability by making infringement of any
provisions of this act or rules there-under.
Answer
The tax officer may charge, following a procedure set by Inland Revenue Department, a penalty up to
25% of the tax payable [section
29(ka1)]

Section 29(2) is also applicable in this case and the tax officer may charge a fine of 100% of the tax
evaded amount.

10) What are the penalties mentioned in section 29 of the act on the following infringements:
(June 2011, 5×1=5)
a) Registration as mentioned in section 10(1) and (2) related infringement.

Answer
Rs. 20,000 every time.

b) An unregistered person issuing an invoice or documents showing collection of tax.


Answer
100% of tax so collected or imprisonment up to Six months or both.

c) On obstruction in inspection by a tax officer.


Answer
Rs. 5,000 every time

d) On infringement of the VAT act and the rules.

The Institute of Chartered Accountants of Nepal 436


CAP-II Paper 7 - Income Tax and VAT
Answer
Rs. 1,000 every time.

e) To erase and edit the data in software of approved computer Billing System.
Answer
Rs. 500,000

11) Ms. Amita is a proprietor of a VAT registered firm. For the month of Shrawan 20X-67, she has collected
VAT of Rs. 200,000 which she has to deposit within 25th Bhadra. But on Bhadra 10,20X-67, she has
delivered a baby boy due to that she could not deposit the VAT amount within Bhadra 25. The tax
officer wants to levy additional fee of 10% per annum on her. She has submitted an application to
Director General IRD on Bhadra 26, 20X-67 for waiver of additional fee levied by the tax officer as the
non-payment of VAT was due to the circumstances beyond her control. Can DG waive that additional
fee levy imposed by the tax officer? Give your opinion. (Dec 2010, 5 Marks)

Answer
Additional fees @ 10% p.a. on payable VAT obligation, as per Sec. 19 (2), can be waived by Director
General in the event of circumstances beyond control.
In case a woman taxpayer who gives birth to a child, a period of 35 days from the date of delivery is
treated as circumstances beyond control.

In order to obtain waiver, the concerned taxpayer shall file an application for waiver within 30 days of
the due date of payment of VAT in front of Director General.

Since, Ms. Amita has filed an application for waiver within 30 days of due date of payment of VAT of
Shrawan 20X-67, Director General may waive additional duty. For this, tax shall be paid after 35th day of
10th Bhadra 20X-67.

12) Calculate the fine payable by a person registered under VAT on delayed in submitting monthly VAT
return by 5 days. (Dec 2009, 2 Marks, CA Inter)

Details of Monthly Sales are as under


Taxable - 7,000,000
Exempted - 3,000,000
Answer
If a person fails to submit Value Added Tax Return, higher of following amount shall be imposed as
penalty [u/s 29 (1) (Ja)]:
(a) 0.05% per day of payable VAT obligation, or
(b) Rs. 1,000 per VAT period

In the given question, the question lacks the information as to the payable VAT obligation for the
month, therefore, penalty cannot be computed precisely.

The Institute of Chartered Accountants of Nepal 437


CAP-II Paper 7 - Income Tax and VAT
13) ABC Ltd, a foreign party got a contract from an organization in Nepal. Before the start of the work, they
have registered themselves with VAT. As a tax professional, they enquire you on the provision for
payment of VAT as per the VAT Act. Advice ABC Ltd. (Dec 2012,4 Marks)

Answer:
Provision regarding payment of VAT (Section 19)
a. A taxpayer shall pay tax for each tax period within twenty five days of the end of the tax period.

b. If a taxpayer does not pay the tax within the time limit specified under (a), an extra charge of ten
percent per annum shall be imposed on the tax due.

c. If a taxpayer applies to the Director General for the exemption of the additional charges as above
stating the reason that the failure to make a timely payment was caused by circumstances beyond
the taxpayer's control, the Director General may, if he finds the reason reasonable, exempt such
charges.
d. The additional charges and the interest pursuant to Section 26 shall be charged from the date on
which the tax first became due.

e. If a tax officer makes tax assessment under section 20 and finds that the amount of tax to be
collected from the tax payer in a tax period is less than the amount he is entitled for refund, then
extra charge and interest shall not collected in that tax period.

f. Tax can be paid through the electronic means or Good for Payment Cheque issued by the Bank. In
such a case, the day tax office receives the payment through electronic means or good for payment
cheque shall be deemed as the day when the tax is collected.

g. Interest shall not be collected on Interest, Extra charge and penalties.

14) In the course of VAT refund audit of M/S Wipro Ltd., an export-oriented undertaking, the tax officer
issued notice under section 20 of the Value Added Tax Act, 2052 and Rule 29 of the Value Added Tax
Regulation, 2053 disallowing certain input tax credit on the ground that it is unrelated to the business.
What are the remedies available to the company? Advice. (June 2012, 5 Marks)
Or
Describe the procedure of Administrative Review in Value Added Tax. (June 2011, 5 Marks)

Answer
The company may apply for administrative review; the procedure is as per follows:

Application for Review & Time Limit [Sec. 31Ka (1)]


Every person who is dissatisfied with the tax officer’s decision of assessment of tax shall make an
application for administrative review at Inland Revenue Department within 30 days of receipt of the Final
Assessment Order.

The Institute of Chartered Accountants of Nepal 438


CAP-II Paper 7 - Income Tax and VAT
Extension of Time Limit for Application [Sec. 31Ka (2)]
In case the taxpayer cannot file application within 30 days of receipt of Final Assessment Order, the tax
payer may file an application for extension of time limit to file Review Application within 7 days of expiry
of such 30 days.

After receiving such extension application, the IRD may extend the deadline for Review Application for
additional 30 days counted from the expiry of first deadline of 30 days, i.e. the maximum time limit to
file a review application is 60 days from the date of receipt of notice.

Formality to File Application [Sec. 31Ka (6) & 31Ka (8)]


The taxpayer shall deposit 100% of undisputed tax amount and one-fourth of disputed tax amount before
filing the review application. Such deposit shall not be refunded unless the final verdict on the issue is
declared.
In case the taxpayer has not submitted any VAT return in connection to the dispute involved in Review
Application, the taxpayer shall file such return prior to submission of Review Application. (Rule 34)

Effect of Application [Sec. 31Ka (3)]


The Director General shall examine the documents and other proofs. In case the matters raised by the
taxpayer is true while examining documents and proofs, the DG shall nullify the decision of the tax officer
and order for reassessment to the concerned tax officer or any other officer.

Time Limit to Decide on Review Application [Sec. 31Ka (4) & 31Ka (5)]
The Department must decide on Review application within 60 days of filing of Review Application. In case
of failure to do so, the taxpayer may deem the application be rejected and choose to appeal in Revenue
Tribunal.

The Institute of Chartered Accountants of Nepal 439


CAP-II Paper 7 - Income Tax and VAT

Chapter 4: Refund of VAT


1) Kathmandu Food Ltd. has the following amounts of sales and purchases excluding VAT. Can VAT refund
be claimed in Baishakh return? (June 2021, 5 marks)
% of export sales to total
Month
Sales Purchase sales
Kartik 500,000.00 700,000.00 43%
Mangsir 800,000.00 800,000.00 65%
Poush 700,000.00 900,000.00 36%
Magh 800,000.00 900,000.00 42%
Falgun 300,000.00 900,000.00 60%
Chaitra 700,000.00 900,000.00 42%
Baishakh 600,000.00 900,000.00 20%

Answer

% of VAT
export Payable /
Month
sales to Export Domestic VAT VAT (Receivable
Sales total sales Sales Sales Payable Purchase Receivable )
Kartik 500,000 43% 215,000 285,000 37,050 700,000 91,000 -53,950
Mangsir 800,000 65% 520,000 280,000 36,400 800,000 104,000 -67,600
Poush 700,000 36% 252,000 448,000 58,240 900,000 117,000 -58,760
Magh 800,000 42% 336,000 464,000 60,320 900,000 117,000 -56,680
Falgun 300,000 60% 180,000 120,000 15,600 900,000 117,000 -101,400
Chaitra 700,000 42% 294,000 406,000 52,780 900,000 117,000 -64,220
Baishakh 600,000 20% 120,000 480,000 62,400 900,000 117,000 -54,600

As per Rule 39(5), if in any month, the export sales of a registered taxpayer is greater than 40%
of total sales, then the taxpayer can claim for refund of vat credit that could not be set off for that
particular month in the format prescribed in Schedule-10.

Since, the portion of export to total sales is more that 40% in the month of Kartik, Mangsir, Magh,
Falgun and Chaitra, the tax payer can claim refund in along with the returns.

The excess input VAT over output VAT for the month of Poush and Baishakh is carried forward for set off
in next four months. If the same cannot be consumed by any payable VAT in succeeding four months, it
may file an application for refund after the elapse of such four months. (Section 24(3))

2) Birat Trading Limited a company dealing in import and export of machineries imported a Machinery
on 5th Magh 2077 and paid value added tax (VAT) Rs. 745,000 and others applicable taxes at the time

The Institute of Chartered Accountants of Nepal 440


CAP-II Paper 7 - Income Tax and VAT
of Import in Nepal. The company do not find any buyer for the imported Machinery in Nepal, The
company sold the same imported Machinery on 5th of Baishakh 2078 to a foreign buyer located at Hong
Kong. Birat Trading Limited received price of Machinery in US $ on 10th Baishakh 2078.

Required: (June 2021, 5 marks)

The management of Birat Trading limited seek your advice as tax professional on:
i) Whether the Birat Trading Limited is eligible to get refund of VAT paid (Rs.745,000) on purchase of
Machinery at the time of import in Nepal as per the provision of value added tax act, 2052?

ii) Is there any period of limitation for claiming refund of VAT as per the provision of value
added tax act, 2052?

Answer:
As a tax professional, my advice to the management of Birat Trading Limited are as follows:

i.) As per section 25(GA) of Vat Act, 2052, If any goods are re-exported and sales consideration thereof
has been received in advance in foreign convertible currency then VAT paid on purchase of such re-
exported goods has to be refunded to the concerned person.
Birat Trading Limited paid Value added tax (VAT) Rs. 745,000 on Machinery at the time of Import in Nepal.
The company do not find any buyer for the imported Machinery in Nepal hence sold to a foreign buyer
and re-exported to Hong Kong on 5th of Baishakh 2078. However, sale consideration in US $ was only
received on 10th Baishakh 2078. Though the sales consideration was received in foreign convertible
currency but not before export (i.e. not in advance) hence, the ABC Limited could not apply for VAT (Rs.
745,000) refund on VAT paid on purchase (import) of such Machinery.

ii.) As per section 25(GHA) of Vat Act, 2052, Notwithstanding anything contained elsewhere in the
provision of the Act , the amount of refund shall not be provided unless application for refund is not
made within the period of three years from the end of relevant tax period.

3) ABC Exporters has following sales and purchase transactions exclusive of VAT for the following months
of 2077:

Month Sales (Rs.) Purchase (Rs.) Export % of Total Sales


Baishakh 800,000 900,000 43%
Jeshtha 200,000 700,000 65%
Ashadh 200,000 700,000 38%
Shrawan 700,000 800,000 42%
Bhadra 300,000 300,000 55%
Ashwin 500,000 600,000 45%
Kartik 200,000 400,000 25%

How much refund can ABC Exporters claim?

The Institute of Chartered Accountants of Nepal 441


CAP-II Paper 7 - Income Tax and VAT
When the tax office refunded the VAT to ABC Exporters, for a particular month, there was
a delay in refund of 90 days from the date of application. Does the tax payer receive any
interest for delay of refund, state the provisions. (Dec 2020, 5 Marks)

Answer:

Domestic VAT on VAT on


Period 2077 Purchase Sales Export Purchase Sales Debit/(Credit)
Baishakh 800,000 456,000.00 344,000 117,000.00 59,280.00 -57,720.00
Jeshtha 200,000 70,000.00 130,000 91,000.00 9,100.00 -81,900.00
Ashadh 200,000 124,000.00 76,000 91,000.00 16,120.00 -74,880.00
Shrawan 700,000 406,000.00 294,000 104,000.00 52,780.00 -51,220.00
Bhadra 300,000 135,000.00 165,000 39,000.00 17,550.00 -21,450.00
Ashwin 500,000 275,000.00 225,000 78,000.00 35,750.00 -42,250.00
Kartik 200,000 150,000.00 50,000 52,000.00 19,500.00 -32,500.00

As per Rule 39(5), if in any month, the export sales of a registered taxpayer is greater than 40%
of total sales, then the taxpayer can claim for refund of vat credit that could not be set off for that
particular month in the format prescribed in Schedule-10.
Since, ABC Exporters sales in the month of Baishakh, Jeshtha, Shrawan, Bhadra and Aswin
exceeds 40% of total sales, it may file an application for refund while filing the VAT return of
respective month.
The excess input VAT over output VAT for the month of Ashad and Kartik is carried forward for
set off in next four months. If the same cannot be consumed by any payable VAT in succeeding
four months, it may file an application for refund after the elapse of such four months. Section
24(3)

It is assumed that all the purchase is qualifying for credit.


As per Rule 47 of the VAT Rules, the rate of interest on refund shall be 15% p.a. Interest shall
only be allowed after 60 days from the day in which the refund was sought. Therefore, for the
particular month (if the excess happened during the month when more than 40% of total sales is
exported) for which there was a delay of 90 days, refund shall only be allowed for 30 days.
However, if the refund was delayed for the month when there was no export over threshold of
more than 40% of sales, the interest of effective for 30 days, as the tax authority is bound to pay
interest if the refund is not made within 60 days of refund application.

4) Banmali Traders has following sales & purchase transactions exclusive of VAT for the following months
of 20X-75:

The Institute of Chartered Accountants of Nepal 442


CAP-II Paper 7 - Income Tax and VAT

Month Sales (Rs.) Purchase (Rs.) Export (% of Total Sales)


Baishakh 500,000 600,000 43%
Jeshtha 825,000 785,000 65%
Ashadh 680,000 675,000 38%
Shrawan 450,000 500,000 40%
Bhadra 300,000 275,000 55%
Ashwin 400,000 500,000 45%
Kartik 600,000 490,000 25%

How much refund can Banmali Traders claim for? (Dec 2018, 5 Marks)

Answer:
As per Sec. 24 of Value Added Tax Act, 2052, unless there is export sales exceeding 40% of total sales
during any month, any excess receivable VAT (i.e. when input VAT exceeds Output VAT) of a month shall
be utilized to set off any payable VAT of next four months. Any remaining excess receivable VAT after
such action for consecutive four months can be refunded by making an application by the concerned
taxpayer along with the VAT return of fourth month or any months thereafter.

In case of export sales exceeding 40 percent of total sales during any month, the taxpayer may make an
application to refund any excess receivable VAT of such month along with the VAT return of the same
month.

Fact of the case


In the given case, the output VAT , input VAT and excess VAT receivable for the respective month are as
follows:

Export Payable/(
Sales Purchase (% of Domesti Output Input Receivabl
Month
(Rs.) (Rs.) Total c Sales VAT VAT e) VAT for
Sales) the month
Baishakh 500,000 600,000 43% 285,000 37,050 78,000 (40,950)
Jeshtha 825,000 785,000 65% 288,750 37,538 102,050 (64,513)
Ashadh 680,000 675,000 38% 421,600 54,808 87,750 (32,942)
Shrawan 450,000 500,000 40% 270,000 35,100 65,000 (29,900)
Bhadra 300,000 275,000 55% 135,000 17,550 35,750 (18,200)
Ashwin 400,000 500,000 45% 220,000 28,600 65,000 (36,400)
Kartik 600,000 490,000 25% 450,000 58,500 63,700 (5,200)

Domestic Sales is computed by using the formula: Total Sales * Percentage of Domestic Sales
Domestic sales is taxed @13%
Export sales is taxed @0%, therefore, the total VAT collected on domestic sales is the amount of
output VAT.

The Institute of Chartered Accountants of Nepal 443


CAP-II Paper 7 - Income Tax and VAT

Conclusion
It is assumed that refund application is filed, as and when the condition for filing of application is fulfilled.
Month Conclusion
Since, export sales exceed 40% of total sales, M/s Banmali Traders may file an application for
Baishakh
VAT refund along with the VAT return of Baisakh itself.
Since, export sales exceed 40% of total sales, M/s Banmali Traders may file an application for
Jeshtha
VAT refund along with the VAT return of Jestha itself
Since, the export sales do not exceed 40% of total sales during the month, the excess VAT shall
be utilized to set off any payable VAT of Shrawan, Bhadra, Ashwin and Kartik. If there is any
Ashadh
excess after that, in the given case, the whole amount of Rs. 32942 is excess, M/s Banmali may
file refund application along with the VAT return of Kartik.
Since, the export sales do not exceed 40% of total sales during the month, the excess VAT shall
be utilized to set off any payable VAT of Bhadra, Ashwin, Kartik and Mangsir. If there is any
Shrawan
excess after such treatment, the remaining excess amount can be claimed as refund by filing an
application along with VAT return of Mangsir.
Since, export sales exceed 40% of total sales, M/s Banmali Traders may file an application for
Bhadra
VAT refund along with the VAT return of Bhadra itself
Since, export sales exceed 40% of total sales, M/s Banmali Traders may file an application for
Ashwin
VAT refund along with the VAT return of Ashwin itself
Since, the export sales do not exceed 40% of total sales during the month, the excess VAT shall
be utilized to set off any payable VAT of Mangsir, Poush, Magh and Falgun. If there is any excess
Kartik
after such treatment, the remaining excess amount can be claimed as refund by filing an application
along with VAT return of Falgun.

5) Silver Imports and Exports, Kathmandu has the following transactions of garments and ginger:
Particulars Jestha, 20X-74 Ashadh, 20X-74 Shrawan 20X-74
(Rs.) (Rs.) (Rs.)

Garments Export 3,00,000 6,00,000 10,00,000

Garments Local Sales 7,00,000 6,00,000 5,00,000

Ginger Export 1,00,000 3,00,000 4,00,000

Ginger Local Sales 4,00,000 5,00,000 6,00,000

Garment Purchase 8,00,000 7,00,000 10,00,000

Ginger Purchase 4,00,000 7,00,000 9,00,000

The company has no previous debit/credit balance in its VAT return. The Company needs your advice
to get refund of the VAT amount for these months. (Dec 2017, 5 Marks)

Answer
As per Sec. 24 of Value Added Tax Act, 2052, unless there is export sales exceeding 40% of total sales
during any month, any excess receivable VAT (i.e. when input VAT exceeds Output VAT) of a month shall
be utilized to set off any payable VAT of next four months. Any remaining excess receivable VAT after

The Institute of Chartered Accountants of Nepal 444


CAP-II Paper 7 - Income Tax and VAT
such action for consecutive four months can be refunded by making an application by the concerned
taxpayer along with the VAT return of fourth month or any months thereafter.

In case of export sales exceeding 40 percent of total sales during any month, the taxpayer may make an
application to refund any excess receivable VAT of such month along with the VAT return of the same
month.

Fact of the case


In the given case, the output VAT , input VAT and excess VAT receivable for the respective month are as
follows:

Particulars Notes Jestha Ashad Shrawan

Garments Export Taxable @ 0% 300,000 600,000 1,000,000


Garments Local Sales Taxable @13% 700,000 600,000 500,000
Ginger Export VAT Exempt 100,000 300,000 400,000
Ginger Local Sales VAT Exempt 400,000 500,000 600,000
Garment Purchase VAT paid @13% 800,000 700,000 1,000,000
Ginger Purchase VAT Exempt 400,000 700,000 900,000

Output VAT

Particulars Notes Jestha Ashad Shrawan


Garments Export Taxable @ 0% - - -
Garments Local Sales Taxable @13% 91,000 78,000 65,000
Ginger Export VAT Exempt - - -
Ginger Local Sales VAT Exempt - - -
Total Output VAT 91,000 78,000 65,000

Input VAT

Particulars Notes Jestha Ashad Shrawan


Garment Purchase VAT paid @13% 104,000 91,000 130,000
Ginger Purchase VAT Exempt - - -
Total Input VAT 104,000 91,000 130,000
Excess VAT

Particulars Jestha Ashad Shrawan


Total Output VAT 91,000 78,000 65,000
Total Input VAT 104,000 91,000 130,000
Payable/(Receivable) VAT for the month (13,000) (13,000) (65,000)

The Institute of Chartered Accountants of Nepal 445


CAP-II Paper 7 - Income Tax and VAT
There is excess Receivable VAT during each of Jestha, Ashad and Shrawan.
The percentage of export sales to total sales during Jestha, Ashad and Shrawan are as follows:

Percentage of Export Sales

Particulars Jestha Ashad Shrawan


Export Sales
(Sum of Export sales of garment and ginger) 400,000 900,000 1,400,000
Total Sales 1,500,000 2,000,000 2,500,000
Percentage of Export Sales 26.67% 45.00% 56.00%

Conclusion
As the export sales during Jestha 20X-74 is less than 40% of total sales, M/s Silver Imports and Exports
shall utilize the excess VAT of Rs. 13,000 during the month to set off any payable VAT of Ashad, Shrawan,
Bhadra and Ashwin. If the amount remains excess even after such utilization, M/s Silver Imports &
Exports may file a refund application along with VAT return of Ashwin 20X-74.

The export sales for Ashad and Shrawan exceeds 40% of total sales, therefore, M/s Silver Imports &
Exports can file a refund application to obtain refund of Rs. 13,000 of Ashad 20X-74 along with VAT return
of Ashad 20X-74 and Rs. 65,000 of Shrawan 20X-74 along with VAT return of Shrawan 20X-74.

6) ABC Oil & Ghee industry having factory at Lalitpur, Nepal producing Mustard oil, Vanaspati ghee and
other processed cooking oil in Nepal as given below. The sales of ABC Oil & Ghee industry to VAT
registered and unregistered person are in the ration of 40% and 60% respectively. The ABC Oil & Ghee
industry had following sales transaction during the month of Falgun, 20X-73:

S.N. Particulars Sales

1. Mustard oil- (Domestic mustard seeds) 6,00,000

2. Mustard oil- (Imported mustard seeds) 4,00,000

3. Vanaspati Ghee- (Domestically produced) 4,00,000

4. Vanaspati Ghee- (Imported) 6,00,000

5. Other processed cooking oil 10,00,000

Total 30,00,000

Whereas XYZ oil industry having factory at Bhaktapur import processed edible cooking oil in bulk
quantities and refill it in small packages before sale. The XYZ oil industry had following sales
transaction during the month of Falgun, 20X-73:
S.N. Particulars Sales

1. Refilled small oil packages (Imported processed 10,00,000


edible cooking oil in bulk quantities)

The Institute of Chartered Accountants of Nepal 446


CAP-II Paper 7 - Income Tax and VAT
Total 30,00,000

Required: (June 2017, 5 Marks)


Compute the amount of VAT refund available, if any, to ABC Oil & Ghee industry and XYZ Oil Industry
with reference to the provision of schedule 1 of VAT Act.

Answer
As per Sec. 24 of Value Added Tax Act, 2052, unless there is export sales exceeding 40% of total sales
during any month, any excess receivable VAT (i.e. when input VAT exceeds Output VAT) of a month shall
be utilized to set off any payable VAT of next four months. Any remaining excess receivable VAT after
such action for consecutive four months can be refunded by making an application by the concerned
taxpayer along with the VAT return of fourth month or any months thereafter.

In case of export sales exceeding 40 percent of total sales during any month, the taxpayer may make an
application to refund any excess receivable VAT of such month along with the VAT return of the same
month.

Conclusion:
In the given case, there is no excess receivable VAT (i.e. no information regarding input VAT is given),
Sec. 24 does not apply. There is no question regarding refund of VAT.

7) Sweets Ltd. which is not registered in VAT purchased 1,000 KG of cheese @ 1,130/KG with VAT from
Kathmandu Dairy. Subsequently Sweets limited sold 50% of cheese to individual customers. You, as a
professional expert advice, Sweets limited whether the company is eligible to claim VAT refund and
limitation there, if any, as per the provision of Value Added Tax Act, 2052. (Dec 2016, 5 Marks)

Answer
As per Sec. 24 of Value Added Tax Act, 2052, unless there is export sales exceeding 40% of total sales
during any month, any excess receivable VAT (i.e. when input VAT exceeds Output VAT) of a month shall
be utilized to set off any payable VAT of next four months. Any remaining excess receivable VAT after
such action for consecutive four months can be refunded by making an application by the concerned
taxpayer along with the VAT return of fourth month or any months thereafter.

In case of export sales exceeding 40 percent of total sales during any month, the taxpayer may make an
application to refund any excess receivable VAT of such month along with the VAT return of the same
month.

Conclusion:
In the given case, there is no excess receivable VAT (i.e. no information regarding input VAT is given),
Sec. 24 does not apply. There is no question regarding refund of VAT.

8) Answer the followings with reference to VAT Act/Rules (June 2017, 4×2.5=10)
a) State the provisions on input tax credit on VAT paid on the lost goods

The Institute of Chartered Accountants of Nepal 447


CAP-II Paper 7 - Income Tax and VAT
Answer
Application for Input Tax Credit & Time Limit for such application
In case the loss of asset by fire, theft, accident, accidental damages, terror, or riot compels a person to
write off the goods (assets) or sale it at lower selling rate, the person shall make an application in writing
to respective Inland Revenue Office along with evidence within 30 days of happening of such events.

Investigation and permission to claim as input tax credit


The tax office shall investigate the matter and finalize the quantum of tax credits to be allowed. On the
basis of such investigation, the tax office may allow the taxpayer to claim Input Tax Credit of VAT paid on
such assets (goods).

Right to claim Input tax Credit without confirmation from Tax Officer
In case the asset is insured, the tax office may allow the taxpayer to claim Input Tax Credit on such goods
to the extent of compensation paid by the Insurance Company, without any investigation from tax
officer.

b) Advise about the threshold of procurement for goods, service, and construction contract that a
public institution requires to procure mandatorily from VAT registered party.
Answer
Rs. 500,000

c) Can an unregistered person collect VAT?


Answer
The person who is not registered as per the provisions of VAT Act cannot collect VAT, except in following
circumstances:
i. Where the law specifically requires collection of VAT under Reverse Charging Mechanism
a. In case of import of service pursuant to Sec. 8 (2), when the buyer shall collect VAT from itself
b. In case of construction of building, apartment, etc. pursuant to Sec. 8 (3), when the buyer shall collect
VAT from itself
ii. Where the law specifically requires collection of VAT by Unregistered persons
a. In case of supply of wood pursuant to Sec. 12Ka, when the national forest, community forest, private
cultivated forest and private forest shall collect VAT even when they are not registered for VAT
b. In case of taxable supply of goods or services by persons specified in Sec. 15 (3), even if they are not
registered for VAT

d) Enumerate the transactions that are VAT attracted.


Answer
Value Added Tax shall be collected in following transactions, except when such goods or services are
listed in Schedule 1 of the Act:
i. Supply of goods or services in Nepal,
ii. Import of goods or services in Nepal, or
iii. Export of goods or services from Nepal.

The Institute of Chartered Accountants of Nepal 448


CAP-II Paper 7 - Income Tax and VAT
9) Horizon Pvt. Ltd. located at Chitawan, has following taxable sales and purchases without VAT in 20X-
72. Can the Pvt. Ltd. claim the refund in Kartik Bhadra return? If yes, how much? (Dec 2015, 5 Marks)
Month Sales (Rs.) Purchase (Rs.)
Baishakh 400,000 600,000
Jestha 300,000 400,000
Ashadh 500,000 550,000
Shrawan 600,000 700,000
Bhadra 400,000 450,000
Aswin 800,000 700,000
Kartik 700,000 500,000

Answer
As per Sec. 24 of Value Added Tax Act, 2052, unless there is export sales exceeding 40% of total sales
during any month, any excess receivable VAT (i.e. when input VAT exceeds Output VAT) of a month shall
be utilized to set off any payable VAT of next four months. Any remaining excess receivable VAT after
such action for consecutive four months can be refunded by making an application by the concerned
taxpayer along with the VAT return of fourth month or any months thereafter.

In case of export sales exceeding 40 percent of total sales during any month, the taxpayer may make an
application to refund any excess receivable VAT of such month along with the VAT return of the same
month.

Conclusion
In the given case, there are no export sales; therefore, M/s Horizon Pvt. Ltd. shall have to utilize excess
Receivable VAT of a month to set off payable VAT of next four months.

Output Input Payable /(Excess) Cumulative


Month Sales Purchase
VAT VAT VAT for the month Excess
Baishakh 400,000 52,000 600,000 78,000 (26,000) (26,000)
Jestha 300,000 39,000 400,000 52,000 (13,000) (39,000)
Ashadh 500,000 65,000 550,000 71,500 (6,500) (45,500)
Shrawan 600,000 78,000 700,000 91,000 (13,000) (58,500)
Bhadra 400,000 52,000 450,000 58,500 (6,500) (65,000)
Aswin 800,000 104,000 700,000 91,000 13,000
Kartik 700,000 91,000 500,000 65,000 26,000

The excess VAT of Baisakh will be utilized to set off VAT payable, if any, of Jestha to Bhadra. Since, there
is no any payable VAT for such months; the company may file a refund application to obtain refund of
Rs, 26,000 belonging to Baisakh along with VAT return of Bhadra 20X-72.
Similarly, excess VAT of Jestha will be utilized to set off VAT payable, if any, of Ashad to Ashwin. Since,
there is no any payable VAT for Ashad, Shrawan and Bhadra, but there is payable VAT for Ashwin. The
excess VAT of Rs. 13,000 will be utilized to set off payable VAT of Ashwin of Rs. 13,000 resulting into nil

The Institute of Chartered Accountants of Nepal 449


CAP-II Paper 7 - Income Tax and VAT
excess in Jestha. Excess of Jestha cannot be refunded if the company applied refund for the excess of
Baisakh along with return of Bhadra 20X-72.

10) Mr. John entered to Nepal via Birgunj with his family on 1st Aswin 20X-72 and visited so many places
in Nepal. The detail of few expenses incurred in Nepal by Mr. John & his family members are as follows:
a) Amount paid to tour operator for family tour package amounted to Rs. 339,000 including Rs.
39,000 applicable VAT.
b) Mr. John purchase one laptop costing Rs. 100,000 and paid applicable VAT on it.
c) Mr. John Paid to Hotel in Kathmandu for cost of lodging, foods & other services Rs. 226,000
including Rs. 26,000 VAT.
d) Mr. John with his family visited orphanage home in Kathmandu and donated a fridge costing Rs.
50,000 and paid applicable VAT on it.
At the time of departure from Tribhuvan International Airport, Kathmandu, Mr. John submitted all
VAT invoices to the counter of IRD and Mr. John is carrying the laptop purchased in Nepal along with
all his other belongings.

Advise Mr. John how much VAT refund he will get. (Dec 2015, 5 Marks)

Answer:
As per Sec. 25Ka of Value Added Tax Act, 2052, in case all the following conditions are satisfied, a foreign
tourist is entitled to refund of Value Added Tax paid in Nepal:
a. The foreign tourist visiting in Nepal shall leave Nepal via air route,
b. The value added tax is refunded only in respect of purchase of goods (and not services),
c. The foreign tourist shall accompany such goods with himself while leaving Nepal, i.e. goods cannot
be gifted in Nepal and it shall be taken away by the tourist; and
d. The cost of such goods shall be more than Rs. 25,000.

Service Fee on such Refund


The revenue authority shall deduct 3% of refundable amount as service charge.

Conclusion
In the given case, Mr. John leaves Nepal via air route. Therefore, the applicability of VAT refund is given
as below:
Particulars Applicability of Reason
refund
Amount paid to tour operator for family No Since, the procured item is not goods
tour package
Purchase one laptop, Mr. John is carrying Yes Since, the cost of goods (i.e. laptop) exceeds Rs.
the laptop purchased in Nepal along with all 25,000 and Mr. John is accompanying laptop with
his other belongings him while leaving Nepal
Paid to Hotel in Kathmandu for cost of No Since, the procured item is not goods
lodging, foods & other services
Donated a fridge to an orphanage home in No Though the cost of goods (i.e. fridge) exceeds Rs.
Kathmandu 25,000; however, Mr. John is not taking away
goods with him

The Institute of Chartered Accountants of Nepal 450


CAP-II Paper 7 - Income Tax and VAT

11) Healthy Oil Industries is manufacturing Mustard Oil. During shortage, the industry also, imports oil and
sell it to its customers to maintain its market. The VAT accounts of Healthy Oil Industries. Showed the
following data for fiscal year 2069/70.

Particulars Trading of Mustard Oil (import Manufacturing of


purchase and sales) Mustard Oil

A. VAT collected on sales

i From VAT registered party 17,00,000 18,00,000

ii From Non-VAT registered party 6,00,000 1,90,000

B. VAT paid on purchase 15,00,000 14,00,000

Calculate the VAT amount which Healthy Oil Industries is facilitated to get refund from Inland Revenue
Department. (Dec 2013, 3 Marks)

Answer:
As per Sec. 24 of Value Added Tax Act, 2052, unless there is export sales exceeding 40% of total sales
during any month, any excess receivable VAT (i.e. when input VAT exceeds Output VAT) of a month shall
be utilized to set off any payable VAT of next four months. Any remaining excess receivable VAT after
such action for consecutive four months can be refunded by making an application by the concerned
taxpayer along with the VAT return of fourth month or any months thereafter.

In case of export sales exceeding 40 percent of total sales during any month, the taxpayer may make an
application to refund any excess receivable VAT of such month along with the VAT return of the same
month.

Conclusion:
In the given case, there is no excess receivable VAT (i.e. output VAT exceeds input VAT), Sec. 24 does not
apply. There is no question regarding refund of VAT.

12) Swasthya Flour Mill Pvt. Ltd. is a Maida producing industry. It is a complete manufacturing unit, and
does not involve in local/import purchase of Maida. For fiscal year 20X-69/X-70, it had following VAT
transactions:
Particulars VAT Amount (Rs.)

A. VAT collected on sales

i. From VAT registered party 15,00,000

ii. From Non-VAT registered party 2,00,000

B. VAT paid on purchase 13,00,000

The Institute of Chartered Accountants of Nepal 451


CAP-II Paper 7 - Income Tax and VAT
What amount of VAT refunds is Swasthya Flour Mill Pvt. Ltd. facilitated?
(Dec 2013, 3 Marks)
Answer
As per Sec. 24 of Value Added Tax Act, 2052, unless there is export sales exceeding 40% of total sales
during any month, any excess receivable VAT (i.e. when input VAT exceeds Output VAT) of a month shall
be utilized to set off any payable VAT of next four months. Any remaining excess receivable VAT after
such action for consecutive four months can be refunded by making an application by the concerned
taxpayer along with the VAT return of fourth month or any months thereafter.

In case of export sales exceeding 40 percent of total sales during any month, the taxpayer may make an
application to refund any excess receivable VAT of such month along with the VAT return of the same
month.

Conclusion:
In the given case, there is no excess receivable VAT (i.e. output VAT exceeds input VAT), Sec. 24 does not
apply. There is no question regarding refund of VAT.

13) State the provisions on Value Added Tax (VAT) refund under Value Added Tax Act, 2052. (Dec 201/
2011, 5 Marks)
Or
State the provisions regarding the refund of VAT excess paid. (Dec 2008, 3 Marks, CA Inter)

Answer
As per Sec. 24 of Value Added Tax Act, 2052, unless there is export sales exceeding 40% of total sales
during any month, any excess receivable VAT (i.e. when input VAT exceeds Output VAT) of a month shall
be utilized to set off any payable VAT of next four months. Any remaining excess receivable VAT after
such action for consecutive four months can be refunded by making an application by the concerned
taxpayer along with the VAT return of fourth month or any months thereafter.
In case of export sales exceeding 40 percent of total sales during any month, the taxpayer may make an
application to refund any excess receivable VAT of such month along with the VAT return of the same
month.

Where a person makes an application for refund of excess VAT, if the VAT refund is due to the reason of
export sales exceeding 40% of total sales during any months, VAT shall be refunded within 30 days of
application and within 60 days in all other cases.

In case of failure to refund within such specified days, interest shall be paid @15% per annum for the
period beginning from the date of expiry of due date and ending on the day of refund.

14) Mr. Cristiano Messi, a foreign tourist, visited Nepal for a month and is now returning to his home
country Japan. He has purchased many goods in Nepal and paid VAT on such purchase. He seeks your
advice on refund of VAT on paid by him on purchase and expenses/costs associated with such refund.
(June 2012, 5 Marks)

The Institute of Chartered Accountants of Nepal 452


CAP-II Paper 7 - Income Tax and VAT
Answer:
As per Sec. 25Ka of Value Added Tax Act, 2052, in case all the following conditions are satisfied, a foreign
tourist is entitled to refund of Value Added Tax paid in Nepal:
a. The foreign tourist visiting in Nepal shall leave Nepal via air route,
b. The value added tax is refunded only in respect of purchase of goods (and not services),
c. The foreign tourist shall accompany such goods with himself while leaving Nepal, i.e. goods cannot
be gifted in Nepal and it shall be taken away by the tourist; and
d. The cost of such goods shall be more than Rs. 25,000.

Service Fee on such Refund


The revenue authority shall deduct 3% of refundable amount as service charge.

Conclusion:
In the given case, if all the above conditions are satisfied, Mr. Cristiano Messi is entitled to refund of VAT
on such goods. The cost for Mr. Messi is 3% of refundable amount.

15) Ginger Cellular Ltd., a cellular mobile Phone Set producing domestic enterprise, is engaged in
producing and selling of cellular mobile phone sets. As a tax expert, Ginger Ltd. seeks your advice on
the refund of VAT paid on purchase of its raw materials. Give your opinion on this. (Dec 2010, 5 Marks)

Answer:
As per Sec. 24 of Value Added Tax Act, 2052, unless there is export sales exceeding 40% of total sales
during any month, any excess receivable VAT (i.e. when input VAT exceeds Output VAT) of a month shall
be utilized to set off any payable VAT of next four months. Any remaining excess receivable VAT after
such action for consecutive four months can be refunded by making an application by the concerned
taxpayer along with the VAT return of fourth month or any months thereafter.

In case of export sales exceeding 40 percent of total sales during any month, the taxpayer may make an
application to refund any excess receivable VAT of such month along with the VAT return of the same
month.

The same provision is applicable to M/s Ginger Cellular Ltd.

16) A dealer manufactured goods worth Rs. 10 million which were exempt under schedule I of VAT Act out
of the goods purchased partly VAT exempt and partly goods liable to VAT. VAT exempt goods
amounting to Rs. 7 million were used for the production of goods which were exported. The Dealer
claims that all the exports were zero rated and he should get full credit for VAT on exempted goods.
Consider his claim and to set off the VAT paid by him on the inputs. (Dec 2010, 4 Marks)

Answer:
As per Sec. 5 (3) of Value Added Tax Act, 2052, no VAT shall be levied on items listed in Schedule 1 of the
Act. Value Added Tax paid in purchases used to produce such non-taxable output can neither be claimed
as input tax credit nor be refunded.

The Institute of Chartered Accountants of Nepal 453


CAP-II Paper 7 - Income Tax and VAT
There shall not be applicability of VAT on domestic or export sales of such items, which are listed in
Schedule1 of the Act.

Therefore, the dealer cannot claim input tax credit or refund of VAT in respect of VAT paid on input that
are used to produce non-taxable output.

17) What are the conditions for refund of VAT to the foreign tourist? Answer citing the provisions of the
Act. (June 2010& Dec 2009, 5 Marks)
Answer
As per Sec. 25Ka of Value Added Tax Act, 2052, in case all the following conditions are satisfied, a foreign
tourist is entitled to refund of Value Added Tax paid in Nepal:
a. The foreign tourist visiting in Nepal shall leave Nepal via air route,
b. The value added tax is refunded only in respect of purchase of goods (and not services),
c. The foreign tourist shall accompany such goods with himself while leaving Nepal, i.e. goods cannot
be gifted in Nepal and it shall be taken away by the tourist; and
d. The cost of such goods shall be more than Rs. 25,000.

Service Fee on such Refund


The revenue authority shall deduct 3% of refundable amount as service charge.

18) ABC Export House has a credit of Rs. 200,000 in the first month representing 30% of its total sales
during that month. However, it has a credit of Rs. 600,000 representing 60% of its total sales in the
following month.

How does ABC Export House claim refund if it does not always have exports over 50 40% of total sales?
(June 2010, 5 Marks)

Answer:
As per Sec. 24 of Value Added Tax Act, 2052, unless there is export sales exceeding 40% of total sales
during any month, any excess receivable VAT (i.e. when input VAT exceeds Output VAT) of a month shall
be utilized to set off any payable VAT of next four months. Any remaining excess receivable VAT after
such action for consecutive four months can be refunded by making an application by the concerned
taxpayer along with the VAT return of fourth month or any months thereafter.
In case of export sales exceeding 40 percent of total sales during any month, the taxpayer may make an
application to refund any excess receivable VAT of such month along with the VAT return of the same
month.
Conclusion
In the given case, the person's total export sales do not exceed 40% of total sales during first month,
therefore, the excess receivable VAT of such month shall be utilized to set off any payable VAT of next
four months. Any remaining excess receivable VAT after such action for consecutive four months can be
refunded by making an application by the concerned taxpayer along with the VAT return of fourth month
or any months thereafter.

The Institute of Chartered Accountants of Nepal 454


CAP-II Paper 7 - Income Tax and VAT
Further, the person's export sales exceed 40% of total sales in the second month; therefore, he can make
an application to claim refund of excess receivable VAT of same month along with VAT return of same
month.

19) A Foreign diplomat to Nepal (recognized by foreign ministry in Nepal) during his stay of last 5 years in
Nepal had purchased goods. He has various invoices / Tax Invoices and seeks your advice whether he
can get refund of VAT on purchases made by him during the stay. Being the tax consultant put forward
your advises as per the VAT Act & Rules. Is there any form designed by IRD for such purpose? (Dec
2009, 3 Marks, CA Inter)

Answer:
As per Sec. 25, a foreign diplomat can avail refund facility on VAT paid by him/her on consumption in
Nepal, if all the following conditions are satisfied:
a. The person is accredited as diplomatic person by Ministry of Foreign Affairs,
b. The country represented by the diplomatic person provides refund facility to Nepali diplomats in
their country, i.e. refund is availed on the basis of reciprocity,
c. The diplomat makes an application within 3 years of the purchase, and
d. The purchase, i.e. individual invoice, belonging to refund application shall not be less than Rs. 10,000.
In the given case, the foreign diplomat can make an application for refund, if he satisfies all the above
conditions.

Form designed by IRD


Yes, a form is prescribed by Government of Nepal in Schedule 17 of Value Added Tax Regulations, 2053.

Further issues of refund:


Such refunds should normally be made within 30 days of application. Amount exceeding Rs. 20,000 shall
be refunded by way of depositing in beneficiary's bank account only.

20) Mr. Shyam, a businessman, submits his VAT return for the month of Falgun according to which total
sales for the month was NRs. 1,000,000 and purchase was NRs. 600,000. Out of the total sales, NRs.
650,000 was export to Tibet (Non L/C). He had no previous debit/credit balance in his VAT return. He
claims the refund of the excess tax paid on purchase on 25th of Chaitra.
State the time and amount of claim he is entitled to. Also mention the documents (if any) he requires
to claim the refund. (June 2009, 5 Marks)

Answer
As per Sec. 24 (3) of Value Added Tax Act, 2052, a person may make an application for refund of VAT of
any month with the VAT return of same month in case the person's export sales is more than 40% of
total sales during the same month.
In the given question, the percentage of export sales is 65%, which is more than 40% of total sales.
Therefore, Mr. Shyam may make an application for VAT refund along with the VAT return of Falgun.

The documents required to be submitted are as follows:


a. Copy of agreement between buyer & seller.

The Institute of Chartered Accountants of Nepal 455


CAP-II Paper 7 - Income Tax and VAT
b. Copy of Pragyapan Patra of export.
c. Copy of Pragyapan Patra of import.
d. Copy of Payment Certificate

The Institute of Chartered Accountants of Nepal 456


CAP-II Paper 7 - Income Tax and VAT

Chapter 5: True False Question


1) Answer the followings with reference to Value Added Tax Act/Rules, whether the statements are
correct or not, with reasons. (December 2020, 5×1=5)
a. While filing an appeal to Revenue Tribunal, the undisputed amount of the assessed Tax due shall
have to be deposited and fifty percent of the amount of the Tax in dispute plus the amount of fine
shall have to be deposited.
b. In case the person has not made the accounts available for inspection under section 16 (1), the
penalty amount is Rs. 5,000 for each time.
c. The color of tax registration number plate for registered person shall be green, for unregistered
person doing Taxable transaction shall be yellow and for person dealing in tax exempted goods
and services shall be white.
d. Nepal Government is not required to collect Value Added Tax even if it engages in supply of goods
or services which attract VAT.
e. No Tax is to be levied for a supply of service by a person residing in Nepal to a person outside Nepal
who has no business transaction in Nepal.

Answer:
a. True.
As per section 33 of Value Added Tax Act, 2052, while filing an appeal to Revenue Tribunal, the
undisputed amount of the assessed tax due shall have to be deposited and fifty percent of the
amount of the tax in dispute plus the amount of fine shall have to be deposited or a bank guarantee
for the same has to be furnished.

b. False.
As per section 29 (1) (nga) in case the person has not made the accounts available for inspection
under section 16 (1), the penalty amount is Rs. 20,000 for each time.

c. True.
As per Rule 14(ka) of Value-Added Rules, 2053, registered person shall have to place tax
registration number plate in the format prescribed by Inland Revenue Department (IRD) in his
place of transaction in a clearly visible manner within thirty days from the effective date of VAT
rule.
Further, as per circular dated 2062.06.21 by IRD, the color of tax registration number plate for
registered person shall be green, for unregistered person doing taxable transaction shall be yellow
and for person dealing in tax exempted goods and services shall be white.

d. False.
Notwithstanding anything contained in subsection (1) or (2) of section 15, local body or
International institution/association/commission based in Nepal or Nepal Government or
corporations engaged in non-VAT transactions shall collect value added tax if they engage in
supply of goods or services which attract VAT.

The Institute of Chartered Accountants of Nepal 457


CAP-II Paper 7 - Income Tax and VAT
e. False.
As per clause 2 of schedule 2 of Value Added Tax, 2052, a supply of service by a person residing in Nepal
to a person outside Nepal, who has a no business transaction, business representative or legally
recognized agent in Nepal, is subjected to zero rate of tax.

2) Answer the followings with reference to Value Added Tax Act/Rules, whether the statements are
correct or not, with reasons. (July 2015, 5×1=5)
a) While filing an appeal to Revenue Tribunal, the undisputed amount of the assessed Tax due shall
have to be deposited and fifty percent of the amount of the Tax in dispute plus the amount of fine
shall have to be deposited.
True

b) In case the person has not made the accounts available for inspection under section 16 (1), the
penalty amount is Rs. 5,000 for each time.
False, the fine is Rs. 20,000 every time.

c) The color of tax registration number plate for registered person shall be green, for unregistered
person doing Taxable transaction shall be yellow and for person dealing in tax exempted goods
and services shall be white.
False, the color of VAT plate shall be as follows:
Persons Color of Alphabet Color of background
VAT registered person White green
Person dealing in taxable transaction, but not Black Yellow
registered due to turnover threshold
Person dealing in Tax Exempt Items Red White

d) Nepal Government is not required to collect Value Added Tax even if it engages in supply of goods
or services which attract VAT.
False, it has to collect VAT on supply of taxable goods or services as per Sec. 15 (3).

e) No Tax is to be levied for a supply of service by a person residing in Nepal to a person outside Nepal
who has no business transaction in Nepal.
Answer
False, the VAT rate is Zero percent. VAT shall be levied.

3) Explain whether the following statements are true or false with reasons: (Dec 2014, 5×1=5)
a) A person shall apply for the close of taxable transaction in the format prescribed in Schedule 10 of
VAT Regulation, 2053.
False, the form is prescribed in Schedule 11.

The Institute of Chartered Accountants of Nepal 458


CAP-II Paper 7 - Income Tax and VAT
b) A person shall be imposed fine of Rs. 10,000 at each time in case of refusal for the inspection of
books of account by the Tax officer.
False, the penalty to be imposed is Rs. 20,000 every time.

c) The Tax paid by a diplomatic body or diplomat on the purchase of taxable goods or services shall
not be refunded if the amount purchase is less than Rs. 5,000 at one time.
False, the amount of purchase shall not be less than Rs. 10,000.

d) Tax Officer may assess Tax as per section 20 if Kathmandu District Development Coordination
Committee collects the tax on taxable transactions and deposit in its internal revenue account.
False, as per Sec. 15 (3), it is the duty of Kathmandu District Coordination Committee to collect VAT on
any taxable supplies even without VAT registration.

e) A foreign tourist can take refund of VAT on the purchase of goods amounting to Rs. 15,000 or more.
False, the amount is more than Rs. 25,000.

4) State with reasons whether the following statements are true or false with reference to Value Added
Tax Act/Rules. (June 2014, 5×2=10)
a) 'No VAT' and 'Zero VAT' have the same meaning as VAT in both the cases is zero.
False, it is due to the following reason:

Basis Zero Rate No VAT


Rate of VAT VAT is levied at Zero Percent VAT is not levied at all
VAT Invoice A VAT Invoice shall be issued as VAT Invoice is not required to be
per the format prescribed by VAT issued for transaction of such items
regulation
VAT credit on Full VAT credit, which means No VAT credit, which means, if
purchases there will not be any load of value value added tax is paid at the time of
(input) added tax on output purchase, such value added tax will
become part of output
Registration The person dealing in Zero rated The person dealing exclusively in No
Requirement items shall be registered for VAT VAT items is relieved from
purpose. registration formality
Other The person dealing in Zero Rated The person dealing exclusively in No
formalities as Items shall observe other VAT items is relieved from all
per the Act formality as per VAT Act, such formalities to be observed as per the
as, maintenance of sales or Act
purchase register duly certified
from Tax Officer, submission of
VAT return, etc.

b) In the case where there is provision of a contract for paying partly the value of goods or services
in more than one day on an installment basis, the time of supply shall be the date of payment.

The Institute of Chartered Accountants of Nepal 459


CAP-II Paper 7 - Income Tax and VAT
False, as per Sec. 6 (3), the time of supply shall be earlier of following:
• Date of payment, or
• Date of due date of installment

c) Roy & Co. is a VAT registered firm engaged in the business of importing passengers' car and selling
them in local market. The firm is claiming full input tax credit on purchase of such cars.
True, since the person's main business is dealing in sales and purchase of cars.

d) If a taxpayer fails to submit tax return as per section 18, the penalty imposed is Rs. 10,000 per
month.
False, penalty to be imposed shall be higher of the following amount:
• 0.05% per day of VAT due, or
• Rs. 1,000 per tax period

e) A separate record for purchase and sale shall be maintained for the used goods which have
purchase price more than Rs. 20,000.
False, such record shall be maintained for purchase of any value.

5) Identify the following questions as True or False with reason:


(June 2012, 10 Marks, CA Inter)

a) In F.Y. 2068/69 Value Added Tax will be imposed and recovered as per Finance Act, 2068.
True, as per Value Added Tax Act, 2052 as amended by Finance Act of each year.

b) VAT is not levied on tea leaf.


True.

c) To make administrative review, the applicant should place one third of disputed amount as bank
guarantee.
False, firstly the amount is one-fourth of disputed amount and 100% of undisputed amount and secondly,
the amount shall be deposited.

d) Rs. 500 thousand penalty will be charged if a person who has taken approval for computer billing
erases or alters the data using another software.
True.

The Institute of Chartered Accountants of Nepal 460


CAP-II Paper 7 - Income Tax and VAT
e) Interest rate for the purpose of VAT is 10 p.c.
False, it is 15 percent.

6) State with reasons whether the followings are True or False. (Dec 2011, 5×2=10, CA Inter)
a) Public corporation who does not transact dealing in vatable VAT attractive goods and services is
not required to be registered with VAT, but if it sells vatable VAT attractive goods and services, it
should collect the VAT and deposit into government revenue account.
Answer
True, it is as per Sec. 15 (3).

b) If any assessee goes for administrative review, he should deposit one third of disputed assessed
amount as bank guarantee.
Answer
False, the amount is one-fourth of disputed tax amount and 100% of undisputed tax amount in cash.

c) If any taxpayer does not pay the self-declared VAT within 25 days of next month of particular tax
period, he should pay 15% p.a. as additional fee on unpaid VAT amount.
Answer
False, the rate for additional fee is 10% p.a. as per Sec. 19 (2).

d) In Nepal, prevailing VAT Rate is 13 % and 0 %.


Answer
True, as per Sec. 7, 13% VAT is applied in domestic transaction of VAT attractive items and 0% VAT is
applicable on items listed in Schedule 2.

e) Abbreviated tax invoice is issued as format prescribed in annexure 6 of VAT Rules, 2053.
Answer
True

7) State with reason whether the following statements are True or False.
(Dec 2010, 10 Marks)
a) A person shall apply for the registration of VAT in the format prescribed in Schedule – 3 of VAT
Regulation, 2053.
Answer
True, the format is prescribed in Schedule 1 of Value Added Tax Regulation, 2053.

b) A shopkeeper can issue abbreviated tax invoice for the transaction up-to Rs.10,000.
Answer
True, only if the shopkeeper obtains approval from tax officer and he is a retailer.

c) A registered person shall preserve all the VAT records up-to 6 years.
Answer
True

The Institute of Chartered Accountants of Nepal 461


CAP-II Paper 7 - Income Tax and VAT

d) A separate record for purchase and sale shall be maintained for the used goods which have
purchase price more than Rs. 20,000.
Answer
False, such record shall be maintained for each goods.

e) Government organizations shall take VAT bill purchasing goods and services above Rs. 5,000.
Answer
False, as per Rule 56 of Value Added Tax Regulation, 2053, the value is Rs. 20,000.

f) The rate of VAT for the import of goods and services will be same as VAT applicable for the
purchase of goods and services in Nepal.
Answer
True

g) In the case of purchase and sale of land and building, VAT is not applicable.
Answer
True, it is listed in Group 12 of Value Added Tax Act, 2052.

h) In the case of import of goods from USA, the place of transaction is USA.
Answer
False, as per Rule 15, place of supply is in customs frontier.

i) A foreign tourist can take refund of VAT on the purchase of goods amounting to Rs. 15,000 or more.
Answer
False, the amount shall be more than Rs. 25,000 as per Sec. 25Ka of Value Added Tax Act, 2052.

j) VAT will not applicable in cable car service.


Answer
False

8) What will be the consequences under VAT Act in the following case?
(June 2004, 5 Marks, CA Inter)
a) Mr. A fails to register his business within the stipulated time mentioned in Sec. 10 of the VAT Act.
Answer
Penalty under Sec. 29 shall be levied @ NRs. 20,000 every time.

b) B, registered in VAT issue the tax invoice and collect tax.


Answer
The person shall pay payable VAT and submit VAT return within 25 days of end of tax period.

c) C, not registered in VAT, issue the tax invoice and collects tax.
Answer

The Institute of Chartered Accountants of Nepal 462


CAP-II Paper 7 - Income Tax and VAT
Tax officer shall recover VAT so collected along with additional amount equal to 100% of VAT so collected.

d) D, registered in VAT does not use certified purchase and sales books.
Answer
Penalty shall be imposed @ Rs. 10,000.

e) E, registered in VAT fails to file return and pay tax in due time.
Answer
For failure to file VAT return, penalty shall be imposed which is higher of following:
a. 0.05% per day of VAT payable, or
b. Rs. 1,000 per tax period

For failure to pay VAT by the due date, penalty shall be imposed as follows:
c. Fees @ 10% p.a. on such payable VAT amount, and
d. Interest @ 15% p.a. on such payable VAT amount

The Institute of Chartered Accountants of Nepal 463


CAP-II Paper 7 - Income Tax and VAT

Chapter 6: Short Notes


1) Define Electronic Media and Registered Person as per the VAT Act, 2052. (Dec 2012, 2
Marks, CA Inter)

Answer
Electronic Media:
“Electronic Media” means computer, internet, email, facsimile, electronic cash register, fiscal
printer, or similar approved media and the term includes any other similar media prescribed by
the Department having similar characteristics.

Registered Person:
“Registered person” means any person who is registered to conduct transactions pursuant to
Section 10 or Section 10A or Section 10B.

2) Define “Person”, “Goods” and “Service” as per VAT Act.


(June 2011, 5 Marks, CA Inter)
Answer
Person: “Person” means any natural person, firm, company, association, institution, partnership
firm, cooperative, joint business, religious endowment, or fund; and the term also includes any
government body, any religious organization, charitable trust or similar other bodies and
branches or sub-branches, with or without any profit motive, involved in Taxable Transactions.

Goods: Goods means both movable and immovable assets.

Service: Service means anything other than goods.

3) Explain about the “Tax Periods” under the Value Added Tax Act/Rules
(June 2013, 5 Marks/(Dec 2016/2017/2020, 2.5 Marks))
Answer
The tax period, as per VAT law, are as follows:
i. Monthly Tax Period (Every Nepalese Calendar Month as Tax Period)
Every registered person, other than those who opt for bimonthly or tri-annual tax period as per
the regulation shall follow monthly tax period for VAT accounting.
ii. Tri-annual Tax period
The following registered persons are eligible to choose trimester tax period:
a. Brick Industry
b. Newspaper publication house
c. Newspaper and electronic publication and transmission house
d. Hotel
e. Tourism Business
f. Cinema Hall

The Institute of Chartered Accountants of Nepal 464


CAP-II Paper 7 - Income Tax and VAT
g. Transport business
These businesses may make an application in front of tax officer to keep their VAT accounting on
trimester basis. Where such application is filed, tax officer must cause to use trimester tax period for
VAT purpose.
It is upon the tax officer to decide the beginning and end of trimester tax period.

iii. Different tax period


In case of a registered person that has maintained its accounts using software cannot generate the
required report in Nepalese Calendar system, it may file an application to Tax Officer to adopt different
tax period with same tax period. If the Tax Office thinks it appropriate, the person may be granted a
different tax period.

iv. First Tax period


First tax period of a registered person shall be period of day of registration to end of concerned tax
period.
Readers should note that this is not a separate tax period, but a provision to clarify that the tax period
may begin at the middle of any month but shall always end in the end of month. This also clarifies that
the tax periods are uniform.

v. No-tax period
In case where any registered person applying cancellation of registration is not prompted with the
decision of tax officer within three months from the date of filing of application, the person is not
required to file any tax return after the elapse of such three months.

4) Write Short Notes on following:


1. No VAT credit (June 2022, 2.5 Marks)
Answer:
In any of the following circumstances, VAT paid on purchases cannot be claimed as input tax credit:
a. If the intended use of purchase is to generate non-taxable output,
b. In case the purchase is made by non-registered person,
c. In case there is no proper Value Added Tax invoice (other than abbreviated tax invoice) to
substantiate VAT paid except when VAT is paid under import (where import documents
substitute VAT invoice), u/s 8 (2), 12Ka and 15 (3).

2. De-registration from VAT (June 2022, 2.5 Marks)


U/s 11, the Tax Officer shall cancel the registration of a registered person in any of the following
circumstances:
a. In the case of body corporate, if the body corporate is closed down, sold or transferred or if the
body corporate in any manner ceases to exist;
b. In the case of an sole proprietorship, if the owner dies;
c. In the case of a partnership firm, if it is dissolved or the partner dies;
d. If a registered person ceases to be engaged in taxable transactions;

The Institute of Chartered Accountants of Nepal 465


CAP-II Paper 7 - Income Tax and VAT
e. If the taxpayer files for zero return or does not file return at all within a consecutive period of
twelve months,
f. If the taxable transaction of the taxpayer does not reach fifty lakhs in case of goods, and twenty
lakhs in case of services and transaction mixed with goods and services in past twelve months,
g. if registered by error.
Taxpayer shall produce his or her documents for auditing within 15 days of the filing of the tax
returns for cancellation of registration; and the Tax Officer shall perform the audit of such a
taxpayer, and inform the taxpayer of cancellation of registration or if the registration is not to be
so cancelled, the Tax Officer shall give information thereof to the taxpayer. If the Tax Officer
does not cancel registration or make any decision to cancel registration within that period, the
taxpayer is not obliged to file the tax returns subsequent to that period.

3. Tax officer (Dec 2019, 2.5 Marks)


Answer
“Tax officer” means any Tax Officer or Chief Tax Officer or Chief Tax Administrator appointed by the
Government of Nepal and the term includes Section Officer or Director or Deputy Director General of
the Department or any other Officer designated by Government of Nepal who is empowered to use the
power of a tax officer in accordance with the provisions of Value Added Tax Act, 2052.

4. Electronic invoice (Dec 2019, 2.5 Marks)


Answer
A taxpayer may issue invoice electronically with prior approval from the Department.The Department
may publish a public notice to cause the taxpayers prescribed in the notice for compulsory issuance of
electronic invoice and order to link the electronic medium of the taxpayer to Central Bill Monitoring
System of Department.

Department shall develop and implement a procedure in relation to integrity and security of software or
equipment that issue electronic invoicing. The concerned Producer, distributor and user shall comply
such procedures.

5. Refund of VAT paid on purchase by foreign tourist (Dec 2019, 2.5 Marks)
Answer
Where a foreign tourist on a visit to Nepal returning from Nepal via air route purchases goods worth
more than Rs. 25,000 and where such goods are accompanied by the tourist at the time of departure
from Nepal, tax paid on such goods shall be refunded as per the procedures prescribed by the
Department.
An amount to the extent of 3% of the refundable amount shall be deducted at the time of refund as
service fee.

6. Suspension of transaction of registered person (Dec 2019, 2.5 Marks)


Answer

The Institute of Chartered Accountants of Nepal 466


CAP-II Paper 7 - Income Tax and VAT
Where a registered person commits any offences as specified in Section 29 of Value Added Tax Act, 2052
for two or more times, the Director General may order the tax officer to suspend such person's place of
transaction up to 7 days so that the transactions are not carried out.

7. Market Value (Dec 2018/ June 2016/June 2022)


Answer
The market value of a supply of goods or services at a given date shall be the consideration in money
which the supply would generally fetch if supplied in similar circumstances at that date, being a supply
freely offered and made between persons who are not related persons.

While determining the Market Value, Tax Officer shall determine the market value based after studying
the price and transaction of similar other registered seller dealing in similar goods.
Where the market value cannot be determined by tax officer, the Director General shall determine the
market value on the basis of information received from the registered suppliers dealing in similar goods.

8. Collection of tax from other than registered person (Dec 2018/2021, 2.5 Marks)
Answer
Value Added tax is collected from both registered and unregistered persons by a registered person while
supplying taxable goods or services as per Sec. 5 of Value Added Tax Act, 2052. Under the circumstances
as mentioned in Sec. 15 (3), any unregistered supplier specified therein shall collect value added tax on
transaction of taxable goods or services.
An unregistered recipient of service from foreign party shall levy and collect VAT under reverse charging
principle, if the foreign party is not registered in Nepal.

Further, an unregistered owner of building, apartments, shopping complex built for commercial purpose
shall levy and collect VAT under reverse charging principle, if the cost of such structures exceed Rs. 50
Lakhs and the builder of such structure is unregistered person under Sec. 8 (3).

9. Zero rated and VAT exempted goods/services (Dec 2018, 2.5 Marks)
Answer

Basis Zero Rate No VAT


Rate of VAT VAT is levied at Zero Percent VAT is not levied at all
VAT Invoice A VAT Invoice shall be issued as VAT Invoice is not required to be
per the format prescribed by VAT issued for transaction of such items
regulation
VAT credit on Full VAT credit, which means there No VAT credit, which means, if value
purchases will not be any load of value added added tax is paid at the time of
(input) tax on output purchase, such value added tax will
become part of output
Registration The person dealing in Zero rated The person dealing exclusively in No
Requirement items shall be registered for VAT VAT items is relieved from
purpose. registration formality

The Institute of Chartered Accountants of Nepal 467


CAP-II Paper 7 - Income Tax and VAT
Other The person dealing in Zero Rated The person dealing exclusively in No
formalities as Items shall observe other formality VAT items is relieved from all
per the Act as per VAT Act, such as- formalities to be observed as per the
maintenance of sales and purchase Act
register duly certified from Tax
Officer, submission of VAT return,
etc.

10. Valuation at market price by tax officer (Dec 2018, 2.5 Marks)
Answer
In the following circumstances, the tax officer uses market value to determine taxable value of any
goods or services:
i. When goods or services are exchanged in consideration of goods or services,
ii. When a person supplies goods or services obtaining partial consideration,
iii. When there is sufficient reason to prove that the supplier is doing under-invoicing,
iv. When a person cancels its registration, the goods and capital items on which it has taken input tax
credit,
v. When a person stops using a capital item on which it has taken input tax credit in taxable
transaction, or
vi. When a tax officer finds there is shortage of stock during physical inspection, and appropriate legal
procedure as per Rule 39Ka is not fulfilled or there is no appropriate documentation to support the
shortage

11. Timing of goods supplied (June 2018, 2.5 Marks)


Answer
In case of supply of goods, earliest of following shall be time for supply under Sec. 6(2):
i. Date of issuance of invoice, or
ii. Date of possession of goods/ date of removal of goods by the recipient from the supplier’s business
place, or
iii. Date of receiving consideration by the supplier
In case of application of one or more conditions at once as mentioned above, the time of
supply shall be determined on subjective basis as prescribed by Director General.

12. Person (June 2018 & Dec 2016, 2.5 Marks)


Answer
“Person” means any natural person, firm, company, association, institution, partnership firm,
cooperative, joint business, religious endowment, or fund; and the term also includes any government
body, any religious organization, charitable trust or similar other bodies and branches or sub-branches,
with or without any profit motive, involved in Taxable Transactions.

13. Proportionate credit (June 2018& June 2016, 2.5 Marks)


Answer

The Institute of Chartered Accountants of Nepal 468


CAP-II Paper 7 - Income Tax and VAT
In case a person deals both in VAT attractive items and Exempted Items, the following principle shall be
applicable:
i. VAT paid on purchase that is specifically identifiable to VAT attractive items is allowed at full while
claiming Input VAT Credit.
ii. VAT paid on purchase that is specifically identifiable to Exempted Goods is not eligible for any Tax
Credit.
iii. VAT paid on such goods or services that are not directly identifiable to either of the Exempted Items
or Tax attracting Items shall be eligible for Proportionate Tax Credit.
The proportion shall be determined on the basis of specific proportion of usage of input for taxable and
nontaxable output. If the proportion cannot be determined on the basis of direct relation between
taxable and nontaxable output, the credit shall be claimed in the proportion of Taxable Sales & Exempted
Sales of the period. The VAT attributable in the proportion of Taxable Sales to Total Sales during the
period shall be eligible for Input Tax Credit, whereas that attributable to Exempted Sales is not allowed
for set off.

14. VAT refund to diplomats (June 2018, 2.5 Marks)


Answer
The following diplomats or persons entitled to diplomatic privileges are eligible to make an application
for refund:
i. VAT paid in Nepal by diplomatic persons privileged on a reciprocal basis from Ministry of Foreign
Affairs, or
ii. VAT paid in Nepal by person having diplomatic privileges engaged in Regional or International
Organization or missions.

These persons shall be entitled to refund of VAT paid by them to the extent of their own consumption.
The application for refund shall be made directly to Inland Revenue Department, along with the
recommendation letter from Ministry of Foreign Affairs.
The tax shall not be refunded for a purchase of less than Rs. 10,000 at a time.
The refund application shall be filed within three years of such transactions.

15. Circumstances beyond Control (Dec 2017/2020, 2.5 Marks)


Answer
The following circumstances shall be deemed as Circumstances beyond Control:
i. In case of incapacity of person required to pay tax due to falling ill, up-to 7- days of the date of
recovery,
ii. In case the person required to pay tax is in death ritual; up to 7- days of the end of the such ritual,
iii. In case a woman required to pay tax delivers a child, up to 35 days of the date of delivery,
iv. In case the person required to pay tax dies or becomes insane or disappears and his/ her heir or
guardian submits an application within 35- days of the date of such incident, up to 7 days of the
receipt of application,
v. In circumstances where the person required to pay tax has not been able to come to the tax office
because of the closure of the path due to floods, landslides or other similar reasons, up to 7 days of
the opening of the road,

The Institute of Chartered Accountants of Nepal 469


CAP-II Paper 7 - Income Tax and VAT
vi. In cases where person cannot come due to transport stoppage, up to the next day of the end of
such stoppage.
vii. In circumstances beyond control created by the act of god-like fire, earth-quake, within 30- days
from such calamity.

Where additional time limit is required to be requested due to circumstances beyond control referred
to in clauses (i) , (ii) , (iii) , (iv) , (v) , and (vii), the recommendation of the concerned rural municipality
or urban municipality shall be submitted.
While requesting for an additional time –limit due to the circumstance referred to in clause (vi), the
recommendation of the rural municipality or urban municipality concerned with the place or stoppage
of means of transport has taken place, shall be submitted.

16. Purchase of Under Invoiced Goods (Dec 2017/2020, 2.5 Marks)


Answer
Where the tax officer suspects there is under-invoicing in case of supply of goods or services, he has
two options as follows:
i. Taxable value shall be re-determined based on market value and VAT shall be assessed by tax
officer, and
ii. buy or cause to buy such goods at the price mentioned in the invoice

Where a person shows the sales of goods at value lower than the prevailing market price through an
under-invoicing scheme, the tax officer may suspend the further transaction of similar remaining stock
of goods and may buy or cause to buy such goods at the price mentioned in the invoice.
Where a person maintaining such stock of goods deny to sell such goods at under-invoiced price, the
tax officer may seize such goods and compute the value of goods taking the base of under-invoiced
price and make payment to the person as and when the person desires to receive it.
The goods acquired or cause to be acquired as above may be sold or cause to be sold at such price and
process as determined by the Director General.

17. Taxable Value for Imported Goods (Dec 2017/2020/June 2020), 2.5 Marks)
Answer
Taxable value for the purpose of import shall be landed cost up-to border (cost paid to vendor, transit
cost or other cost) plus duty charged by the Customs Officer except VAT, which is Transaction Value
determined for the purpose of Customs Duty u/s 13, 14, and 15 of Customs Act plus Customs Duty plus
Excise Duty plus any other duty applicable at customs frontier.

18. Reverse Charge (Dec 2016/June 2021, 2.5/2 Marks)


Answer
As per Value Added Tax Act, 2052, there are two specific circumstances when VAT is levied under
reverse charging principle:
i. Import of Service [Sec. 8 (2)]
Any person, whether registered or not, in Nepal receiving service from person outside Nepal (the
person who is not registered for VAT purpose in Nepal) shall pay VAT for that service.
VAT is levied on taxable value determined by applying Sec. 12 of the Act.

The Institute of Chartered Accountants of Nepal 470


CAP-II Paper 7 - Income Tax and VAT
VAT shall be collected by the service recipient at such time, which is earlier of following:
a. When the service is rendered by the service provider, or
b. When the payment in respect of service is made
When the service recipient pays VAT under this provision in tax office, the tax so paid can be claimed as
input tax credit even without any VAT invoice, subject to other provisions of the Act.

ii. Construction by awarding contract to unregistered person [Sec. 8 (3)]


Any person (registered or not) in Nepal engaged in construction of buildings, apartments, shopping
malls or similar constructions for commercial purpose, the cost of which is more than Rs. 5 millions
shall collect VAT on the construction cost from themselves and deposit it to Revenue Authority in case
the construction work is not carried out through a registered person.

For this purpose, Commercial Purpose means construction of building, apartment, shopping complex or
similar other structures prescribed by IRD for the purpose of sales or using such asset by accounting the
asset as current or fixed asset.

19. Conditions for zero rate of Tax (Dec 2016, 2.5 Marks)
Answer
The conditions for Zero rate of Tax is as follows:
i. Goods exported from Nepal: Where the supply of goods is proved as follows:
a) Goods exported outside Nepal; or
b) Goods shipped as stores on an international flight, the destination of which is outside Nepal; or
c) Goods put on board in an international flight, the destination of which is outside Nepal, for
retail sale or supply or consumption;
ii. Services to be supplied to Persons outside Nepal:
a) A supply of services by a person residing in Nepal to a person outside Nepal that has no
business transaction, business representative or legally recognized agent in Nepal.
b) A supply of goods or services by a person who is residing and is registered in Nepal to a person
who is residing outside Nepal.
iii. Goods or services imported by a person or mission enjoying diplomatic privileges and a person
serving in a diplomatic mission enjoying tariff/duty privileges, on the recommendation of the
Ministry of Foreign affairs, Government of Nepal.
iv. Where any previous treaty or agreement provides for the sales tax exemption on imports, and local
purchase is made from the registered taxpayers, on the recommendation of the concerned project,
the facility of zero rates shall be provided on such supplies, so long as the treaty or agreement is in
effect.
v. Sales of the raw materials and goods manufactured to industries established in Special Economic
Zones pursuant to the laws in force.
vi. Where, on recommendation of the Alternative Energy Promotion Centre, the battery used in the
equipment generating energy from the solar power is produced by any domestic industry and is to
be supplied by that industry, the facility of the zero rate shall be provided to that industry on
recommendation of Alternative Energy Promotion Center and in accordance with the procedure
specified by the Department.

The Institute of Chartered Accountants of Nepal 471


CAP-II Paper 7 - Income Tax and VAT
vii. Where any machinery, equipment, tools and their spare parts , penstock pipes, or iron sheets used
in making thereof required for hydropower projects are produced by any domestic industry and are
to be supplied by that industry, the facility of zero rate shall be provided to that industry on that
transaction, on the recommendation of the Alternative Energy Promotion Centre, in the case of the
project that is operated with the approval of that center, and on the recommendation of the
Department of Electricity Development, in the case of the one other than that operated with the
approval of Alternative Energy Promotion Center in accordance with the procedures specified by
the Department.
viii. The Value Added Tax paid on raw materials used for the manufacturing of painting, handicrafts,
carving and similar other handicrafts by a industrialist in Cottage and Small Scale Industry within
Nepal are exported through an approved Export Trading House of Nepal shall be refunded after
fulfilling the procedures specified by the Inland Revenue Department.
ix. The Value Added Tax paid on the import or local purchase of scooters used by People with
Disabilities shall, if such scooters are registered in their name in the Office of the Transport
Management, be refunded on the recommendation of the Ministry of Women, Children and Social
Welfare, or the Chief District Officer of the concerned district, and in accordance with the
procedures as specified by the Department. If such goods are sold to any persons other than the
People with disabilities, the refunded Value Added Tax shall be recovered.
x. In case any machinery, equipment and construction materials as per approved master list required
for the purpose of operation of projects under bilateral or multilateral treaty that have been
concluded with prior approval of Ministry of Finance in terms of tax exemption and that are
manufactured by domestic industries, if supplied to projects directly or through contractor of such
projects, such supply can be treated as zero rated supply as per the procedure prescribed by the
Department.

20. Temporary registration for VAT (June 2016, 2.5 Marks& Dec 2011, 5 Marks)
Answer:

Temporary Registration as organizer or Participant as exhibitor of Fair, Exhibition, etc.


[sec. 10Ka]
Any fair, exhibition, etc. shall be organized by a registered person. Any person who wants to take part
as exhibitor in such event shall also be registered person, unless the transaction is exempt from VAT.
In case any unregistered person desires to participate as exhibitor, there shall be temporary
registration specifically for the purpose of conducting taxable transaction in such event.
Existing registered person can transfer goods for transaction to the place of exhibition or fair.
Any unregistered person desiring to engage in any short-term taxable transactions of goods or services
at fair, show, demonstration, display, exhibition etc., shall make an application along with the
following:
(a) the recommendation from the organizer in the concerned Tax Office or Taxpayer’s Service Office,
and
(b) Proof of deposit of amount equal to 2% of estimated revenue from the event
The tax officer at concerned Inland Revenue Office or Taxpayers Service Office shall issue a
temporary registration certificate once the application for registration along with deposit is
received.

The Institute of Chartered Accountants of Nepal 472


CAP-II Paper 7 - Income Tax and VAT

21. Transfer of business (June 2016, 2.5 Marks)


Answer
Under either of the following two conditions, value added tax is not be applicable on the transfer of
ownership of a business:
a. When a registered person transfers its business to any other registered person; or
b. A business is transferred to any inheritor after the death of an owner.

Formalities to obtain the exemption


Where the transfer as above is agreed between the buyer/seller or agreed by the inheritor, the parties
shall fill up a form (Annex 4 of VAT regulation) and submit it to concerned Inland Revenue Office/
Taxpayers’ Service Office within seven days of such transfer.

Responsibility of Transferee to maintain safe custody of records as per Law and to bear liabilities of
Tax on assessments of Transactions conducted before such Transfer
In case there is transfer of ownership of business or industries that is registered or required to be
registered under law, the transferee shall bear the tax obligation in relation to the period before the
transfer was affected.

The transferee is also responsible to safely retain the accounts, books of accounts, and documents in
relation to the period before or after such transfer of industry or business so transferred until such
period as prescribed (i.e. six years from the end of tax period)

Where the transaction has been transferred, the rights, power and obligations of the transferor shall be
transferred to the transferee.

The Tax Officer may, summon both the parties related to transfer in his/her presence and give them
necessary instructions in relation to the obligations to be fulfilled by them under the Act and these
rules. It shall be the duty of both the parties to follow the directions so given

22. Input Tax Credit (Dec 2014, 5 Marks)


Answer
The input tax credit, as per Value Added Tax Act, 2052, follows invoice-credit method. In respect of
domestic purchase, there shall be proper value added tax invoice, not an abbreviated tax invoice to claim
input tax credit.

Input tax credit in respect of import, is allowed based on import documents and vouchers proving
payment of VAT at Customs Frontier.

Similarly, input tax credit in respect of VAT paid as per Sec . 8 (2), 12Ka and 15 (3), is allowed based on
vouchers proving payment of VAT.

The Institute of Chartered Accountants of Nepal 473


CAP-II Paper 7 - Income Tax and VAT
There are four different types of Tax Credit:
(i) Full Tax Credit
Where an input is wholly used for the purpose of taxable output, tax paid for such input can be claimed
as input tax credit.

(ii) Proportionate Tax Credit


Where the use of input in production of output cannot be determined specifically, i.e. the input has
been used both for taxable output and non-taxable output, VAT paid on such input to the extent input
is used for taxable output can be claimed as input tax credit.

The ratio of credit claim is taxable sales to total sales.

(iii) Partial Tax Credit


40% of VAT paid on purchase of automobiles used for administrative purpose by VAT registered person
dealing only in taxable items can be claimed as VAT credit.

(iv) No Tax Credit


There shall not be any VAT credit on input, in any of the following circumstances except when the
person's main business is dealing in items mentioned in (b) to (e) below:
a. In case output (Intended or actual sales) is exempted from VAT as per Schedule 1, VAT paid on
purchase is not allowed for credit as per Sec. 5(3).
b. VAT paid on Entertainment expense is not allowed for credit
c. VAT paid for consumption of beverages (soft drink, juice or similar).
d. VAT paid for consumption of liquor items (beer, wine, whiskey, or similar).
e. VAT paid for consumption of diesel, petrol or LPG

23. Applicability of VAT on import of goods and services (Dec 2011, 5 Marks)
Or
Levy of VAT on Import (Dec 2006, 3 Marks, CA Inter)

Answer:
VAT is applicable on import of goods or services.

VAT on Import of Goods:


In case of import of goods, respective customs office shall collect VAT on taxable value.

Taxable value for the purpose of import shall be landed cost up-to border (cost paid to vendor, transit
cost or other cost) plus duty charged by the Customs Officer except VAT, which is Transaction Value
determined for the purpose of Customs Duty u/s 13, 14, and 15 of Customs Act plus Customs Duty plus
Excise Duty plus any other duty applicable at customs frontier.

VAT on Import of Service:


If a person imports service, VAT shall be paid under reverse-charging mechanism as per Sec. 8 (2).

The Institute of Chartered Accountants of Nepal 474


CAP-II Paper 7 - Income Tax and VAT
Any person, whether registered or not, in Nepal receiving service from person outside Nepal (the
person who is not registered for VAT purpose in Nepal) shall pay VAT for that service.
VAT is levied on taxable value determined by applying Sec. 12 of the Act.
VAT shall be collected by the service recipient at such time, which is earlier of following:
(a) When the service is rendered by the service provider, or
(b) When the payment in respect of service is made

When the service recipient pays VAT under this provision in tax office, the tax so paid can be claimed as
input tax credit even without any VAT invoice, subject to other provisions of the Act.

24. Electronic Cash Register (Dec 2008, 1 Mark, CA Inter)


Answer
A taxpayer may issue invoice electronically with prior approval from the Department. The Department
may publish a public notice to cause the taxpayers prescribed in the notice for compulsory issuance of
electronic invoice and order to link the electronic medium of the taxpayer to Central Bill Monitoring
System of Department.
Department shall develop and implement a procedure in relation to integrity and security of software
or equipment that issue electronic invoicing. The concerned Producer, distributor and user shall comply
such procedures.

25. Non-Filer of VAT Return (Dec 2008, 1 Mark, CA Inter)


Answer
As per sub-section (8) of Section 17 of VAT Act, 2052, in case of any person registered under the VAT
do not file VAT return for more than 6 months, then any carry forward VAT credit may be withheld and
the registration may be suspended.

26. Unjust Enrichment (Dec 2008, 1 Mark, CA Inter)


Answer
In the case of indirect taxes, such as VAT, the burden of tax is borne ultimately by the customer. The
seller of the goods shifts the burden of tax to the consumer. Thus, it could happen that having already
transferred the burden of tax to the buyer, a seller of the goods or service may lodge claim for refund of
duty and refund if granted results in double benefits to the seller. This is known as unjust enrichment.
Any person who has collected the tax in the nature of indirect taxes and shifted it to the customer, must
be paid to the government and if found refundable, it should be refunded to the customer and the seller
cannot retain it with him.

27. VAT on sale of used goods (Dec 2006, 3 Marks, CA Inter)


Answer
Taxable value for persons dealing in secondhand or used materials is the difference of sales amount and
the cost of sales that includes the VAT paid on it.
Taxable value in case of used goods (for dealer of used goods:
Selling value of the goods- Cost of goods (where VAT paid on such costs is part of cost)
Further, if a registered person not a dealer of used goods sells used goods, the taxable value shall be cash
consideration received or receivable after due consideration of trade discount and trade rebate.

The Institute of Chartered Accountants of Nepal 475


CAP-II Paper 7 - Income Tax and VAT
28. Registration of joint venture (June 2021, 2 Marks)
Answer:
If two or more than two persons establish a joint venture for operating taxable transaction for a certain
period, such persons shall file an application for registration of the joint venture to the Tax Officer of
one of the Tax Office among the Tax Offices wherein such persons establishing the joint venture are
registered.
The Tax Officer shall issue registration certificate in a prescribed format upon receiving application.
Joint venture thus registered shall have to deregister the registration upon expiry of the time
prescribed in the agreement entered into for establishing the joint venture.
Persons who are parties in the joint venture shall be jointly or severally liable for the purpose of
payment of tax liability of the joint venture they are party to.

29. Confidentiality of taxpayer’s documents (June 2021, 2 Marks)


Answer:
Tax related documents or other information received from any person shall not be disclosed or
published except in the following cases:

(i) To inform the officer collecting revenue in the course of protecting the revenue of Government
of Nepal,
(ii) To produce as evidence in the court pursuant to the laws relating to revenue in the course of
proving the liabilities of the taxpayer
(iii) To maintain as a part of public record in the course of proceedings at the Revenue Tribunal
or a court of law"

30. Advance Ruling (June 2021, 2 Marks)


Answer:
As per section 32(Ka) of VAT Act, if any person makes an application in writing to the department for the
clarification on any of the confusion related to any provision of the act for effective the application of
this act, the department may issue their opinion in writing to the person seeking such advance ruling.
Notwithstanding anything contained in section 32(1), department may issue advance ruling except on
such matters which are pending before the courts for decision or matters already decided by courts.

31. Jeopardy Assessment (Dec 2021, 2 Marks)


Answer:
If there is reason to believe that collection of tax is in jeopardy by way of person leaving Nepal
or transferring property or removing or concealing assets tax officer with approval from the DG
may immediately assess and collect the tax due, or about to become due.

32. Circumstances under which Tax Officer can assess tax (Dec 2021, 2 Marks)
Answer:
A Tax Officer may make an assessment in any of the following circumstances:
(a) If the tax return is not filed within the time limit;
(b) If an incomplete or erroneous tax return is filed;

The Institute of Chartered Accountants of Nepal 476


CAP-II Paper 7 - Income Tax and VAT
(c) If a fraudulent tax return is filed;
(d) If the Tax Officer has a reason to believe that the amount of tax is understated or otherwise
incorrect.
(e) If the Tax Officer has a reason to believe that the price of supply is under-invoiced.
(f) If supply is made within the group company by under-invoicing,
(g) If a person with obligation to register, transacts without registration,
(h) If sale is made without issuing invoice,
(i) If tax is collected by unregistered person,
(j) If tax is not filed pursuant to Sub-section (2) or (3) of Section 8,
(k) If conditions under Sub-section (4) of Section 17 are prevalent.

33. Valuation of Closing stock (Dec 2021, 2 Marks)


Answer:
Valuation of closing stock of a business for income year is done at a lower of the following:
Cost of closing stock that remains at the end of year or
Market price of the closing stock at the end of year;

Where identification of each items stock is not possible IT Act permit first in first out (FIFO) OR
weighted average method for valuation of closing stock.

The Institute of Chartered Accountants of Nepal 477


CAP-II Paper 7 - Income Tax and VAT

Chapter 7: Miscellaneous
1) Finance Manager of ABC Hardware is worrying about the price adjustment with its
supplier. He had heard that he can do the price adjustment through the use of Debit/Credit
Note. You are requested to advise him regarding the content of Debit/Credit Note as per
VAT Rules. (Dec 2014, 5 Marks)
Or
What are Debit / Credit Note? (Dec 2012, 4 Marks)
Or
Discuss the provision of Debit and Credit Note in VAT Rules. (Dec 2010, 5 Marks)

Answer
A person shall issue debit note or credit note to change the value of goods in issued invoice.
Credit Note is issued to credit the debit balance of a debtor, i.e. the person issuing invoice issues a credit
note and vice-versa. Credit note is a crucial matter in tax accounting as many countries has policy of using
pre-printed credit notes issued by government. But, such rule has not been in effect in Nepal.
Debit or credit note include following information:
a. Serial Number of the debit or credit note,
b. Date of issue,
c. Name, address and PAN of the supplier,
d. Recipient's name, address, and PAN if a registered person,
e. Serial number and date of the tax invoice concerned,
f. Particulars of the goods or services and reason of issuing to credit or debit,
g. Amount credited or debited,
h. Tax amount credited or debited.

2) What are the records to be maintained by a registered person dealing in used or second
hand materials? How the tax is assessed in such case? Answer with reference to the Value
Added Tax Rules, 2053. (June 2013, 5Marks)
OR
What are the records to be maintained by registered dealer dealing in used or second hand materials?
(Dec 2010, 5 Marks)

Answer
The Dealers of USED GOODS shall keep separate records for each deal of used goods. In case a registered
person dealing used goods is found not to have satisfactorily maintained the records, Tax Officer may
impose VAT on the total selling price of the goods sold by such taxpayer, and the tax officer may issue a
written order requiring them to pay such tax along with the next tax return.
The following are the matters to be maintained in Sales and Purchase Book of Dealer of Used Goods:
In relation to Purchase
i. Date of purchase
ii. Information as to the complete details of the goods

The Institute of Chartered Accountants of Nepal 478


CAP-II Paper 7 - Income Tax and VAT
iii. Purchase price excluding VAT
iv. Tax Rate
v. Tax Amount
vi. Total Amount paid

In relation to Sales
i. Date of Sales
ii. Selling Price excluding VAT
iii. Difference between Selling Price & Purchase Price
iv. Rate of Tax
v. Tax Amount
vi. Total Amount Received

3) What are the records that are to be maintained by the taxpayer as per rule 23 of VAT rules,
2053? (Dec 2012, 5 Marks)
OR
What are the types of accounts and records to be kept by a taxpayer under VAT Act, 2052? What are
their contents? (June 2010, 5 Marks)

Answer
A taxpayer has to maintain the following records:
1. VAT Account
The format of VAT account prescribed in Schedule 7 of Regulation. It is summary of purchase or import
and sale or export including input VAT and Output VAT.
2. Purchase book
According to Form prescribed in Schedule 8 of Regulation, registered person and unregistered person
shall record all transaction of purchase and import in this book. This record is in mandatory use. In
Purchase book, purchases on which input tax credits (even overheads and consumables) is claimed shall
be recorded as purchase. Goods or services without VAT or with VAT, for which input tax credit is not
allowed, are recorded as exempted purchase.
3. Sales book
According to Form prescribed in Schedule 9 of Regulation, registered person and unregistered person
shall record all transaction of sale and export in this book. This record is in mandatory use.

4. Other Records
Apart from above mentioned statutory VAT books, other documents (or computerized records if allowed
by Tax Officer) to be kept by a person are as follows as per Rule 23:
a. Records regarding transactions, cash, etc
b. Copies of Tax Invoices and Abbreviated Tax Invoices
c. Tax Invoices received on purchases
d. Documents regarding import and export
e. Records and copies of Debit or Credit Notes

The Institute of Chartered Accountants of Nepal 479


CAP-II Paper 7 - Income Tax and VAT
5. Register of free or sample goods as per Rule 24
A registered person shall maintain separate records for any received or distributed free goods or samples.
The Rule is silence in regards of records for free goods or sample received or distributed by an
unregistered person.

4) Explain the term “Abbreviated Tax Invoice” as per Value Added Tax Act, 2052. (June 2012,
5 Marks)
Answer
A retailer may issue abbreviated tax invoice after an expressed written permission from tax officer. An
application must be filed by the retailer to obtain prior approval of tax officer. Tax officer has the right
to accept or deny such permission.

The retailer cannot issue an abbreviated tax invoice in respect of sales exceeding Rs. 10,000 including
VAT and other duties. It is the duty of the retailer to issue normal VAT invoice, if the buyer asks for the
normal tax invoice.

The buyer cannot claim input tax credit through the use of Abbreviated Tax Invoice.

The retailer may not show the name of each goods or rate or VAT amount in the issued abbreviated tax
invoice. However, the particulars of individual goods shall be explicitly mentioned.

The person issuing Abbreviated Tax Invoice shall maintain following records, in the event of failure of
which the permission to issue such invoice may be scrapped:
a. Copy of original invoice,
b. In case of transaction through maintaining copy of till roll, the total of the transaction amount, and
c. Record of each transaction including VAT.

5) List occasions when purchase and sales books are required to be certified by the VAT
authorities (June 2009, 3 Marks)
Answer
The sales and purchase book shall be certified by VAT authorities in following occasions:
a. At the time when the registered person submits an application to the office for the certification;
b. During the period of tax audit or examination, or
c. At the time of inspection.

6) What is the minimum information that must be accompanied in the VAT invoice issued?
What consequences will follow in the case of the required particulars is not properly given
while issuing invoice? (June 2009, 5 Marks, CA Inter)
Answer
As per rule 17 of the Value Added Regulation, 2053 all the sellers conducting transactions of taxable
goods or service shall issue a tax invoice stating the name, address, permanent account number, date,
serial number and particulars of the goods (model, brand, Harmonized number) .

The Institute of Chartered Accountants of Nepal 480


CAP-II Paper 7 - Income Tax and VAT
In the case of the tax invoice issued without giving full description, tax shall be collected on the basis of
prevailing highest market price. (Rule 17(4) of the Value Added Tax Rules, 2053 as amended by seventh
amendment 2062/04/01).
Similarly, in the case of purchaser also the input tax credit under rules 39 shall not be granted.

The Institute of Chartered Accountants of Nepal 481

You might also like